Vous êtes sur la page 1sur 409

NOTES AND CASES IN CONSTITUTIONAL LAW

Prepared by:

ATTY. LARRY D. GACAYAN Professor (Constitutional Law Review, Constitutional Law I, Constitutional Law II and Wills and Succession) COLLEGE OF LAW UNIVERSITY OF THE CORDILLERAS Baguio City MARCH 2012 PRE-BAR REVIEWER

CPRS PRE-BAR REVIEW CENTER (Baguio City, Cagayan de Oro City, Zamboanga City, Iloilo City and Davao City)

EXCELLENT PRE-BAR REVIEW CENTER (Baguio City, Naga City, Cebu City and Tacloban City)

POWERHAUS PRE-BAR REVIEW CENTER (Baguio City, Manila, Santiago City, San Fernando City, [LU] and Tagbilaran City)

HOLY TRINITY COLLEGE PRE-BAR REVIEW CENTER (General Santos City)

COSMOPOLITAN BAR REVIEW CENTER (Baguio City)

LEX REVIEW CENTER University of Pangasinan Dagupan City

CHAPTER I FUNDAMENTAL POWERS OF THE STATE (Police Power) a. ERMITA-MALATE HOTEL VS. MAYOR OF MANILA, July 31, 1967 (Requiring customers of motels to show IDs, etc.) b. WHITE LIGHT CORPORATION VS. CITY OF MANILA, January 20, 2009 (Prohibiting short time)

c. CITY OF MANILA VS. JUDGE LAGUIO, 455 SCRA 308 (Requiring motel owners to convert their motels to flower shops, restaurants, antique shops, souvenir shops, handicrafts display centers; art galleries; records and music shops, coffee shops, etc., within 3 months or transfer the motels to other parts of the City of Manila. It is not a valid exercise of police power because if it is immoral to operate said motels in the ErmitaMalate area, the same will likewise be immoral to operate in any other part of the City of Manila) d. DE LA CRUZ VS. PARAS, 123 SCRA 569 (Nightclubs in Bocaue, Bulacan were ordered closed and prohibited by a Municipal Ordinance. The SC held that nightclubs are not illegal per se and therefore, they could not be prohibited but merely to be regulated) Requisites of a valid ordinance: 1. 2. 3. 4. 5. 6. must not contravene the Constitution or any law; must not be unfair or oppressive; must not be partial or discriminatory; must not prohibit but regulate trade; must be general and consistent with public policy; and must not be unreasonable.

e. JMM Promotions vs. CA, 260 SCRA 319 (Artists Record Book must be accomplished with TESDA before women could be deployed to Japan. Otherwise, if they have no formal training before going to Japan, they will end up as prostitutes. It started after the death of Maricris Sioson in Japan) f. ICHONG VS. HERNANDEZ, 101 Phil. 1155 (Retail Trade Nationalization Act prohibiting foreigners from engaging in retail trade is a valid exercise of police power) g. US VS. TORIBIO, 15 Phil. 85 (Slaughtering of carabaos was prohibited by a law which was declared constitutional to promote general welfare considering the importance of carabaos to our country whicl is agriculture-based) h. VELASCO VS. VILLEGAS, February 13, 1983 i. AGUSTIN VS. EDU, 88 SCRA 195 [Early warning device promotes public safety to the motoring public] j. TAXICAB OPERATORS VS. BOT, 119 SCRA 597 (Phasing out of taxicabs over 6 years old valid . However, even taxicabs more than 6 years are still allowed outside MM because they are not as dilapidated as those operating in Metro manila which are being used 24 hours daily) k. BAUTISTA VS. JUINIO, 127 SCRA 329 (Law prohibiting the use of Heavy and Extra Heavy vehicles during weekends and holidays when there is energy crisis is validfor the general welfare) l. ASSOCIATION OF SMALL LANDOWNERS VS. SECRETARY OF AGRARIAN REFORM, 175 SCRA 343 (CARP Law is valid. Combination of police power and power of eminent domain) m. DECS VS. SAN DIEGO, 180 SCRA 533 n. VILLANUEVA VS. CASTANEDA, September 21, 1987 (Permit earlier issued by the Municipal Mayor for the vendors to have a Talipapa along the National Highway of San Fernando City, Pampanga may be validly revoked by the new mayor and could be justified under the police power to promote public health) o. PRC vs. De Guzman, et al., June 21, 2004 (Fatima College of Medicine graduates had unusually and exceptionally high in the 2 most difficult subjects of the exam, i.e., Biochemistry and Obstetrics and Gynecology. The NBI investigation revealed that they had early access to test
2

questions. They cant compel the PRC to give them their licenses. Valid police power measure to safeguard the heath and general welfare of the people) p. DIDIPIO VS. GOZUN, 485 SCRA 586 (Distinctions between police power and power of eminent domain and taxation) 5-a. Not a valid exercise of police power a. CITY GOVERNMENT OF QUEZON CITY VS. ERICTA, 122 SCRA 759 (Requiring private cemeteries to set aside 6% of their land area to be given free as burial places for paupers) b. YNOT VS. IAC, 148 SCRA 659 (Inter-provincial transport of carabaos and carabeefs. The LOI which gives the Director of Animal Industry of the Chairman of the National Meat Commission to dispose of the confiscated animals in any manner he deems fit as unconstitutional) c. DELA CRUZ VS. PARAS, 123 SCRA 569 (Prohibition on the operation of nightclubs in Bocaue, Bulacan)

CHAPTER II Section 1 DUE PROCESS 1. Requisites of judicial due process. CENTRAL MINDANAO UNIVERSITY VS. EXECUTIVE SECRETARY, G.R. No. 184869, September 21, 2010 There is violation of the right of CMU to due process of law when the President through a Presidential Proclamation took away lands of the University intended for its operation and given by the President to indigenous people of Mindanao---without notice or hearing. a. BANCO ESPANOL VS. PALANCA, 37 Phil. 92 (Four (4) Requisites of judicial due process) b. IMELDA MARCOS VS. SANDIGANBAYAN, October 6, 1998 (Deciding a case with the participation of another justice who is not a member of the division that tried her case violates her right to due process. Likewise, the fact that Justice Garchitorena asked 179 questions to the 73 by the Prosecutor clearly shows partiality and violation of her right to due process. [Rental of P102, 760.00 per month then sub-leased to P734,000.00 per month for 25 years after only 19 days] c. PEOPLE VS. OPIDA, June 13, 1986 [The Supreme Court held that the right of the accused to due process of law and impartial trial were violated when it was the judge who conducted the cross-examination of the accused and his witness instead of the Prosecutor coupled with his sarcastic and insulting remarks and ended with the question, Do you want me to dictate the decision now? The judge likewise required the accused to remove his shirt and described for the record all the tattoos found on his body. Clearly, the judge had allied himself with the prosecution] d. PEOPLE OF THE PHILIPPINES VS. BENANCIO MORTERA, G.R. No. 188104, April 23, 2010 [There is no violation of the right to due process and impartial trial as a result of the comment of the Judge that if you are not telling the truth to your own lawyer, how would I
3

know that you are telling the truth now as a result of his change of defense from a negative defense to self-defense in a murder case. The comment of the judge shall be considered based on the entirety of his questions and the circumstances surrounding said utterance.The accused described the clarificatory questions of the judge as prosecutor-like conduct and therefore, lacked the cold neutrality of an impartial judge. e. DELGADO VS. CA, November 10, 1986 (The accused was represented by a non-lawyer during the trial of her case for estafa. She was not aware that Atty. Ico is not a member of the bar. She is entitled to a new trial since her right to due process was violated) f. Consulta vs. People, February 12, 2009 (He was represented during the presentation of prosecutions evidence by Atty. Jocelyn P. Reyes who turned out to be a non-lawyer. He was represented by a PAO Lawyer during the presentation of his evidence. No violation of his right to due process since he was represented by a real lawyer during the presentation of his evidence.) g. DBP VS. CA, January 29, 1999 h. JAVIER VS. COMELEC, 144 SCRA 194 i.DAVID VS. AQUILIZAN, 94 SCRA 707 J. ZAMBALES CHROMITE MINING VS. CA, 94 SCRA 261 (THE SECRETARY OF THE DENR may not review his own decision when he was still the Director of Mines without violating the parties right to due process of law k. ANZALDO VS. CLAVE, 119 SCRA 353 (The Deputy Executive Secretary may not validly review his very own decision as the Chairman of the Civil Service Commission) l.MAYOR ALONTE VS. JUDGE SAVELLANO, 287 SCRA 245 (December 17, 1997 , there was a notice of Promulgation of Decision and on December 18, 1997, a Decision of conviction was promulgated without trialthe right of mayor Alonte to due process of law was violated) TAN VS. TAN, G.R. No. 167139, February 25, 2010

A party cannot complain of violation of her right to due process when the court awarded custody of their children to the husband because she was notified but failed to appear during the hearing of said motion. TAN VS. JUDGE MARIA CLARITA TABIN, MTC 4, BAGUIO CITY, A.M. No. MTJ-09-1729, January 20, 2009

The Judge violated the right of the petitioner to due process when she ordered for her arrest in a BP 22 case for allegedly failing to appear in court during the scheduled hearing EVEN THOUGH SHE IS FROM QUEZON CITY AND SHE WAS NOT NOTIFIED. 2. Procedural due process before administrative bodies a. ATTY. ROMEO ERECE VS. MACALINGAY, ET AL., G.R. No. 166809, April 22, 2008 (No right to cross-examine the complainant and witnesses nor a formal hearing required as long as already given the opportunity to present evidence, i.e., counter-affidavit and affidavit of witnesses) b. DATUFAX MANGUDADATU VS. HRET, December 18, 2008 (Summons by registered mail is not valid. Violation of the right to due process) c. DEP ED VS. CUANAN, December 16, 2008 (Not furnished a copy of the MR of DepED before the Civil Service Commission even though the private
4

complainants did not question the Decision of the CSC. The right to due process of Cuanan was violated when the CSC reversed its decision based on the Motion for Reconsideration where no notice was given to Cuanan) d. ANG TIBAY VS. CIR, 69 Phil. 635 (Seven (7) cardinal primary requisites of due process before administrative bodies) 4. Procedural due process in disciplinary actions against students A. DE LA SALLE UNIVERSITY VS. COURT OF APPEALS, DECEMBER 19, 2007 (STUDENTS HAVE NO RIGHT TO DEMAND FOR CROSS-EXAMINATION OF THE COMPLAINANT AND HIS WITNESSES. IT IS SUFFICIENT THAT HE WAS GIVEN THE OPPORTUNITY TO FILE HIS ANSWER AND OTHER EVIDENCE. Due process is deemed complied with) B. GUZMAN VS. NU, 142 SCRA 706 (5 REQUISITES) 1. In administrative proceedings, does due process require that a party be assisted by counsel? LUMIQUED VS. EXENEA, 282 SCRA 125 JOSE ATIENZA ET AL VS. COMELEC and MANUEL ROXAS II, FRANKLIN DRILON AND NEREUS ACOSTA, G.R. No. 188920, February 16, 2010

There is no violation of the right to due process when the group of Drilon expelled Atienza and company without notice and hearing because administrative due process in accordance with Ang Tibay could be invoked only in bodies created by the State through which governmental acts or functions are performed. An administrative agency contemplates an authority to which the state delegates governmental powers for the performance of state function. The right to due process is meant to protect ordinary citizens against arbitrary government action but not from acts committed by private individuals or entities. The right to due process guards against unwarranted encroachment by the State into the fundamental rights of its citizens and cannot be invoked in private controversies involving private rights.

CHAPTER III Section 1 THE EQUAL PROTECTION CLAUSE ELEAZAR QUINTO VS. COMELEC, G.R. No. 189698, February 22, 2010, reversing the Decision dated December 1, 2009, Per CJ Puno (Mancuso vs. Taft was abandoned and reversed in US Civil Service Commission vs. National Association of Letter Carriers, AFL-CIO and Broadrick vs. State of Oklahoma) VICTORIA GUTTIERREZ VS. DBM, G.R. No.153266, March 18, 2010. [Inclusion of allowances and other fringe benefits for government workers in the national government, state universities and colleges, including those in the local government under the Compensation and Position classification Act of 1989 while those in the AFP and PNP did not, does not violate the equal protection clause because there is real and substantial distinction. Being charged of the actual defense of the country, uniformed personnel of the government are expected to be stationed virtually anywhere in the country. They are likely to be assigned to a variety of low, moderate and high cost areas. Since their basic pay does not vary on location, the continued grant of COLA is intended to help them offset the effects of living in higher cost areas.
5

LOUIS BAROK C. BIRAOGO, Petitioner, v. THE PHILIPPINE TRUTH COMMISSION OF 2010, Respondent., G.R. No. 192935 : December 7, 2010 (Executive Order No. 1 of President Benigno Aquino III creating the Truth Commission of 2010 to investigate reports of graft and corruption of such scale and magnitude that shock and offend the moral and ethical sensibilities of the people, committed by public officers and employees, their co-principals, accomplices and accessories from the private sector, if any, during the previous administration; and thereafter recommend the appropriate action or measure to be taken thereon to ensure that the full measure of justice shall be served without fear or favor. Is unconstitutional because it singled out the administration of President Gloria Macapagal Arroyo while not investigating the previous administrations as violative of the equal protection of the law. 1. HIMAGAN VS. PEOPLE, October 7, 1994 [Preventive suspension of a policeman charged of a criminal case up to his acquittal while other government officials like those facing graft charges are to be preventively suspended only for 90 days after arraignment is not violative of the equal protection clause, there being real and substantial distinction because policemen are armed and if they are allowed to do so during the pendency of his case, witnesses might be afraid to testify] 2. PHIL. JUDGES ASSOCIATION VS. PRADO, November 11, 1993 [Franking privileges for the courts removed while Congress and legislature were not. It is a violation of the equal protection clause] 3. Gumabon vs. Director of Prisons, 37 SCRA 420 [In 1950, they were convicted of rebellion complexed with murder. In 1954, accused similarly situated like Hernandez were sentenced to the penalty for rebellion only after the SC held that there is no such thing as rebellion complexed with murder. They are entitled to have their penalties be reduced , otherwise, their right to equal protection will be violated.] 4. PANFILO LACSON VS. SANDIGANBAYAN, January 20, 1999 [Only 1 chance of appeal from SB to SC by Certiorari while other accused have several appeals from MTC to RTC to CA then to SC. No violation of equal protection because there is real and substantial distinction between government employees and private employees or individuals] 5. Taxicab Operators vs. BOT, September 30,l982 [Theres a valid distinction when taxicabs in Baguio City and in Manila. The latter are used 24 hours daily while the former are used only for few hours] 6. Dumlao vs. COMELEC, 95 SCRA 392 [Over 65 years old are not allowed to ran for the same position held before. Valid because it infuses young blood to our political system] 7. UNIDO vs. COMELEC, 104 SCRA 38 [Free airtime to the President when he campaigned YES to the proposed amendments to the Constitution while proponents of NO shall pay. No discrimination since there is a real distinction. Marcos was addressing the people as the President of the Philippines, not as a head of a political party] CHAPTER IV Section 2 THE SEARCH AND SEIZURE PROVISION

NOTE: THE ANTI-TERRORISM LAW, RA NO. 9372, Section 18, 19 and 26 on arrest and detention.

NEW CASES:
6

SUSAN ESQUILLO VS. PEOPLE, G.R. No. 182010, August 25, 2010 Stop and Friskvalid exception to a Warrantless search and seizure If the person who claims that he was illegally arrested did not question the validity of her Warrantless arrest before her arraignment nor moved to quash the information but raised said issues for the first time on appeal, the same is already deemed waived. LEVISTE VS. JUDGE ALAMEDA, RTC MAKATI, G,.R. No. 182677, August 3, 2010 There is no waiver of the illegality of the arrest if the accused refused to enter a voluntary plea during arraignment and continued to question the validity of the preliminary investigation which resulted in the filing of the information in court. THERE ARE TWO (2) KINDS OF PROBABLE CAUSE 1. The executive determination of probable cause on whether a crime was allegedly committed and file a criminal case in court; and 2. Judicial determination of probable cause for the issuance of a warrant of arrest.

The place to be searched AS INDICATED in the warrant is controlling

PEOPLE VS. CA, 291 SCRA 400 [4 apartment doors at the rear of Abigail Variety Store in QC] As a general rule: Validity of warrantless searches and seizures as a result of an informers tip. PEOPLE VS. ARUTA, 288 SCRA 626 [On December 13, 1988, Olongapo Police received a tip8.5 kilos of marijuana] PEOPLE VS. MONTILLA, 284 SCRA 703 [Dasmarinas, Cavite Police received a tip on June 19, 1994The marijuana courier will alight at the waiting shed of Barangay Salitran, Dasmarinas, Cavite28 kilos of marijuana] P VS. CLAUDIO, 160 SCRA 646 [There is probable cause in the Warrantless search of a bag (behind him in a bus) by a NARCOM agent when he allegedly smelled marijuana therein. By reason of his training, he could smell marijuana and therefore, he has personal knowledge and therefore, probable cause was present making the search legal] PEOPLE VS. SPO3 SANGKI ARA, G.R. No. 185011, December 23, 2009..[buy-bust operation infront of St. Peters College in Toril, Davao City resulted in 25.6563 grams of shabu. c. General or roving warrants Read: 1. Stonehill vs. Diokno,June 19,1967 [General warrant for violation of CB Laws, TCC, NIRC and RPC]

NOTE: In TAMBASEN VS. PEOPLE, July 14, 1995 and People vs. CA, 216 SCRA 101, the Supreme Court described a search warrant for estafa, robbery, theft and qualified theft was described as SCATTER-SHOT WARRANT 2. Bache vs. Ruiz, 37 SCRA 823 [The clerk of court received the evidence of the applicant for a search warrant] 2-a. Pendon vs. CA, Nov. 16, 1990 [When the questions asked by the judge to the applicant are pre-typed, the search warrant is not valid since there could have been no searching questions] 3. Secretary vs. Marcos, 76 SCRA 301 [Illegal possession of firearm and violation of CB LawsGolden Buddha] 4. Castro vs. Pabalan, April 30, l976 [The search warrant is implemented in an adjoining Barrio of Bangar, La Union] 5. Asian Surety vs. Herrera, 52 SCRA 312 [Search warrant for estafa, falsification, tax evasion and insurance fraud is a general warrant despite two carloads of evidence seized] Define probable cause. Who determines probable cause? a.VICENTE LIM VS. HON. FELIX [Warrant of arrest issued by the RTC Makati based on the Resolution and the Information filed by the Fiscal is not valid since the record of the case with the affidavits of witnesses is still in Masbate] 1. Amarga vs. Abbas, 98 Phil. 739 1-b. Quintero vs. NBI, 162 SCRA 467 [Searching parties searched different rooms simultaneously thereby resulting in no witnesses in the other rooms searched, the search is not valid] 1-d. SOLIVEN & BELTRAN VS. MAKASIAR, NOVEMBER 18, 1988 (Note that this was widely criticized) [There is no need for the judge to examine the complainant and the witnesses face to face in order to determine probable cause. It is enough that he shall personally examine the affidavits of the complainant and the witnesses and if he is convinced that there is probable cause, he can validly issue the warrant of arrest] 2-a. P. vs. Villanueva, 110 SCRA 465 (The judge may validly refuse to issue warrants of arrest if he believes that there is no probable cause to for their issuancedespite the findings of probable cause by the filing Prosecutor since that was for purposes of filing only) 2-b. Placer vs. Villanueva, 126 SCRA 463 3. Tolentino vs. Villaluz,July 27,1987 4. Cruz vs. Gatan, 74 SCRA 226 5. Olaes vs. P., 155 SCRA 486 6. GERONIMO VS. RAMOS, 136 SCRA 435 [Warrants of arrest issued in 70 plus cases against several accused after less than 1 hour from receipt of the records of the cases are not valid. Judge could not have determined probable cause in so short a time] 7.JUAN PONCE ENRILE VS. JUDGE JAIME SALAZAR, ET AL., G.R.NO. 92163, June 5, 1990 [Warrant of arrest issued against Senator Enrile after1 hour and 20 minutes from receipt of the records of the case consisting of several thousands of pages is valid. There is no need to read all the affidavits or evidence therein. It is sufficient that he is convinced of the existence of probable cause] d. Warrantless searches and seizures--when valid or not. Is "Operation Kapkap" valid? Warrantless search of alleged obscene magazines.
8

Read: PEOPLE VS. MENGOTE, G.R. No. 87059, June, 1992 [Arrest without warrant because of a bulging tummy which looks like a gun tucked therein is not valid. There was no personal knowledge nor probable cause] PEOPLE VS. GO, 354 SCRA 338 [Search of a gun which could be seen tucked in the waist of the accused in a nightclub is valid. Likewise, the sachets of shabu seen on the front seat of the car of the accused when he opened the car is admissible under the plain view doctrine] MANALILI VS. PEOPLE, October 9, 1997 [The policemen saw several suspicion-looking men at dawn and when they approached said persons, they ran but were caught. The unlicensed firearm confiscated after the policemen search them is admissible] PEOPLE VS. DEL ROSARIO, July 10, 1994 [The policemen gave P100.00 to the informant to buy marijuana from the accused. After he returned with the marijuana, the policemen went to arrest the accused. The arrest is not valid since the same does not fall under Section 5 Rule 113. They have no personal knowledge] MALACAT VS. CA, 283 SCRA 159 [Mere suspicions by the police are not sufficient to validate a Warrantless search and seizure or Warrantless arrest] PITA VS. CA, 178 SCRA 362 [Requisites before the Mayor could confiscate magazines which are allegedly obscene] Warrantless Search and seizure by a private person. PEOPLE VS. MENDOZA, 301 SCRA 66 SILAHIS INTERNATIONAL VS. SOLUTA, 482 SCRA 660 PEOPLE OF THE PHILIPPINES VS. ANDRE MARTI, G.R. NO. 81561, January 18, 1991 VALID WARRANTLESS SEARCH AND SEIZURE: 1. Search made incidental to a valid arrest a. NOLASCO VS. PANO, 139 SCRA 541 (One who was arrested on board a passenger jeepney may not be brought to her residence and then search the same. That could not be considered as search incidental to a valid arrest) b. P vs. Burgos, 144 SCRA 1 [The arrest of an alleged NPA member while plowing his field is not valid since he is not committing a crime. Likewise, the consent for the search for unlicensed firearm given by his wife is not valid] c. ESPANO VS. CA, 288 SCRA 588 (If accused was arrested on the street in front of his house selling prohibited drugs, the arresting officers may not search his house without warrant based on search incidental to a valid arrest rule. 2. Search of moving vehicles PEOPLE VS. BELEN MARIACOS, G.R. No. 18861, June 16, 2010 (Tip that the accused has with her marijuana on her bag at the topload of a passenger jeepney justifies the police authorities to conduct a valid Warrantless search since they have no more time to secure a search warrant)
9

10

a. b. c. d. e.

Carrol vs. US, 267 US 132 PEOPLE VS. LO HO WING, 190 SCRA 122 PEOPLE VS. MALMSTEDT, 198 SCRA 401 MUSTANG LUMBER VS. CA, 257 SCRA 430 PEOPLE vs. AMINNUDIN Y AHNI, JULY 6,1988 (TIP RECEIVED BY THE ILOILO CITY POLICE FROM INFORMANT IN ZAMBOANGA CITY THAT ACCUSED HAS MARIJUANA IN HIS BAG does not amount to probable cause because the Iloilo Police have no personal knowledge. Further, there was plenty of time to secure a search warrant from the court]

Seizure of goods concealed to avoid duties/taxes a. b. c. d. Papa vs. Mago, 22 SCRA 857 Pacis vs. Pamaran, 56 SCRA 16 HIZON VS. CA, 265 SCRA 517 PEOPLE VS. QUE, 265 SCRA 721

Seize of evidence in plain view a. PEOPLE VS. VALDEZ, 341 SCRA 25 [The policemen were informed that the accused had fully grown marijuana planted in his yard. They went to see for themselves the tip and found the same to be true. They arrested the accused and took pictures of him beside his marijuana plants and thereafter uprooted said plant, again with his pictures. There was no valid Warrantless search under the plain view doctrine since they were looking for it and not INADVERTENTLY COME ACROSS THE ILLEGAL ITEMS] a-1. ELENITA FAJARDO VS. PEOPLE, G.R. No. 190889, January 10, 2011 ..Plain view as a valid exception to a Warrantless searches and seizure.

b.

Under the plain view doctrine, objects falling in the plain view of an officer, who has a right to be in the position to have that view, are subject to seizure and may be presented as evidence. It applies when the following requisites concur: (a) the law enforcement officer in search of the evidence has a prior justification for an intrusion or is in a position from which he can view a particular area; (b) the discovery of the evidence in plain view is inadvertent; and (c) it is immediately apparent to the officer that the item he observes may be evidence of a crime, contraband, or otherwise subject to seizure. The law enforcement officer must lawfully make an initial intrusion or properly be in a position from which he can particularly view the area. In the course of such lawful intrusion, he came inadvertently across a piece of evidence incriminating the accused. The object must be open to eye and hand, and its discovery inadvertent. 7. 8. 9. 10. Harris vs. US, 390 US 234 PEOPLE VS. DAMASO, 212 SCRA 547 PEOPLE VS. VELOSO, 252 SCRA 135 PEOPLE VS. LESANGIN, 252 SCRA 213
10

11

When there is waiver of right or gives his consent; a. VEROY VS. LAYAGUE, 210 SCRA 97 (The consent given by the owner of the house to the soldiers to look for rebel soldiers does not justify them to search for unlicensed firearm inside the house by opening cabinets therein) b. Lopez vs. Commissioner, 65 SCRA 336 c. PEOPLE VS. DAMASO, 212 SCRA 547 (Consent given by the owner of the apartment for the police to search the room being rented to Basilio Damaso is not valid) STOP & FRISK or search in checkpoints 1. RICARDO VALMONTE VS. GEN RENATO DE VILLA, GR No. 83988, September 29, 1989 2. SUSAN ESQUILLO VS. PEOPLE, G.R. No. 182010, August 25, 2010 Stop and Friskvalid exception to a Warrantless search and seizure Read also the RESOLUTION ON THE MOTION RECONSIDERATION dated JUNE 15, 1990, 185 SCRA 665 a. PEOPLE VS. POSADAS, 188 SCRA 288 b. MANALILI VS. PEOPLE, October 9, 1997 c. MALACAT VS. CA, 283 SCRA 159 Search of Hot Logs under the Forestry Code MUSTANG LUMBER VS. CA, 257 SCRA 430 If the judge finds that there's probable cause, must he issue a warrant of arrest as a matter of course? 1. Samulde vs. Salvani, September 26, 1988 2. GOZO VS. TAC-AN, 300 SCRA 265 Searching questions a. DR. NEMESIO PRUDENTE VS. THE HON. EXECUTIVE JUDGE ABELARDO M. DAYRIT, RTC 33, Manila & People of the Philippines, GR No. 82870, December 14, 1989 (En Banc) [a SEARCH WARRANT WAS ISSUED BECAUSE OF A very reliable tip obtained by the police against Dr. Nemesio Prudente, President of Polytechnic University of the Philippines and implemented on November 1, 1987, a Sunday and All Saints Day despite Circular No. 19 of the SC b. Pendon vs. CA, Nov. 16, 1990 [Pre-typed questions as a basis of probable cause not valid. No searching questions] 6. Mata vs. Bayona, 128 SCRA 388 [The deposition of the complainant and the witnesses shall be attached to the search warrant as well as in the record of the case, without which, the search for masiao tickets is not valid] 9. Burgos vs. Chief of Staff, 133 SCRA 800 [Mere conclusions of law by the applicant that there are subversive documents at the offices of the newspaper WE FORUM during martial law does not justify a finding of probable cause] 12. Ponsica vs. Ignalaga, July 31,1987 [If the statements of the complainant and the witnesses of an applicant for a search warrant are mere generalities, mere conclusions of law and not positive statements of particular facts, the search warrant is not valid] 13. Aberca vs. Ver, April 15,1988 [The superiors, including Capt. Panfilo Lacson, are liable civilly for the illegal search conducted by their subordinates]
11

FOR

12

PEOPLE VS. GALVEZ, 355 SCRA 246 [The policemen arrested the accused because of what a private person told them, i.e., he sold marijuana to him. The arrest is illegal but could no longer question the validity of his arrest because he entered a plea during the arraignment] 1. Century Fox vs. CA, 164 SCRA 655 (The master tape must be presented if one alleges that the tapes to be seized are illegal and pirated] 2. COLUMBIA PICTURES VS. CA, 261 SCRA 144

CHAPTER IV-A THE RIGHT TO PRIVACY PLEASE NOTE: Section 7. Surveillance of suspects and interception and recording of communications (Except between lawyer and clients; doctors and patients; and journalists and their sources) Section 27. Judicial authorization to examine bank deposits and records 1. OPLE VS. TORRES, July 23, 1998 [National ID System (Admin. Order No. 308, December 12, 1996 of President Ramos) through biometrics technology where the people will have to furnish the government with their fingerprints, retinal scan, hand geometry, facial features and others to be stored in a super computer. Held: Since there is no safeguard that these informations will not be used illegally, the EO is unconstitutional. It falls short of assuring that personal information gathered from the people will be used for the specified purposes without violating the citizens right to privacy. 2. ZULUETA VS. CA, February 10, 1996 [The intimacies of husband and wife does not justify the latter from breaking cabinets in the clinic of the physician-husband and take the diaries, checks, greeting cards, pictures of his alleged paramours] 3. KMU VS. ERMITA, & BAYAN MUNA VS. ERMITA, April 19, 2006 & June 20, 2006 [Proclamation No. 420 of GMA requiring mandatory ID system is valid because it applies only to national government employees and within her power of control under Section 17, Art. VII of the Constitution] 4. SABIO VS. GORDON, October 17, 2006 [Limited right to privacy of government officials]

CHAPTER V FREEDOM OF SPEECH, PRESS, EXPRESSION, etc.

NOTE: VOID FOR VAGUENESS AND OVERBREADTH DOCTRINES APPLY ONLY TO FREEDOM OF EXPRESSION, NOT TO PENAL STATUTES. PEOPLE VS. SITON, G.R. No. 160364, September 18, 2009. The above doctrine could not be used to declare Art. 202 of the RPC on Vagrancy.
12

13

US vs. Bustos, 37 Phil. 731 Rule on criticisms on private individuals who are public figures Baguio Midland Courier vs. CA & Ramon Labo, Jr., 444 SCRA 28 PABLITO V. SANIDAD VS. COMELEC, G.R. NO. 90878, January 29, 1990 [The State could not dictate the time and place for a citizen to exercise his freedom of speech, expression or of the press UNLESS there is clear and present danger] De la Cruz vs. Ela, 99 Phil. 346 [Transferring the place of rally from the eastern part of the town plaza facing the Roman Catholic Church to the other end of the plaza for the Jehovahs Witnesses to rally is valid since there is clear and present danger like what happened in the previous occasions when there were fights between the Catholics and the members of Jehovahs Witnesses as a result of verbal attacks by the latter to the former] In re: Ramon Tulfo, March 19, 1990 In re: Atty. Emil Jurado, July 12, 1990 Elizalde vs. Gutierrez,76 SCRA 448 [News items of an on-going criminal proceedings in court is valid provided: (1) it must be a true and fair report of the proceedings in court; (2) it must be written in good faith; and (3) there must be no comments or remarks by the author. Policarpio vs. Manila Times, 5 SCRA 148 [Lady Lawyer charged in court] Lopez vs. CA, 34 SCRA 116 [Hoax of the Year---Fidel Cruz] Kapisanan vs. Camara Shoes, 11 SCRA 477 [Under protest written by an employee in the payroll should not be a ground to suspend her for 1 month. It is a valid exercise of freedom of expression] IN RE: Atty. Tipon, 79 SCRA 372 Obscenity; test of

Read: a. P. vs. Kottinger, 45 Phil. 352 b. P vs. GO PIN, August 8, 1955 c. Miller vs. California, 37 L. Ed. 2d 419 Tests of obscenity: a. Whether the average person applying to contemporary community standards would find the work appeals to prurient interest; b. Whether the work depicts or describes a patently offensive sexual conduct; c. Whether the work as a whole lacks serious literary, artistic, political or scientific value. d. Ginsberg vs. New York,390 U.S. 629 e. Pita vs. CA, 178 SCRA 362 [City Mayor Bagatsing could not just order the confiscation of copies of Pinoy Playboy because he considers the same as obscene magazine even without a search warrant. That will make him the complainant, prosecutor and judge rolled into one. He must: i. Secure a copy of the alleged obscene publication; ii. Apply for a search warrant; iii. Convince the court that it is obscene; and

13

14

iv. If the court issues a search warrant, that is the only time that he could go to the sidewalks and confiscate the said magazine. Freedom of assembly and to petition the government for redress of grievances

INTEGRATED BAR OF THE PHILIPPINES VS. MAYOR ATIENZA, G,R. No. 175241, February 24, 2010

The act of the mayor in denying the application for a rally permit by the IBP at the foot of Mendiola Bridge on June 22, 2006 from 230 pm to 530 pm and instead allow them to hold said rally at the Plaza Miranda is unconstitutional. It violates the right to freedom of expression and public assembly. This is so because under Section 6 [e] of BP 880 or the Public Assembly Act of 1985 which provides that if the Mayor is of the view that there is imminent and grave danger of a substantive evil warranting the denial or modification of the permit, HE SHALL IMMEDIATELY INFORM THE APPLICANT WHO MUST BE HEARD ON THE MATTER. The opportunity to be heard precedes the action on the permit, since the applicant may directly go to court after an unfavorable action on the permit. 1-a. Gesite vs. CA, 444 SCRA 28 [Concerted mass actions by government officials, teachers in this case, like walkouts, mass leaves, and other work stoppage or absence from work are prohibited because it is tantamount to strike which is prohibited to government employees. 1-b. Bayan vs. Ermita, 488 SCRA 1 (Calibrated Pre-emptive Response/Maximum Tolerance Rule) 1-c. Randy David vs. Ermita, 489 SCRA 160 2. Evangelista vs. Earnshaw, 57 Phil 255 3. Primicias vs. Fuguso, 80 Phil. 71 4. De la Cruz vs. Ela, 99 Phil. 346 [Transferring the place of rally from the eastern part of the town plaza facing the Roman Catholic Church to the other end of the plaza for the Jehovahs Witnesses to rally is valid since there is clear and present danger like what happened in the previous occasions when there were fights between the Catholics and the members of Jehovahs Witnesses as a result of verbal attacks by the latter to the former] 7. Reyes vs. Bagatsing, 125 SCRA 553; see guidelines but note the same was amended by the Public Assembly Act and IBP vs. Atienza Freedom from prior restraint

Read: 1. Newsounds Broadcasting Network vs. Hon. Ceasar Dy, April 2, 2009 [Closure of the Bombo AM and FM stations allegedly because their building is on an agricultural land and therefore, the Mayor did not issue a Business Permit is tantamount to prior restrain. City of Cauayan shall pay the radio stations P10M in damages and P1M in attorneys fees. 1-a. Bro. Eliseo Soriano vs. MTRCB, April 29, 2009 (Read also the dissenting opinion of Justice Antonio Carpio) [Suspension for 3 months of the program Ang Dating Daan and 3 months suspension of the host Bro. Eli Soriano is not prior restraint but subsequent punishment. August 10, 2004 at 10 p.m. Lehitimong anak ng demonyo, sinungaling.
14

15

Gago ka talaga Michael (referring to the host of the INCs program ang Tamang Daan) , masahol ka pa sa putang babae o di ba? Yung putang babae ang gumagana lang duon ay ang ibaba. Kay Michael, ang gumagana ang itaas, o di ba? Masahol pa sa putang babae yan. Sobra ang kasinungalingan ng mga demonyong ito. The balancing-of-interest test AYER PRODUCTION VS. JUDGE CAPULONG, JUAN PONCE ENRILE, ET AL., 160 SCRA 861 [The Dangerous Life: the 1986 EDSA Peoples Power Revolution] Lagunzad vs. Gonzales, 92 SCRA 476

CHAPTER VI THE NON-ESTABLISHMENT OF RELIGION CLAUSE Estrada vs. Escritur, 408 SCRA 1 and 492 SCRA 1

Requisites: a. Must be a member of the Jehovahs Witnesses and the Watch Tower Society; b. The conjugal arrangement was in conformity with their religious beliefs; c. The conjugal arrangement was with the conformity of the congregation; d. Execute Declaration of Pledging Faithfulness. NOTE: The member must not be the guilty party in the broken marriage and this arrangement is not allowed in countries where divorce is allowed. PEOPLE VS. LAGMAN & ZOSA, 38 O.G. 1676, refusal to join the armed forces on religious grounds is not tenable because of the mandatory requirement of Section 4, Art. II of the 1987 Constitution. INK vs. Gironella, 106 SCRA 1 [Gimmick] American Bible Society vs. City of Manila, 101 Phil. 398 Pamil vs. Teleron, November 20, 1978 Victoriano vs. Elizalde Rope, 59 SCRA 54 [Religious belief not to join unions is superior over the collective bargaining agreement] German vs. Barangan, 135 SCRA 514 Gerona vs. Sec. of Education, 106 Phil. 11 [1959] EBRALINAG VS. SUPT. OF CEBU, March 1, 1993 ANG LADLAD [LGBT---LESBIAN, GAY, BISEXUAL and TRANSGENDER] PARTY VS. COMELEC, G.R. No. 190582, April 8, 2010

The Resolution of the COMELEC disallowing Ang Ladlad as a party-list for the May 10, 2010 elections on moral grounds is unconstitutional. Mixed sexual orientation is not a valid groundUse by the COMELEC of the Bible and the Koran to disqualify Ang Ladlad is a grave violation of the non-establishment clause since there should be GOVERNMENT NEUTRALITY IN RELIGIOUS MATTERS. Further, the enumeration of marginalized groups as labor, peasant, fisherfolk, urban poor, indigenous cultural
15

16

communities, elderly, handicapped, women, youth, veterans, overseas workers and professionals IS NOT EXCLUSIVE as held in Ang Bagong Bayani vs. COMELEC)

CHAPTER VII THE CONSTITUTIONAL RIGHT TO TRAVEL

1. REV. FR. ROBERT REYES VS. CA, SEC. RAUL GONZALES, ET AL., G.R. No. 182161, December 3, 2009Writ of amparo may not be used to justify lifting of the hold departure order since the writ applies only to forced disappearances and extrajudicial killings. 1-a. FERDINAND MARCOS, ET AL. VS. HON. RAUL MANGLAPUS, ET AL., G.R. NO. 88211, September 15, 1989 and the Resolution of the Motion for Reconsideration dated October 27, 1989 [ Arts. 12 and 13 of the Universal Declaration of Human Rights] Manotoc vs. CA, 142 SCRA 149 An accused facing a BAILABLE criminal case in court [RTC or SB] may travel abroad if he can: a. prove urgency of the travel; b. duration of the travel; and consent of the surety. 5. Villavicencio vs. Lukban, 39 Phil. 778 CHAPTER VIII THE CONSTITUTIONAL RIGHT TO INFORMATION NOTE: PROVINCE OF NORTH COTABATO VS. GRP (MOA-AD Case) AKBAYAN VS. THOMAS AQUINO, July 16, 2008 (The JEPEPA Case) HAZEL ANTOLIN VS. ATTY. ABELARDO DOMONDON, ET AL., G.R. No. 165036, July 5, 2010 (Asking to be shown a copy of the test questions, key answers and her test booklet in the 1997 CPA Board Examinations as part of the right to informationSC directed the RTC of Manila to hear the case even if it appears to be moot and academic already having passed the 1998 CPA Board Examinations) 1. Sabio vs. Gordon, 504 SCRA 74 1-a. Valmonte vs. Belmonte, GR No. 74930, February 13, 1989 in relation to the Right to Privacy 1-b. Legaspi vs. CSC, 150 SCRA 530 1-e. Aquino-Sarmiento vs. Manuel Morato, November 13, 1991 1-f. Drilon vs. Ermita, April 20, 2006 2. Tanada vs. Tuvera, 146 SCRA 44 3. Bantay Republika vs. COMELEC, 523 SCRA 1 4. Baldoza vs. Dimaano, 71 SCRA 14 5.Lantaco vs. Lllamas, 108 SCRA 502
16

17

6. Subido vs. Ozaeta, 80 Phil. 383 CHAPTER IX THE CONSTITUTIONAL RIGHT TO FORM AND JOIN ASSOCIATIONS 1. In re: ATTY. EDILLON, 84 SCRA 554 CHAPTER X THE POWER OF EMINENT DOMAIN How about a barangay? 1. Barangay Matictic vs. Elbinias, 148 SCRA 83 Procedure for the exercise of said power and when may a writ of possession be issued in favor of the government BIGLANG-AWA VS. JUDGE BACALLA, 354 SCRA 562 CITY OF ILOILO VS. JUDGE LEGASPI, 444 SCRA 269 REPUBLIC VS. JUDGE GINGOYON, 478 SCRA 474 [RA NO. 8974 APPLIES TO NATIONAL GOVERNMENT PROJECTS, NATIONAL INFRASTRUCTURE PROJECTS, AND BUILD OPERATE TRANSFER PROJECTS OF THE GOVERNMENT] REPUBLIC VS. HOLY TRINITY, April 14, 2008 [EXPANSION OF THE NLEX COVERED BY THE GINGOYON DOCTRINE. THE INTERESTS SHALL INURE TO THE BENEFIT OF THE LANDOWNER] However, if the expropriated land will not be used for that in which it was intended, the landowner may ask for its reversion. MACTAN-CEBU INTERNATIONAL AIRPORT AUTHORITY LOZADA, JR., ET AL., G.R. No. 176625, February 25, 2010 VS.

Taking of private property through expropriation is always subject to the condition that the property be devoted to the specific purpose for which it was taken. Corollarily, if the particular purpose or intent was abandoned, then the former owners if they desire, may seek the reversion of their property by [1] returning the amount of just compensation received [2] plus legal interest [3] plus necessary expenses incurred in maintaining the lot as well as the [4] pecuniary value of their services in managing it to the extent that the landowner will be benefited thereby. When the Lahug Airport in Cebu City was no longer in operation, said lands intended for its expansion can no longer be used for the purpose for which it was intended.

BERKENKOTTER VS. CA, December 14, 1992 Manotok vs. CA, May 21,1987 NAPOOCOR VS. BERNAL, G.R. No. 180979, December 15, 2010
17

18

The Commissioners Report to the Couret on the just compensation when not based on documents like zonal valuation of the BIR but only the alleged value of adjacent properties without supporting sworn statements is not valid for being hearsay. The court should not rely on it. CITY OF ILOILO VS. JUDGE SESANA, G.R. NO. 168967, February 12, 2010 No payment for 25 years, that is a gross violation of the right of the landowner. Liable for interest. Note: There are 2 phases of expropriation. Decision allowing or denying expropriation; and 2nd, determination of just compensation. Requisite of "taking" in eminent domain cases

Read: 1. Rep. vs. Castellvi, 58 SCRA 336 Requisites: 1. The expropriator must enter the property; 2. The entrance must not be just for a momentary period; 3. the entry must be under a warrant or color of title; 4. the property must be devoted for public use; and 5. the owner must be ousted from the beneficial use of his land. 1. City of Manila vs. Chinese Community, 40 Phil.349 2. De Knecht vs. Bautista, 100 SCRA 660 3. REPUBLIC OF THE PHILIPPINES VS. CRISTINA DE KNECHT AND THE COURT OF APPEALS, G.R. NO. 87335, February 12, 1989

CHAPTER XI THE NON-IMPAIRMENT CLAUSE 1. Read: 1. Kabiling, et al., vs. NHA, December 18,l987 2. Clements vs. Nolting, 42 Phil. 702 3. Co vs. PNB, 114 SCRA 842 4. Lozano vs. Martinez,146 SCRA 323 5. Rutter vs. Esteban,93 Phil. 68 6. Ilusorio vs. CAR, 17 SCRA 25 7. Ortigas vs. Feati Bank, 94 SCRA 533 8. Ganzon vs. Insierto, 123 SCRA 713

CHAPTER XII RIGHTS DURING CUSTODIAL INVESTIGATION PEOPLE VS. MAHINAY, February 1, 1999 [11 rights of the suspect under custodial investigation which expanded the Miranda Warning]
18

19

LUMANOG VS. PEOPLE, G.R. No. 182555, September 7, 2010 (ROLANDO ABDILLA CASE) Even if the extrajudicial confession is inadmissible in evidence if there are other evidence sufficient to prove the guilt of the accused beyond reasonable doubt, still, the accused may be convicted. 1-a. Requisites of a valid extrajudicial confession: PEOPLE VS. TUNIACO, G.R. No. 185710, January 10, 2010 1. The confession must be voluntary; 2. The confession must be made with the assistance of a competent and independent counsel; 3. The confession must be express; and 4. The confession must be in writing and signed by the suspectand the counsel. PEOPLE VS. FELIXMINIA, March 20, 2002 [Even if the extrajudicial confession is inadmissible but there is enough circumstantial evidence to prove his guilt beyond reasonable doubt, the accused may still be convicted] P. VS. GALIT, 135 SCRA 465 [SINCE THE EXTRAJUDICIAL CONFESSION (obtained through force and violence) WHICH IS INADMISSIBLE is the only evidence against the accused, even though it speaks of gospel truth, then he should be acquitted] PEOPLE VS. SAMOLDE, 336 SCRA 32 and PEOPLE VS. CABILES, 284 SCRA 199; PEOPLE VS. TAN, 286 SCRA 207 [A suspects confession, whether verbal or in writing, when taken without the assistance of counsel, without a valid waiver of such assistance, regardless of the absence of coercion, or the fact that it was voluntarily given, IS INADMISSIBLE IN EVIDENCE, EVEN IF THE APPELLANT;S CONFESSION WERE GOSPEL TRUTH.

Since the lawyer who assisted them at the Police Station is a PAO Lawyer who was already inside the Police Station when the accused was brought in, and ATTY. BESINGA IS NOT BEHOLDEN TO THE POLICE, the confession is admissible. PEOPLE VS. ANTONIO LAUGA, G.R. No. 186228, March 15, 2010

Confession made to the BANTAY BAYAN without being informed of his rights nor with the assistance of counsel is inadmissible in evidence. The Expanded Miranda Doctrine is not applicable. Bantay Bayan is a group of male residents in the barangay organized for the purpose of keeping peace in the community which is an accredited auxiliary of the PNP. Since it is sanctioned by the local government, any inquiry it makes has the color of a state-related function and objective insofar as the entitlement of a suspect to his constitutional rights provided under Art. III Section 12 of the Constitution, otherwise known as the Miranda Rights, is concerned. PEOPLE VS. VILLARINO, G.R. No. 185012, March 15, 2010

Voluntary and spontaneous confession of a suspect who was arrested in connection with a crime, to a member of the PNP guarding him with offer of
19

20

financial reward to the officer if he would throw the sando or shirt which i s evidence against him into the sea is admissible as evidence even if there is no counsel nor not being informed of his rights. 1-b. When there is no need to inform the accused/suspect of his rights. PEOPLE VS. TAMPUS, MARCH 28, 1980 [New Bilibib confession after killing a member of the other gang therein] P VS. TAYLARAN, 108 SCRA 373 [I killed my mother-in-law since she is a barang is admissible because he uttered it to the police when he voluntarily surrendered] 1. PEOPLE VS. JUDGE RUBEN C. AYSON, RTC 6, BAGUIO CITY, 175 SCRA 216 (Investigator is a private person like in the Philippine Airlines, so it does not apply to administrative investigations) 2. KIMPO VS. CA, 232 SCRA 53 [Confession or investigation by a private individual] 3. PEOPLE VS. ORDONO, 334 SCRA 673 [Radio announcer] 4. PEOPLE VS. ZUELA, 323 SCRA 589; PEOPLE VS. ANDAM [Before the Mayor, with TV coverage] 5. PEOPLE VS. ENDINO, 353 SCRA 307 [Videotaped interview with newsmen] 6. PEOPLE VS. JOSE TING LAN UY, JR., 475 SCRA 248 [The sworn statement or extrajudicial admission used in convicting him was given to a personnel of the National Power Corporation who investigated him is admissible] 2. Guidelines for police investigation: when it is deemed to have started Read: Escobedo vs. Illinois, 378 US 478 Miranda vs. Arizona, 384 US 436 P. vs. Duero, 104 SCRA 379 PEOPLE VS. BARIQUIT, 341 SCRA 600 [Custodial investigation begins when a person is taken into custody for the commission of a crime. THE PLACE OF INTERROGATION IS NOT DETERMINATIVE OF THE EXISTENCE OR ABSENCE OF CUSTODIAL INVESTIGATION BUT THE TONE AND MANNER OF QUESTIONING BY THE POLICE AUTHORITIES. So even while the policemen and the suspect are just walking in the highway towards the police station, there is already custodial investigation in this case as a result of the questions asked of the suspect] 1-a. People vs. Marcos Jimenez, Dec. 10, 1991 1-c. People vs. Judge Ayson, 175 SCRA 216 When shall the constitutional rights of the demandable? During police line-up? accused as mentioned above

20

21

1. P vs. Usman Hassan, 157 SCRA 261 2. Gamboa vs. Judge Cruz, 162 SCRA 642 3. DE LA TORRE VS. CA, 294 SCRA 196 4. PEOPLE VS. HATTON

Read:

1. THE RIGHT TO COUNSEL, 57 SCRA 481

PEOPLE VS. PATUNGAN, 354 SCRA 413 [The suspect was under coercive and uncounselled investigation by the police without a lawyer for 2 and a half days. Then he was brought to the IBP for the signing of his extrajudicial confession. The mere presence of a lawyer is not sufficient compliance of the constitutional requirement of assistance of counsel. The assistance must be effective, vigilant and independent. A lawyer who could barely hear the investigation going on while working on another case hardly satisfies the minimum requirement of an effective assistance of counsel. PEOPLE VS. OBRERO, 332 SCRA 190 [Assistance of Atty. De Los Reyes, the Station Commander of the WPD to the accused while being investigated by the subordinates of Atty. De los Reyes is not in accordance with the Constitution. The position of said lawyer is adverse to the suspect] PEOPLE VS. JEREZ, 285 SCRA 393 [However, a lawyer provided by the investigators to the suspect during custodial investigation is deemed engaged by the accused WHERE HE NEVER RAISED ANY OBJECTION AGAINST THE FORMERS APPOINTMENT DURING THE COURSE OF THE INVESTIGATION AND THE ACCUSED THEREAFTER SUBSCRIBES TO THE VERACITY OF HIS STATEMENT BEFORE THE FISCAL. PEOPLE VS. REYES, G.R. No. 178300, March 17, 2009 [PAOCTF of Col. Cesar Mancao] [Chua Ong Ping Sim and Raymond Yao were strangled to death after they were kidnapped though the parents agreed to the P5M ransom. When arrested, the suspects were provided by the PAOCTF Investigators ATTY. UMINGA and ATTY. ROUS to assist them. Both lawyers are lawyers of PAOCTF. They informed them of their rights and assisted them during the entire time of the investigation. The SC said the confessions are admissible. This is so because a confession is admissible when: a. b. c. where the accused failed to present credible evidence of compulsion or duress or violence on their persons; where they failed to complain to the officers who administered the oaths; where they did not institute any criminal or administrative action against their alleged intimidators for maltreatment; where there appeared to be no marks of violence on their bodies; and where they did not have themselves examined by a reputable physician to buttress their claim. (People vs. PIA, 229 Phil. 577 [1986]

d. e.

The presence of a lawyer is not intended to stop an accused from saying anything which might incriminate him; but rather, it was adopted in our Constitution to preclude the slightest coercion on the accused to admit something else. THE COUNSEL SHOULD NEVER PREVENT AN ACCUSED FROM FREELY AND VOLUNTARILY TELLING THE TRUTH. (PEOPLE VS. BASE, 385 Phil. 803 [2000]

21

22

PEOPLE VS. MARCOS JIMENEZ, G.R. No. 82604, December 10, 1991 [The lawyer who assists the suspect under custodial investigation should be the latters choice and not merely foisted on him by the police. ALSO, THE LAWYER SHALL BE PRESENT TO ASSIST HIM FROM THE FIRST QUESTION ASKED OF HIM] PEOPLE VS. JUANERIO, 267 SCRA 608 [Assisted by a lawyer applying at the NBI, not valid confession] Could the Fiscal also represent the accused during custodial investigation to satisfy the requirement of the Constitution that the accused is assisted by counsel? P. vs. Matos-Viduaya, September 11, 1990

CHAPTER XIII THE CONSTITUTIONAL RIGHT TO BAIL Section 13 Is bail available in extradition cases? 1. SECRETARY OF JUSTICE VS. JUDGE LANTION, 322 SCRA 160 2. US GOVERNMENT VS. JUDGE PURGANAN & MARK JIMENEZ, 389 SCRA 623 [No right to bail in an extradition case since it is not criminal and nature] 3. RODRIGUEZ VS. PRESIDING JUDGE, 483 SCRA 290 4. GOVERNMENT OF HONGKONG VS. JUDGE OLALIA, JR., 521 SCRA 470 [A prospective extraditee may be allowed on bail if he can prove by CLEAR AND CONVINCING EVIDENCE that he is not a flight risk and that he is ready and willing to comply with the processes of the extradition court. A clear and convincing evidence is an evidence greater than mere preponderance of evidence but below proof beyond reasonable doubt] Is bail by recognizance allowed after conviction by final judgment? ATTY. ADALIM-WHITE VS. JUDGE BAGTAS, 475 SCRA 175 2. Ignacio vs. Villaluz, 66 SCRA 38[Purely cash bond, not allowed] 3. Marcos vs. Cruz, 67 and 70 Phil. 4. Villasenor vs. Abano, 21 SCRA 312 [Factors to consider in granting bail] 5. P vs. IAC, January 10,1987, 147 SCRA 219 6. Manotoc vs. CA, May 30,1986 7. Garcia vs. Domingo, 52 SCRA 143 8. P vs. San Diego, 26 SCRA 522 [Bail application should be decided only after the prosecution has presented all its intended witnesses in a bail hearing] 9. PEOPLE VS. BUCALON, G. R. No. 176933, October 2, 2009 After the prosecution rested its case in a Murder case, respondent, with leave of court, filed a Demurrer to Evidence . The Demurrer was denied by Judge Buyser by Order of March 14, 2002, because: The evidence thus presented by the prosecution is sufficient to prove the guilt of the accused beyond reasonable doubt, but only for the crime of homicide and not for murder, as charged. This is because the qualifying circumstance of treachery alleged in the information cannot be appreciated in this case.
22

23

The defense thereupon presented evidence in the course of which respondent filed a Motion to Fix Amount of Bail Bond contending that in view of Judge Buysers ruling that the prosecution evidence is sufficient to prove only Homicide, and as such, he could be released on bail. Held: The Demurrer to Evidence is tantamount to an application for bail. He could be granted bail. CHAPTER XIV DUE PROCESS IN CRIMINAL PROCEEDINGS

1. P vs. Terrobias, 103 SCRA 321 [One week trial and conviction does not violate the right to due process] PEOPLE VS. MALBOG, October 12, 2000 , 342 SCRA 620 [The complainant testified that the accused ejaculated between her thighs; that she did not try to escape when the accused was out for 45 minutes; did not try to shout for help; waited for the accused to return; Medico Legal findings that no external injuries in the body of the complainant and there was no spermatozoa on her vagina, coupled with the fact that there was no injuries on her private organ but healed lacerations which happened more than one week before the alleged rape negates the protestation of the complainant that she was raped. If ever there was sexual intercourse, the evidence is utterly wanting to show that the same was against her will. Mas vale que queden castigar diez presuntos, que se castigue uno inocente.

ERIBERTO MASANGKAY VS. PEOPLE, G.R. No. 164443, June 18, 2010 Every criminal conviction must draw its strength from the prosecutions evidence. The evidence must be such that the constitutional presumption of innocence is overthrown and guilt is established beyond reasonable doubt. The prosecutorial burden is not met when the circumstances can yield to different inferences. Such equivocation betrays a lack of moral certainty to support a judgment of conviction.

MALILLIN VS. PEOPLE, G.R. No. 172953, April 30, 2008 (Search of shabu inside the room was conducted with warrant. Several policemen with witnesses participated. When the witnesses have left the room since no shabu was found, one policeman who was left suddenly said he found a shabu. Presumption of innocence applicable to support acquittal) PEOPLE VS. DELOS SANTOS, 355 SCRA 415 (ENDURANCE RUN FROM Camp Danglay in Fortich, Bukidnon, to CAMP ALAGAR CDO THE PNP TRAINEES WERE BUMPED BY THE ACCUSED---12 DIED, 11 SERIOUSLY INJURED AND 10 MINOR INJURIES--Murder? If susceptible to two interpretations, it should only be reckless imprudence resulting to homicide)

23

24

PEOPLE VS. SATURNO, 355 SCRA 578 (IMMEDIATELY AFTER THE COMMISSION OF THE CRIME, THE WIFE OF ONE OF THE 4 VICTIMS TOLD THE INVESTIGATORS SHE did could not identify the assailant. Later on she said it is the accused. Presumption of innocence dictates that he should be acquitted) P vs. Poblador, 76 SCRA 634 [ The accused, a 24 year old bachelor, could not have embarrassed himself by having sexual intercourse with an old woman of 45 years of age, with 5 children, the youngest is 5 months old, much less do it in the presence of her husband Dumlao vs. Comelec, 95 SCRA 392 [Presumption of innocence makes a law prohibiting all those who are charged of crimes of rebellion and other political crimes from running for public office UNCONSTITUTIONAL. PEOPLE VS. JAIME JOSE, ET AL., 37 SCRA 450

3. Presumption of innocence in the order of trial Read: 1. Alejandro vs. Pepito, 96 SCRA 322 [Reverse Order of trial not allowed] 2. Sacay vs. Sandiganbayan, July 10, l986 [Reverse order of trial is valid if the accused consents thereto] 3.Sec. 3(3), Rule 119 , 2000 Rules on Criminal Procedure , as amended. 3-a. Presumption of innocence in drugs cases/Obligations of arresting officers/Chain of Evidence Rule under Section 21 of RA No. 9165 Read: PEOPLE VS. PAJARIN, G.R. NO. 190460, January 12, 2011 If the alleged shabu was not marked by the arresting officers after the buy-bust operation at the place where the arrest was made but marked by the investigator when it was turned over in the police station, there is violation of Section 21 and accused shall be acquitted. PEOPLE VS. LORENZO, GR NO. 184760. April 23, 2010 (Section 21) PEOPLE OF THE PHILIPPINES VS. SAPIA ANDONGAN, G.R. No. 184595, June 29, 2010

Justice Carpio-Morales Accused was allegedly arrested in a buy-bus operation in Manila particularly in Abad Santos Avenue along Bambang Street, a street with many people at that time, at around 7:50 p.m. of June 25, 2004. She allegedly sold shabu worth P500.00 for one (1) sachet containing 0.146 grams . No other sachet of shabu was confiscated on her person though she allegedly a drug dealer. HELD:

The chain of custody rule under Section 21 of RA No. 9165 was not shown to have been substantially complied with.
24

25

The presumption of regularity in the performance of official duties could not prevail over the presumption of innocence in favor of the accused. For, among other things, it is incredible for an allegedly known drug-peddler to be standing at a corner of a street at 7:50 in the evening instead of plying her trade secretly, and with only a 0.146-gram sachet worth P500.00 of prohibited drugs in her possession the value of which happens to be what a poseur-buyer wants to buy. PEOPLE VS. RONALDO DE GUZMAN, G.R. No. 186498, March 26, 2010 BONDAD VS. PEOPLE, December 10, 2008 (No pictures) JULIUS CACAO VS. PEOPLE, G.R. No. 180870, January 22, 2020. Conflict of the testimonies of prosecution witnesses on who delivered the seized evidence to the evidence custodian PEOPLE OF THE PHILIPPINES vs.TOKOHISA KIMURA and AKIRA KIZAKI, G.R. No. 130805, April 27, 2004, the Supreme Court held that said failure to mark the seized illegal drugs at the place of apprehension is fatal. It was held that: In People vs. Laxa, the policemen composing the buy-bust team failed to mark the confiscated marijuana immediately after the alleged apprehension of the accused-appellant. One policeman admitted that he marked the seized items only after seeing them for the first time in the police headquarters.

ERIBERTO MASANGKAY VS. PEOPLE, G.R. No. 164443, June 18, 2010 Every criminal conviction must draw its strength from the prosecutions evidence. The evidence must be such that the constitutional presumption of innocence is overthrown and guilt is established beyond reasonable doubt. The prosecutorial burden is not met when the circumstances can yield to different inferences. Such equivocation betrays a lack of moral certainty to support a judgment of conviction. PEOPLE VS. OBMIRANIS, December 16, 2008 MALLILLIN VS. PEOPLE, April 30, 2008 PEOPLE VS. MONALYN CERVANTES, March 17, 2009 PEOPLE VS. MARIAN CORECHE, August 14, 2009 PEOPLE VS. PARTOZA, May 8, 2009

3-b. Effect of inconsistent testimonies of arresting/searching officers LEJANO VS. PEOPLE, DECEMBER 14, 2010 (The Hubert Webb Case) Inconsistent testimony of the prime witness results in acquittal. It is also inconsistent to common human experience. AGUSTIN VS. PEOPLE, April 30, 2008 (One witness said, 5 of them entered the room and searched. He said SPO4 Jara searched the room but when the latter testified, he said, he was outside guarding the premises) PEOPLE VS. ANGUS, August 3, 2010 The mere fact that there is evidence showing that the accused and his wife were quarreling the night before her body was found is not enough circumstantial evidence to prove his guilt for parricide. Presumption of innocence justifies his acquittal.
25

26

3-c. The equipoise rule PEOPLE VS. DE LOS SANTOS, 355 SCRA 415 PEOPLE VS. SATURNO, 355 SCRA 578 PEOPLE VS. JOHN HILARIO, G.R. No. 161070, April 14, 2008

Even if the decision of conviction is already final and executory, the case may be re-opened IF THERE IS GROSS NEGLIGENCE COMMITTED BY COUNSEL THEREBY VIOLATING THE RIGHT OF THE SAID ACCUSED TO DUE PROCESS OF LAW. Two counts of murder with conviction, he directed his PAO Lawyer to file appeal but he failed. His Petition for Relief is granted) Requisites: 1. 2. 3. 4. 5. 6. Matter involves life, liberty or property; existence of special or compelling circumstances; merit of the case; not entirely the fault of the party; not dilatory or frivolous; other party not prejudiced.

1. HILARIO VS. PEOPLE, G.R. No. 167756, April 8, 2008 1-a P vs. Holgado,85 Phil. 752 1-b. Delgado vs. CA, 145 SCRA 357 1-c. CONSULTA VS. PEOPLE, FEBRUARY 12, 2009 1. P vs. Cuison, 193 Phil. 296 2. PEOPLE VS. YAMBOT, 343 SCRA 20 3. PEOPLE VS. NADERA, 324 SCRA 490 (Oleby and Maricris Nadera testified with no cross-examination from the accuseds lawyer. Worse, accused did not testify) 4. PEOPLE VS. BERMAS, April 21, 1999 6. The right to be present during trial 1. Aquino vs. Military Commission, 63 SCRA 546 2. P vs. Judge, 125 SCRA 269 The right to a speedy trial

MONICO JACOB VS. SANDIGANBAYAN, G.R. No. 162206, November 17, 2010 The delay in the trial for almost 1 and years while the Ombudsman is resolving the Motion for Re-investigation may not be a valid ground to dismiss the criminal cases based on speedy trial. Further, the verbal Order of dismissal by Justice Nario as the Presiding Justice of the 4th Division is not valid since no written copy of the order signed by the other justices. No violation of the right against double jeopardy for the reinstatement of the case since there was no valid dismissal in the first place. JAIME BERNAT VS. SANDIGANBAYAN, May 20, 2004. [Submitted for decision as of August 23, 1994 but it was only about 8 years later that there was a Notice of Promulgation of the Decision. THE DETERMINATION OF WHETHER THE DELAYS ARE OF SAID NATURE IS RELATIVE AND CANNOT BE BASED ON MERE
26

27

MATHEMATICAL RECKONING OF TIME. PARTICULAR REGARD TO FACTS AND CIRCUMSTANCES SHALL ALSO BE CONSIDERED] The delay must be VEXATIOUS, CAPRICIOUS AND OPPRESSIVE and this is so if the following are satisfied: a. Length of the delay; b. Reasons for the delay; c. Assertion or failure to assert such right by the accused; and d. Prejudiced caused by the delay. 1. 1-e. DANTE TAN VS. PEOPLE, April 21, 2009 [It does not apply if the accused consented to the archiving of the case which lasted for several years] 2. Conde vs. Rivera, 59 Phil. 650 3. P vs. Araula, 111 SCRA 598 (Fiscal with telegram that he is sick. Dismissal not valid under speedy trial) 4. 12. Tatad vs. SB, 159 SCRA 70 5. DUTERTE VS. SANDIGANBAYAN, 289 SCRA 721 (February 18, 1992 submitted for Resolution by OMB and the Information was filed only on February 22, 1996. Violation) 6. ANGCHANGCO VS. OMBUDSMAN, 269 SCRA 301 (October 22, 1999 up to May 26, 2004, no case filed whether admin or criminal. Violation of speedy disposition of cases) CAPT. WILFREDO ROQUERO VS. THE CHANCELLOR OF UP MANILA, G.R. No. 181851, March 9, 2010

No formal offer of evidence by the complainant against him for 5 years is a clear violation of his right to speedy disposition of cases. The administrative case shall be dismissed. The right to an impartial trial

1.GACAYAN VS. PAMINTUAN, 314 SCRA, September 17, 1999 1-a. PEOPLE VS. OPIDA, June 13, 1986 [The Supreme Court held that the right of the accused to due process of law and impartial trial were violated when it was the judge who conducted the cross-examination of the accused and his witness instead of the Prosecutor coupled with his sarcastic and insulting remarks and ended with the question, Do you want me to dictate the decision now? The judge likewise required the accused to remove his shirt and described for the record all the tattoos found on his body. Clearly, the judge had allied himself with the prosecution] 1-b. PEOPLE OF THE PHILIPPINES VS. BENANCIO MORTERA, G.R. No. 188104, April 23, 2010 [There is no violation of the right to due process and impartial trial as a result of the comment of the Judge that if you are not telling the truth to your own lawyer, how would I know that you are telling the truth now as a result of his change of defense from a negative defense to self-defense in a murder case. The comment of the judge shall be considered based on the entirety of his questions and the circumstances surrounding said utterance. The accused described the clarificatory questions of the judge as prosecutor-like conduct and therefore, lacked the cold neutrality of an impartial judge. 2. Olaguer vs. Chief of Staff, May 22, 1987 3. Ignacio vs. Villaluz,May 5, 1979 4. P vs. Sendaydiego, 81 SCRA 120 5. Dimacuha vs. Concepcion, 117 SCRA 630
27

28

Right to a public trial

1. Garcia vs. Domingo, July 25,1973 2. P vs. Tampus, March 28,1980 The right to be informed of the nature and cause of accusation

PEOPLE VS. JERRY NAZARENO, G.R. No. 167756, April 8, 2008

The information reads that sometime from January, 1990 up to December 6, 1998, in Barangay Codon, Municipality of San Andres, province of Catanduanes.by force, intimidationhad sexual intercourse with the complainant without her consent No violation of the right to be informed since the date and time is not an integral part or element of the crime of rape. Further, the accused waived said right when he never questioned the information before he entered his plea. He should have asked for a Bill of Particulars or Moved to quash the Information. He did not. Clearly, he slumbered on his rights and awakened too late. 1-a. P vs. Crisologo, 150 SCRA 653 1-c. P vs. Resavaga, 159 SCRA 426 3. Ko Bu Lin vs. CA, 118 SCRA 573 The right to meet witnesses face to face or the right of confrontation

DELACRUZ VS. PAPA, December 8, 2010 The witness failed to return for cross-examination because he was sickly and never did so up to the time of his death. His testimony shall be expunged from the record. Read: 1. 2. 3. 4. P vs. Valero, 112 SCRA 661 [Poisoned bread. Case vs. deaf-mute] P vs. Bundalian, 117 SCRA 718 [Libel complainant already dead] P vs. Seneris, 99 SCRA 92 Ortigas, JR. vs. Lufthansa, 64 SCRA 610 Trial in absentia Borja vs. Mendoza, 77 SCRA 420 P vs. Salas, 143 SCRA 163; evidence.

13. Right to secure witnesses and production of 1. Cavili vs. Hon. Florendo, 154 SCRA 610 2. Fajardo vs. Garcia, 98 SCRA 514

CHAPTER XVI THE RIGHT AGAINST SELF-INCRIMINATION

28

29

SOCIAL JUSTICE SOCIETY VS. PDEA, November 3, 2008 random drug testing to students in HS & College valid because it is not intended for criminal prosecution but to help the youth through early detection of drug use. RANDOMNESS AND SUSPICIONLESS. ATTY. MANUEL LASERNA, J. VS. DANGEROUS DRUGS BOARD, G.R. No. 158633, November 3, 2008 [Mandatory drug testing for persons charged of an offense punishable by 6 yrs. And 1 day up IS VIOLATIVE OF THE RIGHT AGAINST SELF-INCRIMINATION 1. Chavez vs. CA, 24 SCRA 663 2. Galman vs. Pamaran, 138 SCRA 294, read including the concurring and dissenting opinions 3. Villaflor vs. Summers, 41 Phil. 62 4. Beltran vs. Samson, 50 Phil. 570 5. Bagadiong vs. Gonzales, 94 SCRA 906 6. US vs. Tang Teng, 23 Phil. 145 7. Cabal vs. Kapunan, Jr. December 29, 1962

CHAPTER XVII THE RIGHT AGAINST INVOLUNTARY SERVITUDE 1. Aclaracion vs. Gatmaitan, 64 SCRA 131 2. Caunca vs. Salazar, supra CHAPTER XVIII RIGHT AGAINST CRUEL AND UNUSUAL PUNISHMENT

LEO ECHEGARAY VS. SECRETARY OF JUSTICE [Death through lethal injection is the most humane way of implementing the Death Penalty. It is not cruel and unusual] Death Penalty was abolished: PEOPLE VS. GAVARRA, 155 SCRA 327; PEOPLE VS. MASANGKAY, 155 SCRA 113; P VS. ATENCIO, 156 SCRA 242; PEOPLE VS. INTINO, SEPT. 26, 1988; PEOPLE VS. MUNOZ, 170 SCRA 107 Death penalty not abolished for two reasons: Sec. 19 [1] Art. III. Congress may re-impose for heinous crimes Section 5 (2) [d] Art. VIII punishment?

b. Is death as a penalty a cruel or unusual

1. P vs. Estoista, 93 Phil. 647 [It is cruel and unusual if the penalty is disproportionate to the crime committed and shocking to the conscience of the community] 2. People vs. Villanueva, 128 SCRA 488; 3. VENIEGAS VS. PEOPLE, 115 SCRA 79; 4. PEOPLE VS. CAMANO, 115 SCRA 688
29

30

CHAPTER XX THE RIGHT AGAINST DOUBLE JEOPARDY

IVLER VS. JUDGE PEDRO, Presiding Judge, METC 71 OF PASIG CITY, G.R. No. 172716, November 17, 2010 After the petitioner pleaded guilty to reckless imprudence resulting to slight physical injuries and was sentenced to censure, he could not be tried anymore of the 2 nd case of reckless imprudence resulting to homicide and damage to property which arose from the same act. Double jeopardy has set in. LEJANO VS. PEOPLE, January 18, 2011 and PEOPLE VS. HUBERT WEBB ET AL. After the acquittal of the accused by the SC on December 14, 2010, double jeopardy has set in and no motion for reconsideration may be entertained. MONICO JACOB VS. SB, November 17, 2010 Since the dismissal based on speedy trial was not upheld, then the invocation of double jeopardy is unavailing. When the act is punished by both a law and an ORDINANCE: PEOPLE VS. JUDGE RELOVA, 148 SCRA 292 DAYAP BS. SENDIONG, JANUARY 29, 2009 [Acquittal through demurrer to evidence is final and could not be appealed. Double jeopardy has set in but civil aspect may still proceed if there is no express finding that accused has not committed the crime] PEOPLE VS. DOMINGO, G.R. No. 184343, March 2, 2009 [If an accused of Murder was convicted of Homicide only in the RTC but appeals the same to the higher court, he could not complain of double jeopardy if the appellate court sentences him of Murder since that is the evidence found during the review of the case on Appeal] 1. CUDIA VS. CA, 284 SCRA 173 [Petitioner was arrested of illegal possession of firearm in Mabalacat, Pampanga but it was the City Prosecutor of Angeles City who signed the information] PEOPLE VS. BALISACAN, 17 SCRA 1119 [Entered a plea of guilty to a charge of homicide but prayed for presentation of his evidence for incomplete selfdefense. He was acquitted instead. No double jeopardy since there was a valid plea] PEOPLE VS. GALANO, 75 SCRA 193 [Charged of estafa in Batangas. During trial, it was shown that the elements of the crime actually took place in Manila. No double jeopardy if a new information will be filed in Manila] 8. P vs. Judge Hernando, 108 SCRA 121 9. Esmena vs. Judge Pogoy, 102 SCRA 861 10. Mazo vs. Mun. Court, 113 SCRA 217 [Demurrer to Evidence] 11. Andres vs. Cacdac, 113 SCRA 217 14. P vs. Fuentebella, 100 SCRA 672 15. DIONALDO VS. DACUYCUY, 108 SCRA 736 16. PEOPLE VS. BALADJAY, 17. ESMENA VS. JUDGE POGOY
30

31

18. ANDRES VS. CACDAC 19. PEOPLE VS CITY COURT OF SILAY, 154 SCRA 175 May the government appeal a judgment of acquittal or penalty imposed by the trial court? for the increase of the

PEOPLE VS. JUDGE VELASCO, 340 SCRA 207 The government may not appeal or file a Petition for Certiorari questioning the judgment of acquittal. Double jeopardy has set in. 6. The "Supervening Fact Doctrine." Read: 1. 76 SCRA 469 2. P vs. Tarok, 73 Phil. 260 3. P vs. Villasis, 46 O.G. 268 4. Melo vs. People, 85 Phil. 766 5. P vs. Buling, 107 Phil. 712 5-a. P vs. Adil, 76 SCRA 462 5-b. P. vs. Tac-an, 182 SCRA 601 6. P vs. City Court of Manila, 121 SCRA 637 7. Read also Sec. 7, Rule 117, 2000 Rules on

Criminal Procedure

CHAPTER XXI RIGHT AGAINST EX-POST FACTO LAW, BILL OF ATTAINER, ETC. Ex post facto law applies only to penal laws. LACSON VS. SANDIGANBAYAN, January 20, 1999 a. which makes an act criminal before the passing of the law and which was innocent when committed, and punishes such action; b. which aggravates the crime or makes it greater when it was committed; c. when it changes the punishment and inflicts a greater punishment than the penalty when the crime was committed; d. which alters the legal rules of evidence and now receives less or different testimony to convict the accused; e. a law that alters the situation of a person to his disadvantage, in relation to an offense; f. which assumes or regulates civil rights and remedies but in effect imposes penalty or deprivation of a right which when done was lawful; and f. deprives a person accused of a crime of some lawful protection to which he has become entitled, such as the protection of a former conviction or acquittal, or proclamation of amnesty.

31

32

NOTES & CASES


CHAPTER 1 FUNDAMENTAL POWERS OF THE STATE (Police Power) 1. Define: police power---is the power vested in the legislature by the Constitution to make, ordain, establish all manner of wholesome and reasonable laws for the good and welfare of the State and its people. (ERMITA MALATE HOTEL VS. CITY MAYOR, July 31, 1967) The basic purposes of police power are: a. to promote the general welfare, comfort and convenience of the people; (ASSOCIATION OF SMALL LANDOWNERS VS. SECRETARY, 175 SCRA 343; US VS. TORIBIO, 15 Phil. 85 b. to promote and preserve public health; (VILLANUEVA VS. CASTANEDA, September 21, 1987; DECS VS. SAN DIEGO, 180 SCRA 533 [NMAT]; LORENZO VS. DIRECTOR OF HEALTH, 50 Phil. 595apprehend and confine lepers in a leprosarium) Police Power as a limitation to the right to practice a profession PROFESSIONAL REGULATIONS COMMISSION VS. ARLENE DE GUZMAN, ET AL., June 21, 2004 Facts:

The Board then issued Resolution No. 19 withholding the registration as physicians of all the examinees from Fatima College of Medicine. Compared with other examines from other schools, the results of those from Fatima were not only incredibly high but unusually clustered close to each other. The NBI Investigation found that the Fatima examinees gained early access to the test questions. Held:

But like all rights and freedoms guaranteed by the Constitution, their exercise may be regulated pursuant to the police power of the State to safeguard health, morals, peace, education, order, safety, and general
32

33

welfare of the people. As such, mandamus will not lie to compel the Board of Medicine to issue licenses for the respondents to practice medicine. RA 2382 which prescribes the requirements for admission to the practice of medicine, the qualifications of the candidates for the board examination, the scope and conduct of the examinations, the grounds for the denying of the issuance of a physicians license, or revoking a license that has been issued. It is therefore clear that the examinee must prove that he has fully complied with all the conditions and requirements imposed by law and the licensing authority to be granted the privilege to practice medicine. In short, he shall have all the qualifications and none of the disqualifications. The petition is therefore granted. c. to promote and protect public safety; (AGUSTIN VS. EDU, 88 SCRA 195; TAXICAB OPERATORS VS. JUINIO, 119 SCRA 897 ) d. to maintain and safeguard peace and order; (GUAZON VS. DE VILLA) e. to protect public morals; (CITY OF MANILA VS. JUDGE LAGUIO, JR., 455 SCRA 308; WHITE LIGHT CORPORATION VS. CITY OF MANILA, January 20, 2009; DE LA CRUZ VS. PARAS, 123 SCRA 569; ERMITA MALATE HOTEL VS. CITY MAYOR, July 31, 1967; VILLAVICENCIO VS. MAYOR LUKBAN OF MANILA, 39 Phil. 778; JMM PROMOTIONS VS. CA, 260 SCRA 319; VELASCO VS. VILLEGAS, February 13, 1983) An Ordinance of the City of Manila prohibiting short-time in Motels and Hotels. WHITE LIGHT CORPORATION, TITANIUM CORPORATION and STA. MESA TOURIST & DEVELOPMENT CORPORATION vs. CITY OF MANILA, represented by MAYOR ALFREDO S. LIM, G.R. No. 122846, January 20, 2009 TINGA, J.: With another city ordinance of Manila also principally involving the tourist district as subject, the Court is confronted anew with the incessant clash between government power and individual liberty in tandem with the archetypal tension between law and morality. In City of Manila v. Laguio, Jr, the Court affirmed the nullification of a city ordinance barring the operation of motels and inns, among other establishments, within the Ermita-Malate area. The petition at bar assails a similarly-motivated city ordinance that prohibits those same establishments from offering short-time admission, as well as pro-rated or wash up rates for such abbreviated stays. Our earlier decision tested the city ordinance against our sacred constitutional rights to liberty, due process and equal protection of law. The same parameters apply to the present petition. This Petition challenges the validity of Manila City Ordinance No. 7774 entitled, An Ordinance Prohibiting Short-Time Admission, Short-Time Admission Rates, and Wash-Up Rate Schemes in Hotels,
33

34

Motels, Inns, Lodging Houses, Pension Houses, Establishments in the City of Manila (the Ordinance). The facts are as follows:

and

Similar

On December 3, 1992, City Mayor Alfredo S. Lim (Mayor Lim) signed into law the Ordinance. The Ordinance is reproduced in full, hereunder: SEC. 3. Pursuant to the above policy, short-time admission and rate [sic], wash-up rate or other similarly concocted terms, are hereby prohibited in hotels, motels, inns, lodging houses, pension houses and similar establishments in the City of Manila. SEC. 4. Definition of Term[s]. Short-time admission shall mean admittance and charging of room rate for less than twelve (12) hours at any given time or the renting out of rooms more than twice a day or any other term that may be concocted by owners or managers of said establishments but would mean the same or would bear the same meaning. SEC. 5. Penalty Clause. Any person or corporation who shall violate any provision of this ordinance shall upon conviction thereof be punished by a fine of Five Thousand (P5,000.00) Pesos or imprisonment for a period of not exceeding one (1) year or both such fine and imprisonment at the discretion of the court; Provided, That in case of [a] juridical person, the president, the manager, or the persons in charge of the operation thereof shall be liable: Provided, further, That in case of subsequent conviction for the same offense, the business license of the guilty party shall automatically be cancelled. On December 15, 1992, the Malate Tourist and Development Corporation (MTDC) filed a complaint for declaratory relief with prayer for a writ of preliminary injunction and/or temporary restraining order ( TRO) with the Regional Trial Court (RTC) of Manila, Branch 9 impleading as defendant, herein respondent City of Manila (the City) represented by Mayor Lim. MTDC prayed that the Ordinance, insofar as it includes motels and inns as among its prohibited establishments, be declared invalid and unconstitutional. MTDC claimed that as owner and operator of the Victoria Court in Malate, Manila it was authorized by Presidential Decree (P.D.) No. 259 to admit customers on a short time basis as well as to charge customers wash up rates for stays of only three hours. They contend that the assailed Ordinance is an invalid exercise of police power. II. To students of jurisprudence, the facts of this case will recall to mind not only the recent City of Manila ruling, but our 1967 decision in Ermita-Malate Hotel and Motel Operations Association, Inc., v. Hon. City Mayor of Manila. Ermita-Malate concerned the City ordinance requiring patrons to fill up a prescribed form stating personal information such as name, gender, nationality, age, address and occupation before they could be admitted to a motel, hotel or lodging house. This earlier ordinance was precisely enacted to minimize certain practices deemed harmful to public morals. A purpose similar to the annulled ordinance in City of Manila
34

35

which sought a blanket ban on motels, inns and similar establishments in the Ermita-Malate area. However, the constitutionality of the ordinance in Ermita-Malate was sustained by the Court. The common thread that runs through those decisions and the case at bar goes beyond the singularity of the localities covered under the respective ordinances. All three ordinances were enacted with a view of regulating public morals including particular illicit activity in transient lodging establishments. This could be described as the middle case, wherein there is no wholesale ban on motels and hotels but the services offered by these establishments have been severely restricted. At its core, this is another case about the extent to which the State can intrude into and regulate the lives of its citizens. The test of a valid ordinance is well established. A long line of decisions including City of Manila has held that for an ordinance to be valid, it must not only be within the corporate powers of the local government unit to enact and pass according to the procedure prescribed by law, it must also conform to the following substantive requirements: (1) must not contravene the Constitution or any statute; (2) must not be unfair or oppressive; (3) must not be partial or discriminatory; (4) must not prohibit but may regulate trade; (5) must be general and consistent with public policy; and (6) must not be unreasonable . A. Police power, while incapable of an exact definition, has been purposely veiled in general terms to underscore its comprehensiveness to meet all exigencies and provide enough room for an efficient and flexible response as the conditions warrant. Police power is based upon the concept of necessity of the State and its corresponding right to protect itself and its people.Police power has been used as justification for numerous and varied actions by the State. These range from the regulation of dance halls, movie theaters, gas stations, and cockpits. The awesome scope of police power is best demonstrated by the fact that in its hundred or so years of presence in our nations legal system, its use has rarely been denied. The apparent goal of the Ordinance is to minimize if not eliminate the use of the covered establishments for illicit sex, prostitution, drug use and alike. These goals, by themselves, are unimpeachable and certainly fall within the ambit of the police power of the State. B. The primary constitutional question that confronts us is one of due process, as guaranteed under Section 1, Article III of the Constitution. Due process evades a precise definition. The purpose of the guaranty is to prevent arbitrary governmental encroachment against the life, liberty and property of individuals. The due process guaranty serves as a protection against arbitrary regulation or seizure. Even corporations and partnerships are protected by the guaranty insofar as their property is concerned. The due process guaranty has traditionally been interpreted as imposing two related but distinct restrictions on government, "procedural due process" and "substantive due process." Procedural due process refers
35

36

to the procedures that the government must follow before it deprives a person of life, liberty, or property. Procedural due process concerns itself with government action adhering to the established process when it makes an intrusion into the private sphere. Examples range from the form of notice given to the level of formality of a hearing. Substantive due process completes the protection envisioned by the due process clause. It inquires whether the government has sufficient justification for depriving a person of life, liberty, or property. The question of substantive due process, more so than most other fields of law, has reflected dynamism in progressive legal thought tied with the expanded acceptance of fundamental freedoms. Police power, traditionally awesome as it may be, is now confronted with a more rigorous level of analysis before it can be upheld. The vitality though of constitutional due process has not been predicated on the frequency with which it has been utilized to achieve a liberal result for, after all, the libertarian ends should sometimes yield to the prerogatives of the State. Instead, the due process clause has acquired potency because of the sophisticated methodology that has emerged to determine the proper metes and bounds for its application. C. The general test of the validity of an ordinance on substantive due process grounds is best tested when assessed with the evolved footnote 4 test laid down by the U.S. Supreme Court in U.S. v. Carolene Products. Footnote 4 of the Carolene Products case acknowledged that the judiciary would defer to the legislature unless there is a discrimination against a discrete and insular minority or infringement of a fundamental right. Consequently, two standards of judicial review were established: strict scrutiny for laws dealing with freedom of the mind or restricting the political process, and the rational basis standard of review for economic legislation. A third standard, denominated as heightened or immediate scrutiny, was later adopted by the U.S. Supreme Court for evaluating classifications based on gender and legitimacy, Immediate scrutiny was adopted by the U.S. Supreme Court in Craig, after the Court declined to do so in Reed v. Reed. While the test may have first been articulated in equal protection analysis, it has in the United States since been applied in all substantive due process cases as well. We ourselves have often applied the rational basis test mainly in analysis of equal protection challenges. Using the rational basis examination, laws or ordinances are upheld if they rationally further a legitimate governmental interest. Under intermediate review, governmental interest is extensively examined and the availability of less restrictive measures is considered. Applying strict scrutiny, the focus is on the presence of compelling, rather than substantial, governmental interest and on the absence of less restrictive means for achieving that interest. In terms of judicial review of statutes or ordinances, strict scrutiny refers to the standard for determining the quality and the amount of governmental interest brought to justify the regulation of fundamental freedoms. Strict scrutiny is used today to test the validity of laws dealing with the regulation of speech, gender, or race as well as other fundamental
36

37

rights as expansion from its earlier applications to equal protection. The United States Supreme Court has expanded the scope of strict scrutiny to protect fundamental rights such as suffrage, judicial access and interstate travel. If we were to take the myopic view that an Ordinance should be analyzed strictly as to its effect only on the petitioners at bar, then it would seem that the only restraint imposed by the law which we are capacitated to act upon is the injury to property sustained by the petitioners, an injury that would warrant the application of the most deferential standard the rational basis test. Yet as earlier stated, we recognize the capacity of the petitioners to invoke as well the constitutional rights of their patrons those persons who would be deprived of availing short time access or wash-up rates to the lodging establishments in question. D. It cannot be denied that the primary animus behind the ordinance is the curtailment of sexual behavior. The City asserts before this Court that the subject establishments have gained notoriety as venue of prostitution, adultery and fornications in Manila since they provide the necessary atmosphere for clandestine entry, presence and exit and thus became the ideal haven for prostitutes and thrill-seekers. Whether or not this depiction of a mise-en-scene of vice is accurate, it cannot be denied that legitimate sexual behavior among willing married or consenting single adults which is constitutionally protected will be curtailed as well, as it was in the City of Manila case. Our holding therein retains significance for our purposes: We cannot discount other legitimate activities which the Ordinance would proscribe or impair. There are very legitimate uses for a wash rate or renting the room out for more than twice a day. Entire families are known to choose pass the time in a motel or hotel whilst the power is momentarily out in their homes. In transit passengers who wish to wash up and rest between trips have a legitimate purpose for abbreviated stays in motels or hotels. Indeed any person or groups of persons in need of comfortable private spaces for a span of a few hours with purposes other than having sex or using illegal drugs can legitimately look to staying in a motel or hotel as a convenient alternative. E. Further, it is apparent that the Ordinance can easily be circumvented by merely paying the whole day rate without any hindrance to those engaged in illicit activities. Moreover, drug dealers and prostitutes can in fact collect wash rates from their clientele by charging their customers a portion of the rent for motel rooms and even apartments. The Ordinance needlessly restrains the operation of the businesses of the petitioners as well as restricting the rights of their patrons without sufficient justification. The Ordinance rashly equates wash rates and renting out a room more than twice a day with immorality without accommodating innocuous intentions. To be candid about it, the oft-quoted American maxim that you cannot legislate morality is ultimately illegitimate as a matter of law,
37

38

since as explained by Calabresi, that phrase is more accurately interpreted as meaning that efforts to legislate morality will fail if they are widely at variance with public attitudes about right and wrong. Our penal laws, for one, are founded on age-old moral traditions, and as long as there are widely accepted distinctions between right and wrong, they will remain so oriented. WHEREFORE, the Petition is GRANTED. Ordinance No. 7774 is hereby declared UNCONSTITUTIONAL. ********************************* An Ordinance requiring the motels in Ermita-Malate area to transfer to another place in the City of Manila as well as prohibiting THE ESTABLISHMENT OR OPERATION OF BUSINESSES PROVIDING CERTAIN FORMS OF AMUSEMENT, ENTERTAINMENT, SERVICES AND FACILITIES IN THE ERMITA-MALATE AREA was held unconstitutional CITY OF MANILA, HON. ALFREDO S. LIM as the Mayor of the City of Manila, et al vs. HON. PERFECTO A.S. LAGUIO, JR., as Presiding Judge, RTC, Manila and MALATE TOURIST DEVELOPMENT CORPORATION, GR No. 118127, April 12, 2005

TINGA, J.: FACTS: The City Council of Manila enacted on 9 March 1993 and approved by petitioner City Mayor on 30 March 1993 an Ordinance is entitled

AN ORDINANCE PROHIBITING THE ESTABLISHMENT OR OPERATION OF BUSINESSES PROVIDING CERTAIN FORMS OF AMUSEMENT, ENTERTAINMENT, SERVICES AND FACILITIES IN THE ERMITA-MALATE AREA, PRESCRIBING PENALTIES FOR VIOLATION THEREOF, AND FOR OTHER PURPOSES. The Ordinance is reproduced in full, hereunder: SECTION 1. Any provision of existing laws and ordinances to the contrary notwithstanding, no person, partnership, corporation or entity shall, in the Ermita-Malate area bounded by Teodoro M. Kalaw Sr. Street in the North, Taft Avenue in the East, Vito Cruz Street in the South and Roxas Boulevard in the West, pursuant to P.D. 499 be allowed or authorized to contract and engage in, any business providing certain forms of amusement, entertainment, services and facilities where women are used as tools in entertainment and which tend to disturb the community, annoy the inhabitants, and adversely affect the social and moral welfare of the community, such as but not limited to: 1. Sauna Parlors 2. Massage Parlors
38

39

3. Karaoke Bars 4. Beerhouses 5. Night Clubs 6. Day Clubs 7. Super Clubs 8. Discotheques 9. Cabarets 10. Dance Halls 11. Motels 12. Inns SEC. 2 The City Mayor, the City Treasurer or any person acting in behalf of the said officials are prohibited from issuing permits, temporary or otherwise, or from granting licenses and accepting payments for the operation of business enumerated in the preceding section. SEC. 3. Owners and/or operator of establishments engaged in, or devoted to, the businesses enumerated in Section 1 hereof are hereby given three (3) months from the date of approval of this ordinance within which to wind up business operations or to transfer to any place outside of the Ermita-Malate area or convert said businesses to other kinds of business allowable within the area, such as but not limited to: 1. Curio or antique shop 2. Souvenir Shops 3. Handicrafts display centers 4. Art galleries 5. Records and music shops 6. Restaurants 7. Coffee shops 8. Flower shops 9. Music lounge and sing-along restaurants, with well-defined activities for wholesome family entertainment that cater to both local and foreign clientele. 10. Theaters engaged in the exhibition, not only of motion pictures but also of cultural shows, stage and theatrical plays, art exhibitions, concerts and the like. 11. Businesses allowable within the law and medium intensity districts as provided for in the zoning ordinances for Metropolitan Manila, except new warehouse or open-storage depot, dock or yard, motor repair shop, gasoline service station, light industry with any machinery, or funeral establishments. The Ordinance was questioned as an invalid exercise of police power and violative of the due process and equal protection clause of the 1987 Constitution. HELD: The tests of a valid ordinance are well established. A long line of decisions has held that for an ordinance to be valid, it must not only be within the corporate powers of the local government unit to enact and must be passed according to the procedure prescribed by law, it must also conform to the following substantive requirements: (1) must not contravene the Constitution or any statute; (2) must not be unfair or oppressive; (3) must not be partial or discriminatory; (4) must not prohibit but may regulate trade; (5) must be general and consistent with public policy; and (6) must not be unreasonable. The Ordinance contravenes the Constitution
39

40

The police power of the City Council, however broad and farreaching, is subordinate to the constitutional limitations thereon; and is subject to the limitation that its exercise must be reasonable and for the public good. In the case at bar, the enactment of the Ordinance was an invalid exercise of delegated power as it is unconstitutional and repugnant to general laws. The relevant constitutional provisions are the following: SEC. 5. The maintenance of peace and order, the protection of life, liberty, and property, and the promotion of the general welfare are essential for the enjoyment by all the people of the blessings of democracy. SEC. 14. The State recognizes the role of women in nation-building, and shall ensure the fundamental equality before the law of women and men. SEC. 1. No person shall be deprived of life, liberty or property without due process of law, nor shall any person be denied the equal protection of laws. Sec. 9. Private property shall not be taken for public use without just compensation. A. The Ordinance infringes the Due Process Clause The constitutional safeguard of due process is embodied in the fiat (N)o person shall be deprived of life, liberty or property without due process of law. . . . There is no controlling and precise definition of due process. It furnishes though a standard to which governmental action should conform in order that deprivation of life, liberty or property, in each appropriate case, be valid. This standard is aptly described as a responsiveness to the supremacy of reason, obedience to the dictates of justice, and as such it is a limitation upon the exercise of the police power. The purpose of the guaranty is to prevent governmental encroachment against the life, liberty and property of individuals; to secure the individual from the arbitrary exercise of the powers of the government, unrestrained by the established principles of private rights and distributive justice; to protect property from confiscation by legislative enactments, from seizure, forfeiture, and destruction without a trial and conviction by the ordinary mode of judicial procedure; and to secure to all persons equal and impartial justice and the benefit of the general law. The guaranty serves as a protection against arbitrary regulation, and private corporations and partnerships are persons within the scope of the guaranty insofar as their property is concerned. This clause has been interpreted as imposing two separate limits on government, usually called procedural due process and substantive due process. Procedural due process, as the phrase implies, refers to the procedures that the government must follow before it deprives a person of life, liberty, or property. Classic procedural due process issues are concerned with what kind of notice and what form of hearing the government must provide when it takes a particular action. Substantive due process, as that phrase connotes, asks whether the government has an adequate reason for taking away a persons life, liberty, or property. In other words, substantive due process looks to whether there is a sufficient justification for the governments action. Case law in the United States (U.S.) tells us that whether there is such a justification depends very much on the level of scrutiny used. For
40

41

example, if a law is in an area where only rational basis review is applied, substantive due process is met so long as the law is rationally related to a legitimate government purpose. But if it is an area where strict scrutiny is used, such as for protecting fundamental rights, then the government will meet substantive due process only if it can prove that the law is necessary to achieve a compelling government purpose. The police power granted to local government units must always be exercised with utmost observance of the rights of the people to due process and equal protection of the law. Such power cannot be exercised whimsically, arbitrarily or despotically as its exercise is subject to a qualification, limitation or restriction demanded by the respect and regard due to the prescription of the fundamental law, particularly those forming part of the Bill of Rights. Individual rights, it bears emphasis, may be adversely affected only to the extent that may fairly be required by the legitimate demands of public interest or public welfare. Due process requires the intrinsic validity of the law in interfering with the rights of the person to his life, liberty and property. Requisites for the valid exercise of Police Power are not met To successfully invoke the exercise of police power as the rationale for the enactment of the Ordinance, and to free it from the imputation of constitutional infirmity, not only must it appear that the interests of the public generally, as distinguished from those of a particular class, require an interference with private rights, but the means adopted must be reasonably necessary for the accomplishment of the purpose and not unduly oppressive upon individuals. It must be evident that no other alternative for the accomplishment of the purpose less intrusive of private rights can work. A reasonable relation must exist between the purposes of the police measure and the means employed for its accomplishment, for even under the guise of protecting the public interest, personal rights and those pertaining to private property will not be permitted to be arbitrarily invaded. Lacking a concurrence of these two requisites, the police measure shall be struck down as an arbitrary intrusion into private rights and a violation of the due process clause. The Ordinance was enacted to address and arrest the social ills purportedly spawned by the establishments in the Ermita-Malate area which are allegedly operated under the deceptive veneer of legitimate, licensed and tax-paying nightclubs, bars, karaoke bars, girlie houses, cocktail lounges, hotels and motels. Petitioners insist that even the Court in the case of Ermita-Malate Hotel and Motel Operators Association, Inc. v. City Mayor of Manila had already taken judicial notice of the alarming increase in the rate of prostitution, adultery and fornication in Manila traceable in great part to existence of motels, which provide a necessary atmosphere for clandestine entry, presence and exit and thus become the ideal haven for prostitutes and thrill-seekers. The object of the Ordinance was, accordingly, the promotion and protection of the social and moral values of the community. Granting for the sake of argument that the objectives of the Ordinance are within the scope of the City Councils police powers, the means employed for the accomplishment thereof were unreasonable and unduly oppressive. The Ordinance seeks to legislate morality but fails to address the core issues of morality. Try as the Ordinance may to shape morality, it should not foster the illusion that it can make a moral man out of it because immorality is not a thing, a building or establishment; it is in the hearts of
41

42

men. The City Council instead should regulate human conduct that occurs inside the establishments, but not to the detriment of liberty and privacy which are covenants, premiums and blessings of democracy. While petitioners earnestness at curbing clearly objectionable social ills is commendable, they unwittingly punish even the proprietors and operators of wholesome, innocent establishments. In the instant case, there is a clear invasion of personal or property rights, personal in the case of those individuals desirous of owning, operating and patronizing those motels and property in terms of the investments made and the salaries to be paid to those therein employed. If the City of Manila so desires to put an end to prostitution, fornication and other social ills, it can instead impose reasonable regulations such as daily inspections of the establishments for any violation of the conditions of their licenses or permits; it may exercise its authority to suspend or revoke their licenses for these violations; and it may even impose increased license fees. In other words, there are other means to reasonably accomplish the desired end. Means employed are constitutionally infirm The Ordinance disallows the operation of sauna parlors, massage parlors, karaoke bars, beerhouses, night clubs, day clubs, super clubs, discotheques, cabarets, dance halls, motels and inns in the Ermita-Malate area. In Section 3 thereof, owners and/or operators of the enumerated establishments are given three (3) months from the date of approval of the Ordinance within which to wind up business operations or to transfer to any place outside the Ermita-Malate area or convert said businesses to other kinds of business allowable within the area. Further, it states in Section 4 that in cases of subsequent violations of the provisions of the Ordinance, the premises of the erring establishment shall be closed and padlocked permanently. It is readily apparent that the means employed by the Ordinance for the achievement of its purposes, the governmental interference itself, infringes on the constitutional guarantees of a persons fundamental right to liberty and property. Modality employed is unlawful taking In addition, the Ordinance is unreasonable and oppressive as it substantially divests the respondent of the beneficial use of its property.[77] The Ordinance in Section 1 thereof forbids the running of the enumerated businesses in the Ermita-Malate area and in Section 3 instructs its owners/operators to wind up business operations or to transfer outside the area or convert said businesses into allowed businesses. An ordinance which permanently restricts the use of property that it can not be used for any reasonable purpose goes beyond regulation and must be recognized as a taking of the property without just compensation.[78] It is intrusive and violative of the private property rights of individuals. The Constitution expressly provides in Article III, Section 9, that private property shall not be taken for public use without just compensation. The provision is the most important protection of property rights in the Constitution. This is a restriction on the general power of the government to take property. The constitutional provision is about
42

43

ensuring that the government does not confiscate the property of some to give it to others. In part too, it is about loss spreading. If the government takes away a persons property to benefit society, then society should pay. The principal purpose of the guarantee is to bar the Government from forcing some people alone to bear public burdens which, in all fairness and justice, should be borne by the public as a whole.[79] The second option instructs the owners to abandon their property and build another one outside the Ermita-Malate area. In every sense, it qualifies as a taking without just compensation with an additional burden imposed on the owner to build another establishment solely from his coffers. The proffered solution does not put an end to the problem, it merely relocates it. Not only is this impractical, it is unreasonable, onerous and oppressive. The conversion into allowed enterprises is just as ridiculous. How may the respondent convert a motel into a restaurant or a coffee shop, art gallery or music lounge without essentially destroying its property? This is a taking of private property without due process of law, nay, even without compensation. Petitioners cannot therefore order the closure of the enumerated establishments without infringing the due process clause. These lawful establishments may be regulated, but not prevented from carrying on their business. This is a sweeping exercise of police power that is a result of a lack of imagination on the part of the City Council and which amounts to an interference into personal and private rights which the Court will not countenance. In this regard, we take a resolute stand to uphold the constitutional guarantee of the right to liberty and property. The foregoing premises show that the Ordinance is an unwarranted and unlawful curtailment of property and personal rights of citizens. For being unreasonable and an undue restraint of trade, it cannot, even under the guise of exercising police power, be upheld as valid. B. The Ordinance violates Equal Protection Clause Equal protection requires that all persons or things similarly situated should be treated alike, both as to rights conferred and responsibilities imposed. Similar subjects, in other words, should not be treated differently, so as to give undue favor to some and unjustly discriminate against others.[98] The guarantee means that no person or class of persons shall be denied the same protection of laws which is enjoyed by other persons or other classes in like circumstances.[99] The equal protection of the laws is a pledge of the protection of equal laws.[100] It limits governmental discrimination. The equal protection clause extends to artificial persons but only insofar as their property is concerned.[101] Legislative bodies are allowed to classify the subjects of legislation. If the classification is reasonable, the law may operate only on some and not all of the people without violating the equal protection clause.[103] The classification must, as an indispensable requisite, not be arbitrary. To be valid, it must conform to the following requirements: 1) It must be based on substantial distinctions. 2) It must be germane to the purposes of the law. 3) It must not be limited to existing conditions only. 4) It must apply equally to all members of the class.[104]

43

44

In the Courts view, there are no substantial distinctions between motels, inns, pension houses, hotels, lodging houses or other similar establishments. By definition, all are commercial establishments providing lodging and usually meals and other services for the public. No reason exists for prohibiting motels and inns but not pension houses, hotels, lodging houses or other similar establishments. The classification in the instant case is invalid as similar subjects are not similarly treated, both as to rights conferred and obligations imposed. It is arbitrary as it does not rest on substantial distinctions bearing a just and fair relation to the purpose of the Ordinance. The Court likewise cannot see the logic for prohibiting the business and operation of motels in the Ermita-Malate area but not outside of this area. A noxious establishment does not become any less noxious if located outside the area.

f. to promote the economic security of the people. (ICHONG VS. HERNANDEZ, 101 Phil. 11155) Not a valid exercise of police power: a. CITY GOVERNMENT OF QC VS. ERICTA, 122 SCRA 759; (Requiring private cemeteries to set aside a portion of their land area to be given as burial place for paupers, free of charge, is an invalid exercise of police power. It constitutes taking of a private property for public use without just compensation. The local government units could not validly pass to private cemeteries their obligation under the Local Government Code to provide cemeteries to their constituents) b. YNOT VS. IAC, 148 SCRA 659; the Director of Animal Industry or the Chairman if the National Meat Commission may dispose of the carabaos or carabeef confiscated for violating the executive order prohibiting the inter-provincial transport of said animals without prior permit issued by the government to charitable agencies as he may deem fit. This is oppressive and unreasonable since the owner of the animals is denied due process of law and the Director of Animal Industry or Chairman of the National Meat Commission is given so much discretion as the law is not complete in itself nor is there a standard to guide the official. c. DE LA CRUZ VS. PARAS, 123 SCRA 569 (An Ordinance of Bocaue, Bulacan prohibiting the operation of nightclubs is unconstitutional. It is not a valid exercise of police power. This is so because nightclubs are not illegal per se. They can be regulated but not prohibited) power of eminent domain power of taxation 2. Differences and similarities DIDIPIO EARTH SAVERS MULTI PURPOSE ASSOCIATION VS. DENR SEC. ELISEA GOZU, ET AL., 485 SCRA 586 Chico-Nazario, J. 1. The power of eminent domain is the inherent right of the State to condemn or to take private property for public use upon payment of just
44

45

compensation while police power is the power of the state to promote public welfare by restraining and regulating the use of liberty and property without compensation; 2. In the exercise of police power, enjoyment of a property is restricted because the continued use thereof would be injurious to public welfare. In such case, there is no compensable taking provided none of the property interests is appropriated for the use or for the benefit of the public. Otherwise, there should be compensable taking if it would result to public use. 3. Properties condemned under police power are usually noxious or intended for noxious purpose; hence , no compensation shall be paid. Likewise, in the exercise of police power, property rights of private individuals are subjected to restraints and burdens in order to secure the general comfort, health and prosperity of the state. While the power of eminent domain often results in the appropriation of title to or possession of property, it need not always be the case. Taking may include trespass without actual eviction of the owner, material impairment of the value of the property or prevention of the ordinary uses for which the property was intended such as the establishment of an easement. As such, an imposition of burden over a private property through easement (by the government) is considered taking; hence, payment of just compensation is required. The determination of just compensation, however, is a judicial function (EPZA vs. Dulay, 149 SCRA 305) and initial determinations on just compensation by the executive department and Congress cannot prevail over the courts findings. 3. Limitations in the exercise of said powers 4. Tests for a valid exercise of police power a. the interests of the public, not mere particular class, require the exercise of police power; (LAWFUL SUBJECT) b. the means employed is reasonably necessary for the accomplishment of the purpose and not unduly oppressive to individuals. (LAWFUL MEANS). In short, the end does not justify the means. Illustration: Lawful subject but the means employed is illegal RESTITUTO YNOT VS. THE ITERMEDIATE APPELLATE COURT, G.R. No. 74457,March 20, 1987 Cruz, J. Facts: 1. On January, 13, 1984, Ynot transported six carabaos by using a pumpboat from Masbate to Iloilo. The six carabaos, were, however, confiscated by the Police Station Commander of Baratoc Nuevo, Iloilo for alleged violation of Executive Order No. 626-A which prohibits the interprovincial transporting of carabaos and carabeefs which does not comply with the provisions of Executive No.626;

45

46

2. That Section 1 of the said law provides that "henceforth, no carabaos regardless of age, sex physical condition or purpose and no carabeef shall be transported from one province to another. The carabao or carabeef transported in violation of the said law shall be subjected to confiscation and forfeiture by the government to be distributed to charitable institution and similar institutions as the Chairman of the National meat inspection Commission may see fit in the case of the carabeef, and to deserving farmers through the dispersal of the Director of Animal Industry, in the case of carabaos; 3. Ynot filed a suit for recovery and the carabao were returned to him upon the issuance of a writ of replevin upon his filing of a supersede as bond in the amount of P12,000.00; 4. After trial of the case, the Judge upheld the validity of the act of the Police Station Commander in confiscating the carabaos. Ynot was ordered to returned the carabaos but since he could not do so, the court ordered the confiscation of the bond. The court refused to rule on the constitutionality of the said Executive Order on the ground of lack of authority to do so and also because of its presumed validity; 5. The petitioner appealed to the IAC but the said court upheld the decision of the Trial Court. Hence this petition for review on certiorari before the Supreme Court where YNOT claimed that the penalty of confiscation is INVALID the same was imposed without according the owner the right to be heard before a competent and impartial tribunal as guaranteed by due process. Issues: 1. May a lower court (like the MTC, RTC, of the Court of Appeals) declare a law unconstitutional? 2. Is Executive Order No. 626-A constitutional? Sub-issues under this are: a. Was it a valid police power measure? b. Was there an undue delegation of legislative power? Held: 1. While the lower courts should observe a becoming modesty in examining constitutional question, THEY ARE NOT PREVENTED FROM RESOLVING THE SAME WHENEVER WARRANTED, subject only to review by the supreme court. This is so because under Section 5,[2(a)], Art. VIII, of the 1987 Constitution provides that the Supreme Court has the power to "review, revise, reverse, modify or affirm on appeal" or certiorari as the rules of court may provide, final judgments and orders of the lower courts in all cases involving the constitutionality of certain measures. This simply means that lower courts may declare whether or not a law is constitutional. 2. In order that a measure or law may be justified under the police power of the state, it must meet two tests:
46

47

a. the subject must be lawful; and b. the means employed is lawful. Since the prohibition of the slaughtering of carabaos except where they are at least 7 years old when male and at least 11 years old when female is in furtherance of the public interest since said carabaos are very useful to the work at the farm, it is conceded that the Executive Order meets the first test---- it has lawful subject. But does the law meets the second requisite or test which is lawful method? Executive Order No. 626-A imposes an absolute ban not on the slaughtering of carabaos BUT ON THIER MOVEMENT, providing that "no carabao regardless of age, sex, physical condition or purpose and no carabeef shall be transported from one province to another." The reasonable connection between the means employed and the purpose sought to be achieved by the question measure is missing. We do not see how the prohibition of the inter-provincial transport can prevent their indiscriminate slaughter considering that they can be killed any where, with no less difficulty in one province than in the other. Obviously, retaining a carabao in one province will not prevent their slaughter there, any more than moving them to another province will make it easier to kill them there. The law is unconstitutional because it struck at once and pounced upon the petitioner without giving him a chance to be heard, thus denying him the centuries-old guarantee of elementary fair play. Since the Executive Order in question is a penal law, then violation thereof should be pronounce not by the police BUT BY A COURT OF JUSTICE, WHICH ALONE WOULD HAVE HAD THE AUTHORITY TO IMPOSE THE PRESCRIBED PENALTY, AND ONLY AFTER TRIAL AND CONVICTION OF THE ACCUSED. Also, there is no reasonable guidelines or bases of the Director of Animal Industry or the Chairman of the NATIONAL Meat Inspection Commission in the disposition of the carabaos or carabeef other than what "they may see fit" which is very dangerous and could result to opportunities for partiality and abuse, and even graft and corruption. The Executive Order is, therefore, invalid and unconstitutional and not a valid police power measure because the METHOD EMPLOYED TO CONSERVE CARABAOS IS NOT REASONABLY NECESSARY TO THE PURPOSE OF THE LAW AND, WORSE IS UNDULY OPPRESSIVE. DUE PROCESS IS VIOLATED BECAUSE THE OWNER OF THE PROPERTY CONFISCATED IS DENIED THE RIGHT TO BE HEARD IN HIS DEFENSE AND IS IMMEDIATELY CONDEMNED AND PUNISHED. THE CONFERMENT ON THE ADMINISTRATIVE AUTHORITIES (like the police) OF THE POWER TO ADJUDGE THE GUILT OF THE SUPPOSED OFFENDER IS A CLEAR ENCROACHMENT OF JUDICIAL FUNCTIONS AND MILITATES AGAINST THE DOCTRINE OF SEPARATION OF POWERS. Also, there is undue delegation of legislative power to the officers mentioned therein (Director of Animal Industry and Head of the National
47

48

Meat Commission) because they were given unlimited discretion in the distribution of the property confiscated. 5. Read: q. r. s. t. u. v. w. x. JMM Promotions vs. CA, 260 SCRA 319 ERMITA-MALATE HOTEL VS. MAYOR OF MANILA, July 31, 1967; ICHONG VS. HERNANDEZ, 101 Phil. 1155 CHURCHILL VS. RAFFERTY, 32 Phil. 580 PEOPLE VS. POMAR, 46 Phil. 447 US VS. TORIBIO, 15 Phil. 85 VELASCO VS. VILLEGAS, February 13, 1983 ILOILO ICE & COLD STORAGE VS. MUNICIPAL COUNCIL, 24 Phil. 471 y. AGUSTIN VS. EDU, 88 SCRA 195 z. TAXICAB OPERATORS VS. BOT, 119 SCRA 597 aa. BAUTISTA VS. JUINIO, 127 SCRA 329 A law prohibiting the use of Heavy and Extra Heavy Vehicles on weekends and holidays when there is energy crisis is a valid police power measure. MARY CONCEPCION-BAUTISTA VS. ALFREDO JUINIO, ET AL, 127 SCRA 329 Fernando, C.J. Facts: 1. On May 31, 1979, President Marcos issued Letter of Instruction No. 869 prohibiting the use of private motor vehicles with H (Heavy Vehicles) and EH (Extra Heavy Vehicles) on week-ends and holidays from 12:00 a.m. Saturday morning to 5:00 a.m. Monday morning, or 1:00 a.m. of the holiday to 5:00 a.m. of the day after the holiday. Motor vehicles of the following classifications are however, exempted: 1. S----service; 2. T----Truck; 3. DPL--Diplomatic; 4. CC---Consular Corps; and 5. TC---Tourist Cars 2. On June 11, 1979, the then Commissioner of Land Transportation, ROMEO EDU issued Circular No. 39 imposing "the penalties of fine, confiscation of vehicle and cancellation of registration on owners of the above-specified found violating such letter of Instructions"; 3. Bautista is questioning the constitutionality of the LOI and the Implementing Circular on the grounds that: a. The banning of H and EH vehicles is unfair, discriminatory, and arbitrary and thus contravenes the EQUAL PROTECTION CLAUSE; and b. The LOI denies the owners of H and EH vehicles of due process, more specifically of their right to use and enjoy their private property and of their freedom to travel and hold family gatherings,
48

49

reunions, outings on week-ends and holidays, while those not included in the prohibition are enjoying unrestricted freedom; c. The Circular violates the prohibition against undue delegation of legislative power because the LOI does not impose the penalty of confiscation. HELD: 1. It must be pointed out that the LOI was promulgated to solve the oil crisis which was besetting the country at that time. It was therefore a valid police power measure to ensures the country's economy as a result of spiralling fuel prices. In the interplay of Bautista's right to due process and the exercise of police power by the State, the latter must be given leeway. The police power is intended to promote public health, public morals, public safety and general welfare. 2. The petitioners' claim that their right to equal protection was violated is without basis. This is so because there is a valid classification in this case. Definitely, Heavy and Extra-Heavy vehicles consume more gasoline that the other kinds of vehicles and it is but proper to regulate the use of those which consumes more gasoline. If all the owner of H and EH vehicles are treated in the same fashion, or whatever restrictions cast on some in the group is held equally binding on the rest, there is no violation of the equal protection clause. 3. The penalty of "impounding" the vehicle as embodied in Circular No. 39 has no statutory basis. Therefore, it is not valid being an "ultra vires". bb. ASSOCIATION OF SMALL LANDOWNERS VS. SECRETARY OF AGRARIAN REFORM, 175 SCRA 343 cc. DECS VS. SAN DIEGO, 180 SCRA 533 dd. VILLANUEVA VS. CASTANEDA, September 21, 1987 5-a. Not a valid exercise of police power CITY GOVERNMENT OF QUEZON CITY VS. ERICTA, 122 SCRA 759 CHAPTER IIDUE PROCESS Section 1---NO PERSON SHALL BE DEPRIVED OF LIFE, LIBERTY OR PROPERTY WITHOUT DUE PROCESS OF LAW, NOR SHALL ANY PERSON BE DENIED EQUAL PROTECTION OF THE LAWS. Kinds of Due Process: a. substantive due process---requires the intrinsic validity of the law in interfering with the rights of the person to life, liberty or property. In short, it is to determine whether it has a valid governmental objective like for the interest of the public as against mere particular class. b. Procedural due process---one which hears before it condemns as pointed out by Daniel Webster.
49

50

Due process is a law which hears before it condemns, which proceeds upon inquiry and renders judgment only after trial (Per Daniel Webster in the DARTMOUTH COLLEGE CASE) Due process have different requisites in: 3. 4. 5. 6. Due process before judicial bodies or judicial due process; Due process before administrative bodies; Due process before the labor tribunals; and Due process involving students.

If the proceeding is not covered by any of the above, due process may not be invoked if one was not given the right to be heard. Illustrative case:

DUE PROCESS

ABAD, J.: On July 5, 2005 respondent Franklin M. Drilon , then the president of the Liberal Party (LP), announced his partys withdrawal of support for the administration of President Gloria Macapagal-Arroyo. But petitioner Jose L. Atienza, Jr., LP Chairman, and a number of party members denounced Drilons move, claiming that he made the announcement without consulting his party. On March 2, 2006 petitioner Atienza hosted a party conference to supposedly discuss local autonomy and party matters but, when convened, the assembly proceeded to declare all positions in the LPs ruling body vacant and elected new officers, with Atienza as LP president. Respondent Drilon immediately filed a petition with the Commission on Elections (COMELEC) to nullify the elections. He claimed that it was illegal considering that the partys electing bodies, the National Executive Council (NECO) and the National Political Council (NAPOLCO), were not properly convened. Drilon also claimed that under the amended LP Constitution, party officers were elected to a fixed three-year term that was yet to end on November 30, 2007. On the other hand, petitioner Atienza claimed that the majority of the LPs NECO and NAPOLCO attended the March 2, 2006 assembly. The election of new officers on that occasion could be likened to people power, wherein the LP majority removed respondent Drilon as president by direct action. Atienza also said that the amendments to the original LP Constitution, or the Salonga Constitution, giving LP officers a fixed three-year term, had not been properly
50

51

ratified. Consequently, the term of Drilon and the other officers already ended on July 24, 2006 On October 13, 2006, the COMELEC issued a resolution, partially granting respondent Drilons petition. It annulled the March 2, 2006 elections and ordered the holding of a new election under COMELEC supervision. It held that the election of petitioner Atienza and the others with him was invalid since the electing assembly did not convene in accordance with the Salonga Constitution. But, since the amendments to the Salonga Constitution had not been properly ratified, Drilons term may be deemed to have ended. Thus, he held the position of LP president in a holdover capacity until new officers were elected. Both sides of the dispute went to the Supreme Court to challenge the COMELEC rulings. On April 17, 2007 a divided Court issued a resolution, granting respondent Drilons petition and denying that of petitioner Atienza. The Court held, through the majority, that the COMELEC had jurisdiction over the intra-party leadership dispute; that the Salonga Constitution had been validly amended; and that, as a consequence, respondent Drilons term as LP president was to end only on November 30, 2007. Subsequently, the LP held a NECO meeting to elect new party leaders before respondent Drilons term expired. Fifty-nine NECO members out of the 87 who were supposedly qualified to vote attended. Before the election, however, several persons associated with petitioner Atienza sought to clarify their membership status and raised issues regarding the composition of the NECO. Eventually, that meeting installed respondent Manuel A. Roxas II (Roxas) as the new LP president. On January 11, 2008 petitioners Atienza, Matias V. Defensor, Jr., Rodolfo G. Valencia, Danilo E. Suarez, Solomon R. Chungalao, Salvacion Zaldivar-Perez, Harlin Cast-Abayon, Melvin G. Macusi, and Eleazar P. Quinto, filed a petition for mandatory and prohibitory injunction before the COMELEC against respondents Roxas, Drilon and J.R. Nereus O. Acosta, the party secretary general. Atienza, et al. sought to enjoin Roxas from assuming the presidency of the LP, claiming that the NECO assembly which elected him was invalidly convened. They questioned the existence of a quorum and claimed that the NECO composition ought to have been based on a list appearing in the partys 60th Anniversary Souvenir Program. Both Atienza and Drilon adopted that list as common exhibit in the earlier cases and it showed that the NECO had 103 members. Petitioners Atienza, et al. also complained that Atienza, the incumbent party chairman, was not invited to the NECO meeting and that some members, like petitioner Defensor, were given the status of guests during the meeting. Atienzas allies allegedly raised these issues but respondent Drilon arbitrarily thumbed them down and railroaded the proceedings. He suspended the meeting and moved it to another room, where Roxas was elected without notice to Atienzas allies. On the other hand, respondents Roxas, et al. claimed that Roxas election as LP president faithfully complied with the provisions of the amended LP Constitution. The partys 60th Anniversary Souvenir Program could not be used for determining the NECO members because supervening events changed the bodys number and composition. Some NECO members had died, voluntarily resigned, or had gone on leave after accepting positions in the government. Others had lost their re-election bid or did not run in the May 2007 elections, making them ineligible to serve as NECO members. LP members who got elected to public office also became part of the NECO. Certain persons of
51

52

national stature also became NECO members upon respondent Drilons nomination, a privilege granted the LP president under the amended LP Constitution. In other words, the NECO membership was not fixed or static; it changed due to supervening circumstances. Respondents Roxas, et al. also claimed that the party deemed petitioners Atienza, Zaldivar-Perez, and Cast-Abayon resigned for holding the illegal election of LP officers on March 2, 2006. This was pursuant to a March 14, 2006 NAPOLCO resolution that NECO subsequently ratified. Meanwhile, certain NECO members, like petitioners Defensor, Valencia, and Suarez, forfeited their party membership when they ran under other political parties during the May 2007 elections. They were dropped from the roster of LP members. On June 18, 2009 the COMELEC issued the assailed resolution denying petitioners Atienza, et al.s petition. It noted that the May 2007 elections necessarily changed the composition of the NECO since the amended LP Constitution explicitly made incumbent senators, members of the House of Representatives, governors and mayors members of that body. That some lost or won these positions in the May 2007 elections affected the NECO membership. Petitioners failed to prove that the NECO which elected Roxas as LP president was not properly convened. As for the validity of petitioners Atienza, et al.s expulsion as LP members, the COMELEC observed that this was a membership issue that related to disciplinary action within the political party. The COMELEC treated it as an internal party matter that was beyond its jurisdiction to resolve. Without filing a motion for reconsideration of the COMELEC resolution, petitioners Atienza, et al. filed this petition for certiorari under Rule 65. ISSUE Whether or not respondents Roxas, et al. violated petitioners Atienza, et al.s constitutional right to due process by the latters expulsion from the party. HELD: Petitioners Atienza, et al. argue that their expulsion from the party is not a simple issue of party membership or discipline; it involves a violation of their constitutionally-protected right to due process of law. They claim that the NAPOLCO and the NECO should have first summoned them to a hearing before summarily expelling them from the party. According to Atienza, et al., proceedings on party discipline are the equivalent of administrative proceedings and are, therefore, covered by the due process requirements laid down in Ang Tibay v. Court of Industrial Relations. But the requirements of administrative due process do not apply to the internal affairs of political parties. The due process standards set in Ang Tibay cover only administrative bodies created by the state and through which certain governmental acts or functions are performed. An administrative agency or instrumentality contemplates an authority to which the state delegates governmental power for the performance of a state function. The
52

53

constitutional limitations that generally apply to the exercise of the states powers thus, apply too, to administrative bodies The constitutional limitations on the exercise of the states powers are found in Article III of the Constitution or the Bill of Rights. The Bill of Rights, which guarantees against the taking of life, property, or liberty without due process under Section 1 is generally a limitation on the states powers in relation to the rights of its citizens. The right to due process is meant to protect ordinary citizens against arbitrary government action, but not from acts committed by private individuals or entities. In the latter case, the specific statutes that provide reliefs from such private acts apply. The right to due process guards against unwarranted encroachment by the state into the fundamental rights of its citizens and cannot be invoked in private controversies involving private parties. Although political parties play an important role in our democratic set-up as an intermediary between the state and its citizens, it is still a private organization, not a state instrument. The discipline of members by a political party does not involve the right to life, liberty or property within the meaning of the due process clause. An individual has no vested right, as against the state, to be accepted or to prevent his removal by a political party. The only rights, if any, that party members may have, in relation to other party members, correspond to those that may have been freely agreed upon among themselves through their charter, which is a contract among the party members. Members whose rights under their charter may have been violated have recourse to courts of law for the enforcement of those rights, but not as a due process issue against the government or any of its agencies. But even when recourse to courts of law may be made, courts will ordinarily not interfere in membership and disciplinary matters within a political party. A political party is free to conduct its internal affairs, pursuant to its constitutionally-protected right to free association. In Sinaca v. Mula, the Court said that judicial restraint in internal party matters serves the public interest by allowing the political processes to operate without undue interference. It is also consistent with the state policy of allowing a free and open party system to evolve, according to the free choice of the people. To conclude, the COMELEC did not gravely abuse its discretion when it upheld Roxas election as LP president but refused to rule on the validity of Atienza, et al.s expulsion from the party. While the question of party leadership has implications on the COMELECs performance of its functions under Section 2, Article IX-C of the Constitution, the same cannot be said of the issue pertaining to Atienza, et al.s expulsion from the LP. Such expulsion is for the moment an issue of party membership and discipline, in which the COMELEC cannot intervene, given the limited scope of its power over political parties. 7. Requisites of judicial due process. m. BANCO ESPANOL VS. PALANCA, 37 Phil. 921 Requisites: 1. There must be an impartial court or tribunal clothed with judicial power to hear and decide the matter before it; 2. Jurisdiction must be lawfully acquired over the person of the defendant or over the property subject of the proceedings; 3. The defendant must be given the opportunity to be heard;
53

54

4. Judgment must be rendered only after lawful hearing. a. GALMAN VS. PAMARAN (the 1st case) n. IMELDA MARCOS VS. SANDIGANBAYAN, October 6, 1998 IMELDA R. MARCOS VS. SANDIGANBAYAN, G.R. No. 126995, October 6, 1998 Purisima, J. Facts: 1. On June 8, 1984, IMELDA MARCOS and JOSE DANS, as Chairman and Vice Chairman of the Light Railway Transit Authority (LRTA) entered into a Lease Contract with the Philippine General Hospital Foundation (PGHFI) involving an LRTA property in Pasay City for P102,760.00 per month for 25 years; 2. On June 27,1984, the PGHFI subleased the said property for P734,000.00 per month to the Transnational Construction Corporation represented by one Ignacio Jumenez; 3. After petitioners husband was deposed as President of the Philippines, she and Dans were charged of alleged violation of Section 3 [g] of RA 3019, otherwise known as the Anti-Graft and Corrupt Practices Act before the Sandiganbayan; 4. After trial , the First Division of the Sandiganbayan failed to comply with the legal requirement that all the 3 justices must be unanimous in its Decision because Justice Garchitorena and Justice Jose Balajadia voted for the conviction of both accused while Justice Narciso Atienza voted to acquit them; 5. Thereafter, Justice Garchitorena as Presiding Justice issued Administrative Order No. 288-93 constituting a Special Division of five and designating Justices Augusto Amores and Cipriano del Rosario; 6. On September 21, 1993, Justice Amores wrote Justice Garchitorena that he be given 15 days his Manifestation. On the same date, however, Justice Garchitorena dissolved the division of 5 allegedly because he and Justice Balajadia had agreed to the opinion of Justice del Rosario; 7. On September 24, 1993, a Decision was rendered convicting the petitioner and Dans of violation of Sec. 3 [g] of RA 3019; 8. On June 29, 1998, the Third Division of the Supreme Court by a vote of 32 affirmed the conviction of the petitioner but acquitted DANS; 9. Petitioner then filed a Motion for Reconsideration and at the same time prayed that her Motion be heard by the Supreme Court en banc claiming that her right to due process of law, both substantive and procedural, was violated: a. as a result of the fact that she was convicted as a result of the alleged disparity of the rentals agreed upon with PGHFI and the subsequent sublease contract between PGHFI and Transnational Construction Corporation; and b. the First Division convicted her after Justice Garchitorena dissolved the Special Division of 5 after a lunch in a Quezon City restaurant where they agreed to convict her in one case and acquit her in her other cases. The said meeting was attended by another justice who is not a member of the First Division or the Special Division in violation of the Rules of the Sandiganbayan which requires that sessions of the court shall be done only in its principal office in Manila and that only justices belonging to the division should join the deliberations.
54

55

Held: The petitioner is hereby acquitted. 1. The great disparity between the rental price of the lease agreement signed by the petitioner (P102,760.00 per month) and the sub-lease rental (P734,000.00 per month) does not necessarily render the monthly rate of P102,760.00 manifestly and grossly disadvantageous to the government in the absence of any evidence using rentals of adjacent properties showing that the rentals in the property subject of the lease agreement is indeed very low. NO EVIDENCE WHATSOEVER WAS PRESENTED BY THE PROSECUTION REGARDING THE RENTAL RATE OF ADJACENT PROPERTIES.. As such, the prosecution failed to prove the guilt of the petitioner reasonable doubt. 2. The court notes likewise the bias and prejudice of Presiding Justice Garchitorena against the petitioner as shown by his leading, misleading and baseless hypothetical questions of said justice to RAMON F. CUERVO, witness for the petitioner. Said justice asked 179 questions to the witness as against the prosecutor who cross-examined the witness and asked 73. Said number of questions could no longer be described as clarificatory questions. Another ground therefore for the acquittal of the petitioner is that she was denied IMPARTIAL TRIAL before the Sandiganbayan. This is one reason why the case could no longer be remanded to the Sandiganbayan especially so that the other Sandiganbayan Justices in the Special Division of 5 have retired. There is therefore no compelling reason why the case should still be remanded to the lower court when all the evidence are already with the Supreme Court. (NOTE: The vote was 9-5 for Acquittal. CJ Narvasa, Justices Regalado, Davide, Jr., Romero, and Panganiban voted for conviction while Justice Vitug was the only Justice who voted for the return of the case to the Sandiganbayan to allow the corrections of the perceived irregularities in the proceedings below.) o. DBP VS. CA, January 29, 1999 (Repeated failure of a party to present evidence justifies the court to consider the case submitted for decision and hold that the party has waived the right to present evidence) p. MATUGUINA VS. CA, 263 SCRA 490 q. PEOPLE VS. CA, 262 SCRA 452 r. JAVIER VS. COMELEC, 144 SCRA 194

JAVIER VS. COMELEC G.R. No.L- 68379-812, September 22, 1986 FACTS: 1. The petitioner Evelio Javier and the private respondent Arturo Pacificador were candidates in Antique for the Batasang Pambansa election in May 1984; 2. Alleging serious anomalies in the conduct of the elections and the canvass of the election returns, Javier went to the COMELEC to prevent the impending proclamation of his rival;

55

56

3. On May 18, 1984, the Second Division of the COMELEC directed the provincial board of canvassers to proceed with the canvass but to suspend the proclamation of the winning candidate until further orders; 4. On June 7, 1984, the same Second Division ordered the board to immediately convene and to proclaim the winner without prejudice to the outcome of the petition filed by Javier with the COMELEC; 5. On certiorari with the S.C. the proclamation made by the Board of Canvassers was set aside as premature, having been made before the lapse of the 5 - day period of appeal, which the petitioner seasonably made; 6. On July 23, 1984 the Second Division itself proclaimed Pacificador the elected assemblyman of Antique. ISSUE: Was the Second Division of the COMELEC, authorized to promulgate its decision of July 23, 1984 proclaiming Pacificador the winner in the election ? APPLICABLE PROVISIONS OF THE CONSITUTION: The applicable provisions of the 1973 Constitution are Art. XII-C, secs. 2 and 3, which provide: "Section 2. Be the sole judge of all contests relating to the election, returns and qualifications of all members of the Batasang Pambansa and elective provincial and city officials." "Section 3. The Commission on Elections may sit en banc or in three divisions. All election cases may be heard and decided by divisions except contests involving members of the Batasang Pambansa, which shall be heard and decided en banc. Unless otherwise provided by law, all election cases shall be decided within ninety days from the date of their submission for decision." CONTENTIONS OF THE PARTIES: Petitioner: The proclamation made by the Second Division is invalid because all contests involving members of the Batasang Pambansa come under the jurisdiction of the Commission on Elections en banc. Respondents: Only "contests" need to be heard and decided en banc, all other cases can be - in fact, should be - filed with and decided only by any of the three divisions. There is a difference between "contests" and "cases" and also a difference between "pre-proclamation controversies" and "election protests". The pre-proclamation controversy between the petitioner and the private respondent was not yet a contest at the time and therefore could be validly heard by a mere division of the Commission on elections,
56

57

consonant with Sec. 3. The issue at that stage was still administrative and could be resolved by a division. HELD: a. The S.C. decided to resolve the case even if the Batasang Pambansa had already been abolished by the Aquino government, and even if Javier had already died in the meantime. This was because of its desire for this case to serve as a guidance for the future. Thus it said: "The Supreme Court is not only the highest arbiter of legal questions but also the conscience of the government. The citizen comes to us in quest of law but we must also give him justice. The two are not always the same. There are times when we cannot grant the latter because the issue has been settled and decision is no longer possible according to law. But there are also times when although the dispute has disappeared, as in this case, it nevertheless cries out to be resolved. Justice demands that we act, then, not only for the vindication of the outraged right, though gone, but also for the guidance of and as a restraint upon the future." b. The S.C. held on the main issue that in making the COMELEC the sole judge of all contests involving the election, returns and qualifications of the members of the Batasang Pambansa and elective provincial and city officials, the Constitution intended to give it full authority to hear and decide these cases from beginning to end and on all matter related thereto, including those arising before the proclamation of the winners. The decision rendered by the Second Division alone was therefore set aside as violative of the Constitution. The case should have been decided en banc. c. Pre-proclamation controversies became known and designated as such only because of Sec. 175 of the 1978 Election Code. The 1973 Constitution could not have therefore been intended to have divided contests between pre and post proclamation when that Constitution was written in 1973. d. The word "contests" should not be given a restrictive meaning; on the contrary, it should receive the widest possible scope conformably to the rule that the words used in the Constitution should be interpreted liberally. As employed in the 1973 Constitution, the term should be understood as referring to any matter involving the title or claim of title to an elective office, made before or after the proclamation of the winner, whether or not the contestant is claiming the office in dispute. e. There was also a denial of due process. One of the members of the Second Division, Commissioner Jaime Opinion was a law partner of Pacificador. He denied the motion to disqualify him from hearing the case. The Court has repeatedly and consistently demanded "the cold neutrality of an impartial judge" as the indispensable imperative of due process . To bolster that requirement we have held that the judge must not only be impartial but must also appear to be impartial as an added assurance to the parties that his decision will be just. s. AZUL VS. CASTRO, 133 SCRA 271 t. PADERANGA VS. AZURA, 136 SCRA 266 u. DAVID VS. AQUILIZAN, 94 SCRA 707
57

58

v. LORENZANA VS. CAYETANO, 78 SCRA 485 (respondent was not a party to the ejectment case) so to enforce the decision on her violates her right to due process of law w. ZAMBALES CHROMITE MINING VS. CA, 94 SCRA 261 x. ANZALDO VS. CLAVE, 119 SCRA 353 y. SINGSON VS. NLRC, 273 SCRA 258 z. ANZALDO VS. CLAVE, 119 SCRA 353 aa. MAYOR ALONTE VS. JUDGE SAVELLANO, 287 SCRA 245 MAYOR BAYANI ALONTE VS. JUDGE SAVELLANO, 287 SCRA 245 Vitug, J. Mayor Alonte of Binan, Laguna was charged of rape before Branch 25, RTC of Laguna. However, as a result of a petition for a transfer of venue filed by the prosecution and granted by the SC, his case was transferred to RTC Branch 53, Manila, presided over by the respondent judge. After the petitioners arraignment, the prosecution submitted an AFFIDAVIT OF DESISTANCE signed by the private complainant JUVIE-LYN PUNONGBAYAN where she prayed for the withdrawal of the case because she is no longer interested in pursuing the same with no intention of re-filing the said case in the future. Pending resolution of the said motion to withdraw, the petitioner filed a motion for bail. The same was not resolved despite several motions filed by the petitioner to resolve the same. On December 17, 1997, counsel for the petitioner, ATTY. PHILIP SIGFRID FORTUN, received a notice from the respondent judge notifying him of the promulgation of the decision in this case despite the fact that the prosecution and the defense have not presented their evidence in court. On December 18, 1997, the respondent judge issued a Decision convicting the petitioner of rape and sentenced to suffer a penalty of RECLUSION PERPETUA. Issue: Whether or not the petitioner was denied his right to due process of law. Held: In order that an accused in a criminal proceedings is deemed to have been given the right to due process of law, the following requisites must be complied with before a decision is rendered: 1. the court or tribunal trying the case is clothed with jurisdiction to hear and determine the matter before it; 2. that jurisdiction was lawfully acquired by it over the person of the accused; 3. that the accused is given the opportunity to be heard; and 4. that judgment is rendered only upon lawful hearing (PEOPLE VS. DAPITAN, 197 SCRA 378)
58

59

The act of the respondent judge in rendering a decision without even giving the petitioner the right to adduce evidence in his behalf is a gross violation of his right to due process of law. The Decision rendered is NULL AND VOID for want of due process. As long as there is a notice to a party in the hearing of a motion on the custody of their children in a Declaration of Nullity of Marriage case, there is no violation of the right to due process. SUSIE CHAN-TAN vs. JESSE TAN, G.R. No. 167139, February 25, 2010 ISSUE: Petitioner raises the question of whether the 30 March 2004 decision and the 17 May 2004 resolution of the trial court giving custody to their children have attained finality despite the alleged denial of due process since she was not present during the hearing. HELD:

Petitioner contends she was denied due process when her counsel failed to file pleadings and appear at the hearings for respondents omnibus motion to amend the partial judgment as regards the custody of the children and the properties in her possession. Petitioner claims the trial court issued the 17 May 2004 resolution relying solely on the testimony of respondent. Respondent stresses neither petitioner nor her counsel appeared in court at the hearings on respondent's omnibus motion or on petitioners motion to dismiss. We also ruled in Tuason that notice sent to the counsel of record is binding upon the client and the neglect or failure of the counsel to inform the client of an adverse judgment resulting in the loss of the latters right to appeal is not a ground for setting aside a judgment valid and regular on its face. Further, petitioner cannot claim that she was denied due process. While she may have lost her right to present evidence due to the supposed negligence of her counsel, she cannot say she was denied her day in court. Records show petitioner, through counsel, actively participated in the proceedings below, filing motion after motion. Contrary to petitioners allegation of negligence of her counsel, we have reason to believe the negligence in pursuing the case was on petitioners end, as may be gleaned from her counsels manifestation dated 3 May 2004: Undersigned Counsel, who appeared for petitioner, in the nullity proceedings, respectfully informs the Honorable Court that she has not heard from petitioner since Holy Week. Attempts to call petitioner have failed. Undersigned counsel regrets therefore that she is unable to respond in an intelligent manner to the Motion (Omnibus Motion) filed by respondent.

59

60

Clearly, despite her counsels efforts to reach her, petitioner showed utter disinterest in the hearings on respondents omnibus motion seeking, among others, custody of the children. The trial judge was left with no other recourse but to proceed with the hearings and rule on the motion based on the evidence presented by respondent. Petitioner cannot now come to this Court crying denial of due process. The right of the accused to due process of law was violated when the judge issued a warrant for her arrest even though she has not received any notice for her arraignment before the Municipal Trial Court of Baguio City because the notice was actually sent to her through the Chief of Police of Quezon City. NORYN S. TAN VS. JUDGE MARIA CLARITA CASUGATABIN, A.M. No. MTJ-09-1729, January 20, 2009 AUSTRIA-MARTINEZ, J.: FACTS: Noryn S. Tan (complainant) filed a Complaint dated April 2, 2007 against Judge Maria Clarita Casuga-Tabin (respondent) of the Municipal Trial Court in Cities (MTCC), Branch 4, Baguio City for denial of due process relative to Criminal Case No. 118628. Complainant avers: On November 9, 2006, the Philippine National Police (PNP) Quezon City Police District (QCPD) served her a warrant of arrest dated October 13, 2006, issued by the MTCC Baguio City, Branch 4, presided by respondent, relative to Criminal Case No. 118628 for alleged violation of Batas Pambansa Blg. 22. It was only then that she learned for the first time that a criminal case was filed against her before the court. She was detained at the Quezon City Hall Complex Police Office and had to post bail of P1,000.00 before the Office of the Executive Judge of the Regional Trial Court (RTC) of Quezon City for her temporary release. Upon verification, she learned that respondent issued on August 8, 2006 an Order directing her to appear before the court on October 10, 2006 for arraignment. It was sent by mail to PNP Quezon City for service to her. However, she did not receive any copy of the Order and up to the present has not seen the same; hence, she was not able to attend her arraignment. She also found out that there was no proof of service of the Order or any notice to her of the arraignment. This notwithstanding, respondent issued a warrant for her arrest. Complainant alleges that she was deeply aggrieved and embarrassed by the issuance of the warrant for her arrest despite the fact that she was never notified of her arraignment. Complainant prayed that the appropriate investigation be conducted as to the undue issuance of a warrant for her arrest. In her Comment dated July 5, 2007, respondent answered: She issued the warrant of arrest because when the case was called for appearance, the complainant, as accused therein, failed to appear. Prior to the issuance of the warrant of arrest, her staff sent by registered mail the court's Order dated August 8, 2006 addressed to complainant through the Chief of Police, PNP, 1104, Quezon City directing complainant to appear on October 10, 2006 at 8:30 a.m. for the arraignment and preliminary conference in Criminal Case No. 118628, as proven by Registry Receipt No. 0310. It is true that the return on the court's Order dated August 8, 2006 had not yet been made by the QC Police on or before October 10, 2006. Nonetheless, she issued the warrant of arrest in good faith and upon the following grounds: (a) under Sec. 3 of Rule 131 of the Rules of Court, the court was entitled to presume that on October 10, 2006, after the lapse of a little over two months,
60

61

official duty had been regularly performed and a letter duly directed and mailed had been received in the regular course of mail; and (b) Sec. 12 of the 1983 Rule on Summary Procedure in Special Cases provides that bail may be required where the accused does not reside in the place where the violation of the law or ordinance was committed. The warrant of arrest she issued was meant to implement this provision, which was not repealed by the 1991 Revised Rule on Summary Procedure, since complainant is a resident of Quezon City and not of Baguio City. If her interpretation was erroneous, she (respondent) believes that an administrative sanction for such error would be harsh and unsympathetic. She has nothing personal against complainant and did not want to embarrass or humiliate her. She issued the warrant in the honest belief that her act was in compliance with the rules. She prays that the case against her be dismissed and that a ruling on the interpretation of Secs. 10 & 12, of the 1983 Rule on Summary Procedure in Special Cases, in relation to Sec. 16 of the 1991 Revised Rule on Summary Procedure be made for the guidance of the bench and bar. HELD: Whenever a criminal case falls under the Summary Procedure, the general rule is that the court shall not order the arrest of the accused, unless the accused fails to appear whenever required. This is clearly provided in Section 16 of the 1991 Revised Rule on Summary Procedure which states: Sec. 16. Arrest of accused. - The court shall not order the arrest of the accused except for failure to appear whenever required. Release of the person arrested shall either be in bail or on recognizance by a responsible citizen acceptable to the court. (Emphasis supplied) In this case, respondent claims that the issuance of a warrant for the arrest of complainant was justified, since complainant failed to appear during the arraignment in spite of an order requiring her to do so. Respondent admits, however, that a copy of the Order dated August 8, 2006, was sent to complainant through the Chief of Police, PNP, 1104, Quezon City. While it is true that the Rules of Court provides for presumptions, one of which is that official duty has been regularly performed, such presumption should not be the sole basis of a magistrate in concluding that a person called to court has failed to appear as required, which in turn justifies the issuance of a warrant for her arrest, when such notice was not actually addressed to her residence but to the police in her city. So basic and fundamental is a person's right to liberty that it should not be taken lightly or brushed aside with the presumption that the police through which the notice had been sent, actually served the same on complainant whose address was not even specified. The Court has held that a judge commits grave abuse of authority when she hastily issues a warrant of arrest against the accused in violation of the summary procedure rule that the accused should first be notified of the charges against him and given the opportunity to file his counter-affidavits and countervailing evidence . Hence, complainants right to due process was violated. Considering that this is respondent's first administrative infraction in her more than 8 years of service in the judiciary, which serves to mitigate her liability, the Court holds the imposition of a fine in the amount of P10,000.00 to be proper in this case. 8. Procedural due process before administrative bodies e. TIBAY VS. CIR, 69 Phil. 635
61

62

Requisites: a. b. c. d. e. the right to a hearing which includes the right to present evidence; the tribunal must consider the evidence presented; the decision must have something to support itself; the evidence must be substantial; the decision must be based on the evidence presented during the hearing; f. the tribunal or body must act on its own independent consideration of the law or facts; g. the board or body shall in all controversial questions, render its decision in such a manner that the parties to the proceedings can know the various issues involved. f. AMERICAN TOBACCO VS. DIRECTOR, 67 SCRA 287 g. MANILA ELECTRIC COMPANY VS. NLRC, 263 SCRA 531 h. DELGADO VS. CA, November 10, 1986 If an accused was represented by a non-lawyer during the trial (though she thought that he is a lawyer), her right to due process was violated and therefore entitled to a new trial. e. PEDRO CONSULTA VS. PEOPLE, G.R. No. 179462, February 12, 2009 ISSUE: Whether or not appellant was denied due process having been represented by a fake lawyer during arraignment, pre-trial and presentation of principal witnesses for the prosecution. HELD: On the matter of accused-appellants claim of having been denied due process, an examination of the records shows that while accusedappellant was represented by Atty. Jocelyn P. Reyes, who seems not a lawyer, during the early stages of trial, the latter withdrew her appearance with the conformity of the former as early as July 28, 2000 and subsequently, approved by the RTC in its Order dated August 4, 2000. Thereafter, accused-appellant was represented by Atty. Rainald C. Paggao from the Public Defenders (Attorneys) Office of Makati City. Since the accused-appellant was already represented by a member of the Philippine Bar who principally handled his defense, albeit unsuccessfully, then he cannot now be heard to complain about having been denied of due process. That appellants first counsel may not have been a member of the bar does not dent the proven fact that appellant prevented Nelia and company from proceeding to their destination. Further, appellant was afforded competent representation by the Public Attorneys Office during the presentation by the prosecution of the medico-legal officer and during the presentation of his evidence. People v. Elesterio1[4] enlightens:

1[4]

G.R. No. 63971, May 9, 1989, 173 SCRA 243, 249.

62

63

As for the circumstance that the defense counsel turned out later to be a non-lawyer, it is observed that he was chosen by the accused himself and that his representation does not change the fact that Elesterio was undeniably carrying an unlicensed firearm when he was arrested. At any rate, he has since been represented by a member of the Philippine bar, who prepared the petition for habeas corpus and the appellants brief. 9. Procedural due process in disciplinary actions against students Academic freedom; due process in disciplinary actions involving students DE LA SALLE UNIVERSITY VS. COURT OF APPEALS, HON.WILFREDO D. REYES, in his capacity as Presiding Judge of Branch 36, Regional Trial Court of Manila, THE COMMISSION ON HIGHER EDUCATION, THE DEPARTMENT OF EDUCATION CULTURE AND SPORTS, ALVIN AGUILAR, JAMES PAUL BUNGUBUNG, RICHARD REVERENTE and ROBERTO VALDES, JR., G.R. No. 127980, December 19, 2007

REYES, R.T., J.: THE FACTS: PRIVATE respondents Alvin Aguilar, James Paul Bungubung, Richard Reverente and Roberto Valdes, Jr. are members of Tau Gamma Phi Fraternity who were expelled by the De La Salle University (DLSU) and College of Saint Benilde (CSB) Joint Discipline Board because of their involvement in an offensive action causing injuries to petitioner James Yap and three other student members of Domino Lux Fraternity. The mauling incidents were a result of a fraternity war. The victims, namely: petitioner James Yap and Dennis Pascual, Ericson Cano, and Michael Perez, are members of the Domino Lux Fraternity, while the alleged assailants, private respondents Alvin Aguilar, James Paul Bungubung, Richard Reverente and Roberto Valdes, Jr. are members of Tau Gamma Phi Fraternity, a rival fraternity. The next day, March 30, 1995, petitioner Yap lodged a complaint with the Discipline Board of DLSU charging private respondents with direct assault. Similar complaints were also filed by Dennis Pascual and Ericson Cano against Alvin Lee and private respondents Valdes and Reverente. Thus, cases entitled De La Salle University and College of St. Benilde v. Alvin Aguilar (AB-BSM/9152105), James Paul Bungubung (ABPSM/9234403), Robert R. Valdes, Jr. (BS-BS-APM/9235086), Alvin Lee (EDD/9462325), Richard Reverente (AB-MGT/9153837) and Malvin A. Papio (AB-MGT/9251227) were docketed as Discipline Case No. 94953-25121. The Director of the DLSU Discipline Office sent separate notices to private respondents Aguilar, Bungubung and Valdes, Jr. and Reverente informing them of the complaints and requiring them to answer. Private respondents filed their respective answers. Said notices issued by De La Salle Discipline Board uniformly stated as follows:
63

64

Please be informed that a joint and expanded Discipline Board had been constituted to hear and deliberate the charge against you for violation of CHED Order No. 4 arising from the written complaints of James Yap, Dennis C. Pascual, and Ericson Y. Cano. You are directed to appear at the hearing of the Board scheduled on April 19, 1995 at 9:00 a.m. at the Bro. Connon Hall for you and your witnesses to give testimony and present evidence in your behalf. You may be assisted by a lawyer when you give your testimony or those of your witnesses. During the proceedings before the Board on April 19 and 28, 1995, private respondents interposed the common defense of alibi. No fullblown hearing was conducted nor the students allowed to cross-examine the witnesses against them. On May 3, 1995, the DLSU-CSB Joint Discipline Board issued a Resolution finding private respondents guilty. They were meted the supreme penalty of automatic expulsion pursuant to CHED Order No. 4. The dispositive part of the resolution reads: WHEREFORE, considering all the foregoing, the Board finds respondents ALVIN AGUILAR (AB-BSM/9152105), JAMES PAUL BUNGUBUNG (AB-PSM/9234403), ALVIN LEE (EDD/94623250) and RICHARD V. REVERENTE (AB-MGT/9153837) guilty of having violated CHED Order No. 4 and thereby orders their automatic expulsion. In the case of respondent MALVIN A. MGT/9251227), the Board acquits him of the charge. I SSUE Were private respondents accorded due process of law because there was no full-blown hearing nor were they allowed to cross-examine the witnesses against them? H E L D: Private respondents right to due process of law was not violated. In administrative cases, such as investigations of students found violating school discipline, [t]here are withal minimum standards which must be met before to satisfy the demands of procedural due process and these are: that (1) the students must be informed in writing of the nature and cause of any accusation against them; (2) they shall have the right to answer the charges against them and with the assistance if counsel, if desired; (3) they shall be informed of the evidence against them; (4) they shall have the right to adduce evidence in their own behalf; and (5) the evidence must be duly considered by the investigating committee or official designated by the school authorities to hear and decide the case. Where a party was afforded an opportunity to participate in the proceedings but failed to do so, he cannot complain of deprivation of due process. Notice and hearing is the bulwark of administrative due process,
64

PAPIO

(AB-

65

the right to which is among the primary rights that must be respected even in administrative proceedings. The essence of due process is simply an opportunity to be heard, or as applied to administrative proceedings, an opportunity to explain ones side or an opportunity to seek reconsideration of the action or ruling complained of. So long as the party is given the opportunity to advocate her cause or defend her interest in due course, it cannot be said that there was denial of due process. A formal trial-type hearing is not, at all times and in all instances, essential to due process it is enough that the parties are given a fair and reasonable opportunity to explain their respective sides of the controversy and to present supporting evidence on which a fair decision can be based. To be heard does not only mean presentation of testimonial evidence in court one may also be heard through pleadings and where the opportunity to be heard through pleadings is accorded, there is no denial of due process. Private respondents were duly informed in writing of the charges against them by the DLSU-CSB Joint Discipline Board through petitioner Sales. They were given the opportunity to answer the charges against them as they, in fact, submitted their respective answers. They were also informed of the evidence presented against them as they attended all the hearings before the Board. Moreover, private respondents were given the right to adduce evidence on their behalf and they did. Lastly, the Discipline Board considered all the pieces of evidence submitted to it by all the parties before rendering its resolution in Discipline Case No. 9495-3-25121. Private respondents cannot claim that they were denied due process when they were not allowed to cross-examine the witnesses against them. This argument was already rejected in Guzman v. National University] where this Court held that x x x the imposition of disciplinary sanctions requires observance of procedural due process. And it bears stressing that due process in disciplinary cases involving students does not entail proceedings and hearings similar to those prescribed for actions and proceedings in courts of justice. The proceedings in student discipline cases may be summary; and cross examination is not, x x x an essential part thereof.

GUZMAN VS. NATIONAL UNIVERSITY G.R. No. L-68288, July 11, 1986 FACTS: Petitioners who are students of the National University were barred from enrolment. The school claims that their scholastic standing is poor and that they have been involved in activities that have disrupted classes and had conducted mass actions without the required permits. HELD: a. It is apparent that despite the accusations of alleged violations hurled by the school against the petitioners, the fact is that it had never conducted proceedings of any sort to determine whether or not petitioners-students had indeed led or participated "in activities within the university premises, conducted without prior permit from school authorities, that disturbed or disrupted classes therein".
65

66

Also apparent is the omission of respondents to cite any duly published rule of theirs by which students may be expelled or refused re-enrollment for poor scholastic standing. b. Under the Education Act of 1982, students have the right "to freely choose their field of study subject to existing curricula and to continue their course therein up to graduation, EXCEPT in case of academic deficiency, or violation of disciplinary regulations." The petitioner were denied of this right, and were being disciplined without due process, in violation of the admonition in the Manual of Regulations for Private Schools that "no penalty shall be imposed upon any student except for cause as defined in *** (the) Manuel and/or in the school rules and regulations as duly promulgated and only after due investigation shall have been conducted. It has already been held in Berina vs. Philippine Maritime Institute, 117 SCRA 581, that it is illegal of a school to impose sanctions on students without conducting due investigation. c. Of course, all schools have the power to adopt and enforce its rules. In fact the maintenance of good school discipline is a duty specifically enjoined on every private school. The Manual of Regulations for Private Schools provides that: "* * The school rules governing discipline and the corresponding sanctions therefor must be clearly specified and defined in writing and made known to the students and/or their parents or guardians. Schools shall have the authority and prerogative to promulgate such rules and regulations as they may deem necessary from time to time effective as of the date of their promulgation unless otherwise specified." d. The imposition of disciplinary sanctions requires observance of procedural due process. Due process in disciplinary cases involving students : a. need not entail proceedings and hearing similar to those prescribed for actions and proceedings in court of justice; b. the proceedings may be summary; c. cross-examination is not an essential part thereof. But the S.C. said that the following minimum standards must be met to satisfy the demands of procedural due process: 1. the students must be informed in writing of the nature and cause of any accusation against them; 2. they shall have the right to answer the charges against them, with the assistance of counsel; 3. they shall be informed of the evidence against them; 4. they shall have the right to adduce evidence in their own behalf; 5. the evidence must be duly considered by the investigating committee or official designated by the school authorities to hear and decide the case.
66

67

c. BERINA VS. PMI, September 30, 1982 Due process in the dismissal of employees Requisites of Due Process before the NLRC 1. Notice; and 2. Hearing a. b. c. d. e. f. g. MGG Marine Services vs. NLRC, 259 SCRA 664 Philippine Savings Bank vs. NLRC, 261 SCRA 409 RAYCOR AIR CONTROL VS. NLRC, 261 SCRA 589 WALLEM MARITIME SERVICES VS. NLRC, 263 SCRA 174 SAMILLANO VS. NLRC, 265 SCRA 788 STOLT-NIELSEN VS. NLRC, 264 SCRA 307 GARCIA VS. NLRC, 264 SCRA 261

10. Effect of a Motion for Reconsideration to violation of the right to due process a. CASUELA VS. OFFICE OF THE OMBUDSMAN, 276 SCRA 635 b. CORDENILLO VS. EXECUTIVE SECRETARY, 276 SCRA 652 11. In administrative proceedings, does due process require that [1] a party be assisted by counsel and [2] be able to cross-examine the witnesses? LUMIQUED VS. EXENEA, 282 SCRA 125 There is no law, whether the Civil Service Act or the Administrative Code of 1987, which provides that a respondent in an administrative case should be assisted by counsel in order that the proceedings therein is considered valid. Not only, that, petitioner herein was given the opportunity several times to engage the services of a lawyer to assist him but he confidently informed the investigators that he could protect himself. Administrative Due Process before Civil Service Commission does require cross-examination of complainant and his witnesses by respondent. the not the the

ATTY. ROMEO ERECE VS. LYN MACALINGAY, ET AL., G.R. No. 166809, April 22, 2008 THE FACTS: Petitioner is the Regional Director of the Commission on Human Rights (CHR) Region I, whose office is located in San Fernando City, La Union. Respondent employees of the CHR Region I filed an AffidavitComplaint dated October 2, 1998 against petitioner alleging that he denied them the use of the office vehicle assigned to petitioner, that petitioner still claimed transportation allowance even if he was using the said vehicle, and that he certified that he did not use any government vehicle, when in fact he did, in order to collect transportation allowance.
67

68

Respondent filed his answer denying the allegations against him. After a fact-finding investigation, the CSC Proper in CSC Resolution No. 99-1360 dated July 1, 1999 charged petitioner with Dishonesty and Grave Misconduct for using a government vehicle in spite of his receipt of the monthly transportation allowance and for certifying that he did not use any government vehicle, when in fact, he did, in order to receive the transportation allowance. Pertinent portions of the formal charge read: 1. That despite the regular receipt of Erece of his monthly Representation and Transportation Allowance (RATA) in the amount of P4,000.00, he still prioritizes himself in the use of the office vehicle (Tamaraw FX) in spite of the directive from the Central Office that he cannot use the service vehicle for official purposes and at the same time receive his transportation allowance; 2. That Erece did not comply with the directive of the Central Office addressed to all Regional Human Rights Directors, as follows: to regularize your receipt of the transportation allowance component of the RATA to which you are entitled monthly, you are hereby directed to immediately transfer to any of your staff, preferably one of your lawyers, the memorandum receipt of the vehicle(s) now still in your name; 3. That he certified in his monthly liquidation of his RATA that he did not use any government vehicle for the corresponding month, which is not true because he is the regular user of the government vehicle issued to CHR-Region I. The foregoing facts and circumstances indicate that government service has been prejudiced by the acts of Erece. WHEREFORE, Romeo L. Erece is hereby formally charged with Dishonesty and Grave Misconduct. Accordingly, he is given five (5) days from receipt hereof to submit his Answer under oath and affidavits of his witnesses, if any, to the Civil Service Commission-Cordillera Administrative Region (CSC-CAR). On his Answer, he should indicate whether he elects a formal investigation or waives his right thereto. Any Motion to Dismiss, request for clarification or Bills of Particulars shall not be entertained by the Commission. Any of these pleadings interposed by the respondent shall be considered as an Answer and shall be evaluated as such. Likewise, he is advised of his right to the assistance of counsel of his choice.2[4] After a formal investigation of the case, the CSC issued Resolution No. 020124, dated January 24. 2002, finding petitioner guilty of dishonesty and conduct prejudicial to the best interest of the service and penalizing him with dismissal from the service. Petitioner filed a petition for review of the CSC Resolution with the CA.

2[4]

Id. at 35-36.

68

69

In the Decision promulgated on January 7, 2005, the CA upheld the CSC Resolution, the dispositive portion of which reads: WHEREFORE, in view of the foregoing, the petition is DENIED and the assailed Resolutions of the Civil Service Commission are hereby AFFIRMED. Hence, this petition. I S S U E: Petitioner raised the issue of violation of his right to due process because he was denied the right to cross-examine the respondents on their affidavit-complaint. H E L D: Petitioner contends that he was denied due process as he was not afforded the right to cross-examine his accusers and their witnesses. He stated that at his instance, in order to prevent delay in the disposition of the case, he was allowed to present evidence first to support the allegations in his Counter-Affidavit. After he rested his case, respondents did not present their evidence, but moved to submit their position paper and formal offer of evidence, which motion was granted by the CSC over his (petitioners) objection. Respondents then submitted their Position Paper and Formal Offer of Exhibits. Petitioner submits that although he was allowed to present evidence first, it should not be construed as a waiver of his right to crossexamine the complainants. Although the order of presentation of evidence was not in conformity with the procedure, still petitioner should not be deemed to have lost his right to cross-examine his accusers and their witnesses. This may be allowed only if he expressly waived said right. The Court agrees with the CA that petitioner was not denied due process when he failed to cross-examine the complainants and their witnesses since he was given the opportunity to be heard and present his evidence. In administrative proceedings, the essence of due process is simply the opportunity to explain ones side. Velez v. De Vera it was held that : Due process of law in administrative cases is not identical with judicial process for a trial in court is not always essential to due process. While a day in court is a matter of right in judicial proceedings, it is otherwise in administrative proceedings since they rest upon different principles. The due process clause guarantees no particular form of procedure and its requirements are not technical. Thus, in certain proceedings of administrative character, the right to a notice or hearing are not essential to due process of law. The constitutional requirement of due process is met by a fair hearing before a regularly established administrative agency or tribunal. It is not essential that hearings be had before the making of a determination if thereafter, there is available trial and tribunal before which all objections and defenses to the making of such determination may be raised and considered. One adequate hearing is all that due process requires. . . .
69

70

The right to cross-examine is not an indispensable aspect of due process. Nor is an actual hearing always essential. . . . The dismissal of the petitioner from the government is valid. There is violation of the right to due process of law if a party he is declared as having waived the right to file his answer despite improper service of summons. DATUPAX MANGUDADATU VS. HOUSE OF REPRESENTATIVES ELECTORAL TRIBUNAL (HRET), G.R. No. 179813, December 18, 2008 LEONARDO-DE CASTRO, J.: Datu Pax Pakung S. Mangudadatu (petitioner) and Angelo O. Montilla (private respondent) were congressional candidates for the First District of Sultan Kudarat during the May 14, 2007 national elections. Petitioner won by 17,451 votes and was proclaimed on May 22, 2007 by the Provincial Board of Canvassers as the duly elected Representative of the said congressional district. On May 31, 2007, respondent filed with the HRET a Petition of Protest (Ad Cautelam)3[4] contesting the results of the elections and the proclamation of petitioner. On June 14, 2007, the Secretary of the HRET caused the service of summons4[5] upon petitioner through registered mail at Purok Losaria,5[6] Tamnag (Poblacion), Lutayan, Sultan Kudarat, requiring petitioner to file an Answer to the protest within ten (10) days from receipt thereof.

On July 11, 2007, the HRET received the Registry Return Receipt Card,6[7] showing that a certain Aileen R. Baldenas7[8] (Baldenas) received the summons on June 27, 2007. On August 16, 2007, the HRET issued Resolution No. 07-1798[9] which noted the aforementioned Registry Return Receipt Card and that despite the fact that 43 days from June 27, 2007 had passed since Baldenas received the summons, petitioner had not filed an answer in accordance with Rule 279[10] of the 2004 HRET Rules. In the same Resolution, the HRET considered petitioner to have entered a general denial of the allegations of the protest.

In an Order dated August 17, 2007, the HRET set the preliminary conference on September 27, 2007 at 11:00 a.m.

3[4] 4[5]

Id., pp. 41-76. Id., p. 77. 5[6] The assailed Resolutions state Loria but the Summons and Registry Return Receipt Card correctly state Losaria. 6[7] Rollo, p. 78. 7[8] The assailed Resolutions state Baldena; it should be Baldenas based on the Registry Return Receipt Card. 8[9] See Note 2. 9[10] RULE 27. Failure to Answer; Effect. If no answer is filed to the protest, counter-protest, or the petition for quo warranto within the period fixed in these Rules, a general denial shall be deemed to have been entered. 70

71

Meanwhile, petitioner informally learned of respondents protest, prompting petitioner to request his lawyers to verify the same from the records of the HRET. Thereafter, his lawyers entered their appearance on September 4, 2007 and requested that they be furnished with copies of the petition of protest as well as notices, orders and resolutions pertaining to the protest.

On September 10, 2007, petitioner filed a Motion to Reconsider10[11] Resolution No. 07-179 and Motion to Admit Answer with Counter-Protest, alleging that he never received the summons issued by the HRET. In his affidavit11[12] attached to the motion, petitioner denied that Baldenas was a member of his household or his employee. He further claimed that she was not authorized to receive any important documents addressed to him. And assuming that he had authorized her, the summons received by her was never brought to his attention.

On September 19, 2007, the HRET issued Resolution No. 0730012[13] denying for lack of merit.

Hence, this petition.

Petitioner filed the instant petition imputing grave abuse of discretion amounting to lack of jurisdiction on the part of the HRET for issuing Resolution Nos. 07-179 and 07-300. He also prayed for a temporary restraining order and/or a writ of preliminary injunction for this Court to enjoin the HRET from further proceeding with HRET Case No.07-021. Petitioner contended that the HRET never acquired jurisdiction over his person because of the absence of a valid service of summons. He argued that a substitute service of summons is made only when the defendant cannot be served personally at a reasonable time after efforts to locate him have failed.13[14] In his case, since the process servers return failed to show on its face the impossibility of personal service, then the substituted service was improper and invalid.

In his comment, respondent countered that the HRET did not commit grave abuse of discretion in issuing Resolution Nos. 07-179 dated August 16, 2007 and 07-300 dated September 19, 2007. He argued that Rule 22 of the 2004 HRET Rules merely states that the Secretary of the Tribunal shall issue the corresponding summons to the protestee or respondent, as the case may be. He posited then that the intent of the HRET in not expressly specifying personal service of summons on the protestee or respondent was to give it a reasonable discretion or leeway in serving the summons by other means such as registered mail. Thus, service of summons on petitioner through registered mail did not violate Rule 22 of the 2004 HRET Rules. Further, respondent claimed that Rule 14, Sections 6 and 7 of the Rules of Court were inconsistent with Rule 22
10[11] 11[12] 12[13] 13[14]

Rollo, pp. 26-37. Id., p. 30. See Note 3. Rollo, p.12. 71

72

of the 2004 HRET Rules and therefore should not be given suppletory application to HRET proceedings.

HELD:

Rule 22 of the 2004 HRET Rules provides:

RULE 22. Summons. If the petition is not summarily dismissed in accordance with Rule 21 of these Rules, the Secretary of the Tribunal shall issue the corresponding summons to the protestee or respondent, as the case may be, together with a copy of the petition, requiring him within ten (10) days from receipt thereof to file his answer.

The 2004 HRET Rules on summons is silent on how the summons should be served on the protestee. Significantly, Rule 8014[15] of the 2004 HRET Rules provides that the 1997 Rules of Civil Procedure applies by analogy or suppletorily in so far as the latter may be applicable and not inconsistent therewith as well as with the orders, resolutions and decisions of the HRET. In view of the failure of the HRET Rules to specify the authorized modes of service of summons, resort then is necessary to Sections 6 and 7, Rule 14, 1997 Rules of Civil Procedure, which state: SEC. 6. Service in person on defendant. Whenever practicable, the summons shall be served handling a copy thereof to the defendant in person, or, if he refuses to receive and sign for it, by tendering it to him. SEC. 7. Substituted service. If, for justifiable causes, the defendant cannot be served within a reasonable time as provided in the preceding section, service may be effected (a) by leaving copies of the summons at the defendants residence with some person of suitable age and discretion then residing therein, or (b) by leaving copies at defendants office or regular place of business with some competent person in charge thereof.

In the case at bar, the service of the summons was made through registered mail, which is not among the allowed modes of service under Rule 14 of the Rules of Court.

Indeed, if in ordinary civil cases (which involve only private and proprietary interests) personal service of summons is preferred and service by registered mail is not allowed on jurisdictional and due process grounds, with more reason should election cases (which involve public
14[15]

RULE 80. Applicability. The following shall be applicable by analogy or in suppletory character and effect in so far as they may be applicable and are not inconsistent with these Rules and with the orders, resolutions and decisions of the Tribunal, namely: (1) The Rules of Court; (2) Decisions of the Supreme Court; (3) Decisions of the Electoral Tribunals. 72

73

interest and the will of the electorate) strictly follow the hierarchy of modes of service of summons under the Rules of Court.

The Court sees no reason why the HRET cannot make use of its own process servers to personally serve the summons, or alternatively, delegate the matter to the process server of a court with territorial jurisdiction over the place of residence of the respondent/protestee in the election case, at the expense of the petitioner/protestant. Considering that the proper service of summons on the respondent/protestee is a jurisdictional requirement and goes to heart of due process, we cannot allow service of summons by a method not sanctioned by the HRET Rules in relation to the Rules of Court. In view of the foregoing, we find that the HRET committed grave abuse of discretion in considering petitioner to have entered a general denial of the allegations in respondents petition of protest and in denying his motion to reconsider as well as his motion to admit answer with counter-protest.

The right to due process on the part of the respondent was violated when the Civil Service Commission reconsidered its earlier decision in favor of the former based on a Motion for Reconsideration wherein said respondent was not furnished a copy thereof nor given the chance to comment on it.

DEPARTMENT OF EDUCATION VS. GODOFREDO CUANAN, G.R. No. 169013, December 16, 2008 The factual background of the case is as follows: On March 11, 1996, Luzviminda Borja and Juliana Castro, on behalf of their respective minor daughters, Lily Borja and Charo Castro, filed before the Department of Education, Culture and Sports - Regional Office No. III (DECS-RO No. III), Cabanatuan City, two separate administrative complaints for Sexual Harassment and Conduct Unbecoming a Public Officer against Cuanan, then Principal of Lawang Kupang Elementary School in San Antonio, Nueva Ecija. Acting on the complaints, DECS-RO No. III Regional Director Vilma L. Labrador constituted an Investigating Committee, composed of three DepEd officials from the province, to conduct a formal investigation. Following the investigation, the Investigating Committee submitted its Investigation Report dated December 14, 1999, finding Cuanan guilty of sexual harassment and recommending his forced resignation without prejudice to benefits. In a Decision dated January 28, 2000, Regional Director Labrador concurred in the findings of the Investigating Committee and meted out the penalty of forced resignation to Cuanan without prejudice to benefits. In an Order dated April 13, 2000, then DepEd Secretary Andrew Gonzales affirmed the Decision of Regional Director Labrador. On May 30, 2000, Cuanan filed a Petition for Reconsideration thereof, but the same was denied for lack of merit by Secretary Gonzales in a Resolution dated June 19, 2000.
73

74

Cuanan elevated his case to the CSC. On January 20, 2003, the CSC issued Resolution No. 030069 , which set aside the June 19, 2000 Resolution of Secretary Gonzales and exonerated Cuanan from the charge of sexual harassment. On January 23, 2003, copies of the resolution were duly sent to the parties, including the DepEd , Cuanan received a copy of Resolution No. 030069 on January 31, 2003 . In a Letter dated February 3, 2003, Cuanan requested his reinstatement as Elementary School Principal I . In a 1st Indorsement, the District Supervisor recommended appropriate action. In a 2nd Indorsement dated February 4, 2003, Schools Division Superintendent Dioscorides D. Lusung (Superintendent) recommended that Cuanan be reinstated to duty as School Principal of San Antonio District upon finality of the decision of the CSC . In a Letter dated February 10, 2003, Regional Director Ricardo T. Sibug informed the Superintendent that Cuanan could not be immediately reinstated to the service until an order of implementation was received from the Department Secretary. Sometime in March 2003, DepEd Undersecretary Jose Luis Martin C. Gascon sent a letter to the CSC requesting a copy of CSC Resolution No. 030069 dated January 20, 2003. In a Letter dated March 25, 2003, the CSC informed the DepEd that a copy of the requested resolution was duly sent to it on January 23, 2003. Nonetheless, the CSC sent another copy of the resolution to the DepEd for its reference. The DepEd received said reference copy on March 28, 2003. On April 11, 2003, then DepEd Secretary Edilberto C. de Jesus filed a Petition for Review/Reconsideration with the CSC. No copy of the pleading was served upon Cuanan. On July 29, 2003, Secretary De Jesus filed a Supplemental Petition for Review/Reconsideration reiterating the prayer for reversal of the resolution. Again, no copy of the pleading was served upon Cuanan. Subsequently, pursuant to Division Special Order No. 001 series of 2003 dated June 18, 2003, Cuanan was reinstated to his former position as school principal effective April 30, 2003 In Division Special Order No. 285, series of 2003 dated July 8, 2003, Cuanan was directed to return to duty . Based thereon, Cuanan requested payment of salaries and his inclusion in the payroll, which the Division School Superintendent of Nueva Ecija duly endorsed on November 7, 2003 . However, on October 22, 2004, the CSC issued Resolution No. 041147 setting aside CSC Resolution No. 030069 dated January 20, 2003. It found Cuanan guilty of Sexual Harassment, Grave Misconduct and Conduct Grossly Prejudicial to the Best Interest of the Service and meted out the penalty of dismissal from the service with forfeiture of retirement benefits, cancellation of his service eligibility, and perpetual disqualification from holding public office. Cuanan received a copy of the Resolution on November 9, 2004 . Thirteen days later, or on November 22, 2004, Cuanan filed a petition for certiorari with the CA seeking to annul Resolution No. 041147, alleging that the CSC should not have entertained the petition for review/reconsideration since the DepEd was not the complainant or the party adversely affected by the resolution; that the petition for review/reconsideration was filed out of time; and that Cuanan was not furnished copies of the pleadings filed by the DepEd in violation of procedural due process.
74

75

The DepEd sought the dismissal of the petition on the ground of improper remedy, the mode of review from a decision of the CSC being a petition for review under Rule 43 of the Rules of Court. On May 16, 2005, the CA rendered a Decision granting the petition for certiorari and setting aside CSC Resolution No. 041147 dated October 12, 2004. The CA held that while a motion for reconsideration and a petition for review under Rule 43 were available remedies, Cuanan's recourse to a petition for certiorari was warranted, since the act complained of was patently illegal; that the CSC gravely abused its discretion in granting the petition for review/reconsideration filed by the DepEd without regard for Cuanan's fundamental right to due process, since he was not duly notified of the petition for review/reconsideration, nor was he required by the CSC to file a comment thereon, much less, given a copy of the said petition; that the DepEd failed to establish that the resolution was not yet final and executory when it filed its petition for review/reconsideration. DepEd filed a Motion for Reconsideration but the CA denied the same in its Resolution dated July 18, 2005. Hence, the present petition on the following grounds: I WITH DUE RESPECT, THE COURT OF APPEALS GRAVELY ERRED ON A QUESTION OF LAW IN TAKING COGNIZANCE OF THE PETITION IN CA-G.R. SP NO. 87499, THE SAME NOT BEING THE PROPER REMEDY IN ASSAILING CSC RESOLUTION NO. 041147 DATED OCTOBER 22, 2004. II WITH DUE RESPECT, THE COURT OF APPEALS GRAVELY ERRED ON A QUESTION OF LAW IN ADJUDGING CSC AS HAVING COMMITTED GRAVE ABUSE OF DISCRETION IN ISSUING RESOLUTION NO. 041147 DATED OCTOBER 22, 2004 . DepEd contends that the CA should have dismissed outright the petition for certiorari because CSC decisions are appealable to the CA by petition for review under Rule 43; that the filing of a motion for reconsideration was a precondition to the filing of a petition for certiorari under Rule 65; that the DepEd, even if not the complainant, may question the resolution of the CSC; that Cuanan failed to prove that the CSC's petition for review/reconsideration was not seasonably filed; that even if Cuanan was not served a copy of the pleadings filed by the DepEd, the CSC was not bound by procedural rules. Cuanan, on the other hand, contends that the DepEd cannot file a motion for reconsideration from the CSC Resolution exonerating him, since it is not the complainant in the administrative case and therefore not a party adversely affected by the decision therein; that even if DepEd may seek reconsideration of the CSC Resolution, the petition for review/reconsideration was filed out of time; and that Cuanans right to due process was violated when he was not given a copy of the pleadings filed by the DepEd or given the opportunity to comment thereon. The Court finds it necessary, before delving on the grounds relied upon by the DepEd in support of the petition, to first resolve the question of whether the DepEd can seek reconsideration of the CSC Resolution exonerating Cuanan.
75

76

In a long line of cases, beginning with Civil Service Commission v. Dacoycoy , and reiterated in Philippine National Bank v. Garcia, Jr ., the Court has maintained that the disciplining authority qualifies as a party adversely affected by the judgment, who can file an appeal of a judgment of exoneration in an administrative case. CSC Resolution No. 021600 allows the disciplining authority to appeal from a decision exonerating an erring employee, thus: Section 2. Coverage and Definition of Terms. x x x (l) PARTY ADVERSELY AFFECTED refers to the respondent against whom a decision in a disciplinary case has been rendered or to the disciplining authority in an appeal from a decision exonerating the said employee. (Emphasis supplied) Hence, Cuanan's exoneration under CSC Resolution No. 030069 may be subject to a motion for reconsideration by the DepEd which, as the appointing and disciplining authority, is a real party in interest. Now, as to the merits of DepEd's arguments, the Court finds none. The remedy of an aggrieved party from a resolution issued by the CSC is to file a petition for review thereof under Rule 43 of the Rules of Court within fifteen days from notice of the resolution. Recourse to a petition for certiorari under Rule 65 renders the petition dismissible for being the wrong remedy. Nonetheless, there are exceptions to this rule, to wit: (a) when public welfare and the advancement of public policy dictates; (b) when the broader interest of justice so requires; (c) when the writs issued are null and void; or (d) when the questioned order amounts to an oppressive exercise of judicial authority . As will be shown forthwith, exception (c) applies to the present case. Furthermore, while a motion for reconsideration is a condition precedent to the filing of a petition for certiorari, immediate recourse to the extraordinary remedy of certiorari is warranted where the order is a patent nullity, as where the court a quo has no jurisdiction; where petitioner was deprived of due process and there is extreme urgency for relief; where the proceedings in the lower court are a nullity for lack of due process; where the proceeding was ex parte or one in which the petitioner had no opportunity to object . These exceptions find application to Cuanan's petition for certiorari in the CA. At any rate, Cuanan's petition for certiorari before the CA could be treated as a petition for review, the petition having been filed on November 22, 2004, or thirteen (13) days from receipt on November 9, 2004 of CSC Resolution No. 041147, clearly within the 15-day reglementary period for the filing of a petition for review . Such move would be in accordance with the liberal spirit pervading the Rules of Court and in the interest of substantial justice . Furthermore, CSC Resolution No. 030069 has long become final and executory. It must be noted that the records show that copies of CSC Resolution No. 030069 were duly sent to the parties, including DepEd, on January 23, 2003 . Cuanan received a copy thereof on January 31, 2003 while the DepEd requested a copy sometime in March 2003, or about two months later. Under the Rules of Evidence, it is presumed that official duty has been regularly performed, unless contradicted . This presumption includes that of regularity of service of judgments, final orders or resolutions. Consequently, the burden of proving the irregularity in official conduct -that is, non-receipt of the duly sent copy of CSC Resolution No. 030069 -- is on the part of the DepEd, which in the present case clearly failed to discharge the same .
76

77

Thus, the presumption stands that CSC Resolution No. 030069 dated January 20, 2003 had already become final and executory when the DepEd filed its Petition for Review/Reconsideration on April 11, 2003, more than two months later. It is elementary that once judgment has become final and executory, it becomes immutable and can no longer be amended or modified. In GallardoCorro v. Gallardo , this Court held: Nothing is more settled in law than that once a judgment attains finality it thereby becomes immutable and unalterable. It may no longer be modified in any respect, even if the modification is meant to correct what is perceived to be an erroneous conclusion of fact or law, and regardless of whether the modification is attempted to be made by the court rendering it or by the highest court of the land. Just as the losing party has the right to file an appeal within the prescribed period, the winning party also has the correlative right to enjoy the finality of the resolution of his case. The doctrine of finality of judgment is grounded on fundamental considerations of public policy and sound practice, and that, at the risk of occasional errors, the judgments or orders of courts must become final at some definite time fixed by law; otherwise, there would be no end to litigations, thus setting to naught the main role of courts of justice which is to assist in the enforcement of the rule of law and the maintenance of peace and order by settling justiciable controversies with finality . Moreover, while it is true that administrative tribunals exercising quasijudicial functions are free from the rigidity of certain procedural requirements, they are bound by law and practice to observe the fundamental and essential requirements of due process in justiciable cases presented before them . The relative freedom of the CSC from the rigidities of procedure cannot be invoked to evade what was clearly emphasized in the landmark case of Ang Tibay v. Court of Industrial Relations that all administrative bodies cannot ignore or disregard the fundamental and essential requirements of due process. Furthermore, Section 43.A of the Uniform Rules in Administrative Cases in the Civil Service provides: Section 43.A. Filing of Supplemental Pleadings. - All pleadings filed by the parties with the Commission, shall be copy furnished the other party with proof of service filed with the Commission. Any supplemental pleading to supply deficiencies in aid of an original pleading but which should not entirely substitute the latter can be filed only upon a favorable action by the Commission on the motion of a party to the case. The said motion should be submitted within five (5) days from receipt of a copy of the original pleading and it is discretionary upon the Commission to allow the same or not or even to consider the averments therein.(Emphasis supplied) Cuanan undoubtedly was denied procedural due process. He had no opportunity to participate in the proceedings for the petition for review/ reconsideration filed by the DepEd, since no copy of the pleadings filed by the DepEd were served upon him or his counsel; nor was he even required by the CSC to file his comments thereon. Considering that pleadings filed by the DepEd were not served upon Cuanan, they may be treated as mere scraps of paper which should not have merited the attention or consideration of the CSC. WHEREFORE, the petition is DENIED. The assailed Decision and Resolution of the Court of Appeals in CA-G.R. SP No. 87499 are AFFIRMED.
77

78

There is no violation of the petitioners right to due process when after the election protest against him was already submitted for decision and the ballots transferred to the Senate Electoral Tribunal, the COMELEC went to deliberate on the case at the Senate Electoral Tribunal using the ballots therein in the process without notice to the petitioner. JOSELITO MENDOZA VS. COMELEC and ROBERTO PAGDANGANAN, G.R. No. 188308, October 15, 2009 BRION, J.: The petitioner and the respondent vied for the position of Governor of the Province of Bulacan in the May 14, 2007 elections. The petitioner was proclaimed winning candidate and assumed the office of Governor. The respondent seasonably filed an election protest with the COMELEC, which was raffled to the Second Division and docketed as EPC No. 2007-44. Revision of ballots involving the protested and counter-protested precincts in Angat, Bocaue, Calumpit, Doa Remedios Trinidad, Guiginto, Malolos, Meycauayan, Norzagaray, Pandi, Paombong, Plaridel, Pulilan, San Rafael and San Jose del Monte soon followed. The revision was conducted at the COMELECs office in Intramuros. After revision, the parties presented their other evidence, leading to the parties formal offer of their respective evidence. The COMELEC approved the parties formal offer of evidence and then required the parties to submit their respective memoranda. The parties complied with the COMELECs order. The case was thereafter submitted for resolution. On March 2, 2009 the COMELEC transferred the Bulacan ballot boxes, including those involved in the provincial election contest, to the Senate Electoral Tribunal (SET) in connection with the protest filed by Aquilino Pimentel III against Juan Miguel Zubiri. In light of this development, the petitioner moved to suspend further proceedings. . The COMELECs Second Division denied the petitioners motion in its Order of April 29, 2009, ruling that the COMELEC has plenary powers to find alternative methods to facilitate the resolution of the election protest; thus, it concluded that it would continue the proceedings after proper coordination with the SET. The petitioner moved to reconsider this Order, but the COMELECs Second Division denied the motion in its Order of May 26, 2009. These interrelated Resolutions led to the COMELECs continued action specifically, the appreciation of ballots on the provincial election contest at the SET offices--which the COMELEC did without informing the petitioner. Allegedly alarmed by information on COMELEC action on the provincial election contest within the SET premises without notice to him and without his participation, the petitioners counsel wrote the SET Secretary, Atty. Irene Guevarra, a letter dated June 10, 2009 to confirm the veracity of the reported
78

79

conduct of proceedings.15[2] The SET Secretary responded on June 17, 2009 as follows: x x x please be informed that the conduct of proceedings in COMELEC EPC No. 2007-44 (Pagdanganan vs. Mendoza) within the Tribunal Premises was authorized by then Acting Chairman of the Tribunal, Justice Antonio T. Carpio, upon formal request of the Office of Commissioner Lucenito N. Tagle. Basis of such grant is Section 3, Comelec Resolution No. 2812 dated 17 October 1995, stating that (t)he Tribunals, the Commission and the Courts shall coordinate and make arrangement with each other so as not to delay or interrupt the revision of ballots being conducted. The synchronization of revision of ballots shall be such that the expeditious disposition of the respective protest case shall be the primary concern. While the said provision speaks only of revision, it has been the practice of the Tribunal to allow the conduct of other proceedings in local election protest cases within its premises as may be requested. [emphasis supplied]16[3] ISSUE: A. WHETHER OR NOT THE COMELEC VIOLATED DUE PROCESS BY CONDUCTING PROCEEDINGS WITHOUT GIVING DUE NOTICE TO THE PETITIONER. The petitioner argues that the election protest involves his election as Governor; thus, its subject matter involves him and the people of the Province of Bulacan who elected him. On this basis, he claims entitlement to notice and participation in all matters that involve or are related to the election protest. He further asserts that he had the legitimate expectation that no further proceedings would be held or conducted in the case after its submission for decision. Citing the commentaries of Father Joaquin Bernas,17[4] the petitioner argues that the proceedings before the COMELEC in election protests are judicial in nature and character. Thus, the strictures of judicial due process specifically, (a) opportunity to be heard and (b) that judgment be rendered only after lawful hearing apply. Notices in judicial dispute, he claims, are not really just a matter of courtesy; they are elementary fundamental element of due process, they are part and parcel of a right of a party to be heard. He further cites Justice Isagani A. Cruz,18[5] who wrote: x x x Every litigant is entitled to his day in court. He has a right to be notified of every incident of the proceeding and to be present at every stage thereof so that he may be heard by himself and counsel for the protection of his interest. The petitioner claims that without notice to him of the proceedings, the due process element of the right to have judgment only after lawful hearing is absent. There is no way, he claims, that a judicial proceeding held without notice to the parties could be described as a lawful hearing, especially a proceeding which has as its subject matter the sovereign will of an entire province.
15[2] 16[3]

See Petition, p. 12. Rollo, p. 45. 17[4] J. Bernas, Constitutional Structure and Powers of Government, 2005, pp. 718-719. 18[5] I. Cruz, Constitutional Law, 2003, p. 14.

79

80

He was therefore denied his day in court, he claims, when the COMELEC conducted the examination and appreciation of ballots. The proceedings should be stopped and declared null and void; its future results, too, should be nullified, as nothing derived from the anomalous and unconstitutional clandestine and unilateral proceedings should ever be part of any decision that the COMELEC may subsequently render. The poisonous fruits (derived from the proceedings) should have no part and should not be admitted for any purpose and/or in any judicial proceeding. HELD: The petition is anchored on the alleged conduct of proceedings in the election protest following the completed revision of ballots at the SET premises without notice to and without the participation of the petitioner. Significantly, the conduct of proceedings is confirmed by the SET Secretary in the letter we quoted above.19[8] As the issues raised show the petitioners focus is not really on the COMELEC Orders denying the suspension of proceedings when the ballot boxes and other election materials pertinent to the election contest were transferred to the SET; the focus is on what the COMELEC did after to the issuance of the Resolutions. We read the petition in this context as these COMELEC Orders are now unassailable as the period to challenge them has long passed.20[9] The substantive issue we are primarily called upon to resolve is whether there were proceedings within the SET premises, entitling the petitioner to notice and participation, which were denied to him; in other words, the issue is whether the petitioners right to due process has been violated. A finding of due process violation, because of the inherent arbitrariness it carries, necessarily amounts to grave abuse of discretion. As a preliminary matter, we note that the petitioner has claimed that COMELEC exercises judicial power in its action over provincial election contests and has argued its due process position from this view. We take this opportunity to clarify that judicial power in our country is vested in one Supreme Court and in such lower courts as may be established by law.21[10] This exclusive grant of authority to the Judiciary is reinforced under the second paragraph of Section 1, Article VIII of the Constitution which further states that Judicial power includes the duty of the courts of justice to settle actual controversies involving rights which are legally demandable and enforceable.. ., thus constitutionally locating the situs of the exercise of judicial power in the courts. In contrast with the above definitions, Section 2, Article IX(C) of the Constitution lists the COMELECs powers and functions, among others, as follows: (1) Enforce and administer all laws and regulations relative to the conduct of an election, plebiscite, initiative, referendum, and recall. (2) Exercise exclusive original jurisdiction over all contests relating to the elections, returns and qualifications of all elective
19[8] 20[9]

Supra note 3. See Section 3, Rule 64 of the Rules of Court. The petitioner received the COMELEC Resolution denying his motion for reconsideration on June 1, 2009. Thirty (30) days later or on July 1, 2009, he filed a motion for extension of time to file the petition. The petition cannot but be late because of the remainder rule under Section 3, Rule 64. 21[10] Section 1 (first paragraph), Article VIII, 1987 Constitution.

80

81

regional, provincial, and city officials, and appellate jurisdiction over all contests involving elective municipal officials decided by trial courts of general jurisdiction, or involving elective barangay officials by trial courts of limited jurisdiction. Decisions, final orders, or rulings of the Commission on election contests involving elective municipal and barangay officials shall be final, executory, and not appealable. (3) Decide, except those involving the right to vote, all questions affecting elections, including determination of the number and location of polling places, appointment of election officials and inspectors, and registration of voters. Under these terms, the COMELEC under our governmental structure is a constitutional administrative agency and its powers are essentially executive in nature (i.e., to enforce and administer election laws),22[11] quasi-judicial (to exercise original jurisdiction over election contests of regional, provincial and city officials and appellate jurisdiction over election contests of other lower ranking officials), and quasi-legislative (rulemaking on all questions affecting elections and the promulgation of its rules of procedure). The COMELECs adjudicative function is quasi-judicial since it is a constitutional body, other than a court, vested with authority to decide election contests, and in the course of the exercise of its jurisdiction, to hold hearings and exercise discretion of a judicial nature;23[12] it receives evidence, ascertain the facts from these submissions, determine the law and the legal rights of the parties, and on the basis of all these decides on the merits of the case and renders judgment.24[13] Despite the exercise of discretion that is essentially judicial in character, particularly with respect to election contests, COMELEC is not a tribunal within the judicial branch of government and is not a court exercising judicial power in the constitutional sense;25[14] hence, its adjudicative function, exercised as it is in the course of administration and enforcement, is quasijudicial. The appropriate due process standards that apply to the COMELEC, as an administrative or quasi-judicial tribunal, are those outlined in the seminal case of Ang Tibay v. Court of Industrial Relations,26[16] quoted below: (1) The first of these rights is the right to a hearing, which includes the right of the party interested or affected to present his own case and submit evidence in support thereof. xxx (2) Not only must the party be given an opportunity to present his case and to adduce evidence tending to establish the rights which he asserts but the tribunal must consider the evidence presented. (3) While the duty to deliberate does not impose the obligation to decide right, it does imply a necessity which cannot
22[11] 23[12]

Ututalum v. Commission on Elections, G.R. No. L-25349, December 3, 1965, 15 SCRA 465. See: Presidential Anti-Dollar Salting Task Force v. Court of Appeals, G.R. No. 83578, March 16, 1989, 171 SCRA 348; Midland Insurance Corporation v. IAC , No. L-71905, August 13, 1986, 143 SCRA 458. 24[13] See: Cario v. Commission on Human Rights , G.R. No. 96681, December 2, 1991, 204 SCRA 483, on the activities encompassed by the exercise of quasi-judicial power. 25[14] See: Cipriano v. COMELEC, G.R. No. 158830, August 10, 2004, 436 SCRA 45, citing Sandoval v. COMELEC, 323 SCRA 403 [2000]. 26[16] 69 Phil. 635 (1940).

81

82

be disregarded, namely, that of having something to support its decision. A decision with absolutely nothing to support it is a nullity, a place when directly attached. (4) Not only must there be some evidence to support a finding or conclusion, but the evidence must be "substantial. "Substantial evidence is more than a mere scintilla. It means such relevant evidence as a reasonable mind might accept as adequate to support a conclusion." (5) The decision must be rendered on the evidence presented at the hearing, or at least contained in the record and disclosed to the parties affected. (6) The Court of Industrial Relations or any of its judges, therefore, must act on its or his own independent consideration of the law and facts of the controversy, and not simply accept the views of a subordinate in arriving at a decision. (7) The Court of Industrial Relations should, in all controversial questions, render its decision in such a manner that the parties to the proceeding can know the various issues involved, and the reasons for the decisions rendered. The performance of this duty is inseparable from the authority conferred upon it. These are now commonly referred to as cardinal primary rights in administrative proceedings. The first of the enumerated rights pertain to the substantive rights of a party at hearing stage of the proceedings. The essence of this aspect of due process, we have consistently held, is simply the opportunity to be heard, or as applied to administrative proceedings, an opportunity to explain ones side or an opportunity to seek a reconsideration of the action or ruling complained of.27[17] A formal or trial-type hearing is not at all times and in all instances essential; in the case of COMELEC, Rule 17 of its Rules of Procedure defines the requirements for a hearing and these serve as the standards in the determination of the presence or denial of due process. The second, third, fourth, fifth, and sixth aspects of the Ang Tibay requirements are reinforcements of the right to a hearing and are the inviolable rights applicable at the deliberative stage, as the decision-maker decides on the evidence presented during the hearing. These standards set forth the guiding considerations in deliberating on the case and are the material and substantial components of decision-making. Briefly, the tribunal must consider the totality of the evidence presented which must all be found in the records of the case (i.e., those presented or submitted by the parties); the conclusion, reached by the decision-maker himself and not by a subordinate, must be based on substantial evidence.28[18] Finally, the last requirement, relating to the form and substance of the decision of a quasi-judicial body, further complements the hearing and decisionmaking due process rights and is similar in substance to the constitutional requirement that a decision of a court must state distinctly the facts and the law

27[17] 28[18]

Bautista v. Comelec, G.R. Nos. 154796-97, October 23, 2003, 414 SCRA 299. Supra note 17.

82

83

upon which it is based.29[19] As a component of the rule of fairness that underlies due process, this is the duty to give reason to enable the affected person to understand how the rule of fairness has been administered in his case, to expose the reason to public scrutiny and criticism, and to ensure that the decision will be thought through by the decision-maker. In the present case, the petitioner invokes both the due process component rights at the hearing and deliberative stages and alleges that these component rights have all been violated. We discuss all these allegations below. The Right to Notice and to be Heard. a. At the Hearing and Revision of Ballots. Based on the pleadings filed, we see no factual and legal basis for the petitioner to complain of denial of his hearing stage rights. In the first place, he does not dispute that he fully participated in the proceedings of the election protest until the case was deemed submitted for resolution; he had representation at the revision of the ballots, duly presented his evidence, and summed up his case through a memorandum. These various phases of the proceedings constitute the hearing proper of the election contest and the COMELEC has more than satisfied the opportunity to be heard that the Ang Tibay hearing stage rights require. In these proceedings, the petitioner stood head-to-head with the respondent in an adversarial contest where both sides were given their respective rights to speak, make their presentations, and controvert each others submission, subject only to established COMELEC rules of procedures. Under these undisputed facts, both parties had their day in court, so to speak, and neither one can complain of any denial of notice or of the right to be heard. b. At the Proceedings at the SET. A critical question to be answered in passing upon due process questions at this stage of the election contest is the nature of the so-called proceedings after the ballots and other materials pertinent to the provincial election contest were transferred to the SET. In the petition, the petitioner alleged that there were strange proceedings30[20] which were unilateral, clandestine and surreptitious within the premises of the SET, on documents, ballots and election materials whose possession and custody have been transferred to the SET, and the petitioner was NEVER OFFICIALLY NOTIFIED of the strange on-goings at the SET.31[21] Attached to the petition was the letter of the Secretary of the SET confirming the conduct of proceedings in the provincial election contest, and citing as basis the authority of Acting SET Chairman, Justice Antonio T. Carpio, upon the formal request of the Office of Commissioner Lucenito N. Tagle, and citing Section 3, COMELEC Resolution No. 2812 dated 17 October 1995 on the coordination envisioned among the COMELEC, the SET and the courts so as not to delay or interrupt the revision of ballots being conducted. While the SET letter made the reservation that While the said provision speaks only of revision, it has been the practice of the Tribunal to allow the conduct of other proceedings in local election protest cases within its premises as may be requested, no mention whatsoever was made of the kind of proceedings taking place.

29[19]

CONSTITUTION, Article VIII, Section 14; See Solid Homes, Inc. v. Laserna, G.R. No. 166051, April 8, 2008, 550 SCRA 613. 30[20] Rollo, p. 12. 31[21] Id., p. 13.

83

84

It was at this point that this Court intervened, in response to the petitioners prayer for the issuance of temporary injunctive relief, through the issuance of a Status Quo Order with a non-extendible directive for the respondents to file their comments on the petition; for indeed, any further revision of ballots or other adversarial proceedings after the case has been submitted for resolution, would not only be strange and unusual but would indicate a gross violation of due process rights. After consideration of the respondents Comments and the petitioners petition and Reply, we hold that the contested proceedings at the SET (contested proceedings) are no longer part of the adversarial aspects of the election contest that would require notice of hearing and the participation of the parties. As the COMELEC stated in its Comment and without any contrary or disputing claim in the petitioners Reply:32[22] However, contrary to the claim of petitioner, public respondent in the appreciation of the contested ballots in EPC No. 2007-44 simultaneously with the SET in SET Case No. 001-07 is not conducting further proceedings requiring notice to the parties. There is no revision or correction of the ballots because EPC No. 2007-04 was already submitted for resolution. Public respondent, in coordinating with the SET, is simply resolving the submitted protest case before it. The parties necessarily take no part in said deliberation, which require utmost secrecy. Needless to state, the actual decision-making process is supposed to be conducted only by the designated members of the Second Division of the public respondent in strict confidentiality. In other words, what took place at the SET were the internal deliberations of the COMELEC, as a quasi-judicial body, in the course of appreciating the evidence presented and deciding the provincial election contest on the merits. These deliberations are no different from judicial deliberations which are considered confidential and privileged.33[23] We find it significant that the private respondents Comment fully supported the COMELECs position and disavowed any participation in the contested proceeding the petitioner complained about. The petitioner, on the other hand, has not shown that the private respondent was ever present in any proceeding at the SET relating to the provincial election contest. To conclude, the rights to notice and to be heard are not material considerations in the COMELECs handling of the Bulacan provincial election contest after the transfer of the ballot boxes to the SET; no proceedings at the instance of one party or of COMELEC has been conducted at the SET that would require notice and hearing because of the possibility of prejudice to the other party. The COMELEC is under no legal obligation to notify either party of the steps it is taking in the course of deliberating on the merits of the provincial election contest. In the context of our standard of review for the petition, we see no grave abuse of discretion amounting to lack or excess of jurisdiction committed by the COMELEC in its deliberation on the Bulacan election contest and the appreciation of ballots this deliberation entailed. Alleged Violations of Deliberation Stage Rights.

32[22] 33[23]

COMELEC Comment; rollo, pp. 72-S and 72-T. See Chavez v. Public Estates Authority, G.R. No. 133250, July 9, 2002, 384 SCRA 152.

84

85

On the basis of the above conclusion, we see no point in discussing any alleged violation of the deliberative stage rights. First, no illegal proceeding ever took place that would bear the poisonous fruits that the petitioner fears. Secondly, in the absence of the results of the COMELEC deliberations through its decision on the election protest, no basis exists to apply the Ang Tibay deliberative stage rights; there is nothing for us to test under the standards of the due process deliberative stages rights before the COMELEC renders its decision. Expressed in terms of our standard of review, we have as yet no basis to determine the existence of any grave abuse of discretion. Conduct of COMELEC Deliberations at the SET Premises We turn to the issue of the propriety of the COMELECs consideration of the provincial election contest (specifically its appreciation of the contested ballots) at the SET premises and while the same ballots are also under consideration by the SET for another election contest legitimately within the SETs own jurisdiction. We state at the outset that the COMELEC did not lose jurisdiction over the provincial election contest, as the petitioner seems to imply, because of the transmittal of the provincial ballot boxes and other election materials to the SET. The Constitution conferred upon the COMELEC jurisdiction over election protests involving provincial officials. The COMELEC in this case has lawfully acquired jurisdiction over the subject matter, i.e., the provincial election contest, as well as over the parties. After its jurisdiction attached, this jurisdiction cannot be ousted by subsequent events such as the temporary transfer of evidence and material records of the proceedings to another tribunal exercising its own jurisdiction over another election contest pursuant to the Constitution. This is the rule of adherence of jurisdiction.34[24] Thus, the jurisdiction of the COMELEC over provincial election contest exists side by side with the jurisdiction of the Senate Electoral Tribunal, with each tribunal being supreme in their respective areas of concern (the Senate election contests for the SET, and the regional, provincial and city election contests for the COMELEC), and with neither one being higher than the other in terms of precedence so that the jurisdiction of one must yield to the other. But while no precedence in jurisdiction exists, the COMELEC, vowing to the reality that only a single ballot exists in an election for national and local officials, saw it fit to lay down the rule on the order of preference in the custody and revision of ballots and other documents contained in the ballot boxes. The order, in terms of the adjudicatory tribunal and as provided in COMELEC Resolution No. 2812, runs: 1. 2. 3. 4. 5. Presidential Electoral Tribunal; Senate Electoral Tribunal; House of Representatives Electoral Tribunal; Commission on Elections; and Regional Trial Courts.

This order of preference dictated that the ballot boxes and other election materials in Bulacans provincial election contest, had to be transferred to the SET when the
34[24]

See: Ramos v. Central Bank of the Philippines, No. L-29352, October 4, 1971, 41 SCRA 565; Bengzon v. Inciong, Nos. L-48706-07, June 29, 1079, 91 SCRA 248; Baltazar v. CA, 104 SCRA 619 [1981]; Ramos v. Our Lady of Peace School, No. L-55950, December 26, 1984, 133 SCRA 741; Lee v. Presiding Judge, MTC Legazpi City, No. L-68789, November 10, 1986, 145 SCRA 408.

85

86

latter needed these materials for its revision of ballots. The transfer to the SET, however, did not mean that the Bulacan provincial election contest at that time already submitted for decision had to be suspended as the COMELEC held in its Orders of 29 April 2009 and 26 May 2009 in EPC No. 2007-44.35[25] This is particularly true in Bulacans case as no revision had to be undertaken, the revision having been already terminated. WHEREFORE, premises considered, we DISMISS the petition for certiorari for lack of merit. We accordingly LIFT the STATUS QUO ORDER we issued, effective immediately. CHAPTER III - THE EQUAL PROTECTION CLAUSE Sec. 1nor shall any person be denied the equal protection of the laws. The laws considering appointed officials of the government who filed their certificates of candidacy considered resigned while elected officials are not does not violate the equal protection clause of the Constitution. ELEAZAR P. QUINTO and GERINO TOLENTINO, JR. VS. COMELEC, G.R. No. 189698, February 22, 2010 (Reversing the December 1, 2009 En Banc Decision) Puno, CJ The main issue in this case is whether or not the second proviso in the third paragraph of Section 13 of Republic Act No. 9369, Section 66 of the Omnibus Election Code and Section 4(a) of COMELEC Resolution No. 8678, providing that appointive officials are deemed automatically resigned from their jobs upon the filing of their certificates of candidacy (while the elected officials are not) is unconstitutional mainly on the ground that they violate the equal protection clause of the Constitution and suffer from overbreadth. On December 1, 2009, the Supreme Court held that the questioned provisions of the above-mentioned laws are unconstitutional for being violative of the equal protection clause. On Motion for Reconsideration, the Supreme Court reconsidered its earlier Decision and declared the above laws and COMELEC Resolution constitutional. In support of their respective motions for reconsideration, respondent COMELEC and movants-intervenors submit the following arguments: (1) The assailed Decision is contrary to, and/or violative of, the constitutional proscription against the participation of public appointive officials and members of the military in partisan political activity;

35[25]

Rollo, pp. 29-34.

86

87

(2) The assailed provisions do not violate the equal protection clause when they accord differential treatment to elective and appointive officials, because such differential treatment rests on material and substantial distinctions and is germane to the purposes of the law; (3) The assailed provisions do not suffer from the infirmity of overbreadth; and (4) There is a compelling need to reverse the assailed Decision, as public safety and interest demand such reversal. We find the foregoing arguments meritorious. The assailed December 1, 2009 Decision struck down Section 4(a) of Resolution 8678, the second proviso in the third paragraph of Section 13 of Republic Act (RA) 9369, and Section 66 of the Omnibus Election Code, on the following grounds: (1) They violate the equal protection clause of the Constitution because of the differential treatment of persons holding appointive offices and those holding elective positions; (2) They are overbroad insofar as they prohibit the candidacy of all civil servants holding appointive posts: (a) without distinction as to whether or not they occupy high/influential positions in the government, and (b) they limit these civil servants activity regardless of whether they be partisan or nonpartisan in character, or whether they be in the national, municipal or barangay level; and (3) Congress has not shown a compelling state interest to restrict the fundamental right of these public appointive officials. We grant the motions for reconsideration. We now rule that Section 4(a) of Resolution 8678, Section 66 of the Omnibus Election Code, and the second proviso in the third paragraph of Section 13 of RA 9369 are not unconstitutional, and accordingly reverse our December 1, 2009 Decision. We now hold that Section 4(a) of Resolution 8678, Section 66 of the Omnibus Election Code, and the second proviso in the third paragraph of Section 13 of RA 9369 are not violative of the equal protection clause of the Constitution. i. Farias, et al. v. Executive Secretary, et al. is Controlling

In truth, this Court has already ruled squarely on whether these deemedresigned provisions challenged in the case at bar violate the equal protection clause of the Constitution in Farias, et al. v. Executive Secretary, et al. In Farias, the constitutionality of Section 14 of the Fair Election Act, in relation to Sections 66 and 67 of the Omnibus Election Code, was assailed on the ground, among others, that it unduly discriminates against appointive officials. As Section 14 repealed Section 67 (i.e., the deemed-resigned provision in respect of elected officials) of the Omnibus Election Code, elected officials are no longer considered ipso facto resigned from their respective offices upon their filing of certificates of candidacy. In contrast, since Section 66 was not repealed, the limitation on appointive officials continues to be operative they are deemed resigned when they file their certificates of candidacy.
87

88

The petitioners in Farias thus brought an equal protection challenge against Section 14, with the end in view of having the deemed-resigned provisions apply equally to both elected and appointive officials. We held, however, that the legal dichotomy created by the Legislature is a reasonable classification, as there are material and significant distinctions between the two classes of officials. Consequently, the contention that Section 14 of the Fair Election Act, in relation to Sections 66 and 67 of the Omnibus Election Code, infringed on the equal protection clause of the Constitution, failed muster. We ruled: The petitioners' contention, that the repeal of Section 67 of the Omnibus Election Code pertaining to elective officials gives undue benefit to such officials as against the appointive ones and violates the equal protection clause of the constitution, is tenuous. The equal protection of the law clause in the Constitution is not absolute, but is subject to reasonable classification. If the groupings are characterized by substantial distinctions that make real differences, one class may be treated and regulated differently from the other. The Court has explained the nature of the equal protection guarantee in this manner: The equal protection of the law clause is against undue favor and individual or class privilege, as well as hostile discrimination or the oppression of inequality. It is not intended to prohibit legislation which is limited either in the object to which it is directed or by territory within which it is to operate. It does not demand absolute equality among residents; it merely requires that all persons shall be treated alike, under like circumstances and conditions both as to privileges conferred and liabilities enforced. The equal protection clause is not infringed by legislation which applies only to those persons falling within a specified class, if it applies alike to all persons within such class, and reasonable grounds exist for making a distinction between those who fall within such class and those who do not. Substantial distinctions clearly exist between elective officials and appointive officials. The former occupy their office by virtue of the mandate of the electorate. They are elected to an office for a definite term and may be removed therefrom only upon stringent conditions. On the other hand, appointive officials hold their office by virtue of their designation thereto by an appointing authority. Some appointive officials hold their office in a permanent capacity and are entitled to security of tenure while others serve at the pleasure of the appointing authority. Another substantial distinction between the two sets of officials is that under Section 55, Chapter 8, Title I, Subsection A. Civil Service Commission, Book V of the Administrative Code of 1987 (Executive Order No. 292), appointive officials, as officers and employees in the civil service, are strictly prohibited from engaging in any partisan political activity or take (sic) part in any
88

89

election except to vote. Under the same provision, elective officials, or officers or employees holding political offices, are obviously expressly allowed to take part in political and electoral activities. By repealing Section 67 but retaining Section 66 of the Omnibus Election Code, the legislators deemed it proper to treat these two classes of officials differently with respect to the effect on their tenure in the office of the filing of the certificates of candidacy for any position other than those occupied by them. Again, it is not within the power of the Court to pass upon or look into the wisdom of this classification. Since the classification justifying Section 14 of Rep. Act No. 9006, i.e., elected officials vis--vis appointive officials, is anchored upon material and significant distinctions and all the persons belonging under the same classification are similarly treated, the equal protection clause of the Constitution is, thus, not infringed. The case at bar is a crass attempt to resurrect a dead issue. The miracle is that our assailed Decision gave it new life. We ought to be guided by the doctrine of stare decisis et non quieta movere. This doctrine, which is really adherence to precedents, mandates that once a case has been decided one way, then another case involving exactly the same point at issue should be decided in the same manner. This doctrine is one of policy grounded on the necessity for securing certainty and stability of judicial decisions. As the renowned jurist Benjamin Cardozo stated in his treatise The Nature of the Judicial Process: It will not do to decide the same question one way between one set of litigants and the opposite way between another. If a group of cases involves the same point, the parties expect the same decision. It would be a gross injustice to decide alternate cases on opposite principles. If a case was decided against me yesterday when I was a defendant, I shall look for the same judgment today if I am plaintiff. To decide differently would raise a feeling of resentment and wrong in my breast; it would be an infringement, material and moral, of my rights." Adherence to precedent must then be the rule rather than the exception if litigants are to have faith in the evenhanded administration of justice in the courts. Our Farias ruling on the equal protection implications of the deemedresigned provisions cannot be minimalized as mere obiter dictum. It is trite to state that an adjudication on any point within the issues presented by the case cannot be considered as obiter dictum. This rule applies to all pertinent questions that are presented and resolved in the regular course of the consideration of the case and lead up to the final conclusion, and to any statement as to the matter on which the decision is predicated. For that reason, a point expressly decided does not lose its value as a precedent because the disposition of the case is, or might have been, made on some other ground; or even though, by reason of other points in the case, the result reached might have been the same if the court had held, on the particular point, otherwise than it did. As we held in Villanueva, Jr. v. Court of Appeals, et al.: A decision which the case could have turned on is not regarded as obiter dictum merely because, owing to the disposal of the contention, it was necessary to consider another question, nor can
89

90

an additional reason in a decision, brought forward after the case has been disposed of on one ground, be regarded as dicta. So, also, where a case presents two (2) or more points, any one of which is sufficient to determine the ultimate issue, but the court actually decides all such points, the case as an authoritative precedent as to every point decided, and none of such points can be regarded as having the status of a dictum, and one point should not be denied authority merely because another point was more dwelt on and more fully argued and considered, nor does a decision on one proposition make statements of the court regarding other propositions dicta. (italics supplied) ii. Classification Germane to the Purposes of the Law

The Farias ruling on the equal protection challenge stands on solid ground even if reexamined. To start with, the equal protection clause does not require the universal application of the laws to all persons or things without distinction. What it simply requires is equality among equals as determined according to a valid classification. The test developed by jurisprudence here and yonder is that of reasonableness, which has four requisites: (1) The classification rests on substantial distinctions; (2) It is germane to the purposes of the law; (3) It is not limited to existing conditions only; and (4) It applies equally to all members of the same class. Our assailed Decision readily acknowledged that these deemed-resigned provisions satisfy the first, third and fourth requisites of reasonableness. It, however, proffers the dubious conclusion that the differential treatment of appointive officials vis--vis elected officials is not germane to the purpose of the law, because whether one holds an appointive office or an elective one, the evils sought to be prevented by the measure remain, viz.: For example, the Executive Secretary, or any Member of the Cabinet for that matter, could wield the same influence as the Vice-President who at the same time is appointed to a Cabinet post (in the recent past, elected Vice-Presidents were appointed to take charge of national housing, social welfare development, interior and local government, and foreign affairs). With the fact that they both head executive offices, there is no valid justification to treat them differently when both file their [Certificates of Candidacy] for the elections. Under the present state of our law, the VicePresident, in the example, running this time, let us say, for President, retains his position during the entire election period and can still use the resources of his office to support his campaign. Sad to state, this conclusion conveniently ignores the long-standing rule that to remedy an injustice, the Legislature need not address every manifestation of the evil at once; it may proceed one step at a time. In addressing a societal concern, it must invariably draw lines and make choices, thereby creating some inequity as to those included or excluded. Nevertheless, as long as the bounds of reasonable choice are not exceeded, the courts must defer to the legislative
90

91

judgment. We may not strike down a law merely because the legislative aim would have been more fully achieved by expanding the class. Stated differently, the fact that a legislative classification, by itself, is under inclusive will not render it unconstitutionally arbitrary or invidious. There is no constitutional requirement that regulation must reach each and every class to which it might be applied; that the Legislature must be held rigidly to the choice of regulating all or none. Thus, any person who poses an equal protection challenge must convincingly show that the law creates a classification that is palpably arbitrary or capricious. He must refute all possible rational bases for the differing treatment, whether or not the Legislature cited those bases as reasons for the enactment, such that the constitutionality of the law must be sustained even if the reasonableness of the classification is fairly debatable. In the case at bar, the petitioners failed and in fact did not even attempt to discharge this heavy burden. Our assailed Decision was likewise silent as a sphinx on this point even while we submitted the following thesis: ... [I]t is not sufficient grounds for invalidation that we may find that the statutes distinction is unfair, underinclusive, unwise, or not the best solution from a public-policy standpoint; rather, we must find that there is no reasonably rational reason for the differing treatment. In the instant case, is there a rational justification for excluding elected officials from the operation of the deemed resigned provisions? I submit that there is. An election is the embodiment of the popular will, perhaps the purest expression of the sovereign power of the people. It involves the choice or selection of candidates to public office by popular vote. Considering that elected officials are put in office by their constituents for a definite term, it may justifiably be said that they were excluded from the ambit of the deemed resigned provisions in utmost respect for the mandate of the sovereign will. In other words, complete deference is accorded to the will of the electorate that they be served by such officials until the end of the term for which they were elected. In contrast, there is no such expectation insofar as appointed officials are concerned. The dichotomized treatment of appointive and elective officials is therefore germane to the purposes of the law. For the law was made not merely to preserve the integrity, efficiency, and discipline of the public service; the Legislature, whose wisdom is outside the rubric of judicial scrutiny, also thought it wise to balance this with the competing, yet equally compelling, interest of deferring to the sovereign will. (emphasis in the original) In fine, the assailed Decision would have us equalize the playing field by invalidating provisions of law that seek to restrain the evils from running riot. Under the pretext of equal protection, it would favor a situation in which the evils are unconfined and vagrant, existing at the behest of both appointive and elected officials, over another in which a significant portion thereof is contained. The absurdity of that position is self-evident, to say the least. The concern, voiced by our esteemed colleague, Mr. Justice Nachura, in his dissent, that elected officials (vis--vis appointive officials) have greater
91

92

political clout over the electorate, is indeed a matter worth exploring but not by this Court. Suffice it to say that the remedy lies with the Legislature. It is the Legislature that is given the authority, under our constitutional system, to balance competing interests and thereafter make policy choices responsive to the exigencies of the times. It is certainly within the Legislatures power to make the deemed-resigned provisions applicable to elected officials, should it later decide that the evils sought to be prevented are of such frequency and magnitude as to tilt the balance in favor of expanding the class. This Court cannot and should not arrogate unto itself the power to ascertain and impose on the people the best state of affairs from a public policy standpoint. iii. Mancuso v. Taft Has Been Overruled

Finding no Philippine jurisprudence to prop up its equal protection ruling, our assailed Decision adverted to, and extensively cited, Mancuso v. Taft. This was a decision of the First Circuit of the United States Court of Appeals promulgated in March 1973, which struck down as unconstitutional a similar statutory provision. Pathetically, our assailed Decision, relying on Mancuso, claimed: (1) The right to run for public office is inextricably linked with two fundamental freedoms freedom of expression and association; (2) Any legislative classification that significantly burdens this fundamental right must be subjected to strict equal protection review; and (3) While the state has a compelling interest in maintaining the honesty and impartiality of its public work force, the deemed-resigned provisions pursue their objective in a far too heavy-handed manner as to render them unconstitutional. It then concluded with the exhortation that since the Americans, from whom we copied the provision in question, had already stricken down a similar measure for being unconstitutional[,] it is high-time that we, too, should follow suit. Our assailed Decisions reliance on Mancuso is completely misplaced. We cannot blink away the fact that the United States Supreme Court effectively overruled Mancuso three months after its promulgation by the United States Court of Appeals. In United States Civil Service Commission, et al. v. National Association of Letter Carriers AFL-CIO, et al. and Broadrick, et al. v. State of Oklahoma, et al., the United States Supreme Court was faced with the issue of whether statutory provisions prohibiting federal and state employees from taking an active part in political management or in political campaigns were unconstitutional as to warrant facial invalidation. Violation of these provisions results in dismissal from employment and possible criminal sanctions. The Court declared these provisions compliant with the equal protection clause. It held that (i) in regulating the speech of its employees, the state as employer has interests that differ significantly from those it possesses in regulating the speech of the citizenry in general; (ii) the courts must therefore balance the legitimate interest of employee free expression against the interests of the employer in promoting efficiency of public services; (iii) if the employees expression interferes with the maintenance of efficient and regularly functioning services, the limitation on speech is not unconstitutional; and (iv) the Legislature is to be given some flexibility or latitude in ascertaining which positions are to be
92

93

covered by any statutory restrictions. Therefore, insofar as government employees are concerned, the correct standard of review is an interest-balancing approach, a means-end scrutiny that examines the closeness of fit between the governmental interests and the prohibitions in question. Letter Carriers elucidated on these principles, as follows: Until now, the judgment of Congress, the Executive, and the country appears to have been that partisan political activities by federal employees must be limited if the Government is to operate effectively and fairly, elections are to play their proper part in representative government, and employees themselves are to be sufficiently free from improper influences. The restrictions so far imposed on federal employees are not aimed at particular parties, groups, or points of view, but apply equally to all partisan activities of the type described. They discriminate against no racial, ethnic, or religious minorities. Nor do they seek to control political opinions or beliefs, or to interfere with or influence anyone's vote at the polls. But, as the Court held in Pickering v. Board of Education, the government has an interest in regulating the conduct and the speech of its employees that differ(s) significantly from those it possesses in connection with regulation of the speech of the citizenry in general. The problem in any case is to arrive at a balance between the interests of the (employee), as a citizen, in commenting upon matters of public concern and the interest of the (government), as an employer, in promoting the efficiency of the public services it performs through its employees. Although Congress is free to strike a different balance than it has, if it so chooses, we think the balance it has so far struck is sustainable by the obviously important interests sought to be served by the limitations on partisan political activities now contained in the Hatch Act. It seems fundamental in the first place that employees in the Executive Branch of the Government, or those working for any of its agencies, should administer the law in accordance with the will of Congress, rather than in accordance with their own or the will of a political party. They are expected to enforce the law and execute the programs of the Government without bias or favoritism for or against any political party or group or the members thereof. A major thesis of the Hatch Act is that to serve this great end of Government-the impartial execution of the laws-it is essential that federal employees, for example, not take formal positions in political parties, not undertake to play substantial roles in partisan political campaigns, and not run for office on partisan political tickets. Forbidding activities like these will reduce the hazards to fair and effective government. There is another consideration in this judgment: it is not only important that the Government and its employees in fact avoid practicing political justice, but it is also critical that they appear to the public to be avoiding it, if confidence in the system of representative Government is not to be eroded to a disastrous extent.
93

94

Another major concern of the restriction against partisan activities by federal employees was perhaps the immediate occasion for enactment of the Hatch Act in 1939. That was the conviction that the rapidly expanding Government work force should not be employed to build a powerful, invincible, and perhaps corrupt political machine. The experience of the 1936 and 1938 campaigns convinced Congress that these dangers were sufficiently real that substantial barriers should be raised against the party in power-or the party out of power, for that matter-using the thousands or hundreds of thousands of federal employees, paid for at public expense, to man its political structure and political campaigns. A related concern, and this remains as important as any other, was to further serve the goal that employment and advancement in the Government service not depend on political performance, and at the same time to make sure that Government employees would be free from pressure and from express or tacit invitation to vote in a certain way or perform political chores in order to curry favor with their superiors rather than to act out their own beliefs. It may be urged that prohibitions against coercion are sufficient protection; but for many years the joint judgment of the Executive and Congress has been that to protect the rights of federal employees with respect to their jobs and their political acts and beliefs it is not enough merely to forbid one employee to attempt to influence or coerce another. For example, at the hearings in 1972 on proposed legislation for liberalizing the prohibition against political activity, the Chairman of the Civil Service Commission stated that the prohibitions against active participation in partisan political management and partisan political campaigns constitute the most significant safeguards against coercion . . .. Perhaps Congress at some time will come to a different view of the realities of political life and Government service; but that is its current view of the matter, and we are not now in any position to dispute it. Nor, in our view, does the Constitution forbid it. Neither the right to associate nor the right to participate in political activities is absolute in any event. x x x As we see it, our task is not to destroy the Act if we can, but to construe it, if consistent with the will of Congress, so as to comport with constitutional limitations. (italics supplied) Broadrick likewise definitively stated that the assailed statutory provision is constitutionally permissible, viz.: Appellants do not question Oklahoma's right to place evenhanded restrictions on the partisan political conduct of state employees. Appellants freely concede that such restrictions serve valid and important state interests, particularly with respect to attracting greater numbers of qualified people by insuring their job security, free from the vicissitudes of the elective process, and by protecting them from political extortion. Rather, appellants maintain that however permissible, even commendable, the goals of s 818 may be, its language is unconstitutionally vague and its prohibitions too broad in their sweep, failing to distinguish
94

95

between conduct that may be proscribed and conduct that must be permitted. For these and other reasons, appellants assert that the sixth and seventh paragraphs of s 818 are void in toto and cannot be enforced against them or anyone else. We have held today that the Hatch Act is not impermissibly vague. We have little doubt that s 818 is similarly not so vague that men of common intelligence must necessarily guess at its meaning. Whatever other problems there are with s 818, it is all but frivolous to suggest that the section fails to give adequate warning of what activities it proscribes or fails to set out explicit standards' for those who must apply it. In the plainest language, it prohibits any state classified employee from being an officer or member of a partisan political club or a candidate for any paid public office. It forbids solicitation of contributions for any political organization, candidacy or other political purpose and taking part in the management or affairs of any political party or in any political campaign. Words inevitably contain germs of uncertainty and, as with the Hatch Act, there may be disputes over the meaning of such terms in s 818 as partisan, or take part in, or affairs of political parties. But what was said in Letter Carriers, is applicable here: there are limitations in the English language with respect to being both specific and manageably brief, and it seems to us that although the prohibitions may not satisfy those intent on finding fault at any cost, they are set out in terms that the ordinary person exercising ordinary common sense can sufficiently understand and comply with, without sacrifice to the public interest.' x x x xxxx [Appellants] nevertheless maintain that the statute is overbroad and purports to reach protected, as well as unprotected conduct, and must therefore be struck down on its face and held to be incapable of any constitutional application. We do not believe that the overbreadth doctrine may appropriately be invoked in this manner here. xxxx The consequence of our departure from traditional rules of standing in the First Amendment area is that any enforcement of a statute thus placed at issue is totally forbidden until and unless a limiting construction or partial invalidation so narrows it as to remove the seeming threat or deterrence to constitutionally protected expression. Application of the overbreadth doctrine in this manner is, manifestly, strong medicine. It has been employed by the Court sparingly and only as a last resort. x x x x x x But the plain import of our cases is, at the very least, that facial over-breadth adjudication is an exception to our traditional rules of practice and that its function, a limited one at the outset, attenuates as the otherwise unprotected behavior that it forbids the State to sanction moves from pure speech toward conduct and that conduct-even if expressive-falls within the scope of otherwise valid criminal laws that reflect legitimate state interests in maintaining comprehensive controls over harmful,
95

96

constitutionally unprotected conduct. Although such laws, if too broadly worded, may deter protected speech to some unknown extent, there comes a point where that effect-at best a predictioncannot, with confidence, justify invalidating a statute on its face and so prohibiting a State from enforcing the statute against conduct that is admittedly within its power to proscribe. To put the matter another way, particularly where conduct and not merely speech is involved, we believe that the overbreadth of a statute must not only be real, but substantial as well, judged in relation to the statute's plainly legitimate sweep. It is our view that s 818 is not substantially overbroad and that whatever overbreadth may exist should be cured through case-by-case analysis of the fact situations to which its sanctions, assertedly, may not be applied. Unlike ordinary breach-of-the peace statutes or other broad regulatory acts, s 818 is directed, by its terms, at political expression which if engaged in by private persons would plainly be protected by the First and Fourteenth Amendments. But at the same time, s 818 is not a censorial statute, directed at particular groups or viewpoints. The statute, rather, seeks to regulate political activity in an even-handed and neutral manner. As indicted, such statutes have in the past been subject to a less exacting overbreadth scrutiny. Moreover, the fact remains that s 818 regulates a substantial spectrum of conduct that is as manifestly subject to state regulation as the public peace or criminal trespass. This much was established in United Public Workers v. Mitchell, and has been unhesitatingly reaffirmed today in Letter Carriers. Under the decision in Letter Carriers, there is no question that s 818 is valid at least insofar as it forbids classified employees from: soliciting contributions for partisan candidates, political parties, or other partisan political purposes; becoming members of national, state, or local committees of political parties, or officers or committee members in partisan political clubs, or candidates for any paid public office; taking part in the management or affairs of any political party's partisan political campaign; serving as delegates or alternates to caucuses or conventions of political parties; addressing or taking an active part in partisan political rallies or meetings; soliciting votes or assisting voters at the polls or helping in a partisan effort to get voters to the polls; participating in the distribution of partisan campaign literature; initiating or circulating partisan nominating petitions; or riding in caravans for any political party or partisan political candidate. x x x It may be that such restrictions are impermissible and that s 818 may be susceptible of some other improper applications. But, as presently construed, we do not believe that s 818 must be discarded in toto because some persons arguably protected conduct may or may not be caught or chilled by the statute. Section 818 is not substantially overbroad and it not, therefore, unconstitutional on its face. (italics supplied) It bears stressing that, in his Dissenting Opinion, Mr. Justice Nachura does not deny the principles enunciated in Letter Carriers and Broadrick. He would hold, nonetheless, that these cases cannot be interpreted to mean a reversal of Mancuso, since they pertain to different types of laws and were decided based on a different set of facts, viz.:
96

97

In Letter Carriers, the plaintiffs alleged that the Civil Service Commission was enforcing, or threatening to enforce, the Hatch Acts prohibition against active participation in political management or political campaigns. The plaintiffs desired to campaign for candidates for public office, to encourage and get federal employees to run for state and local offices, to participate as delegates in party conventions, and to hold office in a political club. In Broadrick, the appellants sought the invalidation for being vague and overbroad a provision in the (sic) Oklahomas Merit System of Personnel Administration Act restricting the political activities of the States classified civil servants, in much the same manner as the Hatch Act proscribed partisan political activities of federal employees. Prior to the commencement of the action, the appellants actively participated in the 1970 reelection campaign of their superior, and were administratively charged for asking other Corporation Commission employees to do campaign work or to give referrals to persons who might help in the campaign, for soliciting money for the campaign, and for receiving and distributing campaign posters in bulk. Mancuso, on the other hand, involves, as aforesaid, an automatic resignation provision. Kenneth Mancuso, a full time police officer and classified civil service employee of the City of Cranston, filed as a candidate for nomination as representative to the Rhode Island General Assembly. The Mayor of Cranston then began the process of enforcing the resign-to-run provision of the City Home Rule Charter. Clearly, as the above-cited US cases pertain to different types of laws and were decided based on a different set of facts, Letter Carriers and Broadrick cannot be interpreted to mean a reversal of Mancuso. x x x (italics in the original) We hold, however, that his position is belied by a plain reading of these cases. Contrary to his claim, Letter Carriers, Broadrick and Mancuso all concerned the constitutionality of resign-to-run laws, viz.: (1) Mancuso involved a civil service employee who filed as a candidate for nomination as representative to the Rhode Island General Assembly. He assailed the constitutionality of 14.09(c) of the City Home Rule Charter, which prohibits continuing in the classified service of the city after becoming a candidate for nomination or election to any public office. (2) Letter Carriers involved plaintiffs who alleged that the Civil Service Commission was enforcing, or threatening to enforce, the Hatch Acts prohibition against active participation in political management or political campaigns with respect to certain defined activities in which they desired to engage. The plaintiffs relevant to this discussion are (a) The National Association of Letter Carriers, which alleged that its members were desirous of, among others, running in local elections for offices such as school board member, city council member or mayor;
97

98

(b) Plaintiff Gee, who alleged that he desired to, but did not, file as a candidate for the office of Borough Councilman in his local community for fear that his participation in a partisan election would endanger his job; and (c) Plaintiff Myers, who alleged that he desired to run as a Republican candidate in the 1971 partisan election for the mayor of West Lafayette, Indiana, and that he would do so except for fear of losing his job by reason of violation of the Hatch Act. The Hatch Act defines active participation in political management or political campaigns by cross-referring to the rules made by the Civil Service Commission. The rule pertinent to our inquiry states: 30. Candidacy for local office: Candidacy for a nomination or for election to any National, State, county, or municipal office is not permissible. The prohibition against political activity extends not merely to formal announcement of candidacy but also to the preliminaries leading to such announcement and to canvassing or soliciting support or doing or permitting to be done any act in furtherance of candidacy. The fact that candidacy, is merely passive is immaterial; if an employee acquiesces in the efforts of friends in furtherance of such candidacy such acquiescence constitutes an infraction of the prohibitions against political activity. (italics supplied) Section 9(b) requires the immediate removal of violators and forbids the use of appropriated funds thereafter to pay compensation to these persons. (3) Broadrick was a class action brought by certain Oklahoma state employees seeking a declaration of unconstitutionality of two subparagraphs of Section 818 of Oklahomas Merit System of Personnel Administration Act. Section 818 (7), the paragraph relevant to this discussion, states that [n]o employee in the classified service shall be a candidate for nomination or election to any paid public office Violation of Section 818 results in dismissal from employment, possible criminal sanctions and limited state employment ineligibility. Consequently, it cannot be denied that Letter Carriers and Broadrick effectively overruled Mancuso. By no stretch of the imagination could Mancuso still be held operative, as Letter Carriers and Broadrick (i) concerned virtually identical resign-to-run laws, and (ii) were decided by a superior court, the United States Supreme Court. It was thus not surprising for the First Circuit Court of Appeals the same court that decided Mancuso to hold categorically and emphatically in Magill v. Lynch that Mancuso is no longer good law. As we priorly explained: Magill involved Pawtucket, Rhode Island firemen who ran for city office in 1975. Pawtuckets Little Hatch Act prohibits city employees from engaging in a broad range of political activities. Becoming a candidate for any city office is specifically
98

99

proscribed, the violation being punished by removal from office or immediate dismissal. The firemen brought an action against the city officials on the ground that that the provision of the city charter was unconstitutional. However, the court, fully cognizant of Letter Carriers and Broadrick, took the position that Mancuso had since lost considerable vitality. It observed that the view that political candidacy was a fundamental interest which could be infringed upon only if less restrictive alternatives were not available, was a position which was no longer viable, since the Supreme Court (finding that the governments interest in regulating both the conduct and speech of its employees differed significantly from its interest in regulating those of the citizenry in general) had given little weight to the argument that prohibitions against the coercion of government employees were a less drastic means to the same end, deferring to the judgment of Congress, and applying a balancing test to determine whether limits on political activity by public employees substantially served government interests which were important enough to outweigh the employees First Amendment rights. It must be noted that the Court of Appeals ruled in this manner even though the election in Magill was characterized as nonpartisan, as it was reasonable for the city to fear, under the circumstances of that case, that politically active bureaucrats might use their official power to help political friends and hurt political foes. Ruled the court: The question before us is whether Pawtucket's charter provision, which bars a city employee's candidacy in even a nonpartisan city election, is constitutional. The issue compels us to extrapolate two recent Supreme Court decisions, Civil Service Comm'n v. Nat'l Ass'n of Letter Carriers and Broadrick v. Oklahoma. Both dealt with laws barring civil servants from partisan political activity. Letter Carriers reaffirmed United Public Workers v. Mitchell, upholding the constitutionality of the Hatch Act as to federal employees. Broadrick sustained Oklahoma's Little Hatch Act against constitutional attack, limiting its holding to Oklahoma's construction that the Act barred only activity in partisan politics. In Mancuso v. Taft, we assumed that proscriptions of candidacy in nonpartisan elections would not be constitutional. Letter Carriers and Broadrick compel new analysis. xxxx What we are obligated to do in this case, as the district court recognized, is to apply the Courts interest balancing approach to the kind of nonpartisan election revealed in this record. We believe that the district court found more residual vigor in our opinion in Mancuso v. Taft than remains after Letter Carriers. We have particular
99

100

reference to our view that political candidacy was a fundamental interest which could be trenched upon only if less restrictive alternatives were not available. While this approach may still be viable for citizens who are not government employees, the Court in Letter Carriers recognized that the government's interest in regulating both the conduct and speech of its employees differs significantly from its interest in regulating those of the citizenry in general. Not only was United Public Workers v. Mitchell "unhesitatingly" reaffirmed, but the Court gave little weight to the argument that prohibitions against the coercion of government employees were a less drastic means to the same end, deferring to the judgment of the Congress. We cannot be more precise than the Third Circuit in characterizing the Court's approach as "some sort of 'balancing' process". It appears that the government may place limits on campaigning by public employees if the limits substantially serve government interests that are "important" enough to outweigh the employees' First Amendment rights. x x x (italics supplied) Upholding thus the constitutionality of the law in question, the Magill court detailed the major governmental interests discussed in Letter Carriers and applied them to the Pawtucket provision as follows: In Letter Carriers[,] the first interest identified by the Court was that of an efficient government, faithful to the Congress rather than to party. The district court discounted this interest, reasoning that candidates in a local election would not likely be committed to a state or national platform. This observation undoubtedly has substance insofar as allegiance to broad policy positions is concerned. But a different kind of possible political intrusion into efficient administration could be thought to threaten municipal government: not into broad policy decisions, but into the particulars of administration favoritism in minute decisions affecting welfare, tax assessments, municipal contracts and purchasing, hiring, zoning, licensing, and inspections. Just as the Court in Letter Carriers identified a second governmental interest in the avoidance of the appearance of "political justice" as to policy, so there is an equivalent interest in avoiding the appearance of political preferment in privileges, concessions, and benefits. The appearance (or reality) of favoritism that the charter's authors evidently feared is not exorcised by the nonpartisan character of the formal election process. Where, as here, party support is a key to successful campaigning, and party rivalry is the norm, the city might reasonably fear that politically active bureaucrats would use their official power to help
100

101

political friends and hurt political foes. This is not to say that the city's interest in visibly fair and effective administration necessarily justifies a blanket prohibition of all employee campaigning; if parties are not heavily involved in a campaign, the danger of favoritism is less, for neither friend nor foe is as easily identified. A second major governmental interest identified in Letter Carriers was avoiding the danger of a powerful political machine. The Court had in mind the large and growing federal bureaucracy and its partisan potential. The district court felt this was only a minor threat since parties had no control over nominations. But in fact candidates sought party endorsements, and party endorsements proved to be highly effective both in determining who would emerge from the primary election and who would be elected in the final election. Under the prevailing customs, known party affiliation and support were highly significant factors in Pawtucket elections. The charter's authors might reasonably have feared that a politically active public work force would give the incumbent party, and the incumbent workers, an unbreakable grasp on the reins of power. In municipal elections especially, the small size of the electorate and the limited powers of local government may inhibit the growth of interest groups powerful enough to outbalance the weight of a partisan work force. Even when nonpartisan issues and candidacies are at stake, isolated government employees may seek to influence voters or their co-workers improperly; but a more real danger is that a central party structure will mass the scattered powers of government workers behind a single party platform or slate. Occasional misuse of the public trust to pursue private political ends is tolerable, especially because the political views of individual employees may balance each other out. But party discipline eliminates this diversity and tends to make abuse systematic. Instead of a handful of employees pressured into advancing their immediate superior's political ambitions, the entire government work force may be expected to turn out for many candidates in every election. In Pawtucket, where parties are a continuing presence in political campaigns, a carefully orchestrated use of city employees in support of the incumbent party's candidates is possible. The danger is scarcely lessened by the openness of Pawtucket's nominating procedure or the lack of party labels on its ballots. The third area of proper governmental interest in Letter Carriers was ensuring that employees achieve advancement on their merits and that they be free from both coercion and the
101

102

prospect of favor from political activity. The district court did not address this factor, but looked only to the possibility of a civil servant using his position to influence voters, and held this to be no more of a threat than in the most nonpartisan of elections. But we think that the possibility of coercion of employees by superiors remains as strong a factor in municipal elections as it was in Letter Carriers. Once again, it is the systematic and coordinated exploitation of public servants for political ends that a legislature is most likely to see as the primary threat of employees' rights. Political oppression of public employees will be rare in an entirely nonpartisan system. Some superiors may be inclined to ride herd on the politics of their employees even in a nonpartisan context, but without party officials looking over their shoulders most supervisors will prefer to let employees go their own ways. In short, the government may constitutionally restrict its employees' participation in nominally nonpartisan elections if political parties play a large role in the campaigns. In the absence of substantial party involvement, on the other hand, the interests identified by the Letter Carriers Court lose much of their force. While the employees' First Amendment rights would normally outbalance these diminished interests, we do not suggest that they would always do so. Even when parties are absent, many employee campaigns might be thought to endanger at least one strong public interest, an interest that looms larger in the context of municipal elections than it does in the national elections considered in Letter Carriers. The city could reasonably fear the prospect of a subordinate running directly against his superior or running for a position that confers great power over his superior. An employee of a federal agency who seeks a Congressional seat poses less of a direct challenge to the command and discipline of his agency than a fireman or policeman who runs for mayor or city council. The possibilities of internal discussion, cliques, and political bargaining, should an employee gather substantial political support, are considerable. (citations omitted) The court, however, remanded the case to the district court for further proceedings in respect of the petitioners overbreadth charge. Noting that invalidating a statute for being overbroad is not to be taken lightly, much less to be taken in the dark, the court held: The governing case is Broadrick, which introduced the doctrine of "substantial" overbreadth in a closely analogous case. Under Broadrick, when one who challenges a law has engaged in
102

103

constitutionally unprotected conduct (rather than unprotected speech) and when the challenged law is aimed at unprotected conduct, "the overbreadth of a statute must not only be real, but substantial as well, judged in relation to the statute's plainly legitimate sweep." Two major uncertainties attend the doctrine: how to distinguish speech from conduct, and how to define "substantial" overbreadth. We are spared the first inquiry by Broadrick itself. The plaintiffs in that case had solicited support for a candidate, and they were subject to discipline under a law proscribing a wide range of activities, including soliciting contributions for political candidates and becoming a candidate. The Court found that this combination required a substantial overbreadth approach. The facts of this case are so similar that we may reach the same result without worrying unduly about the sometimes opaque distinction between speech and conduct. The second difficulty is not so easily disposed of. Broadrick found no substantial overbreadth in a statute restricting partisan campaigning. Pawtucket has gone further, banning participation in nonpartisan campaigns as well. Measuring the substantiality of a statute's overbreadth apparently requires, inter alia, a rough balancing of the number of valid applications compared to the number of potentially invalid applications. Some sensitivity to reality is needed; an invalid application that is far-fetched does not deserve as much weight as one that is probable. The question is a matter of degree; it will never be possible to say that a ratio of one invalid to nine valid applications makes a law substantially overbroad. Still, an overbreadth challenger has a duty to provide the court with some idea of the number of potentially invalid applications the statute permits. Often, simply reading the statute in the light of common experience or litigated cases will suggest a number of probable invalid applications. But this case is different. Whether the statute is overbroad depends in large part on the number of elections that are insulated from party rivalry yet closed to Pawtucket employees. For all the record shows, every one of the city, state, or federal elections in Pawtucket is actively contested by political parties. Certainly the record suggests that parties play a major role even in campaigns that often are entirely nonpartisan in other cities. School committee candidates, for example, are endorsed by the local Democratic committee. The state of the record does not permit us to find overbreadth; indeed such a step is not to be taken lightly, much less to be taken in the dark. On the other hand, the entire focus below, in the short
103

104

period before the election was held, was on the constitutionality of the statute as applied. Plaintiffs may very well feel that further efforts are not justified, but they should be afforded the opportunity to demonstrate that the charter forecloses access to a significant number of offices, the candidacy for which by municipal employees would not pose the possible threats to government efficiency and integrity which Letter Carriers, as we have interpreted it, deems significant. Accordingly, we remand for consideration of plaintiffs' overbreadth claim. (italics supplied, citations omitted) Clearly, Letter Carriers, Broadrick, and Magill demonstrate beyond doubt that Mancuso v. Taft, heavily relied upon by the ponencia, has effectively been overruled. As it is no longer good law, the ponencias exhortation that [since] the Americans, from whom we copied the provision in question, had already stricken down a similar measure for being unconstitutional[,] it is high-time that we, too, should follow suit is misplaced and unwarranted. Accordingly, our assailed Decisions submission that the right to run for public office is inextricably linked with two fundamental freedoms those of expression and association lies on barren ground. American case law has in fact never recognized a fundamental right to express ones political views through candidacy, as to invoke a rigorous standard of review. Bart v. Telford pointedly stated that [t]he First Amendment does not in terms confer a right to run for public office, and this court has held that it does not do so by implication either. Thus, ones interest in seeking office, by itself, is not entitled to constitutional protection. Moreover, one cannot bring ones action under the rubric of freedom of association, absent any allegation that, by running for an elective position, one is advancing the political ideas of a particular set of voters. Prescinding from these premises, it is crystal clear that the provisions challenged in the case at bar, are not violative of the equal protection clause. The deemed-resigned provisions substantially serve governmental interests (i.e., (i) efficient civil service faithful to the government and the people rather than to party; (ii) avoidance of the appearance of political justice as to policy; (iii) avoidance of the danger of a powerful political machine; and (iv) ensuring that employees achieve advancement on their merits and that they be free from both coercion and the prospect of favor from political activity). These are interests that are important enough to outweigh the non-fundamental right of appointive officials and employees to seek elective office. En passant, we find it quite ironic that Mr. Justice Nachura cites Clements v. Fashing and Morial, et al. v. Judiciary Commission of the State of Louisiana, et al. to buttress his dissent. Maintaining that resign-to-run provisions are valid only when made applicable to specified officials, he explains: U.S. courts, in subsequent cases, sustained the constitutionality of resign-to-run provisions when applied to specified or particular officials, as distinguished from all others, under a classification that is germane to the purposes of the law. These resign-to-run legislations were not expressed in a general and sweeping provision, and thus did not violate the test of being
104

105

germane to the purpose of the law, the second requisite for a valid classification. Directed, as they were, to particular officials, they were not overly encompassing as to be overbroad. (emphasis in the original) This reading is a regrettable misrepresentation of Clements and Morial. The resign-to-run provisions in these cases were upheld not because they referred to specified or particular officials (vis--vis a general class); the questioned provisions were found valid precisely because the Court deferred to legislative judgment and found that a regulation is not devoid of a rational predicate simply because it happens to be incomplete. In fact, the equal protection challenge in Clements revolved around the claim that the State of Texas failed to explain why some public officials are subject to the resign-to-run provisions, while others are not. Ruled the United States Supreme Court: Article XVI, 65, of the Texas Constitution provides that the holders of certain offices automatically resign their positions if they become candidates for any other elected office, unless the unexpired portion of the current term is one year or less. The burdens that 65 imposes on candidacy are even less substantial than those imposed by 19. The two provisions, of course, serve essentially the same state interests. The District Court found 65 deficient, however, not because of the nature or extent of the provision's restriction on candidacy, but because of the manner in which the offices are classified. According to the District Court, the classification system cannot survive equal protection scrutiny, because Texas has failed to explain sufficiently why some elected public officials are subject to 65 and why others are not. As with the case of 19, we conclude that 65 survives a challenge under the Equal Protection Clause unless appellees can show that there is no rational predicate to the classification scheme. The history behind 65 shows that it may be upheld consistent with the "one step at a time" approach that this Court has undertaken with regard to state regulation not subject to more vigorous scrutiny than that sanctioned by the traditional principles. Section 65 was enacted in 1954 as a transitional provision applying only to the 1954 election. Section 65 extended the terms of those offices enumerated in the provision from two to four years. The provision also staggered the terms of other offices so that at least some county and local offices would be contested at each election. The automatic resignation proviso to 65 was not added until 1958. In that year, a similar automatic resignation provision was added in Art. XI, 11, which applies to officeholders in home rule cities who serve terms longer than two years. Section 11 allows home rule cities the option of extending the terms of municipal offices from two to up to four years. Thus, the automatic resignation provision in Texas is a creature of the State's electoral reforms of 1958. That the State did not go further in applying the automatic resignation provision to those officeholders whose terms were not extended by 11 or 65, absent an invidious purpose, is not the sort of malfunctioning of the State's lawmaking process forbidden by the Equal Protection Clause. A regulation is not devoid of a rational predicate simply because it happens to be incomplete. The Equal Protection Clause does not forbid Texas to restrict one elected officeholder's
105

106

candidacy for another elected office unless and until it places similar restrictions on other officeholders. The provision's language and its history belie any notion that 65 serves the invidious purpose of denying access to the political process to identifiable classes of potential candidates. (citations omitted and italics supplied) Furthermore, it is unfortunate that the dissenters took the Morial line that there is no blanket approval of restrictions on the right of public employees to become candidates for public office out of context. A correct reading of that line readily shows that the Court only meant to confine its ruling to the facts of that case, as each equal protection challenge would necessarily have to involve weighing governmental interests vis--vis the specific prohibition assailed. The Court held: The interests of public employees in free expression and political association are unquestionably entitled to the protection of the first and fourteenth amendments. Nothing in today's decision should be taken to imply that public employees may be prohibited from expressing their private views on controversial topics in a manner that does not interfere with the proper performance of their public duties. In today's decision, there is no blanket approval of restrictions on the right of public employees to become candidates for public office. Nor do we approve any general restrictions on the political and civil rights of judges in particular. Our holding is necessarily narrowed by the methodology employed to reach it. A requirement that a state judge resign his office prior to becoming a candidate for non-judicial office bears a reasonably necessary relation to the achievement of the state's interest in preventing the actuality or appearance of judicial impropriety. Such a requirement offends neither the first amendment's guarantees of free expression and association nor the fourteenth amendment's guarantee of equal protection of the laws. (italics supplied)

Indeed, the Morial court even quoted Broadrick and stated that: In any event, the legislature must have some leeway in determining which of its employment positions require restrictions on partisan political activities and which may be left unregulated. And a State can hardly be faulted for attempting to limit the positions upon which such restrictions are placed. (citations omitted) V. Section 4(a) of Resolution 8678, Section 13 of RA 9369, and Section 66 of the Omnibus Election Code Do Not Suffer from Overbreadth Apart from nullifying Section 4(a) of Resolution 8678, Section 13 of RA 9369, and Section 66 of the Omnibus Election Code on equal protection ground, our assailed Decision struck them down for being overbroad in two respects, viz.: (1) The assailed provisions limit the candidacy of all civil servants holding appointive posts without due regard for the type of position being held by the employee seeking an elective post and the degree of influence that may be attendant thereto; and
106

107

(2) The assailed provisions limit the candidacy of any and all civil servants holding appointive positions without due regard for the type of office being sought, whether it be partisan or nonpartisan in character, or in the national, municipal or barangay level. Again, on second look, we have to revise our assailed Decision. i. Limitation on Candidacy Regardless of Incumbent Appointive Officials Position, Valid According to the assailed Decision, the challenged provisions of law are overly broad because they apply indiscriminately to all civil servants holding appointive posts, without due regard for the type of position being held by the employee running for elective office and the degree of influence that may be attendant thereto. Its underlying assumption appears to be that the evils sought to be prevented are extant only when the incumbent appointive official running for elective office holds an influential post. Such a myopic view obviously fails to consider a different, yet equally plausible, threat to the government posed by the partisan potential of a large and growing bureaucracy: the danger of systematic abuse perpetuated by a powerful political machine that has amassed the scattered powers of government workers so as to give itself and its incumbent workers an unbreakable grasp on the reins of power. As elucidated in our prior exposition: Attempts by government employees to wield influence over others or to make use of their respective positions (apparently) to promote their own candidacy may seem tolerable even innocuous particularly when viewed in isolation from other similar attempts by other government employees. Yet it would be decidedly foolhardy to discount the equally (if not more) realistic and dangerous possibility that such seemingly disjointed attempts, when taken together, constitute a veiled effort on the part of an emerging central party structure to advance its own agenda through a carefully orchestrated use of [appointive and/or elective] officials coming from various levels of the bureaucracy. [T]he avoidance of such a politically active public work force which could give an emerging political machine an unbreakable grasp on the reins of power is reason enough to impose a restriction on the candidacies of all appointive public officials without further distinction as to the type of positions being held by such employees or the degree of influence that may be attendant thereto. (citations omitted)

ii.

Limitation on Candidacy Regardless of Type of Office Sought, Valid

The assailed Decision also held that the challenged provisions of law are overly broad because they are made to apply indiscriminately to all civil servants holding appointive offices, without due regard for the type of elective office being sought, whether it be partisan or nonpartisan in character, or in the national, municipal or barangay level.
107

108

This erroneous ruling is premised on the assumption that the concerns of a truly partisan office and the temptations it fosters are sufficiently different from those involved in an office removed from regular party politics [so as] to warrant distinctive treatment, so that restrictions on candidacy akin to those imposed by the challenged provisions can validly apply only to situations in which the elective office sought is partisan in character. To the extent, therefore, that such restrictions are said to preclude even candidacies for nonpartisan elective offices, the challenged restrictions are to be considered as overbroad. Again, a careful study of the challenged provisions and related laws on the matter will show that the alleged overbreadth is more apparent than real. Our exposition on this issue has not been repudiated, viz.: A perusal of Resolution 8678 will immediately disclose that the rules and guidelines set forth therein refer to the filing of certificates of candidacy and nomination of official candidates of registered political parties, in connection with the May 10, 2010 National and Local Elections. Obviously, these rules and guidelines, including the restriction in Section 4(a) of Resolution 8678, were issued specifically for purposes of the May 10, 2010 National and Local Elections, which, it must be noted, are decidedly partisan in character. Thus, it is clear that the restriction in Section 4(a) of RA 8678 applies only to the candidacies of appointive officials vying for partisan elective posts in the May 10, 2010 National and Local Elections. On this score, the overbreadth challenge leveled against Section 4(a) is clearly unsustainable. Similarly, a considered review of Section 13 of RA 9369 and Section 66 of the Omnibus Election Code, in conjunction with other related laws on the matter, will confirm that these provisions are likewise not intended to apply to elections for nonpartisan public offices. The only elections which are relevant to the present inquiry are the elections for barangay offices, since these are the only elections in this country which involve nonpartisan public offices. In this regard, it is well to note that from as far back as the enactment of the Omnibus Election Code in 1985, Congress has intended that these nonpartisan barangay elections be governed by special rules, including a separate rule on deemed resignations which is found in Section 39 of the Omnibus Election Code. Said provision states: Section 39. Certificate of Candidacy. No person shall be elected punong barangay or kagawad ng sangguniang barangay unless he files a sworn certificate of candidacy in triplicate on any day from the commencement of the election period but not later than the day before the beginning of the campaign period in a form to be prescribed by the Commission. The candidate shall state the barangay office for which he is a candidate. xxxx
108

109

Any elective or appointive municipal, city, provincial or national official or employee, or those in the civil or military service, including those in government-owned or-controlled corporations, shall be considered automatically resigned upon the filing of certificate of candidacy for a barangay office. Since barangay elections are governed by a separate deemed resignation rule, under the present state of law, there would be no occasion to apply the restriction on candidacy found in Section 66 of the Omnibus Election Code, and later reiterated in the proviso of Section 13 of RA 9369, to any election other than a partisan one. For this reason, the overbreadth challenge raised against Section 66 of the Omnibus Election Code and the pertinent proviso in Section 13 of RA 9369 must also fail. In any event, even if we were to assume, for the sake of argument, that Section 66 of the Omnibus Election Code and the corresponding provision in Section 13 of RA 9369 are general rules that apply also to elections for nonpartisan public offices, the overbreadth challenge would still be futile. Again, we explained: In the first place, the view that Congress is limited to controlling only partisan behavior has not received judicial imprimatur, because the general proposition of the relevant US cases on the matter is simply that the government has an interest in regulating the conduct and speech of its employees that differs significantly from those it possesses in connection with regulation of the speech of the citizenry in general. Moreover, in order to have a statute declared as unconstitutional or void on its face for being overly broad, particularly where, as in this case, conduct and not pure speech is involved, the overbreadth must not only be real, but substantial as well, judged in relation to the statutes plainly legitimate sweep. In operational terms, measuring the substantiality of a statutes overbreadth would entail, among other things, a rough balancing of the number of valid applications compared to the number of potentially invalid applications. In this regard, some sensitivity to reality is needed; an invalid application that is farfetched does not deserve as much weight as one that is probable. The question is a matter of degree. Thus, assuming for the sake of argument that the partisan-nonpartisan distinction is valid and necessary such that a statute which fails to make this distinction is susceptible to an overbreadth attack, the overbreadth challenge presently mounted must demonstrate or provide this Court with some idea of the number of potentially invalid elections (i.e. the number of elections that were insulated from party rivalry but were nevertheless closed to appointive employees) that may in all probability result from the enforcement of the statute. The state of the record, however, does not permit us to find overbreadth. Borrowing from the words of Magill v. Lynch, indeed, such a step is not to be taken lightly, much less to be taken in the dark, especially since an overbreadth finding in this case
109

110

would effectively prohibit the State from enforcing an otherwise valid measure against conduct that is admittedly within its power to proscribe. This Court would do well to proceed with tiptoe caution, particularly when it comes to the application of the overbreadth doctrine in the analysis of statutes that purportedly attempt to restrict or burden the exercise of the right to freedom of speech, for such approach is manifestly strong medicine that must be used sparingly, and only as a last resort. In the United States, claims of facial overbreadth have been entertained only where, in the judgment of the court, the possibility that protected speech of others may be muted and perceived grievances left to fester (due to the possible inhibitory effects of overly broad statutes) outweighs the possible harm to society in allowing some unprotected speech or conduct to go unpunished. Facial overbreadth has likewise not been invoked where a limiting construction could be placed on the challenged statute, and where there are readily apparent constructions that would cure, or at least substantially reduce, the alleged overbreadth of the statute. In the case at bar, the probable harm to society in permitting incumbent appointive officials to remain in office, even as they actively pursue elective posts, far outweighs the less likely evil of having arguably protected candidacies blocked by the possible inhibitory effect of a potentially overly broad statute. In this light, the conceivably impermissible applications of the challenged statutes which are, at best, bold predictions cannot justify invalidating these statutes in toto and prohibiting the State from enforcing them against conduct that is, and has for more than 100 years been, unquestionably within its power and interest to proscribe. Instead, the more prudent approach would be to deal with these conceivably impermissible applications through case-by-case adjudication rather than through a total invalidation of the statute itself. Indeed, the anomalies spawned by our assailed Decision have taken place. In his Motion for Reconsideration, intervenor Drilon stated that a number of highranking Cabinet members had already filed their Certificates of Candidacy without relinquishing their posts. Several COMELEC election officers had likewise filed their Certificates of Candidacy in their respective provinces. Even the Secretary of Justice had filed her certificate of substitution for representative of the first district of Quezon province last December 14, 2009 even as her position as Justice Secretary includes supervision over the City and Provincial Prosecutors, who, in turn, act as Vice-Chairmen of the respective Boards of Canvassers. The Judiciary has not been spared, for a Regional Trial Court Judge in the South has thrown his hat into the political arena. We cannot allow the tilting of our electoral playing field in their favor. For the foregoing reasons, we now rule that Section 4(a) of Resolution 8678 and Section 13 of RA 9369, which merely reiterate Section 66 of the Omnibus Election Code, are not unconstitutionally overbroad. IN VIEW WHEREOF, the Court RESOLVES to GRANT the respondents and the intervenors Motions for Reconsideration; REVERSE and SET ASIDE this Courts December 1, 2009 Decision; DISMISS the Petition; and ISSUE this Resolution declaring as not UNCONSTITUTIONAL (1) Section 4(a) of COMELEC Resolution No. 8678, (2) the second proviso in the third paragraph of Section 13 of Republic Act No. 9369, and (3) Section 66 of the Omnibus Election Code.
110

111

1. The scope of the equal protection clause, 95 SCRA 2. Equal protection of the law, 13 SCRA 266 3. Requisites for a valid classificationRead: 1. People vs. Cayat, 68 Phil. 12 a. b. c. d. There must be real and substantial distinctions; It must be germane tot he purposes of the law; It must not be limited to existing conditions only; and It must apply equally to all members of the same class.

420

2. Read again, Association of Small Landowners vs. Sec. of Agrarian reform, July 14, 1989 4. Equal protection in generalRead: 8. P. vs. Vera, 65 Phil. 56 9. TIU VS. CA, 301 SCRA 278 (There is real and substantial distinction between business inside the Subic Special Economic Zone and outside wherein those inside are exempt from other taxes as a result of the policy of the government to accelerate the development of the portion of Subic left by the Americans) 10.IMELDA MARCOS VS. CA, 278 SCRA 843 11.HIMAGAN VS. PEOPLE, October 7, 1994 The fact that policemen charged with a criminal offense punishable by more than 6 years are to be suspended during the entire duration of the case unlike other government employees is valid since it rests on valid classification because policemen carry weapons and the badge of the law which can be used to harass or intimidate witnesses against them. There is no violation of the equal protection clause when the Compensation and Classification Act of 1989 includes certain allowances and fringe benefits into the standardized salaries of most government employees but not to police and military personnel.

ABAD, J.: These consolidated cases question the inclusion of certain allowances and fringe benefits into the standardized salary rates for offices in the national government, state universities and colleges, and local government units as required by the Compensation and Position Classification Act of 1989 and implemented through the challenged National Compensation Circular 59 (NCC
111

112

59) while the said allowances and other fringe benefits are not included insofar as members of the police and military are concerned. The Facts and the Case Congress enacted in 1989 Republic Act (R.A.) 6758, called the Compensation and Position Classification Act of 1989 to rationalize the compensation of government employees. Its Section 12 directed the consolidation of allowances and additional compensation already being enjoyed by employees into their standardized salary rates. But it exempted certain additional compensations that the employees may be receiving from such consolidation. Thus: Section 12. Consolidation of Allowances and Compensation. -- All allowances, except for representation and transportation allowances; clothing and laundry allowances; subsistence allowance of marine officers and crew on board government vessels and hospital personnel; hazard pay; allowances of foreign service personnel stationed abroad; and such other additional compensation not otherwise specified herein as may be determined by the DBM, shall be deemed included in the standardized salary rates herein prescribed. Such other additional compensation, whether in cash or in kind, being received by incumbents only as of July 1, 1989 not integrated into the standardized salary rates shall continue to be authorized. The Issue: Whether or not the grant of COLA to military and police personnel to the exclusion of other government employees violates the equal protection clause. The Courts Ruling At the heart of the present controversy is Section 12 of R.A. 6758 as quoted above. . As will be noted from the first sentence above, all allowances were deemed integrated into the standardized salary rates except the following: (1) (2) (3) representation and transportation allowances; clothing and laundry allowances; subsistence allowances of marine officers and crew on board government vessels; subsistence allowances of hospital personnel; hazard pay; allowances of foreign service personnel stationed abroad; and such other additional compensation not otherwise specified in Section 12 as may be determined by the DBM.
112

(4) (5) (6) (7)

113

But, while the provision enumerated certain exclusions, it also authorized the DBM to identify such other additional compensation that may be granted over and above the standardized salary rates. In Philippine Ports Authority Employees Hired After July 1, 1989 v. Commission on Audit, the Court has ruled that while Section 12 could be considered self-executing in regard to items (1) to (6), it was not so in regard to item (7). The DBM still needed to amplify item (7) since one cannot simply assume what other allowances were excluded from the standardized salary rates. It was only upon the issuance and effectivity of the corresponding implementing rules and regulations that item (7) could be deemed legally completed. Clearly, COLA is not in the nature of an allowance intended to reimburse expenses incurred by officials and employees of the government in the performance of their official functions. It is not payment in consideration of the fulfillment of official duty. As defined, cost of living refers to the level of prices relating to a range of everyday items or the cost of purchasing those goods and services which are included in an accepted standard level of consumption. Based on this premise, COLA is a benefit intended to cover increases in the cost of living. Thus, it is and should be integrated into the standardized salary rates. Petitioners contend that the continued grant of COLA to military and police to the exclusion of other government employees violates the equal protection clause of the Constitution. But as respondents pointed out, while it may appear that petitioners are questioning the constitutionality of these issuances, they are in fact attacking the very constitutionality of Section 11 of R.A. 6758. It is actually this provision which allows the uniformed personnel to continue receiving their COLA over and above their basic pay, thus: Section 11. Military and Police Personnel. - The base pay of uniformed personnel of the Armed Forces of the Philippines and the Integrated National Police shall be as prescribed in the salary schedule for these personnel in R.A. 6638 and R.A. 6648. The longevity pay of these personnel shall be as prescribed under R.A. 6638, and R.A. 1134 as amended by R.A. 3725 and R.A. 6648: Provided, however, That the longevity pay of uniformed personnel of the Integrated National Police shall include those services rendered as uniformed members of the police, jail and fire departments of the local government units prior to the police integration. All existing types of allowances authorized for uniformed personnel of the Armed Forces of the Philippines and Integrated National Police such as cost of living allowance, longevity pay, quarters allowance, subsistence allowance, clothing allowance, hazard pay and other allowances shall continue to be authorized. Nothing is more settled than that the constitutionality of a statute cannot be attacked collaterally because constitutionality issues must be pleaded directly and not collaterally. In any event, the Court is not persuaded that the continued grant of COLA to the uniformed personnel to the exclusion of other national government officials run afoul the equal protection clause of the Constitution. The fundamental right of equal protection of the laws is not absolute, but is subject to reasonable
113

114

classification. If the groupings are characterized by substantial distinctions that make real differences, one class may be treated and regulated differently from another. The classification must also be germane to the purpose of the law and must apply to all those belonging to the same class. To be valid and reasonable, the classification must satisfy the following requirements: (1) it must rest on substantial distinctions; (2) it must be germane to the purpose of the law; (3) it must not be limited to existing conditions only; and (4) it must apply equally to all members of the same class. It is clear from the first paragraph of Section 11 that Congress intended the uniformed personnel to be continually governed by their respective compensation laws. Thus, the military is governed by R.A. 6638, as amended by R.A. 9166 while the police is governed by R.A. 6648, as amended by R.A. 6975. Certainly, there are valid reasons to treat the uniformed personnel differently from other national government officials. Being in charged of the actual defense of the State and the maintenance of internal peace and order, they are expected to be stationed virtually anywhere in the country. They are likely to be assigned to a variety of low, moderate, and high-cost areas. Since their basic pay does not vary based on location, the continued grant of COLA is intended to help them offset the effects of living in higher cost areas. 2-A Gumabon vs. Director of Prisons, 37 SCRA 420 2-b. PANFILO LACSON VS. SANDIGANBAYAN, January 20, 1999 2-b-1. BASCO VS. PAGCOR, May 14, 1991 No violation of the equal protection clause if Congress would legalize cock-fighting and horse racing since police power could regulate gambling. 3. PHILIPPINE JUDGES ASSOCIATION VS. PRADO, November 11, 1993 There is no valid distinction for a law removing the franking privilege of the judiciary while leaving the same to the Executive and Legislative despite the fact that there is considerable volume of mails from the courts. Loss of revenue is not a valid ground unless it would be withdrawn to all government offices. FRANCISCO TATAD vs. THE SECRETARY OF DEPARTMENT OF ENERGY, G. R. No. 124360, November 5, 1997 EDCEL LAGMAN, JOKER ARROYO, ENRIQUE GARCIA, WIGBERTO TANADA, FLAG HUMAN RIGHTS FOUNDATION vs. HON. RUBEN TORRES, HON. FRANCISCO VIRAY, PETRON, FILIPINAS SHELL and CALTEX PHILIPPINES, G.R. No. 127867, November 5, 1997.

PUNO, J. These petitions challenge the constitutionality of Republic Act No. 8180 entitled An Act Deregulating the Downstream Oil Industry and for Other Purposes. RA 8180 seeks to end 26 years of government regulation of the downstream oil industry.
114

115

The facts: 1. Prior to 1971, no government agency was regulating the oil industry. New players were free to enter the oil market without any government interference. There were four (4) refining companies at that time. SHELL, CALTEX, BATAAN REFINING COMPANY and FILOIL MARKETING and six (6) petroleum marketing companies: ESSO, FILOIL, CALTEX, GETTY, MOBIL and SHELL; 2. In 1971, the country was driven to its knees by the crippling oil crisis and in order to remedy the same, the OIL INDUSTRY COMMISSION ACT was enacted REGULATING the oil industry ; 3. On November 9, 1973, then President Marcos created the Philippine national Oil Corporation (PNOC) t break the control of the foreigners to the oil industry. It acquired ownership of ESSO Philippines and Filoil and likewise bought controlling shares of the Bataan Refining Corporation. PNOC then operated under the business name PETRON CORPORATION and for the first time, there was a Filipino presence in the Philippine oil market; 4. In 1984, Pres. Marcos through section 8 of PD 1956 created the OIL PRICE STABILIZATION FUND (OPSF) to cushion the effects of frequent changes in the price of oil caused by the exchange rate adjustments or increase of the world market prices crude oil and imported petroleum products; 5. By 1985, only three (3) oil companies were left operating in the country. These are: CALTEX, FILIPINAS SHELL and PNOC; 6. In May, 1987, Pres. Corazon Aquino signed Executive Order No. 172 creating the ENERGY REULATORY BOARD to regulate the business of importing, exporting, shipping, transporting, processing, refining, marketing and distributing energy resources WHEN WARRANTED AND ONLY WHEN PUBLIC NECESSITY REQUIRES. The Board was empowered to fix and regulate the prices of petroleum products and other related merchandise; 7. In March, 1996, Congress enacted RA 8180 deregulating the Oil Industry not later than March, 1997. The law requires that the implementation of the regulation, shall as far as practicable be made at a time WHEN THE PRICES OF CRUDE OIL AND PETROLEUM PRODUCTS IN THE WORLD ARE DECLINING AND WHEN THE EXCHANGE RATE OF THE PESO IN RELATION TO THE US DOLLAR; IS STABLE; 8. On February 8, 1997, Executive Order No. 372 was issued by President Fidel Ramos implementing full deregulation ON THE GROUND THAT THE OPSF FUND HAS BEEN DEPLETED; 9. The petitioners questioned the constitutionality of RA 8180 on the following grounds: a. Section 5 of RA 8180 violates the equal protection clause of the Constitution; b. The imposition of different tariff rates does not deregulate the oil industry and even bars the entry of other players in the oil industry but instead effectively protects the interest of the oil companies with existing refineries. Thus, it runs counter to the objective of the law to foster a truly competitive market; The inclusion of Sec. 5 [b] providing for tariff differential violates Section 26 [1] of Art. VI of the 1987 Constitution which requires every law to have only one subject which should be expressed in the title thereof; c. Section 15 of RA 8180 and EO No. 392 are unconstitutional for undue delegation of legislative power to the President and the Secretary of Energy;
115

116

d. EO 392 implementing the full deregulation of the oil industry is unconstitutional since it is arbitrary and unreasonable since it was enacted due to the alleged depletion of the OPSF fund, a condition which is not found in RA No. 8180; e. Section 15 of RA 8180 is unconstitutional for it allows the formation of a de facto cartel among three existing oil companies in violation of the Constitution prohibiting against monopolies, combination in restraint of trade and unfair competition. The provisions of the law being questioned as unconstitutional are Section 5 [b] and Section 15 which provide: Section 5 [b] Any law to the contrary notwithstanding and starting with the effectivity of this Act, tariff duty shall be imposed and collected on imported crude oil at the rate of 3% and imported refined petroleum products at the rate of seven (7%) percent, except fuel oil and LPG, the rate for which shall be the same; Provided, that beginning on January 1, 2004, the tariff rate on imported crude oil and refined petroleum products shall be the same; Provided, further, that this provision may be amended only by an Act of Congress. xxx Section 15. Implementation of full deregulation. Pursuant to Section 5 [e] of RA 7638, the DOE, upon approval of the President, implement full deregulation of the downstream oil industry not later than March, 1997. As far as practicable, the DOE shall time the full deregulation when the prices of crude oil and petroleum products in the world market are declining and when the exchange rate of the peso in relation to the US dollar is stable. The issues are: Procedural Issues: a. Whether or not the petitions raise justiciable controversy; and b. Whether or not the petitioners have the standing to question the validity of the subject law and executive order. Substantive Issues: a. Whether or not Section 5 of RA 8180 violates the one titleone subject requirement of the Constitution; b. Whether or not Section 5 of RA 8180 violates the equal protection clause of the Constitution; c. Whether section 15 violates the constitutional prohibition on undue delegation of legislative power; d. Whether or not EO 392 is arbitrary and unreasonable; and e. Whether or not RA 8180 violates the constitutional prohibition against monopolies, combinations in restraint of trade and unfair competition. HELD:

116

117

1. Judicial power includes not only the duty of the courts to settle controversies involving rights but also the duty to determine whether or not there has been grave abuse of discretion amounting to lack or excess of jurisdiction on the part of any agency or branch of the government. The courts, as guardians of the Constitution, have the inherent authority to determine whether a statute enacted by the legislature transcends the limit imposed by the fundamental law. When the statute violates the Constitution, it is not only the right of the judiciary to declare such act as unconstitutional and void. 2. The question of locus standi must likewise fall . As held in KAPATIRAN NG MGA NAGLILINGKOD SA PAMAHALAAN NG PILIPINAS, INC. VS. TAN, it was held that: Objections to taxpayers suit for lack of sufficient personality, standing, or interest are , however, in the main procedural matters. CONSIDERING THE IMPORTANCE OF THE CASES TO THE PUBLIC, AND IN KEEPING WITH THE COURTS DUTY TO DETERMINE WHETHER OR NOT THE OTHER BRANCHEDS OF GOVERNMENT HAVE KEPT THEMSELVES WITHIN THE LIMITS OF THE CONSTITUTION AND THE LAWS AND THAT THEY HAVE NOT ABUSE THE DISCRETION GIVEN TO THEM, THE COURT HAS BRUSHED ASIDE TECHNICALITIES OF PROCEDURE AND HAS TAKEN COGNIZANCE OF THESE PETITIONS. There is no disagreement on the part of the parties as to the farreaching importance of the validity of RA 8180. Thus, there is no good sense in being hyper-technical on the standing of the petitioners for they pose issues which are significant to our people and which deserve our forthright resolution. 3. It is contended that Section 5[b[ of RA 8180 on tariff differentials violates the Constitutional prohibition requiring every law to have only one subject which should be expressed in its title. We do not concur with this contention. As a policy, the Court has adopted a liberal construction of the one title---one subject rule. We have consistently ruled that the title need not mirror, fully index or catalogue all contents and minute details of a law. A law having a single general subject indicated in the title may contain a number of provisions, no matter how diverse they may be, so long as they are not inconsistent with or foreign to the general subject, and may be considered in furtherance of such subject by providing for the method and means of carrying out the general subject. We hold that Section 5 providing for tariff differential is germane to the subject of RA 8180 which is the deregulation of the downstream oil industry. 4. The contention that there is undue delegation of legislative power when it authorized the President to determine when deregulation starts is without merit. The petitioners claim that the phrases as far as practicable, decline of crude oil prices in the world market and stability of the peso exchange rate to the US dollar are ambivalent, unclear and inconcrete in meaning and could not therefore provide the determinate or determinable standards which can guide the President in his decision to fully deregulate the oil industry. The power of Congress to delegate the execution of laws has long been settled by this Court in 1916 in the case of COMPANIA GENERAL DE TABACOS DE FILIPINA VS. THE BOARD OF PUBLIC UTILITY COMMISSIONERS WHERE IT WAS HELD THAT:
117

118

The true distinction is between the delegation of power to make the law , which necessarily involves a discretion as to what it shall be, and conferring authority or discretion as to its execution, to be exercised under and in pursuance of the law. The first cannot be done; to the latter, no valid objection can be made. Two tests have been developed to determine whether the delegation of the power to execute laws does not involve the abdication of the power to make law itself. We delineated the metes and bounds of these tests in EASTERM SHIPPING LINES VS. POEA, thus: There are two accepted tests to determine whether or not there is a valid delegation of legislative power , viz: the completeness test and the sufficiency of standard test. Under the first test, the law must be complete in all its terms and conditions when it leaves the legislative such that when it reaches the delegate, the only thing he will do is enforce it. Under the sufficient standard test, there must be adequate guidelines or limitations in the law to map out the boundaries of the delegates authority and prevent the delegation from running riot. BOTH TESTS ARE INTENDED TO PREVENT A TOTAL TRANSFERENCE OF LEGISLATIVE AUTHORITY TO THE DELEGATE, WHO IS NOT ALLOWED TO STEP INTO THE SHOES OF THE LEGISLATURE AND EXERCISE A POWER ESSENTIALLY LEGISLATIVE. The validity of delegating legislative power is now a quiet area in our constitutional landscape because such has become an inevitability in light of the increasing complexity of the task of government. In fact, in HIRABAYASHI VS. UNITED STATES, the Supreme Court through Justice ISAGANI CRUZ held that even if the law does not expressly pinpoint the standard, THE COURTS WILL BEND BACKWARD TO LOCATE THE SAME ELSEWHERE IN ORDER TO SPARE THE STATUTE; IF IT CAN, FROM CONSTITUTIONAL INFIRMITY. 5. EO No. 392 failed to follow faithfully the standards set by RA 8180 when it considered the extraneous factor of depletion of the OPSF Fund. The misapplication of this extra factor cannot be justified. The executive is bereft of any right to alter either by addition or subtraction the standards set by RA 8180 for it has no power to make laws. To cede to the executive the power to make laws would invite tyranny and to transgress the separation of powers. The exercise of delegated power is given a strict scrutiny by courts for the delegate is a mere agent whose action cannot infringe the terms of the agency. 6. Section 19 of Article XII of the Constitution provides: The state shall regulate or prohibit monopolies when the public interests so requires. No combinations in restraint of trade or unfair competition shall be allowed. A monopoly is a privilege or peculiar advantage vested in one or more persons or companies, consisting of the exclusive right or power to carry on a particular business or trade, manufacture a particular article or control the sale or the whole market structure in which one or only a few firms dominate the total sales of a product or service. On the other hand, a combination in restraint of trade is an agreement or understanding between two or more persons, in the form of contract, trust, pool, holding company,
118

119

for the purpose of unduly restricting competition, monopolizing trade and commerce in a certain commodity, controlling its production, distribution and price or otherwise interfering with freedom of trade without statutory authority. Combination in restraint of trade refers to means while monopoly refers to the end. Respondents aver that the 4% tariff differential is designed to encourage new entrants to invest in refineries. They stress that the inventory requirement is meant to guaranty continuous domestic supply of petroleum and to discourage fly-by-night operators. They also claim that the prohibition against predatory pricing is intended to protect prospective entrants. The validity of the assailed provisions of RA 8180 has to be decided in the light of the letter and spirit of Section 19, Art. XII of the Constitution. While the Constitution embraced free enterprise as an economic creed, it did not prohibit per se the operation of monopolies which can, however, be regulated in the public interest. This distinct free enterprise system is dictated by the need to achieve the goals of our national economy as defined under Section 1, Art. XII of the Constitution which are: more equitable distribution of opportunities, income and wealth; a sustained increase in the amount of goods and services produced by the nation for all, especially the underprivileged . It also calls for the State to protect Filipino enterprises against unfair and trades practices. The provisions on 4% tariff differential, predatory pricing and inventory requirement blocks the entry of other players and give undue advantage to the 3 oil companies resulting to monopolies or unfair competition. This is so because it would take billions for new players to construct refineries, and to have big inventories. This would effectively prevent new players. In the case at bar, it cannot be denied that our oil industry is operated and controlled by an oligopoly (dominated by a handful of players) and a foreign oligopoly at that. As the dominant players, SHELL, CALTEX & PETRON boast of existing refineries of various capacities. The tariff differential of 4% works to their immense advantage. Yet, this is only one edge on tariff differential. THE OTHER EDGE CUTS AND CUTS DEEP IN THE HEART OF THEIR COMPETITORS. IT ERECTS HIGH BARRIERS TO NE PLAYERS. New players in order to equalize must build their refineries worth billions of pesos. Those without refineries had to compete with a higher cost of 4%.They will be competing on an uneven field. The provision on inventory widens the advantage of PETRON, SHELL AND CALTEX against prospective new players. The three (3) could easily comply with the inventory requirement in view of their numerous storage facilities. Prospective competitors again find compliance oft his requirement difficult because of prohibitive cost in constructing new storage facilities. The net effect would be to effectively prohibit the entrance of new players. Now comes the prohibition on predatory pricing or selling or offering to sell any product at a price unreasonably below the industry average cost so as to attract customers to the detriment of the competitors. According to HOVENKAMP:

119

120

The rationale for predatory pricing is the sustaining of losses today that will give a firm monopoly profits in the future. The monopoly profits will never materialize, however, if the market is flooded with new entrants as soon as the successful predator attempts to raise its price. Predatory pricing will be profitable only if the market contains significant barriers to new entry. Coupled with the 4% tariff differential and the inventory requirement, the predatory pricing is a significant barrier which discourage new players to enter the oil market thereby promoting unfair competition, monopoly and restraint of trade which are prohibited by the Constitution. 2-d.LACSON VS. SANDIGANBAYAN, January 20, 1999 3. Taxicab Operators vs. BOT, September 30,l982 4. Bautista vs. Juinio,127 SCRA 329 5. Dumlao vs. COMELEC, 95 SCRA 392 6. Villegas vs. Hiu, 86 SCRA 270 7. Ceniza vs. COMELEC, 95 SCRA 763 8. UNIDO vs. COMELEC, 104 SCRA 38 9. Nunez vs. Sandiganbayan, 111 SCRA 433(Read also the dissenting opinion of Justice Makasiar 10. Sison vs. Ancheta, 130 SCRA 654 11. Citizens Surety vs. Puno, 119 SCRA 216 12. Peralta vs. COMELEC, 82 SCRA 30 13. Hawaiian-Phil. Co. vs. Asociacion, 151 SCRA 306 14. Ormoc Sugar Co. vs. Ormoc City, 22 SCRA 603 15. Flores vs. COMELEC, 184 SCRA 484

CHAPTER IV - THE SEARCH AND SEIZURE PROVISION

Section 2. The right of the people to be secure in their persons, houses, papers and effects against unreasonable searches and seizures of whatever nature and for any purpose shall be inviolable, and no search warrant or warrant of arrest shall issue except upon probable cause to be determined personally by the judge after examination under oath or affirmation of the complainant and the witnesses he may produce, and particularly describing the place to be searched and the persons or things to be seized. NOTE: Applicable provisions of the Human Security Act/AntiTerrorism Law, Republic Act No. 9372, Approved on March 6, 2007 and effective on July 15, 2007 (This Law shall be automatically suspended one (1) month before and two (2) months after the holding of any election) Sec. 18. Period of detention without judicial warrant of arrest.- The provisions of Article 125 of the Revised Penal Code, notwithstanding, any police or law enforcement personnel, who, having been duly authorized in writing by the Anti-Terrorism Council has taken custody of a person charged with or suspected of the crime of terrorism or the crime of conspiracy to commit terrorism shall, WITHOUT INCURRING ANY CRIMINAL LIABILITY FOR DELAY IN THE DELIVERY OF DETAINED
120

121

PERSONS TO THE PROPER JUDICIAL AUTHORITIES, DELIVER SAID CHARGED OR SUSPECTED PERSON TO THE PROPER JUDICIAL AUTHORITY WITHIN A PERIOD OF THREE (3) DAYS counted from the moment said charged or suspected person has been apprehended or arrested, detained, and taken into custody by the said police, or law enforcement personnel: Provided, That the arrest of those suspected of the crime of terrorism or conspiracy to commit terrorism must result from the surveillance under Section 7 and examination of bank deposits under Section 27 pf this Act. The police or law enforcement personnel concerned shall, before detaining the person suspected of the crime of terrorism, present him or her before any judge at the latters residence or office nearest the place where the arrest took place at any time of the day or night. It shall be the duty of the judge, among other things, to ascertain the identity of the police or law enforcement personnel and the person or persons they have arrested and presented before him or her, to inquire of them the reasons why they have arrested the person and determine by questioning and personal observation whether or not the subject has been subjected to any physical, moral or psychological torture by whom and why. The judge shall then submit a written report of what he/she had observed when the subject was brought before him to the proper court that has jurisdiction over the case of the person thus arrested. The judge shall forthwith submit his report within 3 calendar days from the time the suspect was brought to his/her residence or office. Immediately after taking custody of a person charged with or suspected of the crime of terrorism or conspiracy to commit terrorism, the police or law enforcement personnel shall notify in writing the judge of the court nearest the place of apprehension or arrest; provided, That where the arrest is made during Saturdays, Sundays, holidays or after office hours, the written notice shall be served at the residence of the judge nearest the place where the accused was arrested. The penalty of 10 years and 1 day to 12 years imprisonment shall be imposed upon the police or law enforcement personnel who fails to notify any judge as provided in the preceding paragraph. Section 19. Period of Detention in the event of an actual or imminent terrorist attack.- In the vent of an actual or imminent terrorist attack,, suspects may not be detained for more than three days without the written approval of a municipal, city, provincial or regional official of a Human Rights Commission, or judge of the municipal, regional trial court, the Sandiganbayan or a justice of the Court of Appeals nearest the place of arrest. If the arrest is made during Saturdays, Sundays or holidays, or after office hours, the arresting police of law enforcement personnel shall bring the person thus arrested to the residence of any of the officials mentioned above that is nearest the place where the accused was arrested. The approval in writing of any of the said officials shall be secured by the police or law enforcement personnel concerned within five days after the date of the detention of the persons concerned; Provided, however, That within three days after the
121

122

detention the suspects whose connection with the terror attack or threat is not established, shall be released immediately. Section 26 provides that persons who have been charged with terrorism or conspiracy to commit terrorism---even if they have been granted bail because evidence of guilt is not strongcan be: Detained under house arrest; Restricted from traveling; and/or Prohibited from using any cellular phones, computers, or other means of communications with people outside their residence. Section 39. Seizure and Sequestration.- The deposits and their outstanding balances, placements, trust accounts, assets, and records in any bank or financial institution, moneys, businesses, transportation and communication equipment, supplies and other implements, and property of whatever kind and nature belonging: To any person charged with or suspected of the crime of terrorism or conspiracy to commit terrorism; to a judicially declared and outlawed terrorist organization or group of persons; to a member of such judicially declared and outlawed organization, association or group of persons, -shall be seized, sequestered, and frozen in order to prevent their use, transfer or conveyance for purposes that are inimical to the safety and security of the people or injurious to the interest of the State. The accused or suspect may withdraw such sums as are reasonably needed by his family including the services of his counsel and his familys medical needs upon approval of the court. He or she may also use any of his property that is under seizure or sequestration or frozen because of his/her indictment as a terrorist upon permission of the court for any legitimate reason. Section 40. The seized, sequestered and frozen bank depositsshall be deemed property held in trust by the bank or financial institution and that their use or disposition while the case is pending shall be subject to the approval of the court before which the case or cases are pending. Section 41. If the person suspected as terrorist is acquitted after arraignment or his case dismissed before his arraignment by a competent court, the seizureshall be lifted by the investigating body or the competent court and restored to him without delay. The filing of an appeal or motion for reconsideration shall not stay the release of said funds from seizure, sequestration and freezing. If convicted, said seized, sequestered and frozen assets shall automatically forfeited in favor of the government.

122

123

Requisites of a valid search warrant Read: a. Essentials of a valid search warrant,145 SCRA 739

b. Validity of a search warrant and the admissibility of evidence obtained in violation thereof. c. The place to be searched as indicated in the warrant is controlling PEOPLE VS. CA, 291 SCRA 400

Narvasa, CJ In applying for a search warrant, the police officers had in their mind the first four (4) separate apartment units at the rear of ABIGAIL VARIETY STORE in Quezon City to be the subject of their search. The same was not, however, what the Judge who issued the warrant had in mind, AND WAS NOT WHAT WAS ULTIMATELY DESCRIBED IN THE SEARCH WARRANT. As such, any evidence obtained from the place searched which is different from that indicated in the search warrant is inadmissible in evidence for any purpose and in any proceeding. This is so because it is neither licit nor fair to allow police officers to search a place different from that stated in the warrant on the claim that the place actually searched---although not that specified in the search warrant---is exactly what they had in view when they applied for the warrant and had demarcated in their supporting evidence. WHAT IS MATERIAL IN DETERMINING THE VALIDITY OF A SEARCH IS THE PLACE STATED IN THE WARRANT ITSELF, NOT WHAT THE APPLICANTS HAD IN THEIR THOUGHTS, OR HAD REPRESENTED IN THE PROOFS THEY SUBMITTED TO THE COURT ISSUING THE WARRANT. As such, it was not just a case of obvious typographical error, but a clear case of a search of a place different from that clearly and without ambiguity identified in the search warrant. NOTE: Very Important: Where a search warrant is issued by one court and the criminal action base don the results of the search is afterwards commenced in another court, IT IS NOT THE RULE THAT A MOTION TO QUASH THE WARRANT (or to retrieve the things seized) MAY BE FILED ONLY IN THE ISSUING COURT---SUCH A MOTION MAY BE FILED FOR THE FIRST TIME IN EITHER THE ISSUING COURT OR THAT IN WHICH THE CRIMINAL PROCEEDING IS PENDING. d. Validity of a warrantless search and seizure as a result of an informers tip. Note the two (2) conflicting decisions of the Supreme Court. PEOPLE VS. ARUTA, 288 SCRA 626 On December 13, 1988, P/Lt. Abello of the Olongapo PNP was tipped off by an informer that Aling Rosa would be arriving from Baguio City the following day with a large volume of marijuana. As a result of
123

124

the tip, the policemen waited for a Victory Bus from Baguio City near the PNB Olongapo, near Rizal Ave. When the accused got off, she was pointed to by the informer. She was carrying a traveling bag at that time. She was not acting suspiciously. She was arrested without a warrant. The bag allegedly contained 8.5 kilos of marijuana. After trial, she was convicted and imposed a penalty of life imprisonment. Issue: Whether or not the marijuana allegedly taken from the accused is admissible in evidence. Held: Warrantless search is allowed in the following instances: 1. 2. 3. 4. 5. 6. customs searches; searches of moving vehicle; seizure of evidence in plain view; consented searches; search incidental to a lawful arrest; and stop and frisk measures. The above exceptions to the requirement of a search warrant, however, should not become unbridled licenses for law enforcement officers to trample upon the conditionally guaranteed and more fundamental right of persons against unreasonable search and seizures. The essential requisite of probable cause must still be satisfied before a warrantless search and seizure can be lawfully conducted. In order that the information received by the police officers may be sufficient to be the basis of probable cause, it must be based on reasonable ground of suspicion or belief a crime has been committed or is about to be committed. The marijuana obtained as a result of a warrantless search is inadmissible as evidence for the following reasons: a. the policemen had sufficient time to apply for a search warrant but they failed to do so; b. the accused was not acting suspiciously; c. the accuseds identity was previously ascertained so applying for a warrant should have been easy; d. the accused in this case was searched while innocently crossing a street Consequently, there was no legal basis for the police to effect a warrantless search of the accuseds bag, there being no probable cause and the accuseds not having been legally arrested. The arrest was made only after the accused was pointed to by the informant at a time when she was not doing anything suspicious. The arresting officers do not have personal knowledge that the accused was committing a crime at that time. Since there was no valid warrantless arrest, it logically follows that the subsequent search is similarly illegal, it being not incidental to a lawful arrest. This is so because if a search is first undertaken, and an arrest effected based on the evidence produced by the search, both such search and arrest would be unlawful, for being contrary to law.
124

125

This case is similar tot he case of PEOPLE VS. AMINNUDIN, and PEOPLE VS. ENCINADA. PEOPLE VS. MONTILLA, 284 SCRA 703 On June 19, 1994, at about 2 p.m., SPO1 Talingting and SPO1 Clarin of the Dasmarinas, Cavite PNP were informed by an INFORMER that a drug courier would be arriving in Barangay Salitran, Dasmarinas, Cavite, from Baguio City, with an undetermined amount of marijuana. The informer likewise informed them that he could recognize said person. At about 4 in the morning of June 20, 1994, the appellant was arrested by the above-named police officers while alighting from a passenger jeepney near a waiting shed in Salitran, Dasmarinas, Cavite, upon being pointed to by the informer. The policemen recovered 28 kilos of dried marijuana leaves. The arrest was without warrant. The trial court convicted the appellant for transporting marijuana based on the testimonies of the Above-named police officers without presenting the alleged informer. Issue: Was the warrantless arrest valid? Held: The accused claims that the warrantless search and seizure is illegal because the alleged information was received by the police on June 19, 1994 and therefore, they could have applied for a search warrant. The said contention is without merit considering that the information given by the informer is too sketchy and not detailed enough for the obtention of the corresponding arrest or search warrant. While there is indication that the informer knows the courier, the records do not show that he knew his name. On bare information, the police could not have secured a warrant from a judge. Furthermore, warrantless search is allowed in the following instances: 1. 2. 3. 4. 5. 6. customs searches; searches of moving vehicle; seizure of evidence in plain view; consented searches; search incidental to a lawful arrest; and stop and frisk measures. Since the accused was arrested for transporting marijuana, the subsequent search on his person is justified. An arresting officer has the right to validly search and seize from the offender (1) dangerous weapons; and (2) those that may be used as proof of the commission of the offense. In the case at bar, upon being pointed to by the informer as the drug courier, the policemen requested the accused to open and show them the contents of his bag and the cartoon he was carrying and he voluntarily
125

126

opened the same and upon cursory inspection, it was found out that it contains marijuana. Hence the arrest. The accused insists that it is normal for a person traveling with a bag and cartoon which should not elicit the slightest suspicion that he was committing a crime. In short, there was no probable cause for this policemen to think that he was committing a crime. The said contention was considered without merit by the Supreme Court considering the fact that he consented to the search as well as the fact that the informer was a reliable one who had supplied similar information to the police in the past which proved positive. (NOTE: The SC held that the non-presentation of the informer does not affect the case for the prosecution because he is not even the best witness. He is merely a corroborative witness to the arresting officers. ) JUSTICE PANGANIBAN: To say that reliable tips from informers constitute probable cause for a warrantless arrest or search IS A DANGEROUS PRECEDENT AND PLACES IN GREAT JEOPARDY THE DOCTRINES LAID DOWN IN MANY DECISIONS MADE BY THIS COURT. (PEOPLE VS. BURGOS, 144 SCRA 1; PEOPLE VS. AMINNUDIN, 163 SCRA 402; PEOPLE VS. ENCINADA, October 2, 1997; PEOPLE VS. MENGOTE, 220 SCRA). The case is similar to the case of People vs. Encimada where the appellant was searched without a warrant while disembarking from a ship on the strength of a tip from an informer received by the police the previous afternoon that the appellant would be transporting prohibited drugs. The search yielded a plastic package containing marijuana. On Appeal, the SC reversed the decision of conviction and held that Encinada did not manifest any suspicious behavior that would necessarily and reasonably invite the attention of the police. Warrantless Arrest, search and seizure in buy-bust operations.

PEOPLE OF THE PHILIPPINES VS. SPO3 SANGKI ARA, et al., G.R. No. 185011, December 23, 2009 VELASCO, JR., J.: This is an appeal from the December 13, 2007 Decision of the Court of Appeals (CA) in CA-G.R. CR-H.C. No. 00025B entitled People of the Philippines v. SPO3 Sangki Ara y Mirasol, Mike Talib y Mama, Jordan Musa y Bayan, which affirmed the Decision of the Regional Trial Court (RTC), Branch 9 in Davao City, convicting accused-appellants of violation of Republic Act No. (RA) 9165 or the Comprehensive Dangerous Drugs Act of 2002.

126

127

The Facts Criminal Case No. 51,471-2002 against Ara That on or about December 20, 2002, in the City of Davao, Philippines and within the jurisdiction of this Honorable Court, the above-named accused, without being authorized by law, willfully, unlawfully and consciously traded, transported and delivered 26.6563 grams of Methamphetamine Hydrochloride or shabu, which is a dangerous drug, with the aggravating circumstance of trading, transporting and delivering said 26.6563 grams of shabu within 100 meters from [the] school St. Peters College of Toril, Davao City. CONTRARY TO LAW. During their arraignment, accused-appellants all gave a not guilty plea. Version of the Prosecution In the morning of December 20, 2002, a confidential informant (CI) came to the Heinous Crime Investigation Section (HCIS) of the Davao City Police Department and reported that three (3) suspected drug pushers had contacted him for a deal involving six (6) plastic sachets of shabu. He was instructed to go that same morning to St. Peters College at Toril, Davao City and look for an orange Nissan Sentra car. Police Chief Inspector Fulgencio Pavo, Sr. immediately formed a buy-bust team composed of SPO3 Reynaldo Capute, SPO4 Mario Galendez, SPO3 Antonio Balolong, SPO2 Arturo Lascaos, SPO2 Jim Tan, SPO1 Rizalino Aquino, SPO1 Bienvenido Furog, PO2 Vivencio Jumawan, Jr., PO2 Ronald Lao, and PO1 Enrique Ayao, Jr., who would act as poseur-buyer. The team proceeded to the school where PO1 Ayao and the CI waited by the gate. At around 8:45 a.m., an orange Nissan Sentra bearing plate number UGR 510 stopped in front of them. The two men approached the vehicle and the CI talked briefly with an old man in the front seat. PO1 Ayao was then told to get in the back seat as accused-appellant Mike Talib opened the door. The old man, later identified as accused-appellant SPO3 Ara, asked PO1 Ayao if he had the money and the latter replied in the positive. Ara took out several sachets with crystalline granules from his pocket and handed them to PO1 Ayao, who thereupon gave the pre-arranged signal of opening the car door. The driver of the car, later identified as accused-appellant Jordan Musa, tried to drive away but PO1 Ayao was able to switch off the car engine in time. The back-up team appeared and SPO1 Furog held on to Musa while PO2 Lao restrained Talib. PO1 Ayao then asked Ara to get out of the vehicle. Recovered from the group were plastic sachets of white crystalline substance: six (6) big sachets, weighing 26.6563 grams, from Ara by PO1 Ayao; five (5) big sachets, weighing 14.2936 grams, from Musa by SPO1 Furog; and a small sachet, weighing 0.3559 gram, from Talib by PO2 Lao. The three suspects were brought to the HCIS and the seized items indorsed to the Philippine National Police (PNP) Crime Laboratory for examination. Forensic Chemist Austero, who conducted the examination, found that the confiscated sachets all tested positive for shabu.
127

128

Version of the Defense The defense offered the sole testimony of Ara, who said that he had been a member of the PNP for 32 years, with a spotless record. On December 20, 2002, SPO3 Ara was in Cotabato City, at the house of his daughter Marilyn, wife of his co-accused Musa. He was set to go that day to the Ombudsmans Davao City office for some paperwork in preparation for his retirement on July 8, 2003. He recounted expecting at least PhP 1.6 million in retirement benefits. Early that morning, past three oclock, he and Musa headed for Davao City on board the latters car. As he was feeling weak, Ara slept in the back seat. Upon reaching Davao City, he was surprised to see another man, Mike Talib, in the front seat of the car when he woke up. Musa explained that Talib had hitched a ride on a bridge they had passed. When they arrived in Toril, Ara noticed the car to be overheating, so they stopped. Ara did not know that they were near St. Peters College since he was not familiar with the area. Talib alighted from the car and Ara transferred to the front seat. While Talib was getting into the back seat, PO1 Ayao came out of nowhere, pointed his .45 caliber pistol at Ara even if he was not doing anything, and ordered him to get off the vehicle. He saw that guns were also pointed at his companions. As the group were being arrested, he told PO1 Ayao that he was also a police officer. Ara insisted that he was not holding anything and that the shabu taken from him was planted. He asserted that the only time he saw shabu was on television. The Ruling of the Trial Court The dispositive portion of the RTC Decision reads: WHEREFORE, premised on the foregoing the Court finds the following: In Criminal Case No. 51,471-2002, the accused herein SANGKI ARA Y MASOL, Filipino, 55 years old, widower, a resident of Kabuntalan, Cotabato City, is hereby found GUILTY beyond reasonable doubt, and is CONVICTED of the crime of violation of Sec. 5, 1st paragraph of Republic Act 9165. He is hereby imposed the DEATH PENALTY and FINE of TEN MILLION PESOS (PhP 10,000,000) with all the accessory penalties corresponding thereto, including absolute perpetual disqualification from any public office, in view of the provision of section 28 of RA 9165 quoted above. Since the prosecution proved beyond reasonable doubt that the crime was committed in the area which is only five (5) to six (6) meters away from the school, the provision of section 5 paragraph 3 Article II of RA 9165 was applied in the imposition of the maximum penalty against the herein accused. In Criminal Case No. 51,472-2002, the accused herein MIKE TALIB y MAMA, Filipino, of legal age, single and a resident of Parang, Cotabato, is found GUILTY beyond reasonable doubt, and is CONVICTED of the crime of violation of Sec. 11, 3rd paragraph, Article II of Republic Act 9165. He is hereby imposed a penalty of Imprisonment of SIXTEEN (16) YEARS and
128

129

a fine of THREE HUNDRED THOUSAND PESOS (PhP 300,000) with all the accessory penalties corresponding thereto. In Criminal Case No. 51,473-2002 the accused herein JORDAN MUSA Y BAYAN, Filipino, 30 years old, married and a resident of Cotabato City, is hereby found GUILTY beyond reasonable doubt and is CONVICTED of the crime for Violation of Sec. 11, 1st paragraph, Article II of Republic Act No. 9165. He is hereby sentenced to suffer a penalty of LIFE IMPRISONMENT and FINE of FOUR HUNDRED THOUSAND PESOS (PhP 400,000) with all the accessory penalties corresponding thereto. SO ORDERED. As the death penalty was imposed on Ara, the case went on automatic review before this Court. Conformably with People v. Mateo, we, however, ordered the transfer of the case to the CA. The Issue Whether the Court of Appeals erred in holding that the arrest of the accused-appellants was valid based on the affidavits of the complaining witnesses Warrantless Arrest and Seizure Valid In calling for their acquittal, accused-appellants decry their arrest without probable cause and the violation of their constitutional rights. They claim that the buy-bust team had more than a month to apply for an arrest warrant yet failed to do so. Owing to the special circumstances surrounding the drug trade, a buy-bust operation has long been held as a legitimate method of catching offenders. It is a form of entrapment employed as an effective way of apprehending a criminal in the act of commission of an offense. We have ruled that a buy-bust operation can be carried out after a long period of planning. The period of planning for such operation cannot be dictated to the police authorities who are to undertake such operation. It is unavailing then to argue that the operatives had to first secure a warrant of arrest given that the objective of the operation was to apprehend the accused-appellants in flagrante delicto. In fact, one of the situations covered by a lawful warrantless arrest under Section 5(a), Rule 113 of the Rules of Court is when a person has committed, is actually committing, or is attempting to commit an offense in the presence of a peace officer or private person. It is erroneous as well to argue that there was no probable cause to arrest accused-appellants. Probable cause, in warrantless searches, must only be based on reasonable ground of suspicion or belief that a crime has been committed or is about to be committed. There is no hard and fast rule or fixed formula for determining probable cause, for its determination varies according to the facts of each case. Probable cause was provided by information gathered from the CI and from accused-appellants themselves when they instructed PO1 Ayao to enter their vehicle and begin the transaction. The illegal sale of shabu inside accusedappellants vehicle was afterwards clearly established. Thus, as we have previously held, the arresting officers were justified in making the arrests as accused-appellants had just committed a crime when Ara sold shabu to PO1 Ayao. Talib and Musa were also frisked for contraband as it may be logically inferred that they were also part of Aras drug activities inside the vehicle. Thi s
129

130

inference was further strengthened by Musas attempt to drive the vehicle away and elude arrest. Moreover, the trial court correctly denied the Motion to Suppress or Exclude Evidence. We need not reiterate that the evidence was not excluded since the buy-bust operation was shown to be a legitimate form of entrapment. The pieces of evidence thus seized therein were admissible. As the appellate court noted, it was within legal bounds and no anomaly was found in the conduct of the buy-bust operation. There is, therefore, no basis for the assertion that the trial courts order denying said motion was biased and committed with grave abuse of discretion.

ELI LUI, ET AL. VS. MATILLANO, May 27, 2004 Right against unreasonable searches and seizures; Mission Order does not authorize an illegal search. Waiver of the right against an unreasonable search and seizure. In search of the allegedly missing amount of P45,000.00 owned by the employer, the residence of a relative of the suspect was forcibly open by the authorities by kicking the kitchen door to gain entry into the house. Thereafter, they confiscated different personal properties therein which were allegedly part of those stolen from the employer. They were in possession of a mission order but later on claimed that the owner of the house gave his consent to the warrantless search. Are the things admissible in evidence? Can they be sued for damages as a result of the said warrantless search and seizure? Held: The right against unreasonable searches and seizures is a personal right which may be waived expressly or impliedly. BUT A WAIVER BY IMPLICATION CANNOT BE PRESUMED. There must be clear and convincing evidence of an actual intention to relinquish the right. There must be proof of the following: a. that the right exists; b. that the person involved had knowledge, either constructive or actual, of the existence of said right; c. that the said person had an actual intention to relinquish the right. Finally, the waiver must be made voluntarily, knowingly and intelligently in order that the said is to be valid. The search was therefore held illegal and the members of the searching party held liable for damages in accordance with the doctrine laid down in Lim vs. Ponce de Leon and MHP Garments vs. CA. e. General or roving warrants Read: 1. Stonehill vs. Diokno,June 19,1967 Concepcion, C.J.
130

131

The petitioners are questioning the validity of a total of 42 search warrants issued on different dates against them and the corporations in which they are officers, directing the peace officer to search the persons above-named and/or the premises of their offices, warehouses and to seize and take possession of the following personal property, to wit: "Books of accounts, financial records, vouchers, correspondence, receipts, ledgers, journals, typewriters and other documents or papers showing all business transactions including disbursement receipts, balance sheets and profit and loss statements" since they are the subject of the offense of violating the CENTRAL BANK LAWS, TARIFF AND CUSTOMS LAWS, INTERNAL REVENUE CODE AND THE REVISED PENAL CODE. The petitioners claim that the search warrants are void being violative of the Constitutional provision on search and seizure on the ground that: a. The search warrants did not particularly describe the documents, books and things to be seized; b. cash money not mentioned in the warrant were actually seized; c. The warrants were issued to fish evidence in the deportation cases against them; d. the searches and seizures were made in an illegal manner; e. the things seized were not delivered to the court to be disposed of in a manner provided for by law. Issue: Were the searches and seizures made in the offices and residences of the petitioners valid? a. As to the searches made on their offices, they could not question the same in their personal capacities because the corporations have a personality separate and distinct with its officers. An objection to an unlawful search and seizure IS PURELY PERSONAL AND CANNOT BE AVAILED OF BY THIRD PARTIES. CONSEQUENTLY, THE PETITIONERS MAY NOT VALIDLY OBJECT TO THE USE IN EVIDENCE AGAINST THEM OF THE DOCUMENTS, PAPERS AND THINGS SEIZED FROM THE OFFICES AND PREMISES OF THE CORPORATIONS, TO WHOM THE SEIZED EFFECTS BELONG, AND MAY NOT BE INVOKED BY THE CORPORATE OFFICERS IN PROCEEDINGS AGAINST THEM IN THEIR INDIVIDUAL CAPACITY. b. As to the documents seized in the residences of the petitioners, the same may not be used in evidence against them because the warrants issued were in the nature of a general warrant for failure to comply with the constitutional requirement that:

131

132

1. that no warrant shall issue but upon probable cause, to be determined by the judge in the manner set forth in said provision; and 2. that the warrant shall particularly describe the things to be seized. None of these requirements has been complied with in the contested warrants. They were issued upon applications stating that the natural and juridical persons therein named had committed a violation of Central bank Laws, Tariff and Customs Laws, Internal revenue Code and Revised Penal Code. IN OTHER WORDS, NO SPECIFIC OFFENSE HAD BEEN ALLEGED IN SAID APPLICATIONS. THE AVERMENTS THEREOF WITH RESPECT TO THE OFFENSE COMMITTED WERE ABSTRACT. AS A CONSEQUENCE, IT WAS IMPOSSIBLE FOR THE JUDGE WHO ISSUED THE WARRANTS TO HAVE FOUND THE EXISTENCE OF PROBABLE CAUSE, FOR THE SAME PRESUPPOSES THE INTRODUCTION OF COMPETENT PROOF THAT THE PARTY AGAINST WHOM IT IS SOUGHT HAS PERFORMED PARTICULAR ACTS, OR COMMITTED SPECIFIC OMISSIONS, VIOLATING A GIVEN PROVISION OF OUR CRIMINAL LAWS. 2. Bache vs. Ruiz, 37 SCRA 823 3. Secretary vs. Marcos, 76 SCRA 301 4. Castro vs. Pabalan, April 30,l976 5. Asian Surety vs. Herrera, 54 SCRA 312 (A search warrant for estafa, falsification, tax evasion and insurance fraud is a general warrant and therefore not valid) 6. Collector vs. Villaluz, June 18,1976 7. Viduya vs. Verdiago, 73 SCRA 553 8. Dizon vs. Castro, April 12, 1985 9. People vs. Veloso, 48 Phil. 169 10. TAMBASEN VS. PEOPLE, July 14, 1995; PEOPLE VS. CA, 216 SCRA 101. A SCATTER-SHOT WARRANT is a search warrant issued for more than one specific offense like one for estafa, robbery, theft and qualified theft) f. Define probable cause. Who determines probable cause? b. ROBERTS VS. CA, 254 SCRA 307 c. DE LOS SANTOS VS. MONTESA, 247 SCRA 85 VICENTE LIM,SR. AND MAYOR SUSANA LIM VS.HON. N. FELIX (G.R. NO. 99054-57) EN BANC GUTIERREZ, JR. J. Facts: -------Petitioners are suspects of the slaying of Congressman Moises Espinosa, Sr. and three of his security escorts and the wounding of another. They were initially charged, with three others, with the crime of multiple murder with frustrated murder. After conducting a preliminary investigation, a warrant of arrest was issued on July 31, 1989. Bail was fixed at P200,000.
132

133

On September 22, 1989, Fiscal Alfane, designated to review the case, issued a Resolution affirming the finding of a prima facie case against the petitioners but ruled that a case of Murder for each of the killing of the four victims and a physical injuries case for inflicting gunshot wound on the survivor be filled instead against the suspects. Thereafter, four separate informations to that effect were filed with the RTC of Masbate with no bail recommended. On November 21, 1989, a motion for change of venue, filed by the petitioners was granted by the SC. It ordered that the case may be transferred from the RTC of Masbate to the RTC of Makati. Petitioners then moved that another hearing ba conducted to determine if there really exists a prima facie case against them in the light of documents showing recantations of some witnesses in the preliminary investigation. They likewise filed a motion to order the transmittal of initial records of the preliminary investigation conducted by the municipal judge of Barsaga of Masbate. These motions were however denied by the court because the prosecution had declared the existence of probable cause, informations were complete in form in substance , and there was no defect on its face. Hence it found it just and proper to rely on the prosecutors certification in each information. ISSUE: ---------Whether or not a judge may issue a warrant of arrest without bail by simply relying on the prosecutions certification and recommendation that a probable cause exists? Held: ----1. The judge committed a grave abuse of discretion. In the case of Placer vs. Villanueva, the sc ruled that a judge may rely upon the fiscal's certification of the existence of a probable cause and on the basis thereof, issue a warrant of arrest. However, the certification does not bind the judge to come out with the warrant of arrest. This decision interpreted the "search and seizure" provision of the 1973 Constitution. Under this provision, the judge must satisfy himself of the existence of probable cause before issuing a warrant of order of arrest. If on the face of information, the judge finds no probable cause, he may disregard the fiscal's certification and require the submission of the affidavits of witness to aid him at arriving at a conclusion as to the existence of a probable cause. This has been the rule since U.S vs. Ocampo and Amarga vs. Abbas. 2. In the case of Soliven vs. Makasiar, decided under the 1987 Constitution, the Court noted that the addition of the word personally after the word determined and the deletion of the grant of authority by the 1973 Constitution to issue warrants to other respondent officers as to may be authorized by law does not require the judge to personally examine the complainant and his witness in his determination of probable cause for the issuance of a warrant of arrest.What the Constitution underscores is the exclusive and personal responsibility of the issuing judge to satisfy
133

134

himself of the existence of probable cause. Following established doctrine and procedures, he shall: (1) personally evaluate the reports and the supporting documents submitted by the fiscal regarding the existence of probable cause and, on the basis thereof, issue a warrant of arrest; (2) If on the basis thereof he finds no probable cause, he may disregard the fiscal's report and require the submission of supporting affidavits of witnesses to aid him in arriving at a conclusion as to the existence of probable cause. 3. The case of People vs. Honorable Enrique B. Inting reiterates the following doctrines: (1) The determination of probable cause is a function of the judge. It is not for the Provincial Fiscal or Prosecutor nor for the Election Supervisor to ascertain. Only the judge alone makes this detemination. (2) The preliminary inquiry made by the prosecutor does not bind the judge. It merely assist him to make the determination of probable cause. The judge does not have to follow what the prosecutor's present to him. By itself, the prosecutor's certification of probable cause is ineffectual. It is the report, the affidavits, the transcripts of stenographic notes, and all other supporting documents behind the prosecutor's certification which are material in assisting the judge to make his determination. (3) Preliminary inquiry should be distinguished from the preliminary investigation proper. While the former seeks to determine probable cause for the issuance of warrant of arrest, the latter ascertains whether the offender should be held for trial or be released. 4. In the case of Castillo vs. Villaluz, the court ruled that judges of RTC no longer have authority to conduct preliminary investigations: This authority was removed from them by the 1985 Rules on Criminal Procedure, effective on January 1, 1985. 5. In the present case, the respondent judge relies solely on the certification of the prosecutor. Considering that all the records of the investigation are in Masbate, he has not personally determined the existence of probable cause. The determination was made by the provincial prosecutor. The constitutional requirement had not been satisfied. The records of the preliminary investigation conducted by the Municipal Court of Masbate and reviewed by the respondent Fiscal were still in Masbate when the respondent Fiscal issued the warrant of arrest against the petitioners. There was no basis for the respondent judge to make his personal determination regarding the existence of probable cause from the issuance of warrant of arrest as mandated by the Constitution. He could not have possibly known what has transpired in Masbate as he had nothing but a certification. Although the judge does not have to personally examine the complainant and his witnesses (for the prosecutor can perform the same functions as commissioner for taking of evidence) there should be a report and necessary documents supporting the Fiscal's bare certification. All of these should be before the judge.
134

135

1. Amarga vs. Abbas, 98 Phil. 739 1-a. 20th Century Fox vs. CA, 164 SCRA 655 1-b. Quintero vs. NBI, 162 SCRA 467 1-c. The Presidential Anti-Dollar Salting Task Force vs. CA, GR No. 83578, March 16, 1989 SOLIVEN VS. MAKASIAR, 167 SCRA 393 The word personally after the word determined does not necessarily mean that the judge should examine the complainant and his witnesses personally before issuing the search warrant or warrant of arrest but the exclusive responsibility on the part of said judge to satisfy himself of the existence of probable cause. As such, there is no need to examine the complainant and his witnesses face to face. It is sufficient if the judge is convinced of the existence of probable cause upon reading the affidavits or deposition of the complainant and his witnesses. 1-e. Pendon vs. CA, Nov. 16, 1990 1-f. P. vs. Inting, July 25, 1990 1-g. Umil vs. Ramos, et al., July 9, 1990 with the Resolution of the Motion for Reconsideration in November, 1991 1-h. Paderanga vs. Drilon, April 19, 1991 2. Department of Health vs. Sy Chi Siong, Inc., GR No. 85289, February 20, 1989 2-a. P. vs. Villanueva, 110 SCRA 465 2-b. Placer vs. Villanueva, 126 SCRA 463 (Only a judge has the power to determine probable insofar as the issuance of a warrant of arrest is concerned) 3. Tolentino vs. Villaluz,July 27,1987 4. Cruz vs. Gatan, 74 SCRA 226 5. Olaes vs. P., 155 SCRA 486 7. Geronimo vs. Ramos, 136 SCRA 435 JUAN PONCE ENRILE VS. JUDGE JAIME SALAZAR, ET AL., G.R.NO. 92163, June 5, 1990 Due process; right to bail; probable cause for the issuance of a warrant of arrest (Note: This might be useful also in your Criminal Law) Narvasa, J. On February 27, 1990, Senator Juan Ponce Enrile was arrested by law enforcement officers led by NBI Director Alfredo Lim on the strength of a warrant of arrest issued by the respondent judge, HON. JAIME SALAZAR, Regional trial Court, Branch 103, Quezon City in Criminal Case No. 90-10941. The warrant was issued on an information signed and filed earlier in the day by Senior State Prosecutor AURELIO TRAMPE charging Senator Enrile, the spouses Rebecco and Erlinda Panlilio, and Gregorio Honasan with the crime of rebellion with murder and multiple frustrated murder allegedly committed during the period of the failed coup attempt from November 29 to December 10, 1990. Senator Enrile was taken to and held overnight at the NBI Headquarters on Taft Ave., Manila, WITHOUT BAIL, NONE HAVING BEEN RECOMMENDED IN THE
135

136

INFORMATION AND NONE FIXED IN THE WARRANT OF ARREST. On February 28, 1990, petitioner through counsel filed a petition for Habeas Corpus alleging that he was deprived of his constitutional rights in being, or having been: a. held to answer for a criminal offense which does not exist in the statute books; b. charged with a criminal offense in an information for which no complaint was initially filed or preliminary investigation was conducted, hence, he was denied due process; c. denied the right to bail; and d. arrested or detained on the strength of warrant issued without the judge who issued it first having personally determined the existence of probable cause. HELD: The parties' oral and written arguments presented the following options: 1. Abandon the Hernandez Doctrine and adopt the dissenting opinion of Justice Montemayor that "rebellion cannot absorb more serious crimes"; 2. Hold Hernandez Doctrine applicable only to offenses committed in furtherance, or as necessary means for the commission, of rebellion, BUT NOT TO ACTS COMMITTED IN THE COURSE OF A REBELLION WHICH ALSO CONSTITUTE COMMON CRIMES OF GRAVE OR LESS GRAVE CHARACTER; 3. Maintain Hernandez Doctrine as applying to make rebellion absorb all other offenses committed in its course, whether or not necessary to its commission or in furtherance thereof. 1. On the first option, 11 justices voted AGAINST abandoning Hernandez. Two members felt that the doctrine should be re-examined. In view of the majority, THE RULING REMAINS GOOD LAW, ITS SUBSTANTIVE AND LEGAL BASES HAVE WITHSTOOD ALL SUBSEQUENT CHALLENGES AND NO NEW ONES ARE PRESENTED HERE PERSUASIVE ENOUGH TO WARRANT A COMPLETE REVERSAL. This is so because of the fact that the incumbent President (exercising legislative powers under the 1986 Freedom Constitution) repealed PD No. 942 which added a new provision of the Revised Penal Code, particularly Art. 142-A which sought to nullify if not repealed the Hernandez Doctrine. In thus acting, the President in effect by legislative fiat reinstated the Hernandez as a binding doctrine with the effect of law. The Court can do no less than accord it the same recognition, absent any sufficiently powerful reason against so doing. 2. On the second option, the Supreme Court was unanimous in voting to reject the same though four justices believe that the arguments in support thereof is not entirely devoid of merit.
136

137

3. With the rejection of the first two options, the Hernandez Doctrine remains a binding doctrine operating to prohibit the complexing of rebellion with any other offense committed on the occasion thereof, either as a means necessary to its commission or as unintended effect of an activity that constitutes rebellion. On the issues raised by the petitioner: a. By a vote of 11-3, the Court ruled that the information filed against the petitioner does in fact charge an offense despite the objectionable phrasing that would complex rebellion with murder and multiple frustrated murder, that indictment is to be read as charging SIMPLE REBELLION. The petitioner's contention that he was charged with a crime that does not exist in the statute books, WHILE TECHNICALLY CORRECT SO FAR AS THE COURT RULED THAT REBELLION MAY NOT BE COMPLEXED WITH OTHER OFFENSES COMMITTED ON THE OCCASION THEREOF, MUST THEREFORE BE DISMISSED AS A MERE FLIGHT OF RHETORIC. Read in the context of Hernandez, the information does indeed charge the petitioner with a crime defined and punished by the Revised Penal Code: SIMPLE REBELLION. b. Was the petitioner charged without a complaint having been initially filed and/or preliminary investigation conducted? The record shows that a complaint for simple rebellion against petitioner was filed by the NBI Director and that based on the strength of said complaint a preliminary investigation was conducted by the respondent prosecutors culminating in the filing of the questioned information. THERE IS NOTHING INHERENTLY IRREGULAR OR CONTRARY TO LAW IN FILING AGAINST A RESPONDENT AN INDUCTMENT FOR AN OFFENSE DIFFERENT FROM WHAT IS CHARGED IN THE INITIATORY COMPLAINT, IF WARRANTED BY THE EVIDENCE DEVELOPED DURING THE PRELIMINARY INVESTIGATION. c. The petitioner claims that the warrant issued is void because it was issued barely one hour and twenty minutes after the case was raffled to the respondent judge which could hardly gave him sufficient time to personally go over the voluminous records of the preliminary investigation. Also, the petitioner claims that the respondent judge issued the warrant for his arrest without first personally determining the existence of probable cause by examining under oath or affirmation the complainant and his witnesses, in violation of Art. III, Section 2, of the Constitution. This Court has already ruled that it is not unavoidable duty of the judge to make such a personal examination, it being sufficient that he follows established procedure by PERSONALLY EVALUATING THE REPORT AND THE SUPPORTING DOCUMENT SUBMITTED BY THE PROSECUTOR. MEREBY BECAUSE SAID RESPONDENT JUDGE HAD WHAT SOME MIGHT CONSIDER ONLY A RELATIVELY BRIEF PERIOD WITHIN WHICH TO COMPLY WITH THAT DUTY , GIVES NO REASON TO ASSUME THAT HE HAD NOT, OR COULD NOT HAVE, SO COMPLIED; NOR DOES THAT SINGLE CIRCUMSTANCE SUFFICE TO OVERCOME THE LEGAL PRESUMPTION THAT OFFICIAL DUTY HAS BEEN REGULARLY PERFORMED. d. Petitioner also claims that he is denied of his constitutional right to bail. In the light of the Court's affirmation of Hernandez as applicable to petitioner's case, and of the logical and necessary corollary that the
137

138

information against him should be considered as charging only the crime of simple rebellion which is bailable before conviction, THAT MUST NOW BE ACCEPTED AS A CORRECT PROPOSITION. NOTES: This might be useful also in your Remedial Law. Was a petition for Habeas Corpus before the Supreme Court the appropriate vehicle for asserting a right to bail or vindicating its denial? The Supreme Court held that the criminal case before the respondent judge is the normal venue for invoking the petitioner's right to have provisional liberty pending trial and judgment. The correct course was for the petitioner to invoke that jurisdiction by filing a petition to be admitted to bail, claiming a right to bail per se or by reason of the weakness of the evidence against him. ONLY AFTER THAT REMEDY WAS DENIED BY THE TRIAL COURT SHOULD THE REVIEW JURISDICTION OF THE SUPREME COURT BE INVOKED, AND EVEN THEN, NOT WITHOUT FIRST APPLYING TO THE COURT OF APPEALS IF APPROPRIATE RELIEF WAS ALSO AVAILABLE THERE. Even assuming that the petitioner's premise that the information charges a non-existent crime would not excuse or justify his improper choice of remedies. Under either hypothesis, the obvious recourse would have been a motion to quash brought in the criminal action before the respondent judge. g. Warrantless searches and seizures--when valid or not. Is "Operation Kapkap" valid? Read: PEOPLE VS. MENGOTE, G.R. No. 87059, June, 1992, 210 SCRA 174 Warrantless search and seizure Cruz, J. Facts| 1. On August 8, 1987, the Western Police District received a telephone call from an informer that there were three suspicious-looking persons at the corner of Juan Luna and North Bay Blvd., in Tondo, Manila; 2. When the surveilance team arrived therein, they saw the accused "looking from side to side" and "holding his abdomen". They approched these persons and identified themselves as policement that is why they tried to ran away because of the other lawmen, they were unable to escape; 3. After their arrest, a .38 cal. Smith and Wessor revolver was confiscated from the accused and several days later, an information for violation of PD 1866 was filed against him;

138

139

4. After trial, Mengote was convicted of having violated PD 1866 and was sentenced to suffer reclusion perpetua based on the alleged gun as the principal evidence. Hence this automatic appeal. Issue: Was there a valid warrantless search and seizure? Held: There is no question that evidence obtained as a result of an illegal search or seizure is inadmissible in any proceeding for any purpose. That is the absolute prohibition of Article III, Section 3 [2], of the Constitution. This is the celebrated exclusionary rule based on the justification given by Justice Learned Hand that "only in case the prosecution, which itself controls the seizing officials, knows that it cannot profit by their wrong will the wrong be repressed." Section 5, Article 113 of the Rules of Court provides: Sec. 5. Arrest without warrant; when lawful.- A peace officer or private person may, without warrant, arrest a person: (a) When, in his presence, the person to be arrested has committed, is actually committing, or is attempting to commit an offense; (b) When an offense has in fact just been committed, and he has personal knowledge of facts indicating that the person to be arrested has committed it; and (c) When the person to be arrested is a prisoner who has escaped from a penal establishment or place where he is serving final judgment or temporarily confined while his case is pending, or has escaped while being transferred from one confinement to another. x x x We have carefully examined the wording of this Rule and cannot see how we we can agree with the prosecution. Par. (c) of Section 5 is obviously inapplicable as Mengote was not an escapee from a penal institution when he was arrested. We therefore confine ourselves to determining the lawfulness of his arrest under either Par. (a) or Par. (b) of this Section. Par. (a) requires that the person be arrested (1) after he has committed or while he is actually committing or is at least attempting to commit an offense, (2) in the presence of the arresting officer. These requirements have not been established in the case at bar. At the time of the arrest in question, the accused-appellant was merely "looking from side to side" and "holding his abdomen," according to the arresting officers themselves. There was apparently no offense that had just been committed or was being actually committed or at least being attempted by Mengote in thie presence.

139

140

The Solicitor General submits that the actual existence of an offense was not necessary as long as Mengote's acts created a reasonable suspicion on the part of the arresting officers and induced in them the belief that an offense had been committed and that accused-appellant had committed it". The question is, What offense? What offense could possibly have been suggested by a person "looking from side to side" and "holding his abdomen" and in aplace not exactly forsaken. These are certainly not sinister acts. And the setting of the arrest made them less so, if at all. It might have been different if Mengote had been apprehended at an unholy hour and in a place where he had no reason to be, like a darkened alley at 3 o'clock in the morning. But he was arrested at 11:30 in the morning and in a crowded street shortly after alighting from a passenger jeep with his companion.He was not skulking in the shadows but walking in the clear light of day. There was nothing clandestine about his being on that street at that busy hour in the blaze of the noonday sun. On the other hand, there could have been a number of reasons, all of them innoent, why hiseyes were darting from side to sideand he was holding his abdomen. If they excited suspicion in the minds of the arresting officers, as the prosecution suggests, it has nevertheless not been shown what their suspicion was all about. xxx The case before us is different because there was nothing to support the arresting officers' suspicion other than Mengote's darting eyes and his hand on his abdomen. By no stretch of the imagination could it have been inferred from these acts that an offense had just been committed, or was actually being committed, or was at least being attempted in their presence. This is similar to PEOPLE vs. AMMINUIDIN, 163 SCRA 402 where the Court held that a warrantless arrest of the accused was unconstitutional. This was effected while he was coming down the vessel, to all appearances no less innocent than the other disembarking passengers. He had not committed nor was actually committing or attempting to commit an offense in the presence of the arresting officers. He was not even acting suspiciously. In short, there was no probable cause that, as the prosecution incorrectly suggested, dispensed with the constitutional requirement of a warrant. Par. (b) is no less applicable because its no less stringent requirements have also not been satisfied. Theprosecution has not shown that at the time of Mengote's arrest an offense had in fact been committed and that the arresting officers had personal knowldge of facts indicating that Mengote had committed it. All they had was hearsay information from the telephone caller, and about a crime that had yet to bem committed. xxx In the landmark case of People vs. Burgos, 144 SCRA 1, this Court declared:

140

141

Under Section 6(a) of Rule 113, the officer arresting a person who has just committed, is committing, or is about to commit an offense must have personalknowledge of that fact. The offense must also be committed in his presence or within his view. (SAYO vs. CHIEF OF POLICE, 80 Phil. 859). xxx In arrests without a warrant under Section 6(b), however, it is not enough that there is reasonable ground to believe that the person to be arrested has committed a crime. A crime must in fact or actually have been committed first. That a crime has actually been committed is an essential precondition. It is not enough to suspect that a crime may have been committed. The fact of the commission of the offense must be undisputed. The test of reasonable ground applies only to the identity of the perpetrator.. This doctrine was affirmed in Alih vs. Castro, 151 SCRA 279, thus: If the arrest was made under Rule 113, Section 5, of the Rules of Court in connection with a crime about to be committed, being committed, or just committed, what was that crime? There is no allegation in the record of such a falsification. Parenthetically, it may be observed that under the Revised Rule 113, Section 5(b), the officer making the arrest must have personal knowledge of the ground therefor as stressed in the recent case of People vs. Burgos. It would be a sad day, indeed, if any person could be summarily arrested and searched just because he is holding his abdomen, even if it be possibly because of a stomach-ache, or if a peace officer could clamp handcuffs on any person with a shifty look on suspicion that he may have committed a criminal act is actually committing or attempting it. This simply cannot be done in a free society. This is not a police state where order is exalted over liberty or, worse, personal malice on the part ofthe arresting officer may be justified in the name of security. xxx The court feels that if the peace officers had been more mindful of the provisions of the Bill of Rights, the prosecution of the accusedappellant might have succeeded. As it happened, they allowed their over zealousness to get the better of them, resulting in their disregard of the requirements of a valid search and seizure that rendered inadmissible the evidence they had invalidly seized. This should be a lesson to other peace officers. Their impulsiveness may be the very cause of the acquittal of persons who deserve to be convicted, escaping the clutches of the law, because, ironically enough, it has not been observed by those who are supposed to enforce it. When illegal arrest is deemed waived. Warrantless arrest; no personal knowledge of the arresting officer PEOPLE VS. GALVEZ, 355 SCRA 246
141

142

Mendoza, J.

However, by entering a plea of not guilty during the arraignment, the accused-appellant waived his right to raise the issue of illegality of his arrest. IT IS NOW SETTLED THAT OBJECTION TO A WARRANT OF ARREST OR THE PROCEDURE BY WHICH A COURT ACQUIRES JURISDICTION OVER THE PERSON OF AN ACCUSED MUST BE MADE BEFORE HE ENTERS HIS PLEA, OTHERWISE, THE OBJECTION IS DEEMED WAIVED. THE FACT THAT THE ARREST WAS ILLEGAL DOES NOT RENDER THE SUBSEQUENT PROCEEDINGS VOID AND DEPRIVE THE STATE OF ITS RIGHT TO CONVICT THE GUILTY WHEN ALL THE FACTS POINT TO THE CULPABILITY OF THE ACCUSED. g-1. Warrantless Search and seizure by a private person. (Valid since the constitutional provision is not applicable to him; when it is not valid) Read: 1. PEOPLE VS. MENDOZA, 301 SCRA 66 Warrantless searches and seizures by private individuals 2. SILAHIS INTERNATIONAL HOTEL, INC. VS. ROGELIO SOLUTA, ET AL., 482 SCRA 660 Carpio-Morales, J. The petitioner suspects that the respondents who are officers of the Silahis International Hotel Union were using the Union Office located inside the hotel in the sale or use of marijuana, dollar smuggling, and prostitution. They arrived at the said conclusion through surveillance. In the morning of January 11, 1988, while the respondent union officer was opening the Union Office, security officers of the plaintiff entered the union office despite objections thereto by forcibly opening the same. Once inside the union office they started to make searches which resulted in the confiscation of a plastic bag of marijuana. An information for violation of the dangerous drugs act was filed against the respondent before the RTC of Manila which acquitted them on the ground that the search conducted was illegal since it was warrantless and without consent by the respondents. After their acquittal, the respondents filed a case for Malicious Prosecution against the petitioner for violation of Art. 32 of the Civil Code. After trial, the Regional Trial Court held that petitioners are liable
142

143

for damages as a result of an illegal search. The same was affirmed by the Court of Appeals. Issue: Whether the warrantless search conducted by the petitioners (private individual and corporation) on the union office of the private respondents is valid. Held: The search is not valid and they are civilly liable under Art. 32 of the Civil Code. The fact that the union office is part of the hotel owned by the petitioners does not justify the warrantless search. The alleged reports that the said union office is being used by the union officers for illegal activities does not justify their acts of barging into the said office without the consent of the union officers and without a search warrant. If indeed there was surveillance made, then they should have applied for a search warrant. The ruling in People vs. Andre Marti is not applicable here because in Marti, a criminal case, the issue was whether an act of a private individual, allegedly in violation of ones constitutional rights may be invoked against the State. In other words, the issue in Marti is whether the evidence obtained by a private person acting in his private capacity without the participation of the State, is admissible. 3. PEOPLE OF THE PHILIPPINES VS. ANDRE MARTI G.R. NO. 81561, January 18, 1991 Warrantless Search and seizure by a private person Bidin, J. FACTS: Andre Marti and his common-law wife, Shirley Reyes went to Manila Packaging and Export Forwarders to send four (4) packages to Zurich, Switzerland. Anita Reyes, owner of the place (no relation to Shirley), received said goods and asked if she could examine and inspect it. Marti refused. However later, following standard operating procedure, Job Reyes, co-owner and husband of Anita opened the boxes for final inspection, before delivering it to the Bureau of Customs and/or Bureau of Posts. Upon opening, a peculiar odor emanated from the box that was supposed to contain gloves. Upon further perusal, he felt and saw a dried leaves inside the box. Job Reyes then brought samples to the NBI, he told them that the boxes to be shipped were still in his office. In the presence of the NBI agents, Reyes opened the box and discovered that the odor came from the fact that the dried leaves were actually those of the marijuana flowering tops. Two other boxes,marked as containing books and tabacalera cigars; also revealed bricks or case-like marijuana leaves and dried marijuana leaves respectively.
143

144

Marti was later invited by the NBI to shed light on the attempted shipment of the dried leaves. Thereafter an information was filed against the appellant for violating RA 6425 or the Dangerous Drugs Act. The Special Criminal Court of Manila convicted accused Marti of violating sec.21(b) of said RA. ISSUES: 1. Did the search conducted by a private person, violate accused's right against unreasonable searches seizures and invocable against the state? 2. Was the evidence procured from the search admissible? Held: 1. No, constitutional protection on search and seizure is imposable only against the state and not to private persons. Since Art. III,2 of the 1987 constitution is almost verbatim from the United States constitution, the SC may consider US Fed. SC cases as likewise doctrinal in this jurisdiction. Hence, in US cases, the constitutional provision against unreasomable searches and seizure was intended as a restraint upon the activities of the sovereign authority and NOT intended against private persons. If a search was initiated by a private person the provision does not apply since it only proscribes government action. This view is supported by the deliberations by the 1986 Constitutional Commission. In short, the protection against unreasonable searches and seizures cannot be extended to acts comitted by private individuals so as to bring it within the ambit of alleged unlawful intrusion. Case at bar will show that it was Job Reyes` initiative that perpetrated the search. He opened the packages and took the samples to NBI. All the NBI agents did was to observe and look in plain sight. This did not convert it to a search as contemplated by the constitution. 2. Yes, since the search was valid, the evidence from therein is admissible evidence. Art.III [2], on the admissibility of evidence in violation of the right against unreasonable searches and seizures, likewise applies only to the government and its agencies and not to private persons. (U.S. cases cited: Burdeau v. McDowell (256 us 465 [1921], state v. Bryan (457 p 2d 661 [1968], Walker v. state (429 s.w 2d 121 [1969]), Barnes v. us (373 F 2d 517 [1967]), Chadwick v. state (329 sw 2d 135). VALID WARRANTLESS SEARCH AND SEIZURE: 3. Search made incidental to a valid arrest d. Moreno vs. Ago Chi, 12 Phil. 439 e. PEOPLE VS. ANG CHUN KIT, 251 SCRA 660 f. PEOPLE VS. LUA, 256 SCRA 539
144

145

g. PEOPLE VS. Figueroa, 248 SCRA 679 h. NOLASCO VS. PANO, 139 SCRA 541 (A search incidental to a valid arrest must be done at the place where the accused is arrested. As such, if accused was arrested while inside a jeepney, there is no valid search incidental to a valid arrest if she will be brought to her residence and thereafter search the said place) i. ESPANO VS. CA, 288 SCRA 588 (If the accused was arrested in the street during a buy-bust operation, the search of his house nearby is not a valid search incidental to a valid arrest) PEOPLE VS. GO, 354 SCRA 338 Where the gun tucked in a persons waist is plainly visible to the police, no search warrant is necessary and in the absence of any license for said firearm, he may be arrested at once as he is in effect committing a crime in the presence of the police officers. No warrant is necessary in such a situation, it being one of the recognized exceptions under the Rules. As a consequence of the accuseds valid warrantless arrest inside the nightclub, he may be lawfully searched for dangerous weapons or anything which may be used as proof of the commission of an offense, without a search warrant in accordance with Section 12, Rule 126. This is a valid search incidental to a lawful arrest. In fact, the subsequent discovery in his car which was parked in a distant place from where the illegal possession of firearm was committed [after he requested that he will bring his car to the Police Station after his warrantless arrest) , of a drug paraphernalia and shabu, CANNOT BE SAID TO HAVE BEEN MADE DURING AN ILLEGAL SEARCH. As such, the items do not fall under the exclusionary rule and the unlicensed firearms, drug paraphernalia and the shabu, can be used as evidence against the accused. 4. Search of moving vehicles f. P. VS. MARIACOS, G.R. No. 188611, June 16, 2010 g. Carrol vs. US, 267 US 132 h. PEOPLE VS. LO HO WING, et al. (G. R. No. 88017) January 21, 1991 i. MUSTANG LUMBER VS. CA, 257 SCRA 430 j. PEOPLE VS. CFI, 101 SCRA 86 k. PEOPLE VS. MALMSTEDT198 SCRA 401 l. PEOPLE VS. LO HO WING, 193 SCRA 122 FACTS: In July 1987, the Special Operations Group of the CIS received a tip from one of its informers about an organized group engaged in importation of illegal drugs and smuggling of contraband items. To infiltrate the crime syndicate, they recruited confidential men and "deep penetration agents" under OPLAN SHARON 887. One such agent was Reynaldo Tia (the dicharged/accused). As an agent, he submitted regular reports of undercover activities of suspected syndicates. CAPTAIN PALMERA, head of oplan sharon 887, in turned informed the Dan gerous Drugs Board of Tia's activities.
145

146

Tia was introduced to his co-accused Lim Cheng Huat by another agent named George. Lim wanted a male travelling companion for his business trips abroad. Tia offered his services and was hired by Lim. Later, Tia was introduced to Peter Lo (alias of accused/appellant Lo Ho Wing), the later turning out to be Tia's intended companion. Appellant Lo Ho Wing and Tia left for Hongkong on October 4, 1987. Tia telephoned Capt. Palmera that they would return to the Philippines on October 6. From Hongkong, the two proceeded to Guangzhou in mainland China. There, appeallant Lo Ho Wing bought six (6) cans of tea.Tia saw these 6 bags when they were opened for examination. That evening, they went to Lo Ho Wing's room and he saw two other men with him. One was fixing the tea bags, while the other was burning a substance on a piece of aluminum foil using a lighter. Appellant Lo Ho Wing joined the second man and sniffed the smoke emitted by the burning substance. When Tia asked Lo Ho Wing what cargo they would bring to Manila, the latter replied that they would be bringing Chinese drugs. The next day en route to Manila, customs examiners inspected the bags containing the tin cans of tea. Since the bags were not closely examined, appellant Lo Ho Wing and Tia were cleared. In Manila, They were met by Lim Cheng Huat. Appelant Lo Ho Wing and Tia boarded a taxi from the airport and loaded their luggage in the taxi's compartment. Lim Cheng Huat followed them in another taxi. Meamwhile, a team composed by Capt. Palmera positioned themselves in strategic areas around the airport. The CIS men who first saw Lo Ho and Tia followed them. Along Imelda Avenue, the CIS car overtook the taxi ridden by Lo Ho Wing and Tia , forcing the taxi driver to stop his vehicle. The CIS team asked the taxi driver to open the baggage compartment. The CIS team asked permission to search their luggage. A tin can of tea was taken out of the compartment. Sgt. Cayabyab of the CIS pried the lid open and pressed it in the middle to pull out the contents. Crystalline white powder resmbling crushed alum came out. Suspecting the crystalline powder to be a dangerous drug, he had the three travelling bags opened for inspection. All the bags threshed out a total of six tin cans. Tia and appellant were taken to the CIS headquarters for questioning. Meanwhile, the second taxi carrying Lim Cheng Huat sped in attempt to escape. However, they were later captured. Samples from the bag tested positive for metamphetamine. The three suspects were indicted for violating Art. III, sec.15 of the Dangerous Drug Act. Appellant Lo Ho Wing and Lim Cheng Huat were sentenced to suffer life imprisonment and to pay a fine of P25,000 each. Reynaldo Tia was discharged as a state witness. The trial court gave full credence to the testimonies of government agents since the presumption of regularity in the performance of official duties were in their favor. ISSUES: 1. Was the warrantless search valid? 2. Are the effects taken admissible as evidence?

146

147

HELD: 1. This is a case of search on a moving vehicle which is one of the well-known exceptions to the valid warrantless search and seizure. To stilol get a search warrant from a judge would allow the accused go scotfree. 2. Since the search and seizure are valid, the evidence obtained is admissible as evidence in any proceeding. 5. Seizure of goods concealed to avoid duties/taxes (Valid) e. f. g. h. Papa vs. Mago, 22 SCRA 857 Pacis vs. Pamaran, 56 SCRA 16 HIZON VS. CA, 265 SCRA 517 PEOPLE VS. QUE, 265 SCRA 721

6. Seize of evidence in plain view a. b. c. d. 7. d. e. f. Harris vs. US, 390 US 234 PEOPLE VS. DAMASO, 212 SCRA 547 PEOPLE VS. VELOSO, 252 SCRA 135 PEOPLE VS. LESANGIN, 252 SCRA 213

When there is waiver of right or gives his consent; De Garcia vs. Locsin, 65 Phil. 689 Lopez vs. Commissioner, 65 SCRA 336 PEOPLE VS. DAMASO, 212 SCRA (In order that there is a valid waiver to a warrantless search, the waiver or consent should be given by the person affected, not just anybody. Example: The landlady could not give a valid consent to the search of a room occupied by a tenant. Said tenant himself should give the consent in order to be valid. The doctrine in Lopez vs. Commissioner to the effect that it could be given by any occupant of a hotel room being rented by the respondent is deemed abandoned) g. VEROY VS. LAYAGUE, 210 SCRA 97. (If the owner of the house allowed the policemen/soldiers to enter his house because they are searching for rebel soldiers but once inside the house, they instead seized an unlicensed firearm,) 8. a. b. c. STOP AND FRISK. People vs. Mengote, June, 1992 PEOPLE VS. POSADAS, 188 SCRA 288 MANALILI VS. PEOPLE, October 9, 1997. (The policemen saw several suspicious looking men at dawn who ran when they went near them. As the policemen ran after them, an unlicensed firearm was confiscated. The search is valid) d. MALACAT VS. CA, 283 SCRA 159. (Mere suspicions not sufficient to validate warrantless arrest) 6. EDDIE GUAZON, ET AL. VS. MAJ. GEN. RENATO DE VILLA, ET AL., GR NO. 80508, January 30, 1990 Warrantless searches; "zonings" and "saturation drives" Section 17, Art. VII of the Constitution
147

148

Gutierrez, Jr., J. Facts: This is a petition for Prohibition with preliminary injunction to prohibit military and police officers from conducting "Areal target zonings" or "saturation drive" in Metro Manila particularly in places where they suspect that the subversives are hiding. The 41 petitioners claim that the saturation drives conducted by the military is in violation of their human rights because with no specific target house in mind, in the dead of the night or early morning hours, police and military officers without any search warrant cordon an area of more than one residence and sometimes the whole barangay. Most of them are in civilian clothes and w/o nameplates or identification cards; that the raiders rudely rouse residents from their sleep by banging on the walls and windows of their homes, shouting, kicking their doors open (destroying some) and ordering the residents to come out; the residents are herded like cows at the point of high powered guns, ordered to strip down to their briefs and examined for tattoo marks; that while examination of the bodies of the men are being conducted, the other military men conduct search and seizures to each and every house without civilian witnesses from the neighbors; some victims complained that their money and other valuables were lost as a result of these illegal operations. The respondents claim that they have legal authority to conduct saturation drives under Art. VII, Sec. 17 of the Constitution which provides: The respondents would want to justify said military operation on the following constitutional provisions: The President shall be the Commander-in-Chief of all the armed forces of the Philippines and whenever it becomes necessary, he may call out such armed forces to prevent or suppress lawless violence, invasion or rebellion x x x xxxx The President shall have control of all the executive departments, bureaus and offices. He shall ensure that the laws are faithfully executed. Held: There can be no question that under ordinary circumstances, the police action of the nature described by the petitioners would be illegal and blatantly violative of the Bill of Rights. If the military wants to flush out subversive and criminal elements, the same must be consistent with the constitutional and statutory rights of the people. However, nowhere in the Constitution can we see a provision which prohibits the Chief Executive from ordering the military to stop unabated criminality, rising lawlessness and alarming communist activities. However, all police actions are governed by the limitations of the Bill of Rights. The government cannot adopt the same reprehensible methods of authoritarian systems both of the right and of the left. This is so because Art. III, Section 3 of the Constitution is very clear as explained in Roan vs. Gonzales, 145 SCRA 687 and Century Fox vs. Court of Appeals, 164 SCRA 655. Also, it must be pointed out that police actions should not be characterized by methods that offend one's sense of justice (Rochin vs. California, 342 US 165).
148

149

The Court believes it highly probable that some violations were actually committed. But the remedy is not to stop all police actions, including the essential and legitimate ones. A show of force is sometimes necessary as long as the rights of people are protected and not violated. However, the remedy of the petitioners is not an original action for prohibition since not one victim complains and not one violator is properly charged. It is basically for the executive department and the trial courts. The problem is appropriate for the Commission of Human Rights. The petition was therefore remanded to the Regional Trial Courts of Manila, Malabon and Pasay City where the petitioners may present evidence supporting their allegations so that the erring parties may be pinpointed and prosecuted. In the meantime, the acts violative of human rights alleged by the petitioners as committed during the police actions are ENJOINED until such time as permanent rules to govern such actions are promulgated. ******************** Cruz, Padilla and Sarmiento, JJ. , Dissenting The ruling of the majority that the petitioners are not proper parties is a specious pretext for inaction. We have held that technical objections may be brushed aside where there are constitutional questions that must be met (RODRIGUEZ VS. GELLA, 92 PHIL. 603; TOLENTINO VS. COMELEC, 41 SCRA 702; PHILCONSA VS. JIMENEZ, 65 SCRA 479; EDU VS. ERICTA, 35 SCRA 481; GONZALES VS. COMELEC, 27 SCRA 835; LAGUNZAD VS. CA, 154 SCRA 199; DEMETRIA VS. ALBA,148 SCRA 208). Lozada was in fact an aberration. Where liberty is involved, every person is a proper party even if he may not be directly injured. Each of us has a duty to protect liberty and that alone makes him a proper party. It is not only the owner of a burning house who has the right to call the firemen. Section 2, Art. III of the constitution is very clear: Unreasonable searches and seizures of whatever nature and for whatever purpose is prohibited. Saturation drives are NOT AMONG THE ACCEPTED INSTANCES WHEN A SEARCH OR AN ARREST MAY BE MADE WITHOUT A WARRANT. THEY COME UNDER THE CONCEPT OF THE FISHING EXPEDITIONS STIGMATIZED BY LAW AND DOCTRINE X X X I submit that this court should instead categorically and emphatically that these saturation drives are violative of human rights and individual liberty and should be stopped immediately. While they may be allowed in the actual theater of military operations against the insurgents, the Court should also make it clear that Metro Manila is not such a battleground. 7. IN THE MATTER OF THE PETITION FOR HABEAS CORPUS OF ROBERTO UMIL, ROLANDO DURAL and RENATO VILLANUEVA. MANOLITA UMIL and NICANOR DURAL, FELICITAS SESE VS. FIDEL RAMOS, ET AL. and companion cases, G.R. No. 81567, July 9, 1990 (An NPA may be arrested without warrant while sleeping or being treated in a hospital because his being a communist rebel is a continuing crime)
149

150

h. If the judge finds that there's probable cause, must he issue a warrant of arrest as a matter of course? See the distinctions. Read: 1. SAMULDE VS. SALVANI, SEPTEMBER 26, 1988 (No because a warrant is issued in order to have jurisdiction of the court over the person of an accused and to assure the court of his presence whenever his case is called in court. As such, if the court believes that the presence of the accused could be had even without a warrant of arrest, then he may not issue said warrant. Note: This case involves a minor offense) 2. GOZO VS. TAC-AN, 300 SCRA 265. If the offense committed is a serious one like that obtaining in this case for murder, the Judge must issue a warrant of arrest after determining the existence of probable cause) i. Searching questions Read: DR. NEMESIO PRUDENTE VS. THE HON. EXECUTIVE JUDGE ABELARDO M. DAYRIT, RTC 33, Manila & People of the Philippines, GR No. 82870, December 14, 1989 (En Banc) Search and seizure; requirements/requisites of a valid search warrant; searching questions Padilla, J. This is a petition to annul and set aside the Order of respondent Judge DENYING the motion of the petitioner to quash Search Warrant No. 8714 as well as its Order denying the petitioner's Motion for Reconsideration. Facts: 1. On October 31, 1987, P/Major Alladin Dimagmaliw, Chief of the Intelligence Special Action Division (ISAD) of the Western Police District (WPD) filed with the Regional Trial Court of Manila, Branch 33, presided by the respondent Judge, an application for the issuance of a Search Warrant for violation of PD 1866 against the petitioner; 2. In his application for search warrant, P/Major Dimagmaliw alleged that:

"1. That he has been informed and has good and sufficient reasons to believe that NEMESIO PRUDENTE who may be found at the Polytechnic University of the Philippines x x x has in his control or possession firearms, explosives, hand grenades and ammunition intended to be used as the means of committing an offense x x x; "2. That the undersigned has verified the report and found it to be a fact x x x ".

In support of said application, P/Lt. Florencio Angeles executed a "Deposition of Witness dated October 31, 1987 .
150

151

3. On November 1, 1987, a Sunday and All Saints Day, the search warrant was enforced by some 200 WPD operatives led by Col. Edgar Dula Torre and Major Maganto; 4. On November 2, 1987, Ricardo Abando, a member of the searching team executed an affidavit alleging that he found in the drawer of a cabinet inside the wash room of Dr. Prudente's office a bulging brown envelope with three live fragmentation hand grenades separately with old newspapers; 5. On November 6, 1987, the petitioner moved to quash the search warrant on the grounds that: a. the complainant's lone witness, Lt. Angeles had no personal knowledge of the facts which formed the basis for the issuance of the search warrant; b. the examination of said witness was not in the form of searching questions and answers; c. the search warrant was a general warrant, for the reason that it did not particularly describe the place to be searched and that it failed to charge one specific offense; and d. the warrant was issued in violation of Circular No. 19 of the Supreme Court in that the complainant failed to allege that the issuance of the search warrant on a Saturday was urgent. 6. On March 9, 1986, the respondent judge denied the motion to quash and on April 20, 1988, the same judge denied petitioner's motion for reconsideration. Hence this petition. Issue: Was the Search Warrant issued by the respondent judge valid? Was there probable cause? Held: a. For a valid search warrant to issue, there must be probable cause, which is to be determined by the judge, after examination under oath or affirmation of the complainant and the witnesses he may produce, and particularly describing the place to be searched and the persons or things to be seized. The probable cause must be in connection with one specific offense and the judge must, before issuing the warrant, personally examine in the form of searching questions and answers, in writing and under oath, the complainant and the witnesses he may produce, on facts personally known to them and attach to the record their sworn statements together with any affidavits submitted. The "probable cause" for a valid search warrant, has been defined "as such facts and circumstances which would lead a reasonably discreet and prudent man to believe that an offense has been committed, and that the objects sought in connection with the offense are in the place sought to be searched". (Quintero vs. NBI, June 23, 1988). This probable cause must be shown to be within the personal knowledge of the complainant or the witnesses he may produce and not based on mere hearsay. (P. VS. SY
151

152

JUCO, 64 PHIL. 667; ALVAREZ VS. CFI, 64 PHIL. 33; US VS. ADDISON, 28 PHIL. 566). In his affidavit, Major Dimagmaliw declared that "he has been informed" that Nemesio Prudente "has in his control and possession" the firearms and explosivees described therein, and that he "has verified the report and found it to be a fact." On the other hand, Lt. Angeles declared that as a result of continuous surveillance for several days, they "gathered informations from verified sources" that the holders of said firearms and explosives are not licensed t possess them. It is clear from the foregoing that the applicant and his witness HAD NO PERSONAL KNOWLEDGE OF THE FACTS AND CIRCUMSTANCES which became the basis for issuing the questioned search warrant, but acquired knowledge thereof only through information from other sources or persons. Despite the fact that Major Dimagmaliw stated in his affidavit that "he verified the information he had earlier received and found it to be a fact, YET THERE IS NOTHING IN THE RECORD TO SHOW OR INDICATE HOW AND WHEN SAID APPLICANT VERIFIED THE EARLIER INFORMATION ACQUIRED BY HIM AS TO JUSTIFY HIS CONCLUSION. He might have clarified this point if there had been searching questions and answers, but there were none. In fact, the records yield no questions and answers, whether searching or not, vis-a-vis the said applicant. In ALVAREZ VS. CFI, 64 PHIL. 33, it was held that the following test must be complied with in an application for search warrant or in a supporting deposition based on personal knowledge or not"The true test of sufficiency of a deposition or affidavit to warrant issuance of a search warrant is whether it was drawn in a manner that perjury could be charged thereon and the affiant be held liable for damage caused. The oath required must refer to the truth of the facts within the personal knowledge of the applicant of a search warrant and/or his witnesses, not of the facts merely reported by a person whom one considers to be reliable."

Tested by the above standards, the allegation of the witness, Lt. Angeles, do not come up to the level of facts based on his personal knowledge so much so that he cannot be held liable for perjury for such allegations in causing the issuance of the questioned search warrant. Besides, respondent judge did not take the deposition of the applicant as required by the Rules of Court. As held in Roan vs. Gonzales, 145 SCRA 694, "mere affidavits of the complainant and his witnesses are thus insufficient. The examining judge has to take the depositions in writing of the complainant and the witnesses he may produce and attach them to the record." b. There was also no searching questions asked by the respondent judge because as shown by the record, his questions were too brief and short and did not examine the complainant and his witnesses in the form of searching questions and answers. On the contrary, the questions asked were leading as they called for a simple "yes" or "no" answer. As held in Quintero vs. NBI, June 23, 1988, "the questions propounded are not sufficiently searching to establish probable cause. Asking of leading
152

153

questions to the deponent in an application for search warrant and conducting of examination in a general manner would not satisfy the requirements for the issuance of a valid search warrant." The Court avails of this decision to reiterate the strict requirements for determination of probable cause in the valid issuance of a search warrant as enunciated in earlier cases. True, this requirements are stringent but the purpose is to assure that the constitutional right of the individual against unreasonable search and seizure shall remain both meaningful and effective. c. The rule is, that a description of a place to be searched is sufficient if the officer with the warrant can with reasonable effort ascertain and identify the place intended (P VS. VELOSO, 48 PHIL. 180). In the case at bar, the warrant described the place to be searched as the premises of the PUP, more particularly the offices of the Department of Science and Tactics as well as the Office of the President, Nemesio Prudente. There is also no violation of the "one specific offense" requirement considering that the application for a search warrant explicitly described the offense: illegal possession of firearms and ammunitions under PD 1866. d. CIRCULAR NO. 19 OF THE SUPREME COURT merely provides for a guideline, departure from which would not necessarily affect the validity of the search warrant provided the constitutional requirements are complied with. c. HUBERT WEBB VS. DE LEON, 247 SCRA 650 Read also: 1. Alvarez vs. CFI, 64 Phil. 33 (When the applicant is basing his knowledge from an informant, the same is not valid) 2. Luna vs. Plaza, 26 SCRA 313 3. De Mulata vs. Irizari, 62 SCRA 210 4. Marinas vs. Siochi, 104 SCRA 423 5. Roan vs. Gonzales, 145 687 6. Mata vs. Bayona, 128 SCRA 388 (Depositions of the applicants and witnesses should be attached to the record of the case) 7. Corro vs. Lising, 137 SCRA 541 8. Nolasco vs Pano, 147 SCRA 509 9. Burgos vs. Chief of Staff, 133 SCRA 800 10. P. vs. Burgos, September 14,1986 11. P. vs. Aminnudin Y Ahni, July 6,1988 12. Ponsica vs. Ignalaga, July 31,1987 (When the statements in the affidavits of witnesses are mere generalities, mere conclusions of law, and not positive statements of particular acts, the warrant is not valid) 13. Aberca vs. Ver, April 15,1988 2. Panganiban vs. Cesar, 159 SCRA 599 3. PENDON VS. CA, November 16, 1990. (When the questions asked to the applicant for a search warrant was pre-typed, the same is not valid since there could have been no searching questions) j. Warrantless searches and seizures--when valid or not.
153

154

Read: 1. RICARDO VALMONTE VS. GEN RENATO DE VILLA, GR No. 83988, September 29, 1989 Warrantless searches and seizures; validity of checkpoints Padilla, J. Facts: 1. On January 20, 1987, the National Capital Region District Command (NCRDC) was activated with the mission of conducting security operations within its area of responsibility for the purpose of maintaining peace and order. As part of its duty to maintain peace and order, the NCRDC installed checkpoints in various parts of Valenzuela, Metro Manila. Petitioners claim that because of these checkpoints, the residents of Valenzuela, MM are worried of being harassed and of their safety being placed at the arbitrary, capricious and whimsical disposition of the military authorities manning the checkpoints considering that their cars and vehicles are being subjected to regular searches and check-ups, especially at night or dawn, without the benefit of a search warrant and/or court order. 2. On July 9, 1988 at dawn, the apprehensions of the residents of Valenzuela increased because Benjamin Parpon, the supply officer of the Municipality of Valenzuela was gunned down in cold blood by the military men manning the checkpoints for ignoring or refusing to submit himself to the checkpoint and for continuing to speed off inspite of several warning shots fired in the air. Issue: Whether or not the existence of said checkpoints as well as the periodic searches and seizures made by the military authorities without search warrant valid? Held: Petitioners' concern for their safety and apprehension at being harassed by the military manning the checkpoints are not sufficient grounds to declare the checkpoints as per se illegal. Not all searches and seizures are prohibited. Those which are reasonable are not forbidden. A reasonable search is not to be determined by any fixed formula but is to be resolved according to the facts of each case. Where, for example, the officer merely draws aside the curtain of a vacant vehicle which is parked on a public fair grounds (People vs. Case, 190 MW 289), or simply looks into a vehicle (State vs. Gaina, 97 SE 62), or flashes a light therein (Rowland vs. Commonwealth, 259 SW 33), these do not constitute unreasonable search.
154

155

The setting up of checkpoints in Valenzuela, Metro Manila may be considered as security measure to effectively maintain peace and order and to thwart plots to destabilize the government. In this connection, the Court may take judicial notice of the shift to urban centers and their suburbs of the insurgency movement, so clearly reflected in the increased killings in cities of police and military men by NPA's "sparrow units," not to mention the abundance of unlicensed firearms. BETWEEN THE INHERENT RIGHT OF THE STATE TO PROTECT ITS EXISTENCE AND PROMOTE PUBLIC WELFARE AND AN INDIVIDUAL'S RIGHT AGAINST A WARRANTLESS SEARCH WHICH IS HOWEVER REASONABLY CONDUCTED, THE FORMER SHALL PREVAIL. True, the manning of these checkpoints by the military is susceptible of abuse by the men in uniform, in the same manner that all governmental power is susceptible to abuse. BUT , AT THE COST OF OCCASIONAL INCONVENIENCE, DISCOMFORT AND EVEN IRRITATION TO THE CITIZEN, THE CHECKPOINTS DURING THESE ABNORMAL TIMES ARE PART OF THE PRICE WE PAY FOR AN ORDERLY SOCIETY AND PEACEFUL COMMUNITY. Finally, it must be emphasized that on July 17, 1988, the military checkpoints in Metro Manila were temporarily lifted and a review and refinement of the rules in the conduct of the police and military manning the checkpoints upon order of the NCRDC Chief. Cruz and Sarmiento, JJ., dissenting: The bland declaration by the majority that individual rights must yield to the demands of national security ignores the fact that the Bill of Rights was intended precisely to limit the authority of the State even if asserted on the ground of national security. RESOLUTION ON THE MOTION RECONSIDERATION, JUNE 15, 1990 Very Important: The Supreme Court in its Resolution of the Motion for Reconsideration dated 15 June, 1990, held that military and police checkpoints are not illegal as these measures to protect the government and safeguards the lives of the people. The checkpoints are legal as where the survival of the organized government is on the balance, or where the lives and safety of the people are in grave peril. However, the Supreme Court held further that the military officers manning the checkpoints may conduct VISUAL SEARCH ONLY, NOT BODILY SEARCH. Read also: 1-a. Rizal Alih vs. Gen. Castro, June 23,1987 1-b. P s. Cendana, October 17, 1990 1-c. P. vs. Castiller, August 6, 1990 1-d. P. vs. Olaes, July 30, 1990 2. Papa vs. Mago, 22 SCRA 857 3. Roldan vs. Arca, 65 SCRA 336 4. P. vs. CFI, 101 SCRA 86
155

FOR

156

5. Pacis vs. Pamaran, 56 SCRA 16 6. Lopez vs. Commisioner, 65 SCRA 336 7. P vs. Cruz, 165 SCRA 135 8. Nolasco vs. Pano, 147 SCRA 509 & 139 SCRA 152 9. P vs. Claudio, 160 SCRA 646 (There is a valid warrantless search when a NARCOM (now PDEA) officer arrests the person who owns a bag which contains marijuana which he found out when he smelled the same. Here , there is a probable cause since he has personal knowledge due to his expertise on drugs) 11. PEOPLE VS. DEL ROSARIO, July 10, 1994. (After the informant was given by the police the amount of P100.00, he went to buy marijuana from the accused then returned to the police headquarters with said article. Thereafter, the policemen went to arrest the accused without warrant. The arrest is not valid since it does not fall under Section 5 Rule 113) Likewise, after securing a search warrant authorizing the seizure of shabu and its paraphernalia and an unlicensed firearm was seized instead, said gun is inadmissible in evidence. k. May a non-judicial officer issue a warrant of arrest? (NO) Read: 1. 2. 3. 4. Harvey vs. Miriam Defensor-Santiago, June 26,1988 Moreno vs. Vivo, 20 SCRA 562 Lim vs. Ponce de Leon, 66 SCRA 299 HORTENCIA SALAZAR VS. HON TOMAS ACHACOSO, G.R. NO. 81510, March 14, 1990 (En banc) 5. Presidential Anti_Dollar Salting Task Force vs. CA, March 16, 1989 l. Properties subject to seizure Read: 1. Sec. 2, Rule 126,1985 Rules on Crimial Procedure, as amended 2. ESPANO VS. CA, 288 SCRA 558 m. Warrantless searches and arrests Read: 1. P. vs. Bati, August 27, 1990 1-a. Manuel et al., vs. Judge Tirso Velasco, GR No. 84666, February 9, 1989 1-b. Garcia-Padilla vs. Enrile,121 SCRA 47 & 137 SCRA 647 1-c. P. vs. Maspil, Jr., August 20, 1990 (Compare with P. vs. Aminnudin, July 6, 1988, supra) 1-d. Posadas vs. CA, Aug. 2, 1990 1-e. P. vs. De la Cruz 1-f. P. vs. ortiz, Dec. 3, 1990 1-g. Rolito Go vs. CA, Feb. 11, 1992 1-h. People vs. Mati, January 18, 1991 2. Morales vs. Ponce Enrile, 121 SCRA 538 2-a. P vs. Burgos, 144 SCRA 1 2-b. People vs. de la Cruz, 184 SCRA 416 2-c. Gatchalian vs. Board, May 31, 1991
156

157

2-d. People vs. Sucro, March 18, 1991 2-e. PEOPLE VS. SOLAYAO, 262 SCRA 255 2-f. PEOPLE VS. CUISON, 256 SCRA 325 2-g. PEOPLE VS. DAMASO, 212 SCRA 547 2-h. OPOSADAS VS. CA, 258 SCRA 188 2-i. PEOPLE VS. JUATAN, 260 SCRA 532 (Buy-bust operation) 3. Sec. 6, Rule 113, 1985 Rules on Criminal Procedure, as amended n. Effect posting bail or entering a plea during the arraignment, if the arrest was illegal. (The alleged illegality of the arrest is deemed waived upon posting of the bond by the accused)

PEOPLE VS. GALVEZ, 355 SCRA 246 Mendoza, J. The policeman arrested the accused-appellant on the basis solely of what Reynaldo Castro had told him and not because he saw the accused-appellant commit the crime charged against him. Indeed, the prosecution admitted that there was no warrant of arrest issued against accused-appellant when the latter was taken into custody. Considering that the accused-appellant was not committing a crime at the time he was arrested nor did the arresting officer have any personal knowledge of facts indicating that accused-appellant committed a crime, his arrest without a warrant cannot be justified. However, by entering a plea of not guilty during the arraignment, the accused-appellant waived his right to raise the issue of illegality of his arrest. IT IS NOW SETTLED THAT OBJECTION TO A WARRANT OF ARREST OR THE PROCEDURE BY WHICH A COURT ACQUIRES JURISDICTION OVER THE PERSON OF AN ACCUSED MUST BE MADE BEFORE HE ENTERS HIS PLEA, OTHERWISE, THE OBJECTION IS DEEMED WAIVED. THE FACT THAT THE ARREST WAS ILLEGAL DOES NOT RENDER THE SUBSEQUENT PROCEEDINGS VOID AND DEPRIVE THE STATE OF ITS RIGHT TO CONVICT THE GUILTY WHEN ALL THE FACTS POINT TO THE CULPABILITY OF THE ACCUSED. Read: 1. 2. 3. 4. 5. 6. Callanta vs. Villanueva, 77 SCRA 377 PEOPLE VS. NAZARENO, 260 SCRA 256 FILOTEO VS. SANDIGANBAYAN, 263 SCRA 222 PEOPLE VS. NAZARENO, 260 SCRA 256 PEOPLE VS. LAPURA, 255 SCRA 85 PEOPLE VS. SILAN, 254 SCRA 491 o . Penalty for illegal arrest Read: Palon vs. NAPOLCOM, May 28, 1989 p. Judicial pronouncements on illegally seized 336 evidence, 106 SCRA
157

158

q. The exclusionary rule,155 SCRA 494 n. What is the status of a document obtained through subpoena? Read: Dianalan vs. Pros., Office of the Tanodbayan, Nov. 27, 1990 r. Search warrant for pirated video tapes 3. Century Fox vs. CA, 164 SCRA 655 (The master copy of the allegedly pirated tape should be presented before the judge in order to convince him of the existence of probable cause) 4. COLUMBIA PICTURES VS. CA, 261 SCRA 144 UY VS. BIR, 344 SCRA 36 The following are the requisites of a valid search warrant: 1. The warrant must be issued upon probable cause; 2. The probable cause must be determined by the judge himself and not by applicant or any other person; 3. In determining probable cause, the judge must examine under oath and affirmation the complainant and such witnesses as the latter may produce; and 4. The warrant issued must particularly describe the place to be searched and the person or things to be seized. A description of the place to be searched is sufficient if the officer with the warrant can, with reasonable effort, ascertain and identify the place intended and distinguish it from other places in the community. Search warrants are not issued on loose, vague or doubtful basis of fact, nor on mere suspicion or belief. In this case, most of the items listed in the warrants fail to meet the test of particularity, especially since the witness had furnished the judge photocopies of the documents sought to be seized. THE SEARCH WARRANT IS SEPARABLE, AND THOSE ITEMS NOT PARTICULARLY DESCRIBED MAY BE CUT OFF WITHOUT DESTROYING THE WHOLE WARRANT.

PEOPLE VS. VALDEZ, 341 SCRA 25 The protection against unreasonable search and seizure covers both innocent and guilty alike against any form of highhandedness of law enforces. The plain view doctrine, which may justify a search without warrant, APPLIES ONLY WHERE THE POLICE OFFICER IS NOT SEARCHING FOR EVIDENCE AGAINST THE ACCUSED, BUT INADVERTENTLY COMES ACROSS AN INCRIMINATING OBJECT. Just because the marijuana plants were found in an unfenced lot does nor prevent the appellant from invoking the protection afforded by the Constitution. The right against unreasonable search and seizure is the immunity of ones person,
158

159

which includes his residence, papers and other possessions. For a person to be immune against unreasonable searches and seizures, he need not be in his home or office, within a fenced yard or private place. PEOPLE VS. BAULA, 344 SCRA 663 In case of consented searches or waiver of the constitutional guarantee against obtrusive searches, it is fundamental that to constitute waiver, IT MUST APPEAR THAT THE RIGHT EXISTS; THE PERSONS INVOLVED HAD KNOWLEDGE, EITHER ACTUAL OR CONSTRUCTIVE, of the existence of such right. The third condition did not exist in the instant case. Neither was the search incidental to a valid warrantless arrest. (PEOPLE VS. FIFUEROA, July 6, 2000) An alleged consent to a warrantless search and seizure cannot be based merely on the presumption of regularity in the performance of official duty. THE PRESUMPTION BY ITSELF, CANNOT PREVAIL AGAINST THE CONSTITUTIONALLY PROTECTED RIGHTS OF AN INDIVIDUAL, AND ZEAL IN THE PURSUIT OF CRIMINALS CANNOT ENNOBLE THE USE OF ARBITRARY METHODS THAT THE CONSTITUTION ITSELF ABHORS.

CHAPTER IV THE RIGHT TO PRIVACY Section 3. The privacy of communication and correspondence shall be inviolable except upon lawful order of the court, or when public safety or order requires otherwise as prescribed by law. Any evidence obtained in violation of this or the preceding section shall be inadmissible for any purpose in any proceeding. Read: Read: NOTE: Applicable provisions of the Human Security Act/Anti-Terrorism Law, Republic Act No. 9372, Approved on March 6, 2007 and effective on July 15, 2007 (This Law shall be automatically suspended one (1) month before and two (2) months after the holding of any election) Please observe the procedure in obtaining the The Warrant [or Order] of Surveillance, not found in the 1987 Philippine Constitution. SURVEILLANCE OF SUSPECTS AND INTERCEPTION AND RECORDING OF COMMUNICATIONS OF SUSPECTS OR CHARGED OF TERRORISM Section 7. Surveillance of suspects and interception and recording of communications. The provisions of RA 4200 (Anti-Wiretapping Law) to the contrary notwithstanding, a police or law enforcement official and the members of his team may, upon a written order of the Court of
159

160

Appeals, listen to, intercept and record, with the use of any mode, form or kind or type of electronic or other surveillance equipment or intercepting and tracking devices, or with the use of any other suitable ways or means for that purpose, any communication, message, conversation, discussion, or spoken or written words between members of a judicially declared and outlawed terrorist organization, association, or group of persons or of any person charged with or suspected of the crime of terrorism or conspiracy to commit terrorism. Provided, That surveillance, interception and recording of communications between lawyers and clients, doctors and patients, journalists and their sources and confidential business correspondence shall not be authorized. Section 8. Formal Application for Judicial Authorization.- The written order of the authorizing division of the Court of Appeals to track down, tap, listen, intercept, and record communications, messages, conversations, discussions, or spoken or written words of any person suspected of the crime of terrorism or the crime of conspiracy to commit terrorism, shall only be granted by the authorizing division of the Court of Appeals UPON AN EX-PARTE written application of a police or law enforcement official who has been duly authorized in writing by the AntiTerrorism Council created in Section 53 of this Act to file such ex-parte application, and upon examination under oath and affirmation of the applicant and the witnesses who may produce to establish: That there is probable cause to believe based on personal knowledge of facts and circumstances that the said crime of terrorism or conspiracy to commit terrorism has been committed, or is being committed, or is about to be committed; That there is probable cause to believe based on personal knowledge of facts and circumstances that evidence which is essential to the conviction of any charged or suspected person for, or to the solution or prevention of any such crimes, will be obtained; and That there is no other effective means readily available for acquiring such evidence. Sec. 9. Classification and Contents of the Order of the Court. The written order granted by the authorizing division of the Court of Appeals as well as its order, if any, to extend or renew the same, the original application of the applicant, including his application to extend or renew, if any, and the written authorizations of the Anti-Terrorism Council shall be deemed and are hereby declared as classified information: Provided, That the person being surveilled or whose communications, letters, papers, messages, conversations, discussions, spoken or written words and effects have been monitored, listened to, bugged or recorded by law enforcement authorities has the right to be informed of the acts done by the law enforcement authorities in the premises or to challenge, if he or she intends to do so, the legality of the interference before the Court of Appeals which issued said written order. The written order of the authorizing division of the court of Appeals shall specify the following: The identity, such as name and address, if known, of the charged of suspected persons whose communications, messages, conversations, discussions, or spoken or written words are to be tracked down, tapped, listened to, intercepted or recorded and, in case of radio, electronic, or telephone (whether wireless or otherwise) communications, messages,
160

161

conversations, discussions, or spoken or written words, the electronic transmission systems or the telephone numbers to be tracked down, tapped, listened to, intercepted, and recorded and their locations if the person suspected of the crime of terrorism or conspiracy to commit terrorism is not fully known, such person shall be subject to continuous surveillance provided there is reasonable ground to do so; The identity (name and address, and the police or law enforcement organization) of the members of his team judicially authorized to track down, tap, listen to, intercept, and record the communications, messages, conversations, discussions, or spoken or written words; The offense or offenses committed, or being committed, or sought to be prevented; and The length of time which the authorization shall be used or carried out. Section. 10. Effective Period of Judicial Authorization. Any authorization granted by the authorizing division of the court of Appealsshall only be effective for the length of time specified in the written order of the authorizing division of the Court of Appeals, which shall not exceed 30 days from the date of receipt of the written order of the authorizing division of the court of Appeals by the applicant police or law enforcement official. The CA may extend or renew the said authorization for another non-extendible period, which shall not exceed 30 days from the expiration of the original periodThe ex-parte application for renewal has been duly authorized by the Anti-terrorism Council in writing. If no case is filed within the 30-day period, the applicant police or law enforcement official shall immediately notify the person subject of the surveillance, interception, and recording of the termination of the said surveillance, interception and recording. [Penalty to be imposed on the police official who fails to inform the person subject of surveillance of the termination of the surveillance, monitoring, interception and recording shall be penalized to 10 years and 1 day to 12 years. Section 15. Evidentiary Value of Deposited Materials. Any listened to, intercepted, and recorded communications, messages, conversationsWHICH HAVE BEEN SECURED IN VIOLATION OF THE PERTINENT PROVISIONS OF THIS ACT, SHALL ABSOLUTELY NOT BE ADMISSIBLE AND USABLE AS EVIDENCE AGAINST ANYBODY IN ANY JUDICIAL, QUASI-JUDICIAL, LEGISLATIVE, OR ADMINISTRATIVE INVESTIGATION, INQUIRY, PROCEEDING, OR HEARING. JUDICIAL AUTHORIZATION TO EXAMINE BANK DEPOSITS, ACCOUNTS, AND RECORDS OF SUSPECTED OR CHARGED TERRORISTS Section 27. judicial authorization required to examine bank deposits, accounts and records. The justices of CA designated as special court to handle antiterrorism cases after satisfying themselves of the existence of probable cause in a hearing called for that purpose that:

A person charged with or suspected of the crime of terrorism or conspiracy to commit terrorism;
161

162

Of a judicially declared and outlawed terrorist organization or group of persons; Of a member of such judicially declared and outlawed organization, association or group of persons, may authorize in writing any police or law enforcement officer and the members of his team duly authorized in writing by the anti-terrorism council to: 1. examine or cause the examination of, the deposits, placements, trust accounts, assets, and records in a bank or financial institution; and 2. gather or cause the gathering of any relevant information about such deposits, placements, trust accounts, assets, and records from a bank or financial institution. The bank or financial institution shall not refuse to allow such examination or to provide the desired information, when so ordered by and served with the written order of the Court of Appeals. Sec. 28. Application to examine deposits, accounts and records. The written order of the CA authorizing the examination of bank deposits, placements, trust accounts, assets and records: A person charged with or suspected of the crime of terrorism or conspiracy to commit terrorism; Of a judicially declared and outlawed terrorist organization or group of persons; Of a member of such judicially declared and outlawed organization, association or group of persons, in a bank or financial institution-SHALL ONLY BE GRANTED BY THE AUTHORIZING DIVISION OF THE CA UPON AN EX-PARTE APPLICATION TO THAT EFFECT OF A POLICE OR LAW ENFORCEMENT OFFICIAL who has been duly authorized by the Anti-Terrorism Council to file such ex-parte application and upon examination under oath or affirmation of the applicant and his witnesses he may produce to establish the facts that will justify the need and urgency of examining and freezing the bank deposits, placements, trust accounts, assets and records: Of A person charged with or suspected of the crime of terrorism or conspiracy to commit terrorism; Of a judicially declared and outlawed terrorist organization or group of persons; Of a member of such judicially declared and outlawed organization, association or group of persons. Section 35. Evidentiary value of deposited bank materials.- Any information, data, excerpts, summaries, notes, memoranda, work sheets, reports or documents acquired from the examination of the bank deposits, placements, trust accounts, assets and records of: A person charged with or suspected of the crime of terrorism or conspiracy to commit terrorism; Of a judicially declared and outlawed terrorist organization or group of persons; Of a member of such judicially declared and outlawed organization, association or group of persons, -which have been secured in violation of the provisions of this Act, shall absolutely not be admissible and usable as evidence against anybody
162

163

in any judicial, quasi-judicial, legislative or administrative investigation, inquiry, proceeding or hearing. 1. PEOPLE VS. CABALQUINTO, September 19, 2006, 502 SCRA 419 2. ZULUETA VS. CA, February 10, 1996 The wife forcibly opened the drawers at the clinic of her doctorhusband and took diaries, checks and greeting cards of his alleged paramours. Thereafter, she used the same in their legal separation case. Said documents are inadmissible in evidence. This is so because the intimacies of husband and wife does not justify the breaking of cabinets to determine marital infidelity. 3. OPLE VS. TORRES, July 23, 1998 Puno, J. Facts: On December 12, 1996, then President FIDEL V. RAMOS issued Administrative Order No. 308 entitled ADOPTION OF A NATIONAL COMPUTERIZED IDENTIFICATION REFERENCE SYSTEM. The AO seeks to have all Filipino citizens and foreign residents to have a Population Reference Number (PRN) generated by the National Statistics Office (NSO) through the use of BIOMETRICS TECHNOLOGY . The AO was questioned by Senator Ople on the following grounds: 1. The establishment of the PRN without any law is an unconstitutional usurpation of the legislative powers of the Congress of the Philippines; 2. The appropriation of public funds for the implementation of the said AO is unconstitutional since Congress has the exclusive authority to appropriate funds for such expenditure; and 3. The AO violates the citizens right to privacy protected by the Bill of Rights of the Constitution. Held: 1. The AO establishes a system of identification that is all-encompassing in scope, affects the life and liberty of every Filipino citizens and foreign residents and therefore, it is supposed to be a law passed by Congress that implements it, not by an Administrative Order issued by the President. Administrative Power, which is supposed to be exercised by the President, is concerned with the work of applying policies and enforcing orders as determined by proper governmental organs. It enables the President to fix a uniform standard of administrative efficiency and check the official conduct of his agents. Prescinding from the foregoing precepts, AO 308 involves a subject that is not appropriate to be covered by an Administrative Order. An administrative order is an ordinance issued by the President which relates to specific aspects in the administrative operation of the government. It must be in harmony with the law and should be for the sole purpose of implementing the law and carrying out
163

164

the legislative policy. The subject of AO 308 therefore is beyond the power of the President to issue and it is a usurpation of legislative power. 2. The AO likewise violates the right to privacy since its main purpose is to provide a common reference number to establish a linkage among concerned agencies through the use of BIOMETRICS TECHNOLOGY. Biometry is the science of the application of statistical methods to biological facts; a mathematical analysis of a biological data. It is the confirmation of an individuals identity through a fingerprint, retinal scan, hand geometry or facial features. Through the PRN, the government offices has the chance of building a huge and formidable information base through the electronic linkage of the files of every citizen. The data, however, may be gathered for gainful and useful government purposes; but the existence of this vast reservoir of personal information constitutes a covert invitation to misuse, a temptation that may be too great for some of our authorities to resist. Further, the AO does not even tells us in clear and unequivocal terms how these informations gathered shall be handled. It does not provide who shall control and access the data and under what circumstances and for what purpose. These factors are essential to safeguard the privacy and guaranty the integrity of the information. The computer linkage gives other government agencies access to the information. YET, THERE ARE NO CONTROLS TO GUARD AGAINST LEAKAGE OF INFORMATIONS. WHEN THE ACCESS CODE OF THE CONTROL PROGRAMS OF THE PARTICULAR COMPUTER SYSTEM IS BROKEN, AN INTRUDER, WITHOUT FEAR OF SANCTION OR PENALTY, CAN MAKE USE OF THE DATA FOR WHATEVER PURPOSE, OR WORSE, MANIPULATE THE DATA STORED WITHIN THE SYSTEM. AO No. 308 is unconstitutional since it falls short of assuring that personal information gathered about our people will be used only for specified purposes thereby violating the citizens right to privacy.

KILUSANG MAYO UNO VS. EXECUTIVE SECRETARY EDUARDO ERMITA, ET AL., April 19, 2006 & June 20, 2006 BAYAN MUNA VS. EXECUTIVE SECRETARY EDUARDO ERMITA, ET AL., April 19, 2006 & June 20, 2006

Carpio, J. President Gloria Macapagal-Arroyo issued Presidential Proclamation No. 420 that mandates the Adoption of a Unified, Multi-purpose Identification System by all Government Agencies in the Executive Department. This is so despite the fact that the Supreme Court held in an En Banc decision in 1998 OPLE VS. EXECUTIVE SECRETARY RUBEN TORRES Administrative Order No. 308[National computerized Identification Reference System] issued by then President Fidel V. Ramos that the same is unconstitutional because a national ID card system requires legislation because it creates a new national data collection and card issuance system, where none existed before. The Supreme Court likewise held that EO 308 as unconstitutional for it violates the citizens right to privacy.
164

165

Based on the Ople ruling, the petitioners claimed that Proclamation No. 420 is unconstitutional on two (2) grounds: a. usurpation of legislative powers; and b. it infringes on the citizens right to privacy Held: The said Executive Order No. 420 does not violate the citizens right to privacy since it does not require all the citizens to be issued a national ID as what happened in AO 308. Only those dealing or employed with the said government entities who are required to provide the required information for the issuance of the said ID.

CAMILO L. SABIO vs. GORDON, G.R. No. 174340, October 17, 2006, 504 SCRA 704 Sandoval-Gutierrez, J. The Facts:

On February 20, 2006, Senator Miriam Defensor Santiago introduced Philippine Senate Resolution No. 455 (Senate Res. No. 455),36[4] directing an inquiry in aid of legislation on the anomalous losses incurred by the Philippines Overseas Telecommunications Corporation (POTC), Philippine Communications Satellite Corporation (PHILCOMSAT), and PHILCOMSAT Holdings Corporation (PHC) due to the alleged improprieties in their operations by their respective Board of Directors.

On May 8, 2006, Chief of Staff Rio C. Inocencio, under the authority of Senator Richard J. Gordon, wrote Chairman Camilo L. Sabio of the PCGG, one of the herein petitioners, inviting him to be one of the resource persons in the public meeting jointly conducted by the Committee on Government Corporations and Public Enterprises and Committee on Public Services. The purpose of the public meeting was to deliberate on Senate Res. No. 455.37[6]

On May 9, 2006, Chairman Sabio and other commissioners of the PCGG declined the invitation because of prior commitment.38[7] At the same time, they invoked Section 4(b) of E.O. No. 1 earlier quoted.

On September 12, 2006, at around 10:45 a.m., Major General Balajadia arrested Chairman Sabio in his office at IRC Building, No. 82 EDSA, Mandaluyong City and brought him to the Senate premises where he was detained.
36[4] 37[6] 38[7]

Annex E of the Petition in G.R. No. 174318. Annex F of the Petition in G.R. No. 174318. Annex G of the Petition in G.R. No. 174318.

165

166

Hence, Chairman Sabio filed with the Supreme Court a petition for habeas corpus against the Senate Committee on Government Corporations and Public Enterprises and Committee on Public Services, their Chairmen, Senators Richard Gordon and Joker P. Arroyo and Members.

I S S U E S:

Is the investigation conducted on the petitioners violative of their right to privacy?

H E L D: Zones of privacy are recognized and protected in our laws. 39[46] Within these zones, any form of intrusion is impermissible unless excused by law and in accordance with customary legal process. The meticulous regard we accord to these zones arises not only from our conviction that the right to privacy is a constitutional right and the right most valued by civilized men,40[47] but also from our adherence to the Universal Declaration of Human Rights which mandates that, no one shall be subjected to arbitrary interference with his privacy and everyone has the right to the protection of the law against such interference or attacks.41[48] Our Bill of Rights, enshrined in Article III of the Constitution, provides at least two guarantees that explicitly create zones of privacy. It highlights a persons right to be let alone or the right to determine what, how much, to whom and when information about himself shall be disclosed.42[49] Section 2 guarantees the right of the people to be secure in their persons, houses, papers and effects against unreasonable searches and seizures of whatever nature and for any purpose. Section 3 renders inviolable the privacy of communication and correspondence and further cautions that any evidence obtained in violation of this or the preceding section shall be inadmissible for any purpose in any proceeding.

In evaluating a claim for violation of the right to privacy, a court must determine whether a person has exhibited a reasonable expectation of privacy and, if so, whether that expectation has been violated by unreasonable government intrusion.43[50] Applying this determination to these cases, the important inquiries are: first, did the directors and
39[46] 40[47] 41[48]

Marquez v. Desierto, G.R. No. 135882, June 27, 2001, 359 SCRA 772. See Morfe v. Mutuc No. L-20387, January 31, 1968, 22 SCRA 424. Article 12 of the Universal Declaration of Human Rights. See also Article 17 (1) and (2) of the International Covenant on Civil and Political Rights. Constitutional and Legal Systems of ASEAN Countries, Sison, Academy of ASEAN Law and Jurisprudence, 1990, at 221, citing I.R. Cortes, The Constitutional Foundations of Privacy, 7 (1970). Burrows v. Superior Court of San Bernardino County , 13 Cal. 3d 238, 529 P 2d 590 (1974). See Katz v. United states (1967), 389 U.S. 347, 350-352, 88 S. Ct. 507, 19 L. Ed. 2d 576; People v. Krivda (1971) 5 Cal. 3d 357, 364, 96 Cal. Rptr. 62, 486 P. 2d 1262; 8 Cal. 3d 623-624,105 Cal. Rptr. 521, 504 P. 2d 457. INSERT Herreras Handbook on Arrest, Search and Seizure.

42[49]

43[50]

166

167

officers of Philcomsat Holdings Corporation exhibit a reasonable expectation of privacy?; and second, did the government violate such expectation?

The answers are in the negative. Petitioners were invited in the Senates public hearing to deliberate on Senate Res. No. 455, particularly on the anomalous losses incurred by the Philippine Overseas Telecommunications Corporation (POTC), Philippine Communications Satellite Corporation (PHILCOMSAT), and Philcomsat Holdings Corporations (PHC) due to the alleged improprieties in the operations by their respective board of directors. Obviously, the inquiry focus on petitioners acts committed in the discharge of their duties as officers and directors of the said corporations, particularly Philcomsat Holdings Corporation. Consequently, they have no reasonable expectation of privacy over matters involving their offices in a corporation where the government has interest. Certainly, such matters are of public concern and over which the people have the right to information.

This goes to show that the right to privacy is not absolute where there is an overriding compelling state interest. In Morfe v. Mutuc,44[51] the Court, in line with Whalen v. Roe,45[52] employed the rational basis relationship test when it held that there was no infringement of the individuals right to privacy as the requirement to disclosure information is for a valid purpose, i.e., to curtail and minimize the opportunities for official corruption, maintain a standard of honesty in public service, and promote morality in public administration.46[53] In Valmonte v. Belmonte,47[54] the Court remarked that as public figures, the Members of the former Batasang Pambansa enjoy a more limited right to privacy as compared to ordinary individuals, and their actions are subject to closer scrutiny. Taking this into consideration, the Court ruled that the right of the people to access information on matters of public concern prevails over the right to privacy of financial transactions.

Under the present circumstances, the alleged anomalies in the PHILCOMSAT, PHC and POTC, ranging in millions of pesos, and the conspiratorial participation of the PCGG and its officials are compelling reasons for the Senate to exact vital information from the directors and officers of Philcomsat Holdings Corporations, as well as from Chairman Sabio and his Commissioners to aid it in crafting the necessary legislation to prevent corruption and formulate remedial measures and policy determination regarding PCGGs efficacy. There being no reasonable expectation of privacy on the part of those directors and officers over the subject covered by Senate Res. No. 455, it follows that their right to privacy has not been violated by respondent Senate Committees.

44[51] 45[52] 46[53] 47[54]

Supra. 429 U.S. 589 (1977). Justice Puno, Lecture on Legislative Inquiry and Right to Privacy, p. 60. 170 SCRA 256 (1989)

167

168

Let it be stressed at this point that so long as the constitutional rights of witnesses, like Chairman Sabio and his Commissioners, will be respected by respondent Senate Committees, it their duty to cooperate with them in their efforts to obtain the facts needed for intelligent legislative action. The unremitting obligation of every citizen is to respond to subpoenae, to respect the dignity of the Congress and its Committees, and to testify fully with respect to matters within the realm of proper investigation. In fine, PCGG Chairman Camilo Sabio and Commissioners Ricardo Abcede, Narciso Nario, Nicasio Conti, and Tereso Javier; and Manuel Andal and Julio Jalandoni, PCGGs nominees to Philcomsat Holdings Corporation, as well as its directors and officers, must comply with the Subpoenae Ad Testificandum issued by respondent Senate Committees directing them to appear and testify in public hearings relative to Senate Resolution No. 455.

CHAPTER V - FREEDOM OF SPEECH, PRESS, EXPRESSION, etc.

Section 4. No law shall be passed abridging the freedom of speech, of expression, or of the press, or the right of the people peaceably to assemble and petition the government for the redress of their grievances. NOTE: Applicable provisions of the Human Security Act/Anti-Terrorism Law, Republic Act No. 9372, Approved on March 6, 2007 and effective on July 15, 2007 (This Law shall be automatically suspended one (1) month before and two (2) months after the holding of any election) Section 26 provides that persons who have been charged with terrorism or conspiracy to commit terrorism---even if they have been granted bail because evidence of guilt is not strongcan be: Detained under house arrest; Restricted from traveling; and/or Prohibited from using any cellular phones, computers, or other means of communications with people outside their residence. 1. Rule on criticisms against acts of public officers Read: 1. Espuelas vs. People, 90 Phil. 524 2. US vs. Bustos, 37 Phil. 731 (A public official should not be onionskinned with reference to comments upon his official acts. The interest of the government and the society demands full discussion of public affairs) 3. P. vs. Perez, 45 Phil. 599 4. Mercado vs. CFI, 116 SCRA 93 2. Freedom of the press, in general
168

169

Read: BAGUIO MIDLAND COURIER & CECILLE AFABLE VS. COURT OF APPEALS & RAMON LABO, JR., 444 SCRA 28 [November 25, 2004] Freedom of Expression; the public has the right to be informed on the mental, moral and physical fitness of candidates for public office. FACTS: 1. In the January 3, 1988 issue of the Baguio Midland Courier (BMC), Cecille Afable, the Editor-in-Chief, in her column In and Out of Baguio made the following comments: Of all the candidates for Mayor of Baguio City), Labo has the most imponderables about him. People would ask: can he read and write? Why is he always talking about his Japanese father-in-law? Is he really a Japanes Senator or a barrio Kapitan? Is it true that he will send P18M aid to Baguio? Somebody wanted to put an advertisement of Labo in the Midland Courier but was refused because he has not yet paid his account of the last time he was a candidate for Congress. We will accept all advertisements for him if he pays his old account first. 2. In the same column, Cecille Afable wrote the following comments in her January 10, 1988 column at the Courier: I heard that the Dumpty in the Egg is campaigning for Cortes. Not fair. Some real doctors are also busy campaigning against Labo because he has not also paid their medical services with them. Since he is donating millions he should also settle his small debts like the reportedly insignificant amount of P27,000 only. If he wins, several teachers were signifying to resign and leave Baguio forever, and Pangasinan will be the franca-liqua of Baguio. 3. As a result of the above articles, Ramon Labor, Jr. filed a complaint for Damages before the regional trial Court of Baguio City as he claimed said articles were libelous. He likewise filed a separate criminal complaint before the Office of the City Prosecutor of Baguio but was dismissed; 4. Labo claimed that the said articles were tainted with malice because he was allegedly described as Dumpty in the Egg or one who is a failure in his business which is false because he is a very successful businessman or to mean zero or a big lie; that he is a balasubas due to his alleged failure to pay his medical expenses; 5. The petitioners, however, were able to prove that Labo has an unpaid obligation to the Courier in the amount of P27,415.00 for the ads placed by his campaigners for the 1984 Batasang Pambansa elections; 6. The Regional Trial Court, Branch 6, Baguio City, in its Decision dated June 14, 1990 dismissed Labos complaint for damages on the ground that the article of petitioner Afable was privileged and constituted fair comment on matters of public interest as it dealt with
169

170

the integrity, reputation and honesty of private respondent Labo who was a candidate for Mayor of Baguio City; 7. On January 7, 1992, the Court of Appeals reversed the RTC Decision and ordered the petitioners to pay Ramon Labo, Jr. damages in the total amount of P350,000.00 after concluding that the Dumpty in the Egg refers to no one but Labo himself. Hence, the Petition to the Supreme Court. ISSUES: Was Labo the Dumpty in the Egg described in the questioned article/ Were the articles subject of the case libelous or privileged/ HELD: 1. The Court of Appeals is wrong when it held that Labo is the Dumpty in the Egg in the questioned article. This is so because the article stated that The Dumpty in the Egg is campaigning for Cortes, another candidate for mayor and opponent of Labo himself. It is unbelievable that Labo campaigned for his opponent and against himself. Although such gracious attitude on the part of Labo would have been commendable, it is contrary to common human experience. As pointed out by the petitioners, had he done that, it is doubtful whether he could have won as City Mayor of Baguio in the 1988 elections, which he actually did. In line with the doctrine in BORJAL VS. CA, 310 SCRA 1, that it is also not sufficient that the offended party recognized himself as the person attacked or defamed, but it must be shown that at least a 3rd person could identify him as the object of the libelous publication, the case should be dismissed since Labo utterly failed to dispose of this responsibility. 2. Labo claims that the petitioners could not invoke public interest to justify the publication since he was not yet a public official at that time. This argument is without merit since he was already a candidate for City mayor of Baguio. As such, the article is still within the mantle of protection guaranteed by the freedom of expression provided in the Constitution since it is the publics right to be informed of the mental, moral and physical fitness of candidates for public office. This was recognized as early as the case of US VS. SEDANO, 14 Phil. 338 [1909] and the case of NEW YORK TIMES VS. SULLIVAN, 376 U.S. 254 where the US Supreme Court held: it is of the utmost consequence that the people should discuss the character and qualifications of candidates for their suffrages. The importance to the State and to society of such discussions is so vast, and the advantages derived so great, that they more than counterbalance the inconvenience of private persons whose conduct may be involved, and occasional injury to the reputations of individuals must yield to the public welfare, although at times such injury may be great. The public benefit from publicity is so great and the chance of injury to private character so small, that such discussion must be privileged. Clearly, the questioned articles constitute fair comment on a matter of public interest as it dealt with the character of the private
170

171

respondent who was running for the top elective post in Baguio City at that time.

PABLITO V. SANIDAD VS. COMELEC, G.R. NO. 90878, January 29, 1990 Freedom of expression and of the press Medialdea, J. Facts: 1. On October 23, 1989, RA 6766, entitled "AN ACT PROVIDING FOR AN ORGANIC ACT FOR THE CORDILLERA AUTONOMOUS REGION" was enacted into law; 2. Pursuant to said law, the City of Baguio and Provinces of Benguet, Abra, Mt. Province, Ifugao and Kalinga-Apayao, all comprising the autonomous region shall take part in a plebiscite originally scheduled for December 27, 1989 but was reset to January 30, 1990 specifically for the ratification or rejection of the said act; 3. By virtue of the 1987 Constitution and the Omnibus Election Code (BP 881), the Comelec issued Comelec Resolution No. 2167, Section 19 of which provides: "Section 19. Prohibition on columnist, commentators or announcers.- During the plebiscite campaign period, on the day before and on plebiscite day, no mass media columnist, commentator, announcer or personality shall use his column or radio or television time to campaign for or against the plebiscite issues." 4. On November 20, 1989, petitioner PABLITO V. SANIDAD who is a columnist ("OVERVIEW") for the Baguio Midland Courier, a weekly newspaper circulated in the City of Baguio and the Cordilleras, filed a petition for Prohibition with prayer for the issuance of a temporary restraining order or a writ of preliminary injunction against the Comelec to enjoin the latter from enforcing Section 19 of resolution No. 2167. Petitioner claims that the said provision is violative of his constitutional freedom of expression and of the press and it also constitutes a prior restraint because it imposes subsequent punishment for those who violate the same; 5. On November 28, 1989, the Supreme Court issued a temporary restraining order enjoining the respondent from enforcing Section 19 of Resolution No. 2167; 6. On January 9, 1990, Comelec through the Solicitor General filed its Comment and moved for the dismissal of the petition on the ground that Section 19 of Resolution No. 2167 does not absolutely bar the petitioner from expressing his views because under Section 90 and 92 of BP 881, he may still express his views or campaign for or against the act through the Comelec space and airtime. Held:
171

172

What is granted by Art. IX-C of the Constitution to the Comelec is the power to supervise and regulate the use and enjoyment of franchises, permits or other grants issued for the operation of transportation or other public utilities to the end that equal opportunity, time and space, and the right to reply, including reasonable, equal rates therefor, for public information campaigns and forums among candidates are insured. The evil sought to be prevented by this provision is the possibility that a franchise holder may favor or give undue advantage to a candidate in terms of advertising time and space. This is also the reason why a columnist, commentator or announcer is required to take a leave of absence from his work during the campaign period if he is a candidate. HOWEVER, NEITHER ARTICLE IX-C OF THE CONSTITUTION NOR SECTION 11(B), 2ND PAR. OF RA 6646 CAN BE CONSTRUED TO MEAN THAT THE COMELEC HAS ALSO BEEN GRANTED THE RIGHT TO SUPERVISE AND REGULATE THE EXERCISE BY MEDIA PRACTITIONERS THEMSELVES OF THEIR RIGHT TO EXPRESSION DURING THE PLEBISCITE PERIODS. Media practitioners exercising their freedom of expression during the plebiscite periods are neither the franchise holders nor the candidates. In fact, there are no candidates in a plebiscite. While it is true that the petitioner is not absolutely barred from campaigning for or against the Organic Act, said fact does not cure the constitutional infirmity of Section 19, Comelec Resolution No. 2167. This is so because IT IS STILL A RESTRICTION ON HIS CHOICE OF THE FORUM WHERE HE MAY EXPRESS HIS VIEW. Plebiscite issues are matters of public concern and importance. The people's right to be informed and to be able to freely and intelligently make a decision would be better served by access to an unabridged discussion of the issues, INCLUDING THE FORUM. The people affected by the issues presented in a plebiscite should not be unduly burdened by restrictions on the forum where the right to expression may be exercised. ACCORDINGLY, Section 19 of Comelec Resolution No. 2167 is hereby declared UNCONSTITUTIONAL. Read also: 1. 2. 3. 4. 5. 6. In re: Ramon Tulfo,March 19, 199 In re: Atty. Emil Jurado, July 12, 1990 Burgos vs. Chief of Staff, 133 SCRA 800 Corro vs. Lising, 137 SCRA 448 Babst vs. NIB, 132 SCRA 316 Elizalde vs. Gutierrez,76 SCRA 448 (In order that any news item relating to a judicial proceeding will not be actionable for being libelous, the same must be [a] a true and fair report of the actual proceedings; [b] must be done in good faith; and [c] no comments nor remarks shall be made by the writer. 7. Policarpio vs. Manila Times, 5 SCRA 148 8. Lopez vs. CA, 34 SCRA 116 9. New York Times vs. Sullivan,376 U.S.254 10. Liwayway Publishing vs. PCGG, April 15,l988 3. Freedom of expression in general
172

173

Read: 1. RANDY DAVID VS. ARROYO, May 3, 2006, 489 SCRA 160; 2. Adiong vs. Comelec, March 31, 1992 (putting of decals and stickers in ones car is within the protected freedom of expression) 3. National Press Club vs. Comelec, March 5, 1992. Real also the dissenting and separate opinions of the justices. (Preventing campaigns through radio, TV and newspapers is valid in order to even the playing field between rich and poor candidates) 4. Zaldivar vs. Sandiganbayan, GR No. 7960-707 & Zaldivar vs. Gonzales, GR No. 80578, February 1, 1989 5. Eastern Broadcasting vs. Dans,137 SCRA 628 6. Newsweek vs. IAC, 142 SCRA 171 7. Kapisanan vs. Camara Shoes, 11 SCRA 477 8. IN RE: Atty. Tipon, 79 SCRA 372 9. Lacsa vs. IAC, May 23,1988 10. Kapunan vs. De Villa, December 6, 1988 4. Not within the protection of the freedom of Constitution 1. Obscenity; test of Read: a. P. vs. Kottinger, 45 Phil. 352 d. P vs. GO PIN, August 8, 1955 Tests: a. Whether the average person applying to contemporary community standards would find the work appeals to prurient interest; b. Whether the work depicts or describes a patently offensive sexual conduct; c. Whether the work as a whole lacks serious literary , artistic, political or scientific value. c. Miller vs. California, 37 L. Ed. 2d 419 d. Ginsberg vs. New York,390 U.S. 629 e. Pita vs. CA, 178 SCRA 362 (A City Mayor may not order the warrantless seizure of magazines which he believes to be obscene; otherwise, he will become the complainant, prosecutor and judge at the same time. He should obtain a search warrant from a judge) 2. Libel or slander; test ofRead: a. Lopez and Manila Times cases, supra b. Quisumbing vs. Lopez, 96 Phil. 510 3. Cases undersub-judice Read: a. P. vs. Alarcon, 69 Phil. 265
173

expression clause of the

174

5. Freedom of assembly and to petition the government grievances

for redress of

INTEGRATED BAR OF THE PHILIPPINES VS. MANILA MAYOR JOSE LITO ATIENZA, G.R. No. 175241, February 24, 2010 CARPIO MORALES, J.: Petitioners Integrated Bar of the Philippines (IBP) and lawyers H. Harry L. Roque and Joel R. Butuyan appeal the June 28, 2006 Decision and the October 26, 2006 Resolution of the Court of Appeals that found no grave abuse of discretion on the part of respondent Jose Lito Atienza, the then mayor of Manila, in granting a permit to rally in a venue other than the one applied for by the IBP. On June 15, 2006, the IBP, through its then National President Jose Anselmo Cadiz (Cadiz), filed with the Office of the City Mayor of Manila a letter application for a permit to rally at the foot of Mendiola Bridge on June 22, 2006 from 2:30 p.m. to 5:30 p.m. to be participated in by IBP officers and members, law students and multi-sectoral organizations. Respondent issued a permit dated June 16, 2006 allowing the IBP to stage a rally on given date but indicated therein Plaza Miranda as the venue, instead of Mendiola Bridge, which permit the IBP received on June 19, 2006. Aggrieved, petitioners filed on June 21, 2006 before the Court of Appeals a petition for certiorari docketed as CA-G.R. SP No. 94949. The petition having been unresolved within 24 hours from its filing, petitioners filed before this Court on June 22, 2006 a petition for certiorari docketed as G.R. No. 172951 which assailed the appellate courts inaction or refusal to resolve the petition within the period provided under the Public Assembly Act of 1985. The Court, by Resolutions of July 26, 2006, August 30, 2006 and November 20, 2006, respectively, denied the petition for being moot and academic, denied the relief that the petition be heard on the merits in view of the pendency of CA-G.R. SP No. 94949, and denied the motion for reconsideration. The rally pushed through on June 22, 2006 at Mendiola Bridge, after Cadiz discussed with P/Supt. Arturo Paglinawan whose contingent from the Manila Police District (MPD) earlier barred petitioners from proceeding thereto. Petitioners allege that the participants voluntarily dispersed after the peaceful conduct of the program. The MPD thereupon instituted on June 26, 2006 a criminal action, docketed as I.S. No. 06I-12501, against Cadiz for violating the Public Assembly Act in staging a rally at a venue not indicated in the permit, to which charge Cadiz filed a Counter-Affidavit of August 3, 2006. In the meantime, the appellate court ruled, in CA-G.R. SP No. 94949, by the first assailed issuance, that the petition became moot and lacked merit. The appellate court also denied petitioners motion for reconsideration by the second assailed issuance.

174

175

Hence, the filing of the present petition for review on certiorari, to which respondent filed his Comment of November 18, 2008 which merited petitioners Reply of October 2, 2009. ISSUE: The main issue is whether the appellate court erred in holding that the modification of the venue in IBPs rally permit does not constitute grave abuse of discretion. Petitioners assert that the partial grant of the application runs contrary to the Pubic Assembly Act and violates their constitutional right to freedom of expression and public assembly. HELD: The Court shall first resolve the preliminary issue of mootness. Undoubtedly, the petition filed with the appellate court on June 21, 2006 became moot upon the passing of the date of the rally on June 22, 2006. A moot and academic case is one that ceases to present a justiciable controversy by virtue of supervening events, so that a declaration thereon would be of no practical use or value. Generally, courts decline jurisdiction over such case or dismiss it on ground of mootness. However, even in cases where supervening events had made the cases moot, this Court did not hesitate to resolve the legal or constitutional issues raised to formulate controlling principles to guide the bench, bar and public. Moreover, as an exception to the rule on mootness, courts will decide a question otherwise moot if it is capable of repetition, yet evading review. In the present case, the question of the legality of a modification of a permit to rally will arise each time the terms of an intended rally are altered by the concerned official, yet it evades review, owing to the limited time in processing the application where the shortest allowable period is five days prior to the assembly. The susceptibility of recurrence compels the Court to definitively resolve the issue at hand. Section 6 of the Public Assembly Act reads: Section 6. Action to be taken on the application (a) It shall be the duty of the mayor or any official acting in his behalf to issue or grant a permit unless there is clear and convincing evidence that the public assembly will create a clear and present danger to public order, public safety, public convenience, public morals or public health. (b) The mayor or any official acting in his behalf shall act on the application within two (2) working days from the date the application was filed, failing which, the permit shall be deemed granted. Should for any reason the mayor or any official acting in his behalf refuse to accept the application for a permit, said application shall be posted by the applicant on the premises of the office of the mayor and shall be deemed to have been filed.

175

176

(c) If the mayor is of the view that there is imminent and grave danger of a substantive evil warranting the denial or modification of the permit, he shall immediately inform the applicant who must be heard on the matter. (d) The action on the permit shall be in writing and served on the application [sic] within twenty-four hours. (e) If the mayor or any official acting in his behalf denies the application or modifies the terms thereof in his permit, the applicant may contest the decision in an appropriate court of law. (f) In case suit is brought before the Metropolitan Trial Court, the Municipal Trial Court, the Municipal Circuit Trial Court, the Regional Trial Court, or the Intermediate Appellate Court, its decisions may be appealed to the appropriate court within fortyeight (48) hours after receipt of the same. No appeal bond and record on appeal shall be required. A decision granting such permit or modifying it in terms satisfactory to the applicant shall, be immediately executory. (g) All cases filed in court under this Section shall be decided within twenty-four (24) hours from date of filing. Cases filed hereunder shall be immediately endorsed to the executive judge for disposition or, in his absence, to the next in rank. (h) In all cases, any decision may be appealed to the Supreme Court. (i) Telegraphic appeals to be followed by formal appeals are hereby allowed. (underscoring supplied) In Bayan, Karapatan, Kilusang Magbubukid ng Pilipinas (KMP) v. Ermita, the Court reiterated: x x x Freedom of assembly connotes the right of the people to meet peaceably for consultation and discussion of matters of public concern. It is entitled to be accorded the utmost deference and respect. It is not to be limited, much less denied, except on a showing, as is the case with freedom of expression, of a clear and present danger of a substantive evil that the state has a right to prevent. Even prior to the 1935 Constitution, Justice Malcolm had occasion to stress that it is a necessary consequence of our republican institutions and complements the right of free speech. To paraphrase the opinion of Justice Rutledge, speaking for the majority of the American Supreme Court in Thomas v. Collins, it was not by accident or coincidence that the rights to freedom of speech and of the press were coupled in a single guarantee with the rights of the people peaceably to assemble and to petition the government for redress of grievances. All these rights, while not identical, are inseparable. In every case, therefore, where there is a limitation placed on the exercise of this right, the judiciary is called upon to examine the effects of the challenged governmental actuation. The sole justification for a limitation on the exercise of this right, so fundamental to the maintenance of democratic institutions, is the danger, of a character both grave and imminent, of a serious evil to public safety, public morals,
176

177

public health, or any other legitimate public interest. (emphasis supplied) The Court in Bayan stated that the provisions of the Public Assembly Act of 1985 practically codified the 1983 ruling in Reyes v. Bagatsing. In juxtaposing Sections 4 to 6 of the Public Assembly Act with the pertinent portion of the Reyes case, the Court elucidated as follows: x x x [The public official concerned shall] appraise whether there may be valid objections to the grant of the permit or to its grant but at another public place. It is an indispensable condition to such refusal or modification that the clear and present danger test be the standard for the decision reached. If he is of the view that there is such an imminent and grave danger of a substantive evil, the applicants must be heard on the matter. Thereafter, his decision, whether favorable or adverse, must be transmitted to them at the earliest opportunity. Thus if so minded, they can have recourse to the proper judicial authority. (italics and underscoring supplied) In modifying the permit outright, respondent gravely abused his discretion when he did not immediately inform the IBP who should have been heard first on the matter of his perceived imminent and grave danger of a substantive evil that may warrant the changing of the venue. The opportunity to be heard precedes the action on the permit, since the applicant may directly go to court after an unfavorable action on the permit. Respondent failed to indicate how he had arrived at modifying the terms of the permit against the standard of a clear and present danger test which, it bears repeating, is an indispensable condition to such modification. Nothing in the issued permit adverts to an imminent and grave danger of a substantive evil, which blank denial or modification would, when granted imprimatur as the appellate court would have it, render illusory any judicial scrutiny thereof. It is true that the licensing official, here respondent Mayor, is not devoid of discretion in determining whether or not a permit would be granted. It is not, however, unfettered discretion. While prudence requires that there be a realistic appraisal not of what may possibly occur but of what may probably occur, given all the relevant circumstances, still the assumption especially so where the assembly is scheduled for a specific public place is that the permit must be for the assembly being held there. The exercise of such a right, in the language of Justice Roberts, speaking for the American Supreme Court, is not to be "abridged on the plea that it may be exercised in some other place. (emphasis and underscoring supplied) Notably, respondent failed to indicate in his Comment any basis or explanation for his action. It smacks of whim and caprice for respondent to just impose a change of venue for an assembly that was slated for a specific public place. It is thus reversible error for the appellate court not to have found such grave abuse of discretion. The Court DECLARES that respondent committed grave abuse of discretion in modifying the rally permit issued on June 16, 2006 insofar as it altered the venue from Mendiola Bridge to Plaza Miranda.

GESITE et al. vs. COURT OF APPEALS, 444 SCRA 51


177

178

Freedom of public school teachers to peaceably assemble and petition the government for redress of grievances; right of public school teachers to form union. The petitioners admitted that they participated in concerted mass actions in Metro Manila from September to the first half of October, 1990 which temporarily disrupted classes in Metro Manila but they claimed that they were not on strike. They claimed that they were merely exercising their constitutional right to peaceably assemble and petition the government for redress of their grievances. Thus, they may not be penalized administratively. HELD: The issue of whether or not the mass action launched by the public school teachers during the period from September up to the 1 st half of October, 1990 was a strike or not has been decided in the case of MANILA PUBLIC SCHOOL TEACHERS ASSOCIATION VS. LAGUIO, 200 SCRA 323 where it was held that these mass actions were to all intents and purposes a strike; they constituted a concerted and unauthorized stoppage of, or absence from, work which it was the teachers duty to perform, undertaken for essentially economic reasons. It is undisputed fact that there was a work stoppage and that petitioners purpose was to realize their demands by withholding their services. The fact that the conventional term strike was not used by the striking employees to describe their common course of action is inconsequential, SINCE THE SUBSTANCE OF THE SITUATION, AND NOT ITS APPEARANCE, WILL BE DEEMED CONTROLLING. Despite the constitutional right to form associations under the Constitution, employees in the public service may not engage in strikes, mass leaves, walkouts and other forms of mass actions that will lead to temporary stoppage or disruption of public service. The right of government employees to organize IS LIMITED TO THE FORMATIONS OF UNIONS OR ASSOCIATIONS ONLY, WITHOUT INCLUDING THE RIGHT TO STRIKE. (Bangalisan vs. CA, 276 SCRA 619) The petitioners are not therefore entitled to their salaries during their suspension because the general proposition is that a public official is not entitled to any compensation if he had not rendered any service.

BAYAN, KARAPATAN, KILUSANG MAGBUBUKID NG PILIPINAS (KMP), and GABRIELA vs. EDUARDO ERMITA, in his capacity as Executive Secretary, Manila City Mayor LITO ATIENZA, Chief of the Philippine National Police, Gen. ARTURO M. LOMIBAO, NCRPO Chief Maj. Gen. VIDAL QUEROL, and Western Police District Chief Gen. PEDRO BULAONG, G.R. No. 169848, May, 2006 AZCUNA, J.:
178

179

The Facts: Petitioners come in three groups. The first petitioners, Bayan, et al., in G.R. No. 169838, allege that they are citizens and taxpayers of the Philippines and that their rights as organizations and individuals were violated when the rally they participated in on October 6, 2005 was violently dispersed by policemen implementing Batas Pambansa (B.P.) No. 880. The second group consists of 26 individual petitioners, Jess del Prado, et al., in G.R. No. 169848, who allege that they were injured, arrested and detained when a peaceful mass action they held on September 26, 2005 was preempted and violently dispersed by the police. They further assert that on October 5, 2005, a group they participated in marched to Malacaang to protest issuances of the Palace which, they claim, put the country under an undeclared martial rule, and the protest was likewise dispersed violently and many among them were arrested and suffered injuries. The third group, Kilusang Mayo Uno (KMU), et al., petitioners in G.R. No. 169881, allege that they conduct peaceful mass actions and that their rights as organizations and those of their individual members as citizens, specifically the right to peaceful assembly, are affected by Batas Pambansa No. 880 and the policy of Calibrated Preemptive Response (CPR) being followed to implement it. KMU, et al., claim that on October 4, 2005, a rally KMU cosponsored was to be conducted at the Mendiola bridge but police blocked them along C.M. Recto and Lepanto Streets and forcibly dispersed them, causing injuries to several of their members. They further allege that on October 6, 2005, a multi-sectoral rally which KMU also co-sponsored was scheduled to proceed along Espaa Avenue in front of the University of Santo Tomas and going towards Mendiola bridge. Police officers blocked them along Morayta Street and prevented them from proceeding further. They were then forcibly dispersed, causing injuries on one of them. Three other rallyists were arrested. All petitioners assail Batas Pambansa No. 880, some of them in toto and others only Sections 4, 5, 6, 12, 13(a), and 14(a), as well as the policy of CPR. They seek to stop violent dispersals of rallies under the no permit, no rally policy and the CPR policy recently announced. B.P. No. 880, The Public Assembly Act of 1985, provides: Batas Pambansa Blg. 880 An Act Ensuring The Free Exercise By The People Of Their Right Peaceably To Assemble And Petition The Government [And] For Other Purposes Be it enacted by the Batasang Pambansa in session assembled: SECTION 1. Title . This Act shall be known as The Public Assembly Act of 1985.
179

180

SEC. 2. Declaration of policy. The constitutional right of the people peaceably to assemble and petition the government for redress of grievances is essential and vital to the strength and stability of the State. To this end, the State shall ensure the free exercise of such right without prejudice to the rights of others to life, liberty and equal protection of the law. SEC. 3. Definition of terms. For purposes of this Act: (b) Public place shall include any highway, boulevard, avenue, road, street, bridge or other thoroughfare, park, plaza square, and/or any open space of public ownership where the people are allowed access. (c) Maximum tolerance means the highest degree of restraint that the military, police and other peace keeping authorities shall observe during a public assembly or in the dispersal of the same. SEC. 4. Permit when required and when not required.-- A written permit shall be required for any person or persons to organize and hold a public assembly in a public place. However, no permit shall be required if the public assembly shall be done or made in a freedom park duly established by law or ordinance or in private property, in which case only the consent of the owner or the one entitled to its legal possession is required, or in the campus of a government-owned and operated educational institution which shall be subject to the rules and regulations of said educational institution. Political meetings or rallies held during any election campaign period as provided for by law are not covered by this Act. SEC. 5. Application requirements.-- All applications for a permit shall comply with the following guidelines: 1. The applications shall be in writing and shall include the names of the leaders or organizers; the purpose of such public assembly; the date, time and duration thereof, and place or streets to be used for the intended activity; and the probable number of persons participating, the transport and the public address systems to be used. 2. The application shall incorporate responsibility of applicant under Section 8 hereof. the duty and

3. The application shall be filed with the office of the mayor of the city or municipality in whose jurisdiction the intended activity is to be held, at least five (5) working days before the scheduled public assembly. 4. Upon receipt of the application, which must be duly acknowledged in writing, the office of the city or municipal mayor shall cause the same to immediately be posted at a conspicuous place in the city or municipal building. SEC. 6. Action to be taken on the application. 1. It shall be the duty of the mayor or any official acting in his behalf to issue or grant a permit unless there is clear and convincing evidence that the public assembly will create a clear and present danger to
180

181

public order, public safety, public convenience, public morals or public health. 2. The mayor or any official acting in his behalf shall act on the application within two (2) working days from the date the application was filed, failing which, the permit shall be deemed granted. Should for any reason the mayor or any official acting in his behalf refuse to accept the application for a permit, said application shall be posted by the applicant on the premises of the office of the mayor and shall be deemed to have been filed. 3. If the mayor is of the view that there is imminent and grave danger of a substantive evil warranting the denial or modification of the permit, he shall immediately inform the applicant who must be heard on the matter. 4. The action on the permit shall be in writing and served on the applica[nt] within twenty-four hours. 5. If the mayor or any official acting in his behalf denies the application or modifies the terms thereof in his permit, the applicant may contest the decision in an appropriate court of law. 6. In case suit is brought before the Metropolitan Trial Court, the Municipal Trial Court, the Municipal Circuit Trial Court, the Regional Trial Court, or the Intermediate Appellate court, its decisions may be appealed to the appropriate court within forty-eight (48) hours after receipt of the same. No appeal bond and record on appeal shall be required. A decision granting such permit or modifying if in terms satisfactory to the applicant shall be immediately executory. 7. All cases filed in court under this section shall be decided within twenty-four (24) hours from date of filing. Cases filed hereunder shall be immediately endorsed to the executive judge for disposition or, in his absence, to the next in rank. 8. Court. CPR, on the other hand, is a policy set forth in a press release by Malacaang dated September 21, 2005, shown in Annex A to the Petition in G.R. No. 169848, thus: Malacaang Manila, Philippines Release No. 2 Official NEWS September 21, 2005 In all cases, any decision may be appealed to the Supreme

STATEMENT OF EXECUTIVE SECRETARY EDUARDO ERMITA On Unlawful Mass Actions In view of intelligence reports pointing to credible plans of antigovernment groups to inflame the political situation, sow disorder and incite people against the duty constituted authorities, we have instructed the PNP as well as the local government units to strictly enforce a no
181

182

permit, no rally policy, disperse groups that run afoul of this standard and arrest all persons violating the laws of the land as well as ordinances on the proper conduct of mass actions and demonstrations. The rule of calibrated preemptive response is now in force, in lieu of maximum tolerance. The authorities will not stand aside while those with ill intent are herding a witting or unwitting mass of people and inciting them into actions that are inimical to public order, and the peace of mind of the national community. Unlawful mass actions will be dispersed. The majority of lawabiding citizens have the right to be protected by a vigilant and proactive government. We appeal to the detractors of the government to engage in lawful and peaceful conduct befitting of a democratic society. The Presidents call for unity and reconciliation stands, based on the rule of law. Petitioners Bayan, et al., contend that Batas Pambansa No. 880 is clearly a violation of the Constitution and the International Covenant on Civil and Political Rights and other human rights treaties of which the Philippines is a signatory. They argue that B.P. No. 880 requires a permit before one can stage a public assembly regardless of the presence or absence of a clear and present danger. It also curtails the choice of venue and is thus repugnant to the freedom of expression clause as the time and place of a public assembly form part of the message for which the expression is sought. Furthermore, it is not content-neutral as it does not apply to mass actions in support of the government. The words lawful cause, opinion, protesting or influencing suggest the exposition of some cause not espoused by the government. Also, the phrase maximum tolerance shows that the law applies to assemblies against the government because they are being tolerated. As a content-based legislation, it cannot pass the strict scrutiny test. Furthermore, the law delegates powers to the Mayor without providing clear standards. The two standards stated in the laws (clear and present danger and imminent and grave danger) are inconsistent. Regarding the CPR policy, it is void for being an ultra vires act that alters the standard of maximum tolerance set forth in B.P. No. 880, aside from being void for being vague and for lack of publication. Finally, petitioners KMU, et al., argue that the Constitution sets no limits on the right to assembly and therefore B.P. No. 880 cannot put the prior requirement of securing a permit. And even assuming that the legislature can set limits to this right, the limits provided are unreasonable: First, allowing the Mayor to deny the permit on clear and convincing evidence of a clear and present danger is too comprehensive. Second, the five-day requirement to apply for a permit is too long as certain events require instant public assembly, otherwise interest on the issue would possibly wane.
182

183

As to the CPR policy, they argue that it is preemptive, that the government takes action even before the rallyists can perform their act, and that no law, ordinance or executive order supports the policy. Furthermore, it contravenes the maximum tolerance policy of B.P. No. 880 and violates the Constitution as it causes a chilling effect on the exercise by the people of the right to peaceably assemble. I s s u e s: 4. On the constitutionality of Batas Pambansa No. 880, specifically Sections 4, 5, 6, 12 13(a) and 14(a) thereof, and Republic Act No. 7160: Are these content-neutral or content-based regulations? Are they void on grounds of overbreadth or vagueness? Do they constitute prior restraint? Are they undue delegations of powers to Mayors? Do they violate international human rights treaties and the Universal Declaration of Human Rights? On the constitutionality and legality of the policy of Calibrated Preemptive Response (CPR): Is the policy void on its face or due to vagueness? Is it void for lack of publication? Is the policy of CPR void as applied to the rallies of September 26 and October 4, 5 and 6, 2005? H e l d: Petitioners standing cannot be seriously challenged. Their right as citizens to engage in peaceful assembly and exercise the right of petition, as guaranteed by the Constitution, is directly affected by B.P. No. 880 which requires a permit for all who would publicly assemble in the nations streets and parks. They have, in fact, purposely engaged in public assemblies without the required permits to press their claim that no such permit can be validly required without violating the Constitutional guarantee. Respondents, on the other hand, have challenged such action as contrary to law and dispersed the public assemblies held without the permit. Section 4 of Article III of the Constitution provides: SEC. 4. No law shall be passed abridging the freedom of speech, of expression, or of the press, or the right of the people peaceably to assemble and petition the government for redress of grievances. The first point to mark is that the right to peaceably assemble and petition for redress of grievances is, together with freedom of speech, of expression, and of the press, a right that enjoys primacy in the realm of constitutional protection. For these rights constitute the very basis of a functional democratic polity, without which all the other rights would be meaningless and unprotected. As stated in Jacinto v. CA, the Court, as early as the onset of this century, in U.S. v. Apurado already upheld the right to assembly and petition, as follows: There is no question as to the petitioners rights to peaceful assembly to petition the government for a redress of grievances and, for
183

1. 2. 3. 4. 5.

5.

1. 2. 3.

184

that matter, to organize or form associations for purposes not contrary to law, as well as to engage in peaceful concerted activities. These rights are guaranteed by no less than the Constitution, particularly Sections 4 and 8 of the Bill of Rights, Section 2(5) of Article IX, and Section 3 of Article XIII. Jurisprudence abounds with hallowed pronouncements defending and promoting the peoples exercise of these rights. As early as the onset of this century, this Court in U.S. vs. Apurado, already upheld the right to assembly and petition and even went as far as to acknowledge: It is rather to be expected that more or less disorder will mark the public assembly of the people to protest against grievances whether real or imaginary, because on such occasions feeling is always wrought to a high pitch of excitement, and the greater, the grievance and the more intense the feeling, the less perfect, as a rule will be the disciplinary control of the leaders over their irresponsible followers. But if the prosecution be permitted to seize upon every instance of such disorderly conduct by individual members of a crowd as an excuse to characterize the assembly as a seditious and tumultuous rising against the authorities, then the right to assemble and to petition for redress of grievances would expose all those who took part therein to the severest and most unmerited punishment, if the purposes which they sought to attain did not happen to be pleasing to the prosecuting authorities. If instances of disorderly conduct occur on such occasions, the guilty individuals should be sought out and punished therefor, but the utmost discretion must be exercised in drawing the line between disorderly and seditious conduct and between an essentially peaceable assembly and a tumultuous uprising. Again, in Primicias v. Fugoso, the Court likewise sustained the primacy of freedom of speech and to assembly and petition over comfort and convenience in the use of streets and parks. Next, however, it must be remembered that the right, while sacrosanct, is not absolute. In Primicias, this Court said: The right to freedom of speech, and to peacefully assemble and petition the government for redress of grievances, are fundamental personal rights of the people recognized and guaranteed by the constitutions of democratic countries. But it is a settled principle growing out of the nature of well-ordered civil societies that the exercise of those rights is not absolute for it may be so regulated that it shall not be injurious to the equal enjoyment of others having equal rights, nor injurious to the rights of the community or society. The power to regulate the exercise of such and other constitutional rights is termed the sovereign police power, which is the power to prescribe regulations, to promote the health, morals, peace, education, good order or safety, and general welfare of the people. This sovereign police power is exercised by the government through its legislative branch by the enactment of laws regulating those and other constitutional and civil rights, and it may be delegated to political subdivisions, such as towns, municipalities and cities by authorizing their legislative bodies called municipal and city councils enact ordinances for purpose Reyes v. Bagatsing further expounded on the right and its limits, as follows: 1. It is thus clear that the Court is called upon to protect the exercise of the cognate rights to free speech and peaceful assembly,
184

185

arising from the denial of a permit. The Constitution is quite explicit: No law shall be passed abridging the freedom of speech, or of the press, or the right of the people peaceably to assemble and petition the Government for redress of grievances. Free speech, like free press, may be identified with the liberty to discuss publicly and truthfully any matter of public concern without censorship or punishment. There is to be then no previous restraint on the communication of views or subsequent liability whether in libel suits, prosecution for sedition, or action for damages, or contempt proceedings unless there be a clear and present danger of a substantive evil that [the State] has a right to prevent. Freedom of assembly connotes the right of the people to meet peaceably for consultation and discussion of matters of public concern. It is entitled to be accorded the utmost deference and respect. It is not to be limited, much less denied, except on a showing, as is the case with freedom of expression, of a clear and present danger of a substantive evil that the state has a right to prevent. Even prior to the 1935 Constitution, Justice Malcolm had occasion to stress that it is a necessary consequence of our republican institutions and complements the right of free speech. To paraphrase the opinion of Justice Rutledge, speaking for the majority of the American Supreme Court in Thomas v. Collins, it was not by accident or coincidence that the rights to freedom of speech and of the press were coupled in a single guarantee with the right of the people peaceably to assemble and to petition the government for redress of grievances. All these rights, while not identical, are inseparable. In every case, therefore, where there is a limitation placed on the exercise of this right, the judiciary is called upon to examine the effects of the challenged governmental actuation. The sole justification for a limitation on the exercise of this right, so fundamental to the maintenance of democratic institutions, is the danger, of a character both grave and imminent, of a serious evil to public safety, public morals, public health, or any other legitimate public interest. 2. Nowhere is the rationale that underlies the freedom of expression and peaceable assembly better expressed than in this excerpt from an opinion of Justice Frankfurter: It must never be forgotten, however, that the Bill of Rights was the child of the Enlightenment. Back of the guaranty of free speech lay faith in the power of an appeal to reason by all the peaceful means for gaining access to the mind. It was in order to avert force and explosions due to restrictions upon rational modes of communication that the guaranty of free speech was given a generous scope. But utterance in a context of violence can lose its significance as an appeal to reason and become part of an instrument of force. Such utterance was not meant to be sheltered by the Constitution. What was rightfully stressed is the abandonment of reason, the utterance, whether verbal or printed, being in a context of violence. It must always be remembered that this right likewise provides for a safety valve, allowing parties the opportunity to give vent to their views, even if contrary to the prevailing climate of opinion. For if the peaceful means of communication cannot be availed of, resort to non-peaceful means may be the only alternative. Nor is this the sole reason for the expression of dissent. It means more than just the right to be heard of the person who feels aggrieved or who is dissatisfied with things as they are. Its value may lie in the fact that there may be something worth hearing from the dissenter. That is to ensure a true ferment of ideas. There are, of course, well-defined limits. What is guaranteed is peaceable assembly. One may not advocate disorder in the name of protest, much less preach rebellion under the cloak of dissent. The Constitution frowns on disorder or tumult attending a rally or assembly. Resort to force is ruled out and outbreaks of
185

186

violence to be avoided. The utmost calm though is not required. As pointed out in an early Philippine case, penned in 1907 to be precise, United States v. Apurado: It is rather to be expected that more or less disorder will mark the public assembly of the people to protest against grievances whether real or imaginary, because on such occasions feeling is always wrought to a high pitch of excitement, and the greater the grievance and the more intense the feeling, the less perfect, as a rule, will be the disciplinary control of the leaders over their irresponsible followers. It bears repeating that for the constitutional right to be invoked, riotous conduct, injury to property, and acts of vandalism must be avoided. To give free rein to ones destructive urges is to call for condemnation. It is to make a mockery of the high estate occupied by intellectual liberty in our scheme of values. There can be no legal objection, absent the existence of a clear and present danger of a substantive evil, on the choice of Luneta as the place where the peace rally would start. The Philippines is committed to the view expressed in the plurality opinion, of 1939 vintage of, Justice Roberts in Hague v. CIO: Whenever the title of streets and parks may rest, they have immemorially been held in trust for the use of the public and, time out of mind, have been used for purposes of assembly, communicating thoughts between citizens, and discussing public questions. Such use of the streets and public places has, from ancient times, been a part of the privileges, immunities, rights and liberties of citizens. The privilege of a citizen of the United States to use the streets and parks for communication of views on national questions may be regulated in the interest of all; it is not absolute, but relative, and must be exercised in subordination to the general comfort and convenience, and in consonance with peace and good order; but must not, in the guise of respondents, be abridged or denied. The above excerpt was quoted with approval in Primicias v. Fugoso. Primicias made explicit what was implicit in Municipality of Cavite v. Rojas, a 1915 decision, where this Court categorically affirmed that plazas or parks and streets are outside the commerce of man and thus nullified a contract that leased Plaza Soledad of plaintiff-municipality. Reference was made to such plaza being a promenade for public use, which certainly is not the only purpose that it could serve. To repeat, there can be no valid reason why a permit should not be granted for the proposed march and rally starting from a public park that is the Luneta. 4. Neither can there be any valid objection to the use of the streets to the gates of the US embassy, hardly two blocks away at the Roxas Boulevard. Primicias v. Fugoso has resolved any lurking doubt on the matter. In holding that the then Mayor Fugoso of the City of Manila should grant a permit for a public meeting at Plaza Miranda in Quiapo, this Court categorically declared: Our conclusion finds support in the decision in the case of Willis Cox v. State of New Hampshire, 312 U.S., 569. In that case, the statute of New Hampshire P.L. chap. 145, section 2, providing that no parade or procession upon any ground abutting thereon, shall be permitted unless a special license therefor shall first be obtained from the selectmen of the town or from licensing committee, was construed by the Supreme Court of New Hampshire as not conferring upon the licensing board unfettered discretion to refuse to grant the license, and held valid. And the Supreme Court of the United States, in its decision (1941) penned by Chief Justice Hughes affirming the judgment of the State Supreme Court, held that a statute requiring persons using the public streets for a parade or procession to procure a special license
186

187

therefor from the local authorities is not an unconstitutional abridgment of the rights of assembly or of freedom of speech and press, where, as the statute is construed by the state courts, the licensing authorities are strictly limited, in the issuance of licenses, to a consideration of the time, place, and manner of the parade or procession, with a view to conserving the public convenience and of affording an opportunity to provide proper policing, and are not invested with arbitrary discretion to issue or refuse license, * * *. Nor should the point made by Chief Justice Hughes in a subsequent portion of the opinion be ignored: Civil liberties, as guaranteed by the Constitution, imply the existence of an organized society maintaining public order without which liberty itself would be lost in the excesses of unrestricted abuses. The authority of a municipality to impose regulations in order to assure the safety and convenience of the people in the use of public highways has never been regarded as inconsistent with civil liberties but rather as one of the means of safeguarding the good order upon which they ultimately depend. The control of travel on the streets of cities is the most familiar illustration of this recognition of social need. Where a restriction of the use of highways in that relation is designed to promote the public convenience in the interest of all, it cannot be disregarded by the attempted exercise of some civil right which in other circumstances would be entitled to protection. xxx 6. x x x The principle under American doctrines was given utterance by Chief Justice Hughes in these words: The question, if the rights of free speech and peaceable assembly are to be preserved, is not as to the auspices under which the meeting is held but as to its purpose; not as to the relations of the speakers, but whether their utterances transcend the bounds of the freedom of speech which the Constitution protects. There could be danger to public peace and safety if such a gathering were marked by turbulence. That would deprive it of its peaceful character. It is true that the licensing official, here respondent Mayor, is not devoid of discretion in determining whether or not a permit would be granted. It is not, however, unfettered discretion. While prudence requires that there be a realistic appraisal not of what may possibly occur but of what may probably occur, given all the relevant circumstances, still the assumption especially so where the assembly is scheduled for a specific public place is that the permit must be for the assembly being held there. The exercise of such a right, in the language of Justice Roberts, speaking for the American Supreme Court, is not to be abridged on the plea that it may be exercised in some other place. xxx 8. By way of a summary. [a] The applicants for a permit to hold an assembly should inform the licensing authority of the date, the public place where and the time when it will take place. [b] If it were a private place, only the consent of the owner or the one entitled to its legal possession is required. [c] Such application should be filed well ahead in time to enable the public official concerned to appraise whether there may be valid objections to the grant of the permit or to its grant but at another public place. [d] It is an indispensable condition to such refusal or modification that the clear and present danger test be the standard for the decision reached. [e] If he is of the view that there is such an imminent and grave danger of a substantive evil, the applicants must be heard on the matter. [f] Thereafter, his decision, whether favorable or adverse, must be transmitted to them
187

188

at the earliest opportunity. Thus if so minded, they can have recourse to the proper judicial authority. Free speech and peaceable assembly, along with the other intellectual freedoms, are highly ranked in our scheme of constitutional values. It cannot be too strongly stressed that on the judiciary, -- even more so than on the other departments rests the grave and delicate responsibility of assuring respect for and deference to such preferred rights. No verbal formula, no sanctifying phrase can, of course, dispense with what has been so felicitously termed by Justice Holmes as the sovereign prerogative of judgment. Nonetheless, the presumption must be to incline the weight of the scales of justice on the side of such rights, enjoying as they do precedence and primacy. x x x. B.P. No. 880 was enacted after this Court rendered its decision in Reyes. The provisions of B.P. No. 880 practically codify the ruling in Reyes:

Reyes v. Bagatsing (G.R. No. L-65366, November 9, 1983, 125 SCRA 553, 569) 8. By way of a summary. The applicants for a permit to hold an assembly should inform the licensing authority of the date, the public place where and the time when it will take place. If it were a private place, only the consent of the owner or the one entitled to its legal possession is required. Such application should be filed well ahead in time to enable the public official concerned to appraise whether there may be valid objections to the grant of the permit or to its grant but at another public place. It is an indispensable condition to such refusal or modification that the clear and present danger test be the standard for the decision reached. If he is of the view that there is such an imminent and grave danger of a substantive evil, the applicants must be heard

B.P. No. 880

SEC. 4. Permit when required and when not required.-- A written permit shall be required for any person or persons to organize and hold a public assembly in a public place. However, no permit shall be required if the public assembly shall be done or made in a freedom park duly established by law or ordinance or in private property, in which case only the consent of the owner or the one entitled to its legal possession is required, or in the campus of a governmentowned and operated educational institution which shall be subject to the rules and regulations of said educational institution. Political meetings or rallies held during any election campaign period as provided for by law are not covered by this Act. SEC. 5. Application requirements.-All applications for a permit shall comply with the following guidelines: (a) The applications shall
188

189

on the matter. Thereafter, his decision, whether favorable or adverse, must be transmitted to them at the earliest opportunity. Thus if so minded, they can have recourse to the proper judicial authority.

be in writing and shall include the names of the leaders or organizers; the purpose of such public assembly; the date, time and duration thereof, and place or streets to be used for the intended activity; and the probable number of persons participating, the transport and the public address systems to be used. (b) The application shall incorporate the duty and responsibility of applicant under Section 8 hereof. (c) The application shall be filed with the office of the mayor of the city or municipality in whose jurisdiction the intended activity is to be held, at least five (5) working days before the scheduled public assembly. (d) Upon receipt of the application, which must be duly acknowledged in writing, the office of the city or municipal mayor shall cause the same to immediately be posted at a conspicuous place in the city or municipal building. SEC. 6. Action to be taken on the application. (a) It shall be the duty of the mayor or any official acting in his behalf to issue or grant a permit unless there is clear and convincing evidence that the public assembly will create a clear and present danger to public order, public safety, public convenience, public morals or public health. (b) The mayor or any official acting in his behalf shall act on the application within two (2) working days from the date the application was filed, failing which, the permit shall be deemed granted. Should for any reason the mayor or any
189

190

official acting in his behalf refuse to accept the application for a permit, said application shall be posted by the applicant on the premises of the office of the mayor and shall be deemed to have been filed. (c) If the mayor is of the view that there is imminent and grave danger of a substantive evil warranting the denial or modification of the permit, he shall immediately inform the applicant who must be heard on the matter. (d) The action on the permit shall be in writing and served on the applica[nt] within twenty-four hours. (e) If the mayor or any official acting in his behalf denies the application or modifies the terms thereof in his permit, the applicant may contest the decision in an appropriate court of law. (f) In case suit is brought before the Metropolitan Trial Court, the Municipal Trial Court, the Municipal Circuit Trial Court, the Regional Trial Court, or the Intermediate Appellate Court, its decisions may be appealed to the appropriate court within forty-eight (48) hours after receipt of the same. No appeal bond and record on appeal shall be required. A decision granting such permit or modifying it in terms satisfactory to the applicant shall be immediately executory. (g) All cases filed in court under this section shall be decided within twenty-four (24) hours from date of filing. Cases filed hereunder shall be immediately endorsed to the executive judge for disposition or, in his absence, to the next in rank.
190

191

(h) In all cases, any decision may be appealed to the Supreme Court. (i) Telegraphic appeals to be followed by formal appeals are hereby allowed.

It is very clear, therefore, that B.P. No. 880 is not an absolute ban of public assemblies but a restriction that simply regulates the time, place and manner of the assemblies. This was adverted to in Osmea v. Comelec,48 where the Court referred to it as a content-neutral regulation of the time, place, and manner of holding public assemblies.49 A fair and impartial reading of B.P. No. 880 thus readily shows that it refers to all kinds of public assemblies50 that would use public places. The reference to lawful cause does not make it content-based because assemblies really have to be for lawful causes, otherwise they would not be peaceable and entitled to protection. Neither are the words opinion, protesting and influencing in the definition of public assembly content based, since they can refer to any subject. The words petitioning the government for redress of grievances come from the wording of the Constitution, so its use cannot be avoided. Finally, maximum tolerance is for the protection and benefit of all rallyists and is independent of the content of the expressions in the rally. Furthermore, the permit can only be denied on the ground of clear and present danger to public order, public safety, public convenience, public morals or public health. This is a recognized exception to the exercise of the right even under the Universal Declaration of Human Rights and the International Covenant on Civil and Political Rights. Neither is the law overbroad. It regulates the exercise of the right to peaceful assembly and petition only to the extent needed to avoid a clear and present danger of the substantive evils Congress has the right to prevent. There is, likewise, no prior restraint, since the content of the speech is not relevant to the regulation. As to the delegation of powers to the mayor, the law provides a precise and sufficient standard the clear and present danger test stated in Sec. 6(a). The reference to imminent and grave danger of a substantive evil in Sec. 6(c) substantially means the same thing and is not an inconsistent standard. As to whether respondent Mayor has the same power independently under Republic Act No. 716051 is thus not necessary
48 49 50

51

G.R. No. 132231, March 31, 1998, 288 SCRA 447. Ibid, p. 478. Except picketing and other concerted action in strike areas by workers and employees resulting from a labor dispute, which are governed by the Labor Code and other labor laws; political meeting or rallies held during any election campaign period, which are governed by the Election Code and other election related laws; and public assemblies in the campus of a government-owned and operated educational institution, which shall be subject to the rules and regulations of said educational institution. (Sec. 3[a] and Sec. 4 of B.P. No. 880). The Local Government Code. Specifically, Section 16 stating the general welfare clause, thus: SEC. 16. General Welfare. Every local government unit shall exercise the powers expressly granted, those necessarily implied therefrom, as well as powers necessary, appropriate, or incidental for its efficient and effective governance, and those which are essential to the promotion of the general welfare. Within their respective territorial jurisdictions, local government units shall ensure and support among other things, the preservation and enrichment of culture, promote health and safety, enhance the right of the people to a

191

192

to resolve in these proceedings, and was not pursued by the parties in their arguments. Finally, for those who cannot wait, Section 15 of the law provides for an alternative forum through the creation of freedom parks where no prior permit is needed for peaceful assembly and petition at any time: SEC. 15. Freedom parks. Every city and municipality in the country shall within six months after the effectivity of this Act establish or designate at least one suitable freedom park or mall in their respective jurisdictions which, as far as practicable, shall be centrally located within the poblacion where demonstrations and meetings may be held at any time without the need of any prior permit. In the cities and municipalities of Metropolitan Manila, the respective mayors shall establish the freedom parks within the period of six months from the effectivity this Act. 2 The Court now comes to the matter of the CPR. As stated earlier, the Solicitor General has conceded that the use of the term should now be discontinued, since it does not mean anything other than the maximum tolerance policy set forth in B.P. No. 880. This is stated in the Affidavit of respondent Executive Secretary Eduardo Ermita, submitted by the Solicitor General, thus: The truth of the matter is the policy of calibrated preemptive response is in consonance with the legal definition of maximum tolerance under Section 3 (c) of B.P. Blg. 880, which is the highest degree of restraint that the military, police and other peacekeeping authorities shall observe during a public assembly or in the dispersal of the same. Unfortunately, however, the phrase maximum tolerance has acquired a different meaning over the years. Many have taken it to mean inaction on the part of law enforcers even in the face of mayhem and serious threats to public order. More so, other felt that they need not bother secure a permit when holding rallies thinking this would be tolerated. Clearly, the popular connotation of maximum tolerance has departed from its real essence under B.P. Blg. 880. It should be emphasized that the policy of maximum tolerance is provided under the same law which requires all pubic assemblies to have a permit, which allows the dispersal of rallies without a permit, and which recognizes certain instances when water cannons may be used. This could only mean that maximum tolerance is not in conflict with a no permit, no rally policy or with the dispersal and use of water cannons under certain circumstances for indeed, the maximum amount of tolerance required is dependent on how peaceful or unruly a mass action is. Our law enforcers should calibrate their response based on the circumstances on the ground with the view to preempting the outbreak of violence. Thus, when I stated that calibrated preemptive response is being enforced in lieu of maximum tolerance I clearly was not referring to its
balanced ecology, encourage and support the development of appropriate and self-reliant scientific and technological capabilities, improve public morals, enhance economic prosperity and social justice, promote full employment among their residents, maintain peace and order, and preserve the comfort and convenience of their inhabitants.

192

193

legal definition but to the distorted and much abused definition that it has now acquired. I only wanted to disabuse the minds of the public from the notion that law enforcers would shirk their responsibility of keeping the peace even when confronted with dangerously threatening behavior. I wanted to send a message that we would no longer be lax in enforcing the law but would henceforth follow it to the letter. Thus I said, we have instructed the PNP as well as the local government units to strictly enforce a no permit, no rally policy . . . arrest all persons violating the laws of the land . . . unlawful mass actions will be dispersed. None of these is at loggerheads with the letter and spirit of Batas Pambansa Blg. 880. It is thus absurd for complainants to even claim that I ordered my corespondents to violate any law. In sum, this Court reiterates its basic policy of upholding the fundamental rights of our people, especially freedom of expression and freedom of assembly. For this reason, the so-called calibrated preemptive response policy has no place in our legal firmament and must be struck down as a darkness that shrouds freedom. It merely confuses our people and is used by some police agents to justify abuses. On the other hand, B.P. No. 880 cannot be condemned as unconstitutional; it does not curtail or unduly restrict freedoms; it merely regulates the use of public places as to the time, place and manner of assemblies. Far from being insidious, maximum tolerance is for the benefit of rallyists, not the government. The delegation to the mayors of the power to issue rally permits is valid because it is subject to the constitutionally-sound clear and present danger standard. WHEREFORE, the petitions are GRANTED in part, and Calibrated Preemptive Response (CPR), insofar as it would purport to differ from or be in lieu of maximum tolerance, is NULL and VOID and respondents are ENJOINED to REFRAIN from using it and to STRICTLY OBSERVE the requirements of maximum tolerance. Read: 1. Right of assembly..31 SCRA 734 and 742 2. Evangelista vs. Earnshaw, 57 Phil 255 3. Primicias vs. Fuguso, 80 Phil. 71 4. De la Cruz vs. Ela, 99 Phil. 346 5. Navarro vs. Villegas, 31 SCRA 731 6. Philippine Blooming Mills Case,51 SCRA 189 7. Reyes vs. Bagatsing, 125 SCRA 553;see guidelines 8. Ruiz vs. Gordon, 126 SCRA 233 9. Villar vs. TIP, 135 SCRA 705 10. Malabanan vs. Ramento, 129 SCRA 359 11. Carpio vs. Guevara, 106 SCRA 685 12. Nestle' Phils. vs. Sanchez, 154 SCRA 542 13. Arreza vs. Araneta University Foundation, 137 SCRA 94 6. Freedom from prior restraint Closing a radio station is definitely prior restraint NEWSOUNDS BROADCASTING NETWORK INC. and CONSOLIDATED BROADCASTING SYSTEM, INC.HON. CEASAR G. DY, FELICISIMO G. MEER, BAGNOS MAXIMO, RACMA FERNANDEZ-GARCIA and THE CITY OF CAUAYAN, G.R. Nos. 170270 &179411, April 2, 2009
193

194

TINGA, J.: Bombo Radyo Philippines (Bombo Radyo) operates several radio stations under the AM and FM band throughout the Philippines. These stations are operated by corporations organized and incorporated by Bombo Radyo, particularly petitioners Newsounds Broadcasting Network, Inc. (Newsounds) and Consolidated Broadcasting System, Inc. (CBS). Among the stations run by Newsounds is Bombo Radyo DZNC Cauayan (DZNC), an AM radio broadcast station operating out of Cauayan City, Isabela. CBS, in turn, runs Star FM DWIT Cauayan (Star FM), also operating out of Cauayan City, airing on the FM band. The service areas of DZNC and Star FM extend from the province of Isabela to throughout Region II and the Cordillera region.52[6] THE ONLY OTHER STATION OPERATING IN CAUAYAN CITY, ISABELA, is owned by the family of respondent Mayor Dy. In 1996, Newsounds commenced relocation of its broadcasting stations, management office and transmitters on property located in Minante 2, Cauayan City, Isabela. The property is owned by CBS Development Corporation (CDC), an affiliate corporation under the Bombo Radyo network which holds title over the properties used by Bombo Radyo stations throughout the country.53[7] On 28 June 1996, CDC was issued by the then municipal government of Cauayan a building permit authorizing the construction of a commercial establishment on the property.54[8] On 5 July 1996, the Housing and Land Use Regulatory Board (HLURB) issued a Zoning Decision certifying the property as commercial.55[9] That same day, the Office of the Municipal Planning and Development Coordinator (OMPDC) of Cauayan affirmed that the commercial structure to be constructed by CDC conformed to local zoning regulations, noting as well that the location is classified as a Commercial area.56[10] Similar certifications would be issued by OMPDC from 1997 to 2001.57[11] A building was consequently erected on the property, and therefrom, DZNC and Star FM operated as radio stations. Both stations successfully secured all necessary operating documents, including mayors permits from 1997 to 2001.58[12] During that period, CDC paid real property taxes on the property based on the classification of the land as commercial.59[13] All that changed beginning in 2002. On 15 January of that year, petitioners applied for the renewal of the mayors permit. The following

52[6]

Rollo (G.R. No. 179411), p. 13. Id. Id. at 90. Id. at 91. Id. at 92. Id. at 93-97. Id. at 98-102. Id. at 103-110.

53[7]

54[8]

55[9]

56[10]

57[11]

58[12]

59[13]

194

195

day, the City Assessors Office in Cauayan City noted on CDCs Declaration of Real Property filed for 2002 confirmed that based on the existing file, CDCs property was classified as commercial.60[14] On 28 January, representatives of petitioners formally requested then City Zoning Administrator-Designate Bagnos Maximo (Maximo) to issue a zoning clearance for the property.61[15] Maximo, however, required petitioners to submit either an approved land conversion papers from the Department of Agrarian Reform (DAR) showing that the property was converted from prime agricultural land to commercial land, or an approved resolution from the Sangguniang Bayan or Sangguniang Panglungsod authorizing the re-classification of the property from agricultural to commercial land.62[16] Petitioners had never been required to submit such papers before, and from 1996 to 2001, the OMPDC had consistently certified that the property had been classified as commercial. THEREAFTER, THE MUNICIPAL OFFICIALS PADLOCKED THE RADIO STATIONS BASED ON THE GROUND THAT THE PETITIONERS FAILED TO SUBMIT THE requisite zoning clearance needed for the issuance of the mayors permit because there was allegedly no DAR Decision converting said land from agricultural to commercial. Petitioners filed a petition for mandamus, docketed as SCA No. 20-171, with the RTC of Cauayan City, Branch 20 to compel the municipality to allow the radio stations to operate. The petition was accompanied by an application for the issuance of temporary restraining order and writ of preliminary prohibitory injunction, both provisional reliefs being denied by the RTC through an Order dated 20 April 2004. Thereafter, the petition was dismissed by the RTC as well as the Court of Appeals. Hence, this case before the Supreme Court. I S S U E: Is constitutional? HELD: The closure constitutes prior restraint. The fundamental constitutional principle that informs our analysis of both petitions is the freedom of speech, of expression or the press.63[32] Free speech and free press may be identified with the liberty to discuss publicly and truthfully any matter of public interest without censorship and punishment. There is to be no previous restraint on the communication of views or subsequent liability whether in libel suits, prosecution for sedition, or action for damages, or contempt proceedings unless there be a the closure of the petitioners radio stations

60[14]

Id. at 103. Id. at 111.

61[15]

62[16]

Id. at 18-19. Article 3, Sec. 4.

63[32]

195

196

clear and present danger of substantive evil that Congress has a right to prevent.64[33] Petitioners have taken great pains to depict their struggle as a textbook case of denial of the right to free speech and of the press. In their tale, there is undeniable political color. They admit that in 2001, Bombo Radyo was aggressive in exposing the widespread election irregularities in Isabela that appear to have favored respondent Dy and other members of the Dy political dynasty.65[34] Respondent Ceasar Dy is the brother of Faustino Dy, Jr., governor of Isabela from 2001 until he was defeated in his re-election bid in 2004 by Grace Padaca, a former assistant station manager at petitioners own DZNC Bombo Radyo.66[35] A rival AM radio station in Cauayan City, DWDY, is owned and operated by the Dy family.67[36] Petitioners likewise direct our attention to a 20 February 2004 article printed in the Philippine Daily Inquirer where Dy is quoted as intending to file disenfranchisement proceedings against DZNCAM.68[37] The following undisputed facts bring the issue of free expression to fore. Petitioners are authorized by law to operate radio stations in Cauayan City, and had been doing so for some years undisturbed by local authorities. Beginning in 2002, respondents in their official capacities have taken actions, whatever may be the motive, that have impeded the ability of petitioners to freely broadcast, if not broadcast at all. These actions have ranged from withholding permits to operate to the physical closure of those stations under color of legal authority. While once petitioners were able to broadcast freely, the weight of government has since bore down upon them to silence their voices on the airwaves. An elementary school child with a basic understanding of civics lessons will recognize that free speech animates these cases. Without taking into account any extenuating circumstances that may favor the respondents, we can identify the bare acts of closing the radio stations or preventing their operations as an act of prior restraint against speech, expression or of the press. Prior restraint refers to official governmental restrictions on the press or other forms of expression in advance of actual publication or dissemination.69[38] While any system of prior restraint comes to court bearing a heavy burden against its constitutionality,70[39] not all prior restraints on speech are invalid.71[40]

64[33]

Gonzales v. COMELEC, 137 Phil. 471, 492 (1969). Rollo (G.R. No. 170270), p. 27.

65[34]

See TJ Burgonio, Isabela gov who ended a dynasty wins RM prize, Philippine Daily Inquirer (1 August 2008), at http://opinion.inquirer.net/inquireropinion/letterstotheeditor/view /20080801-151950/Isabelagov-who-ended-a-dynasty-wins-RM-prize
67[36]

66[35]

Rollo (G.R. No. 170270), p. 17. Rollo (G.R. No. 179411), p. 142. Chavez v. Gonzales, G.R. No. 168338, 15 February 2008, 545 SCRA 441, 491.

68[37]

69[38]

70[39] Social Weather Stations, Inc. v. COMELEC, 409 Phil. 571, 585 (2001); citing New York Times v. United States, 403 U.S. 713, 714, 29 L. Ed. 2d 822, 824 (1971). 71[40]

Chavez v. Gonzales, G.R. No. 168335, 15 February 2008, 545 SCRA 441, 492.

196

197

That the acts imputed against respondents constitute a prior restraint on the freedom of expression of respondents who happen to be members of the press is clear enough. There is a long-standing tradition of special judicial solicitude for free speech, meaning that governmental action directed at expression must satisfy a greater burden of justification than governmental action directed at most other forms of behavior.72[41] We had said in SWS v. COMELEC: Because of the preferred status of the constitutional rights of speech, expression, and the press, such a measure is vitiated by a weighty presumption of invalidity. Indeed, any system of prior restraints of expression comes to this Court bearing a heavy presumption against its constitutional validity. . . . The Government 'thus carries a heavy burden of showing justification for the enforcement of such restraint. There is thus a reversal of the normal presumption of validity that inheres in every legislation.73[42] At the same time, jurisprudence distinguishes between a contentneutral regulation, i.e., merely concerned with the incidents of the speech, or one that merely controls the time, place or manner, and under well defined standards; and a content-based restraint or censorship, i.e., the restriction is based on the subject matter of the utterance or speech. 74[43] Content-based laws are generally treated as more suspect than contentneutral laws because of judicial concern with discrimination in the regulation of expression.75[44] Content-neutral regulations of speech or of conduct that may amount to speech, are subject to lesser but still heightened scrutiny.76[45] Ostensibly, the act of an LGU requiring a business of proof that the property from which it operates has been zoned for commercial use can be argued, when applied to a radio station, as content-neutral since such a regulation would presumably apply to any other radio station or business enterprise within the LGU. However, the circumstances of this case dictate that we view the action of the respondents as a content-based restraint. In their petition for mandamus filed with the RTC, petitioners make the following relevant allegations: 6.1. With specific reference to DZNC, Newsounds, to this date, is engaged in discussing public issues that include, among others, the conduct of public officials that are detrimental to the constituents of Isabela, including Cauayan City. In view of its wide coverage, DZNC has been a primary medium for the exercise of the people of Isabela of their constitutional right to free speech. Corollarily, DZNC has always been at the forefront of the struggle to maintain and uphold freedom of the press, and the peoples corollary right to freedom of speech, expression and petition the government for redress of grievances.

72[41]

GUNTHER, et al., CONSTITUTIONAL LAW (14th ed., 2001), at 964. SWS v. COMELEC, supra note 39. Chavez v. Gonzales, G.R. No. 168338, 15 February 2008, 545 SCRA 441, 493. GUNTHER, et al., supra note 44. Id. at 957.

73[42]

74[43]

75[44]

76[45]

197

198

6.2. Newsounds only rival AM station in Cauayan and the rest of Isabela, DWDY, is owned and operated by the family of respondent Dy.77[46] xxxx 35. Respondents closure of petitioners radio stations is clearly tainted with ill motives. 35.1. It must be pointed out that in the 2001 elections, Bombo Radyo was aggressive in exposing the widespread election irregularities in Isabela that appear to have favored respondent Dy and other members of the Dy political dynasty. It is just too coincidental that it was only after the 2001 elections (i.e., 2002) that the Mayors Office started questioning petitioners applications for renewal of their mayors permits. 35.2. In an article found in the Philippine Daily inquirer dated 20 February 2004, respondent Dy was quoted as saying that he will disenfranchise the radio station. Such statement manifests and confirms that respondents denial of petitioners renewal applications on the ground that the Property is commercial is merely a pretext and that their real agenda is to remove petitioners from Cauayan City and suppress the latters voice. This is a blatant violation of the petitioners constitutional right to press freedom. 35.3. The timing of respondents closure of petitioners radio stations is also very telling. The closure comes at a most critical time when the people are set to exercise their right of suffrage. Such timing emphasizes the ill motives of respondents.78[47] All those circumstances lead us to believe that the steps employed by respondents to ultimately shut down petitioners radio station were ultimately content-based. The United States Supreme Court generally treats restriction of the expression of a particular point of view as the paradigm violation of the First Amendment.79[53] The facts confronting us now could have easily been drawn up by a constitutional law professor eager to provide a plain example on how free speech may be violated. The Court is of the position that the actions of the respondents warrant heightened or strict scrutiny from the Court, the test which we have deemed appropriate in assessing content-based restrictions on free speech, as well as for laws dealing with freedom of the mind or restricting the political process, of laws dealing with the regulation of speech, gender, or race as well as other fundamental rights as expansion from its earlier applications to equal protection.80[54] The immediate implication of the application of the strict scrutiny test is that the burden falls upon respondents as agents of government to prove that their actions do not infringe upon petitioners constitutional rights. As content regulation cannot be done in the absence of any compelling reason, 81[55] the burden
77[46]

Rollo (G.R. No. 179411), p. 170. Id. at 178-179. GUNTHER et. al., supra note 44. See White Light v. Court of Appeals, G.R. No. 122846, 20 January 2009.

78[47]

79[53]

80[54]

198

199

lies with the government to establish such compelling reason to infringe the right to free expression. It is thus evident that respondents had no valid cause at all to even require petitioners to secure approved land conversion papers from the DAR showing that the property was converted from prime agricultural land to commercial land. That requirement, assuming that it can be demanded by a local government in the context of approving mayors permits, should only obtain upon clear proof that the property from where the business would operate was classified as agricultural under the LGUs land use plan or zoning ordinances and other relevant laws. No evidence to that effect was presented by the respondents either to the petitioners, or to the courts. Having established that respondents had violated petitioners legal and constitutional rights, let us now turn to the appropriate reliefs that should be granted. We turn to the issue of damages. Petitioners had sought to recover from respondents P8 Million in temperate damages, P1 Million in exemplary damages, and P1 Million in attorneys fees. Given respondents clear violation of petitioners constitutional guarantee of free expression, the right to damages from respondents is squarely assured by Article 32 (2) of the Civil Code, which provides: Art. 32. Any public officer or employee, or any private individual, who directly or indirectly obstructs, defeats, violates or in any manner impedes or impairs any of the following rights and liberties of another person shall be liable to the latter for damages: x x x x (2) Freedom of speech; We noted in Lim v. Ponce de Leon that [p]ublic officials in the past have abused their powers on the pretext of justifiable motives or good faith in the performance of their duties [and] the object of [Article 32 of the Civil Code] is to put an end to official abuse by plea of the good faith.82[85] The application of Article 32 not only serves as a measure of pecuniary recovery to mitigate the injury to constitutional rights, it likewise serves notice to public officers and employees that any violation on their part of any persons guarantees under the Bill of Rights will meet with final reckoning. The present prayer for temperate damages is premised on the existence of pecuniary injury to petitioner due to the actions of respondents, the amount of which nevertheless being difficult to prove.83[86] Temperate damages avail when the court finds that some pecuniary loss has been suffered but its amount can not, from the nature of the case, be proved with certainty.84[87] The existence of pecuniary injury
81[55]

Osmea v. COMELEC, 351 Phil. 692, 711 (1998).

160 Phil. 991, 1001 (1975). See also MHP Garments, Inc., v. Court of Appeals, G.R. No. 86720, 2 September 1994, 236 SCRA 227, 235.
83[86]

82[85]

Rollo (G.R. No. 179411), p. 183. See CIVIL CODE, Art. 2224.

84[87]

199

200

at bar cannot be denied. Petitioners had no way of knowing it when they filed their petition, but the actions of respondents led to the closure of their radio stations from June 2004 until this Court issued a writ of preliminary injunction in January 2006.85[88] The lost potential income during that one and a half year of closure can only be presumed as substantial enough. Still, despite that fact, possibly unanticipated when the original amount for claimed temperate damages was calculated, petitioners have maintained before this Court the same amount, P8 Million, for temperate damages. The said amount is reasonable under the circumstances.86[89] Exemplary damages can be awarded herein, since temperate damages are available. Public officers who violate the Constitution they are sworn to uphold embody a poison of wickedness that may not run through the body politic.87[90] Respondents, by purposely denying the commercial character of the property in order to deny petitioners the exercise of their constitutional rights and their business, manifested bad faith in a wanton, fraudulent, oppressive and malevolent manner. 88[91] The amount of exemplary damages need not be proved where it is shown that plaintiff is entitled to temperate damages,89[92] and the sought for amount of P1 Million is more than appropriate. We likewise deem the prayer for P1 Million in attorneys fees as suitable under the circumstances. WHEREFORE, the petitions are GRANTED. The assailed decisions of the Court of Appeals and the Regional Trial Court of Cauayan City, Branch 24, are hereby reversed and set aside. Suspension for three (3) months of TV Host, as well as his own TV Program held not prior restraint.

BRO. ELISEO SORIANO VS. MOVIE AND TELEVISION REVIEW AND CLASSIFICATION BOARD, G.R. NO. 164785, APRIL 29, 2009 VELASCO, JR., J.: On August 10, 2004, at around 10:00 p.m., petitioner, as host of the program Ang Dating Daan, aired on UNTV 37, made the following remarks: Lehitimong anak ng demonyo; sinungaling;
According to an article posted on the official website of Bombo Radyo, DZNC accordingly resumed broadcast on 8 February 2006. See http://www.bomboradyo.com/archive/ new/stationprofile /bombocauayan/index.htm (last visited, 6 March 2009)
86[89] 85[88]

See CIVIL CODE, Art. 2225.

[Exemplary damages] are an antidote so that the poison of wickedness may not run through the body politic. Octot v. Ybaez, etc., et al., 197 Phil. 76, 82 (1982). [The award of exemplary damages] would be allowed only if the guilty party acted in a wanton, fraudulent, reckless, oppressive or malevolent manner. Octot v. Ybaez, supra note 87, at 85; citing Ong Yiu v. CA, 91 SCRA 223.
89[92] 88[91]

87[90]

Patricio v. Hon. Leviste, G.R. No. L-51832, 26 April 1989.

200

201

Gago ka talaga Michael, masahol ka pa sa putang babae o di ba. Yung putang babae ang gumagana lang doon yung ibaba, [dito] kay Michael ang gumagana ang itaas, o di ba! O, masahol pa sa putang babae yan. Sabi ng lola ko masahol pa sa putang babae yan. Sobra ang kasinungalingan ng mga demonyong ito. Two days after, before the MTRCB, separate but almost identical affidavit-complaints were lodged by Jessie L. Galapon and seven other private respondents, all members of the Iglesia ni Cristo (INC), against petitioner in connection with the above broadcast. Respondent Michael M. Sandoval, who felt directly alluded to in petitioners remark, was then a minister of INC and a regular host of the TV program Ang Tamang Daan.90[3] Forthwith, the MTRCB sent petitioner a notice of the hearing on August 16, 2004 in relation to the alleged use of some cuss words in the August 10, 2004 episode of Ang Dating Daan.91[4] After a preliminary conference in which petitioner appeared, the MTRCB, by Order of August 16, 2004, preventively suspended the showing of Ang Dating Daan program for 20 days, in accordance with Section 3(d) of Presidential Decree No. (PD) 1986, creating the MTRCB, in relation to Sec. 3, Chapter XIII of the 2004 Implementing Rules and Regulations (IRR) of PD 1986 and Sec. 7, Rule VII of the MTRCB Rules of Procedure.92[5] The same order also set the case for preliminary investigation. The following day, petitioner sought reconsideration of the preventive suspension order, praying that Chairperson Consoliza P. Laguardia and two other members of the adjudication board recuse themselves from hearing the case.93[6] Two days after, however, petitioner sought to withdraw94[7] his motion for reconsideration, followed by the filing with this Court of a petition for certiorari and prohibition,95[8] docketed as G.R. No. 164785, to nullify the preventive suspension order thus issued. On September 27, 2004, in Adm. Case No. 01-04, the MTRCB issued a decision, disposing as follows: WHEREFORE, in view of all the foregoing, a Decision is hereby rendered, finding respondent Soriano liable for his utterances and thereby imposing on him a penalty of three (3) months suspension from his program, Ang Dating Daan. SO ORDERED.96[9] Petitioner then filed this petition for certiorari and prohibition with prayer for injunctive relief, docketed as G.R. No. 165636. In G.R. No. 164785, petitioner raises the following issues:
90[3] 91[4]

Id. at 924, Private Respondents Memorandum. Id. at 110. 92[5] Id. at 112-113, Rules of Procedure in the Conduct of Hearing for Violations of PD 1986 and the IRR.
93[6] 94[7]

Id. at 141-151. Id. at 152-154. 95[8] Id. at 166-252. 96[9] Id. at 378.

201

202

THE ORDER OF PREVENTIVE SUSPENSION PROMULGATED BY RESPONDENT [MTRCB] DATED 16 AUGUST 2004 AGAINST THE TELEVISION PROGRAM ANG DATING DAAN x x x IS NULL AND VOID FOR BEING ISSUED WITH GRAVE ABUSE OF DISCRETION AMOUNTING TO LACK OR EXCESS OF JURISDICTION (A) BY REASON THAT THE [IRR] IS INVALID INSOFAR AS IT PROVIDES FOR THE ISSUANCE OF PREVENTIVE SUSPENSION ORDERS; (B) BY REASON OF LACK OF DUE HEARING IN THE CASE AT BENCH; (C) FOR BEING VIOLATIVE OF EQUAL PROTECTION UNDER THE LAW; (D) FOR BEING VIOLATIVE OF FREEDOM OF RELIGION; AND (E) FOR BEING VIOLATIVE OF FREEDOM OF SPEECH AND EXPRESSION.97[10] In G.R. No. 165636, petitioner relies on the following grounds: SECTION 3(C) OF [PD] 1986, IS PATENTLY UNCONSTITUTIONAL AND ENACTED WITHOUT OR IN EXCESS OF JURISDICTION x x x CONSIDERING THAT: I SECTION 3(C) OF [PD] 1986, AS APPLIED TO PETITIONER, UNDULY INFRINGES ON THE CONSTITUTIONAL GUARANTEE OF FREEDOM OF RELIGION, SPEECH, AND EXPRESSION AS IT PARTAKES OF THE NATURE OF A SUBSEQUENT PUNISHMENT CURTAILING THE SAME; CONSEQUENTLY, THE IMPLEMENTING RULES AND REGULATIONS, RULES OF PROCEDURE, AND OFFICIAL ACTS OF THE MTRCB PURSUANT THERETO, I.E. DECISION DATED 27 SEPTEMBER 2004 AND ORDER DATED 19 OCTOBER 2004, ARE LIKEWISE CONSTITUTIONALLY INFIRM AS APPLIED IN THE CASE AT BENCH; II SECTION 3(C) OF [PD] 1986, AS APPLIED TO PETITIONER, UNDULY INFRINGES ON THE CONSTITUTIONAL GUARANTEE OF DUE PROCESS OF LAW AND EQUAL PROTECTION UNDER THE LAW; CONSEQUENTLY, THE [IRR], RULES OF PROCEDURE, AND OFFICIAL ACTS OF THE MTRCB PURSUANT THERETO, I.E., DECISION DATED 27 SEPTEMBER 2004 AND ORDER DATED 19 OCTOBER 2004, ARE LIKEWISE CONSTITUTIONALLY INFIRM AS APPLIED IN THE CASE AT BENCH; AND III [PD] 1986 IS NOT COMPLETE IN ITSELF AND DOES NOT PROVIDE FOR A SUFFICIENT STANDARD FOR ITS IMPLEMENTATION THEREBY RESULTING IN AN UNDUE DELEGATION OF LEGISLATIVE POWER BY REASON THAT IT DOES NOT PROVIDE FOR THE PENALTIES FOR VIOLATIONS OF ITS PROVISIONS. CONSEQUENTLY, THE [IRR], RULES OF PROCEDURE, AND OFFICIAL ACTS OF THE MTRCB PURSUANT
97[10]

Id. at 182.

202

203

THERETO, I.E. DECISION DATED 27 SEPTEMBER 2004 AND ORDER DATED 19 OCTOBER 2004, ARE LIKEWISE CONSTITUTIONALLY INFIRM AS APPLIED IN THE CASE AT BENCH98[11] G.R. No. 164785 We shall first dispose of the issues in G.R. No. 164785, regarding the assailed order of preventive suspension, although its implementability had already been overtaken and veritably been rendered moot by the equally assailed September 27, 2004 decision. It is petitioners threshold posture that the preventive suspension imposed against him and the relevant IRR provision authorizing it are invalid inasmuch as PD 1986 does not expressly authorize the MTRCB to issue preventive suspension. Petitioners contention is untenable. Administrative agencies have powers and functions which may be administrative, investigatory, regulatory, quasi-legislative, or quasijudicial, or a mix of the five, as may be conferred by the Constitution or by statute.99[12] They have in fine only such powers or authority as are granted or delegated, expressly or impliedly, by law. 100[13] And in determining whether an agency has certain powers, the inquiry should be from the law itself. But once ascertained as existing, the authority given should be liberally construed.101[14] A perusal of the MTRCBs basic mandate under PD 1986 reveals the possession by the agency of the authority, albeit impliedly, to issue the challenged order of preventive suspension. And this authority stems naturally from, and is necessary for the exercise of, its power of regulation and supervision. Sec. 3 of PD 1986 pertinently provides the following: Section 3. Powers and Functions.The have the following functions, powers and duties: xxxx c) To approve or disapprove, delete objectionable portions from and/or prohibit the x x x production, x x x exhibition and/or television broadcast of the motion pictures, television programs and publicity materials subject of the preceding paragraph, which, in the judgment of the board applying contemporary Filipino cultural values as standard, are objectionable for being immoral, indecent, contrary to law and/or good customs, injurious to the prestige of the Republic of the Philippines or its people, or with a dangerous tendency to encourage the commission of violence or of wrong or crime such as but not limited to: xxxx
Id. at 46. Azarcon v. Sandiganbayan, G.R. No. 116033, February 26, 1997, 268 SCRA 747. 100[13] Pimentel v. COMELEC, Nos. L-53581-83, December 19, 1980, 101 SCRA 769. 101[14] Agpalo, ADMINISTRATIVE LAW (2005); citing Matienzon v. Abellera, G.R. No. 77632, June 8, 1988, 162 SCRA 1.
99[12] 98[11]

BOARD

shall

203

204

vi) Those which are libelous or defamatory to the good name and reputation of any person, whether living or dead; xxxx (d) To supervise, regulate, and grant, deny or cancel, permits for the x x x production, copying, distribution, sale, lease, exhibition, and/or television broadcast of all motion pictures, television programs and publicity materials, to the end that no such pictures, programs and materials as are determined by the BOARD to be objectionable in accordance with paragraph (c) hereof shall be x x x produced, copied, reproduced, distributed, sold, leased, exhibited and/or broadcast by television; xxxx k) To exercise such powers and functions as may be necessary or incidental to the attainment of the purposes and objectives of this Act x x x. (Emphasis added.) The issuance of a preventive suspension comes well within the scope of the MTRCBs authority and functions expressly set forth in PD 1986, more particularly under its Sec. 3(d), as quoted above, which empowers the MTRCB to supervise, regulate, and grant, deny or cancel, permits for the x x x exhibition, and/or television broadcast of all motion pictures, television programs and publicity materials, to the end that no such pictures, programs and materials as are determined by the BOARD to be objectionable in accordance with paragraph (c) hereof shall be x x x exhibited and/or broadcast by television. Surely, the power to issue preventive suspension forms part of the MTRCBs express regulatory and supervisory statutory mandate and its investigatory and disciplinary authority subsumed in or implied from such mandate. Any other construal would render its power to regulate, supervise, or discipline illusory. Preventive suspension, it ought to be noted, is not a penalty by itself, being merely a preliminary step in an administrative investigation.102[15] And the power to discipline and impose penalties, if granted, carries with it the power to investigate administrative complaints and, during such investigation, to preventively suspend the person subject of the complaint.103[16] To reiterate, preventive suspension authority of the MTRCB springs from its powers conferred under PD 1986. The MTRCB did not, as petitioner insinuates, empower itself to impose preventive suspension through the medium of the IRR of PD 1986. It is true that the matter of imposing preventive suspension is embodied only in the IRR of PD 1986. Sec. 3, Chapter XIII of the IRR provides: Sec. 3. PREVENTION SUSPENSION ORDER.Any time during the pendency of the case, and in order to prevent or stop further violations or for the interest and welfare of the public, the Chairman of the
Lastimoso v. Vasquez, G.R. No. 116801, April 6, 1995, 243 SCRA 497. Alonzo v. Capulong, G.R. No. 110590, May 10, 1995, 244 SCRA 80; Beja v. Court of Appeals, G.R. No. 97149, March 31, 1992, 207 SCRA 689.
103[16] 102[15]

204

205

Board may issue a Preventive Suspension Order mandating the preventive x x x suspension of the permit/permits involved, and/or closure of the x x x television network, cable TV station x x x provided that the temporary/preventive order thus issued shall have a life of not more than twenty (20) days from the date of issuance. But the mere absence of a provision on preventive suspension in PD 1986, without more, would not work to deprive the MTRCB a basic disciplinary tool, such as preventive suspension. Recall that the MTRCB is expressly empowered by statute to regulate and supervise television programs to obviate the exhibition or broadcast of, among others, indecent or immoral materials and to impose sanctions for violations and, corollarily, to prevent further violations as it investigates. Contrary to petitioners assertion, the aforequoted Sec. 3 of the IRR neither am ended PD 1986 nor extended the effect of the law. Neither did the MTRCB, by imposing the assailed preventive suspension, outrun its authority under the law. Far from it. The preventive suspension was actually done in furtherance of the law, imposed pursuant, to repeat, to the MTRCBs duty of regulating or supervising television programs, pending a determination of whether or not there has actually been a violation. In the final analysis, Sec. 3, Chapter XIII of the 2004 IRR merely formalized a power which PD 1986 bestowed, albeit impliedly, on MTRCB. Sec. 3(c) and (d) of PD 1986 finds application to the present case, sufficient to authorize the MTRCBs assailed action. Petitioners restrictive reading of PD 1986, limiting the MTRCB to functions within the literal confines of the law, would give the agency little leeway to operate, stifling and rendering it inutile, when Sec. 3(k) of PD 1986 clearly intends to grant the MTRCB a wide room for flexibility in its operation. Sec. 3(k), we reiterate, provides, To exercise such powers and functions as may be necessary or incidental to the attainment of the purposes and objectives of this Act x x x. Indeed, the power to impose preventive suspension is one of the implied powers of MTRCB. As distinguished from express powers, implied powers are those that can be inferred or are implicit in the wordings or conferred by necessary or fair implication of the enabling act.104[17] As we held in Angara v. Electoral Commission, when a general grant of power is conferred or a duty enjoined, every particular power necessary for the exercise of one or the performance of the other is also conferred by necessary implication.105[18] Clearly, the power to impose preventive suspension pending investigation is one of the implied or inherent powers of MTRCB. We cannot agree with petitioners assertion that the aforequoted IRR provision on preventive suspension is applicable only to motion pictures and publicity materials. The scope of the MTRCBs authority extends beyond motion pictures. What the acronym MTRCB stands for would suggest as much. And while the law makes specific reference to the closure of a television network, the suspension of a television program is a far less punitive measure that can be undertaken, with the purpose of stopping further violations of PD 1986. Again, the MTRCB would regretfully be rendered ineffective should it be subject to the restrictions petitioner envisages.

104[17] Chavez v. National Housing Authority, G.R. No. 164527, August 15, 2007, 530 SCRA 235, 295296; citing Azarcon, supra note 12, at 761; Radio Communications of the Philippines, Inc. v. Santiago , Nos. L29236 & 29247, August 21, 1974, 58 SCRA 493, 497. 105[18] 63 Phil. 139, 177 (1936).

205

206

Just as untenable is petitioners argument on the nullity of the preventive suspension order on the ground of lack of hearing. As it were, the MTRCB handed out the assailed order after petitioner, in response to a written notice, appeared before that Board for a hearing on private respondents complaint. No less than petitioner admitted that the order was issued after the adjournment of the hearing,106[19] proving that he had already appeared before the MTRCB. Under Sec. 3, Chapter XIII of the IRR of PD 1986, preventive suspension shall issue [a]ny time during the pendency of the case. In this particular case, it was done after MTRCB duly apprised petitioner of his having possibly violated PD 1986107[20] and of administrative complaints that had been filed against him for such violation.108[21] At any event, that preventive suspension can validly be meted out even without a hearing.109[22] Petitioner next faults the MTRCB for denying him his right to the equal protection of the law, arguing that, owing to the preventive suspension order, he was unable to answer the criticisms coming from the INC ministers. Petitioners position does not persuade. The equal protection clause demands that all persons subject to legislation should be treated alike, under like circumstances and conditions both in the privileges conferred and liabilities imposed.110[23] It guards against undue favor and individual privilege as well as hostile discrimination.111[24] Surely, petitioner cannot, under the premises, place himself in the same shoes as the INC ministers, who, for one, are not facing administrative complaints before the MTRCB. For another, he offers no proof that the said ministers, in their TV programs, use language similar to that which he used in his own, necessitating the MTRCBs disciplinary action. If the immediate result of the preventive suspension order is that petitioner remains temporarily gagged and is unable to answer his critics, this does not become a deprivation of the equal protection guarantee. The Court need not belabor the fact that the circumstances of petitioner, as host of Ang Dating Daan, on one hand, and the INC ministers, as hosts of Ang Tamang Daan, on the other, are, within the purview of this case, simply too different to even consider whether or not there is a prima facie indication of oppressive inequality. Petitioner next injects the notion of religious freedom, submitting that what he uttered was religious speech, adding that words like putang babae were said in exercise of his religious freedom. The argument has no merit. The Court is at a loss to understand how petitioners utterances in question can come within the pale of Sec. 5, Article III of the 1987 Constitution on religious freedom. The section reads as follows:

Rollo (G.R. No. 164785), p. 12. Id. at 94. 108[21] Id. at 95. 109[22] Beja, supra note 16; Espiritu v. Melgar, G.R. No. 100874, February 13, 1992, 206 SCRA 256. 110[23] 1 De Leon, PHILIPPINE CONSTITUTIONAL LAW 274 (2003). 111[24] Tiu v. Guingona, G.R. No. 127410, January 20, 1999, 301 SCRA 278; citing Ichong v. Hernandez, 101 Phil. 1155 (1957) and other cases.
107[20]

106[19]

206

207

No law shall be made respecting the establishment of a religion, or prohibiting the free exercise thereof. The free exercise and enjoyment of religious profession and worship, without discrimination or preference, shall forever be allowed. No religious test shall be required for the exercise of civil or political rights. There is nothing in petitioners statements subject of the complaints expressing any particular religious belief, nothing furthering his avowed evangelical mission. The fact that he came out with his statements in a televised bible exposition program does not automatically accord them the character of a religious discourse. Plain and simple insults directed at another person cannot be elevated to the status of religious speech. G.R. No. 165636 Petitioner urges the striking down of the decision suspending him from hosting Ang Dating Daan for three months on the main ground that the decision violates, apart from his religious freedom, his freedom of speech and expression guaranteed under Sec. 4, Art. III of the Constitution, which reads: No law shall be passed abridging the freedom of speech, of expression, or of the press, or the right of the people peaceably to assemble and petition the government for redress of grievance.

He would also have the Court declare PD 1986, its Sec. 3(c) in particular, unconstitutional for reasons articulated in this petition. It is settled that expressions by means of newspapers, radio, television, and motion pictures come within the broad protection of the free speech and expression clause.112[25] Each method though, because of its dissimilar presence in the lives of people and accessibility to children, tends to present its own problems in the area of free speech protection, with broadcast media, of all forms of communication, enjoying a lesser degree of protection.113[26] Just as settled is the rule that restrictions, be it in the form of prior restraint, e.g., judicial injunction against publication or threat of cancellation of license/franchise, or subsequent liability, whether in libel and damage suits, prosecution for sedition, or contempt proceedings, are anathema to the freedom of expression. Prior restraint means official government restrictions on the press or other forms of expression in advance of actual publication or dissemination.114[27] The freedom of expression, as with the other freedoms encased in the Bill of Rights, is, however, not absolute. It may be regulated to some extent to serve important public interests, some forms of speech not being protected. As has been held, the limits of the freedom of expression are reached when the expression touches upon matters of essentially private concern.115[28] In the oft-quoted expression of Justice Holmes, the constitutional guarantee obviously was not intended to give immunity for
US v. Paramount Pictures, 334 U.S. 131; Eastern Broadcasting Corporation v. Dans , Jr., No. L59329, July 19, 1985, 137 SCRA 628. 113[26] Eastern Broadcasting Corporation v. Dans, Jr ., supra note 25; citing FCC v. Pacifica Foundation, 438 U.S. 726; Gonzales v. Kalaw Katigbak, No. L-69500, July 22, 1985, 137 SCRA 717. 114[27] J.G. Bernas, S.J., THE CONSTITUTION OF THE REPUBLIC OF THE PHILIPPINES: A COMMENTARY 205 (1996). 115[28] Lagunsad v. Soto vda. De Gonzales, No. L-32066, August 6, 1979, 92 SCRA 476.
112[25]

207

208

every possible use of language.116[29] From Lucas v. Royo comes this line: [T]he freedom to express ones sentiments and belief does not grant one the license to vilify in public the honor and integrity of another. Any sentiments must be expressed within the proper forum and with proper regard for the rights of others.117[30] Indeed, as noted in Chaplinsky v. State of New Hampshire,118[31] there are certain well-defined and narrowly limited classes of speech that are harmful, the prevention and punishment of which has never been thought to raise any Constitutional problems. In net effect, some forms of speech are not protected by the Constitution, meaning that restrictions on unprotected speech may be decreed without running afoul of the freedom of speech clause.119[32] A speech would fall under the unprotected type if the utterances involved are no essential part of any exposition of ideas, and are of such slight social value as a step of truth that any benefit that may be derived from them is clearly outweighed by the social interest in order and morality.120[33] Being of little or no value, there is, in dealing with or regulating them, no imperative call for the application of the clear and present danger rule or the balancing-of-interest test, they being essentially modes of weighing competing values,121[34] or, with like effect, determining which of the clashing interests should be advanced. Petitioner asserts that his utterance in question is a protected form of speech. The Court rules otherwise. It has been established in this jurisdiction that unprotected speech or low-value expression refers to libelous statements, obscenity or pornography, false or misleading advertisement, insulting or fighting words, i.e., those which by their very utterance inflict injury or tend to incite an immediate breach of peace and expression endangering national security. The Court finds that petitioners statement can be treated as obscene, at least with respect to the average child. Hence, it is, in that context, unprotected speech. In Fernando v. Court of Appeals, the Court expressed difficulty in formulating a definition of obscenity that would apply to all cases, but nonetheless stated the ensuing observations on the matter: There is no perfect definition of obscenity but the latest word is that of Miller v. California which established basic guidelines, to wit: (a) whether to the average person, applying contemporary standards would find the work, taken as a whole, appeals to the prurient interest; (b) whether the work depicts or describes, in a patently offensive way, sexual conduct specifically defined by the applicable state law; and (c) whether the work, taken as a whole, lacks serious literary, artistic, political, or scientific value. But, it would be a serious misreading of Miller to conclude that the trier of facts has the unbridled discretion in determining what is patently offensive. x x x What remains clear is that obscenity is

116[29]

Trohwerk v. United States, 249 U.S. 204 (1919); cited in Bernas, supra at 218. G.R. No. 136185, October 30, 2000, 344 SCRA 481, 490. 118[31] 315 U.S. 568 (1942). 119[32] Agpalo, PHILIPPINE CONSTITUTIONAL LAW 358 (2006). 120[33] Chaplinsky, supra note 31; cited in Bernas, supra note 27, at 248. 121[34] Bernas, supra note 27, at 248.
117[30]

208

209

an issue proper for judicial determination and should be treated on a case to case basis and on the judges sound discretion.122[35] Following the contextual lessons of the cited case of Miller v. California,123[36] a patently offensive utterance would come within the pale of the term obscenity should it appeal to the prurient interest of an average listener applying contemporary standards. A cursory examination of the utterances complained of and the circumstances of the case reveal that to an average adult, the utterances Gago ka talaga x x x, masahol ka pa sa putang babae x x x. Yung putang babae ang gumagana lang doon yung ibaba, [dito] kay Michael ang gumagana ang itaas, o di ba! may not constitute obscene but merely indecent utterances. They can be viewed as figures of speech or merely a play on words. In the context they were used, they may not appeal to the prurient interests of an adult. The problem with the challenged statements is that they were uttered in a TV program that is rated G or for general viewership, and in a time slot that would likely reach even the eyes and ears of children. While adults may have understood that the terms thus used were not to be taken literally, children could hardly be expected to have the same discernment. Without parental guidance, the unbridled use of such language as that of petitioner in a television broadcast could corrupt impressionable young minds. The term putang babae means a female prostitute, a term wholly inappropriate for children, who could look it up in a dictionary and just get the literal meaning, missing the context within which it was used. Petitioner further used the terms, ang gumagana lang doon yung ibaba, making reference to the female sexual organ and how a female prostitute uses it in her trade, then stating that Sandoval was worse than that by using his mouth in a similar manner. Children could be motivated by curiosity and ask the meaning of what petitioner said, also without placing the phrase in context. They may be inquisitive as to why Sandoval is different from a female prostitute and the reasons for the dissimilarity. And upon learning the meanings of the words used, young minds, without the guidance of an adult, may, from their end, view this kind of indecent speech as obscene, if they take these words literally and use them in their own speech or form their own ideas on the matter. In this particular case, where children had the opportunity to hear petitioners words, when speaking of the average person in the test for obscenity, we are speaking of the average child, not the average adult. The average child may not have the adults grasp of figures of speech, and may lack the understanding that language may be colorful, and words may convey more than the literal meaning. Undeniably the subject speech is very suggestive of a female sexual organ and its function as such. In this sense, we find petitioners utterances obscene and not entitled to protection under the umbrella of freedom of speech. Even if we concede that petitioners remarks are not obscene but merely indecent speech, still the Court rules that petitioner cannot avail himself of the constitutional protection of free speech. Said statements were made in a medium easily accessible to children. With respect to the young minds, said utterances are to be treated as unprotected speech.

122[35] 123[36]

G.R. No. 159751, December 6, 2006, 510 SCRA 351, 360-361. 413 U.S. 15.

209

210

No doubt what petitioner said constitutes indecent or offensive utterances. But while a jurisprudential pattern involving certain offensive utterances conveyed in different mediums has emerged, this case is veritably one of first impression, it being the first time that indecent speech communicated via television and the applicable norm for its regulation are, in this jurisdiction, made the focal point. Federal Communications Commission (FCC) v. Pacifica Foundation,124[37] a 1978 American landmark case cited in Eastern Broadcasting Corporation v. Dans, Jr.125[38] and Chavez v. Gonzales,126[39] is a rich source of persuasive lessons. Foremost of these relates to indecent speech without prurient appeal component coming under the category of protected speech depending on the context within which it was made, irresistibly suggesting that, within a particular context, such indecent speech may validly be categorized as unprotected, ergo, susceptible to restriction. In FCC, seven of what were considered filthy words127[40] earlier recorded in a monologue by a satiric humorist later aired in the afternoon over a radio station owned by Pacifica Foundation. Upon the complaint of a man who heard the pre-recorded monologue while driving with his son, FCC declared the language used as patently offensive and indecent under a prohibiting law, though not necessarily obscene. FCC added, however, that its declaratory order was issued in a special factual context, referring, in gist, to an afternoon radio broadcast when children were undoubtedly in the audience. Acting on the question of whether the FCC could regulate the subject utterance, the US Supreme Court ruled in the affirmative, owing to two special features of the broadcast medium, to wit: (1) radio is a pervasive medium and (2) broadcasting is uniquely accessible to children. The US Court, however, hastened to add that the monologue would be protected speech in other contexts, albeit it did not expound and identify a compelling state interest in putting FCCs contentbased regulatory action under scrutiny. The Court in Chavez128[41] elucidated on the distinction between regulation or restriction of protected speech that is content-based and that which is content-neutral. A content-based restraint is aimed at the contents or idea of the expression, whereas a content-neutral restraint intends to regulate the time, place, and manner of the expression under well-defined standards tailored to serve a compelling state interest, without restraint on the message of the expression. Courts subject content-based restraint to strict scrutiny. With the view we take of the case, the suspension MTRCB imposed under the premises was, in one perspective, permissible restriction. We make this disposition against the backdrop of the following interplaying factors: First, the indecent speech was made via television, a pervasive medium that, to borrow from Gonzales v. Kalaw Katigbak,129[42] easily reaches every home where there is a set [and where] [c]hildren will likely be among the avid viewers of the programs therein shown; second, the broadcast was aired at the time of the day when there was a reasonable risk that children might be in the audience; and third, petitioner uttered his speech on a G or for general patronage rated program. Under Sec.
124[37] 125[38]

438 U.S. 726. Supra note 25. 126[39] G.R. No. 168338, February 15, 2008, 545 SCRA 441. 127[40] Shit, piss, fuck, tits, etc. 128[41] Supra note 39. 129[42] Supra note 26.

210

211

2(A) of Chapter IV of the IRR of the MTRCB, a show for general patronage is [s]uitable for all ages, meaning that the material for television x x x in the judgment of the BOARD, does not contain anything unsuitable for children and minors, and may be viewed without adult guidance or supervision. The words petitioner used were, by any civilized norm, clearly not suitable for children. Where a language is categorized as indecent, as in petitioners utterances on a general patronage rated TV program, it may be readily proscribed as unprotected speech. A view has been advanced that unprotected speech refers only to pornography,130[43] false or misleading advertisement,131[44] advocacy of imminent lawless action, and expression endangering national security. But this list is not, as some members of the Court would submit, exclusive or carved in stone. Without going into specifics, it may be stated without fear of contradiction that US decisional law goes beyond the aforesaid general exceptions. As the Court has been impelled to recognize exceptions to the rule against censorship in the past, this particular case constitutes yet another exception, another instance of unprotected speech, created by the necessity of protecting the welfare of our children. As unprotected speech, petitioners utterances can be subjected to restraint or regulation. Despite the settled ruling in FCC which has remained undisturbed since 1978, petitioner asserts that his utterances must present a clear and present danger of bringing about a substantive evil the State has a right and duty to prevent and such danger must be grave and imminent.132[45] Petitioners invocation of the clear and present danger doctrine, arguably the most permissive of speech tests, would not avail him any relief, for the application of said test is uncalled for under the premises. The doctrine, first formulated by Justice Holmes, accords protection for utterances so that the printed or spoken words may not be subject to prior restraint or subsequent punishment unless its expression creates a clear and present danger of bringing about a substantial evil which the government has the power to prohibit.133[46] Under the doctrine, freedom of speech and of press is susceptible of restriction when and only when necessary to prevent grave and immediate danger to interests which the government may lawfully protect. As it were, said doctrine evolved in the context of prosecutions for rebellion and other crimes involving the overthrow of government.134[47] It was originally designed to determine the latitude which should be given to speech that espouses anti-government action, or to have serious and substantial deleterious consequences on the security and public order of the community.135[48] The clear and present danger rule has been applied to this jurisdiction.136[49] As a standard of limitation on free speech and press, however, the clear and present danger test is not a magic incantation that wipes out all problems and does away with analysis and judgment in the testing of the legitimacy of claims to free speech and which compels a court to release a defendant from liability
Gonzales v. Kalaw Katigbak, supra. Pharmaceutical and Health Care Association of the Philippines v. Health Secretary Francisco T. Duque III, G.R. No. 173034, October 9, 2007, 535 SCRA 265. 132[45] Bayan v. Ermita, G.R. No. 169838, April 25, 2006, 488 SCRA 226. 133[46] 16A Am Jur. 2d Constitutional Law Sec. 493; Schenck v. United States, 249 U.S. 47. 134[47] Bernas, supra note 27, at 219-220. 135[48] Gonzales v. COMELEC, No. L-27833, April 18, 1969, 27 SCRA 835. 136[49] ABS-CBN Broadcasting Corp. v. COMELEC, G.R. No. 133486, January 28, 2000, 323 SCRA 811; Adiong v. COMELEC, G.R. No. 103956, March 31, 1992, 207 SCRA 712.
131[44] 130[43]

211

212

the moment the doctrine is invoked, absent proof of imminent catastrophic disaster.137[50] As we observed in Eastern Broadcasting Corporation, the clear and present danger test does not lend itself to a simplistic and all embracing interpretation applicable to all utterances in all forums.138[51] To be sure, the clear and present danger doctrine is not the only test which has been applied by the courts. Generally, said doctrine is applied to cases involving the overthrow of the government and even other evils which do not clearly undermine national security. Since not all evils can be measured in terms of proximity and degree the Court, however, in several casesAyer Productions v. Capulong139[52] and Gonzales v. COMELEC,140[53] applied the balancing of interests test. Former Chief Justice Fred Ruiz Castro, in Gonzales v. COMELEC, elucidated in his Separate Opinion that where the legislation under constitutional attack interferes with the freedom of speech and assembly in a more generalized way and where the effect of the speech and assembly in terms of the probability of realization of a specific danger is not susceptible even of impressionistic calculation,141[54] then the balancing of interests test can be applied. The Court explained also in Gonzales v. COMELEC the balancing of interests test: When particular conduct is regulated in the interest of public order, and the regulation results in an indirect, conditional, partial abridgment of speech, the duty of the courts is to determine which of the two conflicting interests demands the greater protection under the particular circumstances presented. x x x We must, therefore, undertake the delicate and difficult task x x x to weigh the circumstances and to appraise the substantiality of the reasons advanced in support of the regulation of the free enjoyment of rights x x x. In enunciating standard premised on a judicial balancing of the conflicting social values and individual interests competing for ascendancy in legislation which restricts expression, the court in Douds laid the basis for what has been called the balancing-of-interests test which has found application in more recent decisions of the U.S. Supreme Court. Briefly stated, the balancing test requires a court to take conscious and detailed consideration of the interplay of interests observable in a given situation or type of situation. xxx Although the urgency of the public interest sought to be secured by Congressional power restricting the individuals freedom, and the social importance and value of the freedom so restricted, are to be judged in the concrete, not on the basis of abstractions, a wide range of factors are necessarily relevant in ascertaining the point or line of equilibrium. Among these are (a) the social value and importance of the specific aspect of the particular freedom restricted by the legislation; (b) the specific thrust of the restriction, i.e., whether the restriction is direct or indirect, whether or not the persons affected are few; (c) the value and importance
137[50] 138[51]

Zaldivar v. Sandiganbayan, G.R. Nos. 79690-707 & 80578, February 1, 1989, 170 SCRA 1. Supra note 25, at 635. 139[52] No. L-82380, April 29, 1988, 160 SCRA 861. 140[53] Supra note 48. 141[54] Supra at 898.

212

213

of the public interest sought to be secured by the legislationthe reference here is to the nature and gravity of the evil which Congress seeks to prevent; (d) whether the specific restriction decreed by Congress is reasonably appropriate and necessary for the protection of such public interest; and (e) whether the necessary safeguarding of the public interest involved may be achieved by some other measure less restrictive of the protected freedom.142[55] This balancing of interest test, to borrow from Professor Kauper,143[56] rests on the theory that it is the courts function in a case before it when it finds public interests served by legislation, on the one hand, and the free expression clause affected by it, on the other, to balance one against the other and arrive at a judgment where the greater weight shall be placed. If, on balance, it appears that the public interest served by restrictive legislation is of such nature that it outweighs the abridgment of freedom, then the court will find the legislation valid. In short, the balance-of-interests theory rests on the basis that constitutional freedoms are not absolute, not even those stated in the free speech and expression clause, and that they may be abridged to some extent to serve appropriate and important interests.144[57] To the mind of the Court, the balancing of interest doctrine is the more appropriate test to follow. In the case at bar, petitioner used indecent and obscene language and a three (3)-month suspension was slapped on him for breach of MTRCB rules. In this setting, the assertion by petitioner of his enjoyment of his freedom of speech is ranged against the duty of the government to protect and promote the development and welfare of the youth. After a careful examination of the factual milieu and the arguments raised by petitioner in support of his claim to free speech, the Court rules that the governments interest to protect and promote the interests and welfare of the children adequately buttresses the reasonable curtailment and valid restraint on petitioners prayer to continue as program host of Ang Dating Daan during the suspension period. No doubt, one of the fundamental and most vital rights granted to citizens of a State is the freedom of speech or expression, for without the enjoyment of such right, a free, stable, effective, and progressive democratic state would be difficult to attain. Arrayed against the freedom of speech is the right of the youth to their moral, spiritual, intellectual, and social being which the State is constitutionally tasked to promote and protect. Moreover, the State is also mandated to recognize and support the vital role of the youth in nation building as laid down in Sec. 13, Art. II of the 1987 Constitution. The Constitution has, therefore, imposed the sacred obligation and responsibility on the State to provide protection to the youth against illegal or improper activities which may prejudice their general well-being. The Article on youth, approved on second reading by the Constitutional Commission, explained that the State shall extend social protection to minors against all forms of neglect, cruelty, exploitation, immorality, and

Supra at 899-900. Kauper, CIVIL LIBERTIES AND THE CONSTITUTION 113 (1966); cited in Gonzales v. COMELEC, supra note 48; also cited in J.G. Bernas, S.J., THE 1987 CONSTITUTION OF THE REPUBLIC OF THE PHILIPPINES: A COMMENTARY (2003). 144[57] Id.
143[56]

142[55]

213

214

practices which may foster racial, religious or other forms of discrimination.145[58] Indisputably, the State has a compelling interest in extending social protection to minors against all forms of neglect, exploitation, and immorality which may pollute innocent minds. It has a compelling interest in helping parents, through regulatory mechanisms, protect their childrens minds from exposure to undesirable materials and corrupting experiences. The Constitution, no less, in fact enjoins the State, as earlier indicated, to promote and protect the physical, moral, spiritual, intellectual, and social well-being of the youth to better prepare them fulfill their role in the field of nation-building.146[59] In the same way, the State is mandated to support parents in the rearing of the youth for civic efficiency and the development of moral character.147[60] Petitioners offensive and obscene language uttered in a television broadcast, without doubt, was easily accessible to the children. His statements could have exposed children to a language that is unacceptable in everyday use. As such, the welfare of children and the States mandate to protect and care for them, as parens patriae,148[61] constitute a substantial and compelling government interest in regulating petitioners utterances in TV broadcast as provided in PD 1986. FCC explains the duty of the government to act as parens patriae to protect the children who, because of age or interest capacity, are susceptible of being corrupted or prejudiced by offensive language, thus: [B]roadcasting is uniquely accessible to children, even those too young to read. Although Cohens written message, [Fuck the Draft], might have been incomprehensible to a first grader, Pacificas broadcast could have enlarged a childs vocabulary in an instant. Other forms of offensive expression may be withheld from the young without restricting the expression at its source. Bookstores and motion picture theaters, for example, may be prohibited from making indecent material available to children. We held in Ginsberg v. New York that the governments interest in the well-being of its youth and in supporting parents claim to authority in their own household justified the regulation of otherwise protected expression. The ease with which children may obtain access to broadcast material, coupled with the concerns recognized in Ginsberg, amply justify special treatment of indecent broadcasting. Moreover, Gonzales v. Kalaw Katigbak likewise stressed the duty of the State to attend to the welfare of the young: x x x It is the consensus of this Court that where television is concerned, a less liberal approach calls for observance. This is so because unlike motion pictures where the patrons have to pay their way, television reaches every home where there is a set. Children then will likely will be among the avid viewers of the programs therein shown. As was observed by Circuit Court of Appeals Judge Jerome Frank, it is hardly the concern of the law to deal with the sexual fantasies of the adult population. It

145[58] 146[59]

Bernas, supra note 27, at 81. CONSTITUTION, Art. II, Sec. 13. 147[60] Id., id., Sec. 12. 148[61] Id.

214

215

cannot be denied though that the State as parens patriae is called upon to manifest an attitude of caring for the welfare of the young.149[62] The compelling need to protect the young impels us to sustain the regulatory action MTRCB took in the narrow confines of the case. To reiterate, FCC justified the restraint on the TV broadcast grounded on the following considerations: (1) the use of television with its unique accessibility to children, as a medium of broadcast of a patently offensive speech; (2) the time of broadcast; and (3) the G rating of the Ang Dating Daan program. And in agreeing with MTRCB, the court takes stock of and cites with approval the following excerpts from FCC: It is appropriate, in conclusion, to emphasize the narrowness of our holding. This case does not involve a two-way radio conversation between a cab driver and a dispatcher, or a telecast of an Elizabethan comedy. We have not decided that an occasional expletive in either setting would justify any sanction. x x x The [FFCs] decision rested entirely on a nuisance rationale under which context is all important. The concept requires consideration of a host of variables. The time of day was emphasized by the [FFC]. The content of the program in which the language is used will affect the composition of the audience x x x. As Mr. Justice Sutherland wrote a nuisance may be merely a right thing in the wrong place, like a pig in the parlor instead of the barnyard. We simply hold that when the [FCC] finds that a pig has entered the parlor, the exercise of its regulatory power does not depend on proof that the pig is obscene. (Citation omitted.) There can be no quibbling that the remarks in question petitioner uttered on prime-time television are blatantly indecent if not outright obscene. It is the kind of speech that PD 1986 proscribes necessitating the exercise by MTRCB of statutory disciplinary powers. It is the kind of speech that the State has the inherent prerogative, nay duty, to regulate and prevent should such action served and further compelling state interests. One who utters indecent, insulting, or offensive words on television when unsuspecting children are in the audience is, in the graphic language of FCC, a pig in the parlor. Public interest would be served if the pig is reasonably restrained or even removed from the parlor. Ergo, petitioners offensive and indecent language can be subjected to prior restraint. Petitioner theorizes that the three (3)-month suspension is either prior restraint or subsequent punishment that, however, includes prior restraint, albeit indirectly. After a review of the facts, the Court finds that what MTRCB imposed on petitioner is an administrative sanction or subsequent punishment for his offensive and obscene language in Ang Dating Daan. To clarify, statutes imposing prior restraints on speech are generally illegal and presumed unconstitutional breaches of the freedom of speech. The exceptions to prior restraint are movies, television, and radio broadcast censorship in view of its access to numerous people, including the young who must be insulated from the prejudicial effects of
149[62]

Supra note 26, at 729.

215

216

unprotected speech. PD 1986 was passed creating the Board of Review for Motion Pictures and Television (now MTRCB) and which requires prior permit or license before showing a motion picture or broadcasting a TV program. The Board can classify movies and television programs and can cancel permits for exhibition of films or television broadcast. The power of MTRCB to regulate and even impose some prior restraint on radio and television shows, even religious programs, was upheld in Iglesia Ni Cristo v. Court of Appeals. Speaking through Chief Justice Reynato S. Puno, the Court wrote: We thus reject petitioners postulate that its religious program is per se beyond review by the respondent Board. Its public broadcast on TV of its religious program brings it out of the bosom of internal belief. Television is a medium that reaches even the eyes and ears of children. The Court iterates the rule that the exercise of religious freedom can be regulated by the State when it will bring about the clear and present danger of some substantive evil which the State is duty bound to prevent, i.e., serious detriment to the more overriding interest of public health, public morals, or public welfare. x x x xxx While the thesis has a lot to commend itself, we are not ready to hold that [PD 1986] is unconstitutional for Congress to grant an administrative body quasi-judicial power to preview and classify TV programs and enforce its decision subject to review by our courts. As far back as 1921, we upheld this setup in Sotto vs. Ruiz, viz: The use of the mails by private persons is in the nature of a privilege which can be regulated in order to avoid its abuse. Persons possess no absolute right to put into the mail anything they please, regardless of its character.150[63] Bernas adds: Under the decree a movie classification board is made the arbiter of what movies and television programs or parts of either are fit for public consumption. It decides what movies are immoral, indecent, contrary to law and/or good customs, injurious to the prestige of the Republic of the Philippines or its people, and what tend to incite subversion, insurrection, rebellion or sedition, or tend to undermine the faith and confidence of the people in their government and/or duly constituted authorities, etc. Moreover, its decisions are executory unless stopped by a court.151[64] Moreover, in MTRCB v. ABS-CBN Broadcasting Corporation,152[65] it was held that the power of review and prior approval of MTRCB extends to all television programs and is valid despite the freedom of speech guaranteed by the Constitution. Thus, all broadcast networks are regulated by the MTRCB since they are required to get a permit before they air their television programs. Consequently, their right to enjoy their freedom of speech is subject to that requirement. As lucidly explained by Justice Dante O. Tinga, government regulations through the
150[63] 151[64]

G.R. No. 119673, July 26, 1996, 259 SCRA 529, 544, 552. Supra note 56, at 235. 152[65] G.R. No. 155282, January 17, 2005, 448 SCRA 575.

216

217

MTRCB became a necessary evil with the government taking the role of assigning bandwidth to individual broadcasters. The stations explicitly agreed to this regulatory scheme; otherwise, chaos would result in the television broadcast industry as competing broadcasters will interfere or co-opt each others signals. In this scheme, station owners and broadcasters in effect waived their right to the full enjoyment of their right to freedom of speech in radio and television programs and impliedly agreed that said right may be subject to prior restraintdenial of permit or subsequent punishment, like suspension or cancellation of permit, among others. The three (3) months suspension in this case is not a prior restraint on the right of petitioner to continue with the broadcast of Ang Dating Daan as a permit was already issued to him by MTRCB for such broadcast. Rather, the suspension is in the form of permissible administrative sanction or subsequent punishment for the offensive and obscene remarks he uttered on the evening of August 10, 2004 in his television program, Ang Dating Daan. It is a sanction that the MTRCB may validly impose under its charter without running afoul of the free speech clause. And the imposition is separate and distinct from the criminal action the Board may take pursuant to Sec. 3(i) of PD 1986 and the remedies that may be availed of by the aggrieved private party under the provisions on libel or tort, if applicable. As FCC teaches, the imposition of sanctions on broadcasters who indulge in profane or indecent broadcasting does not constitute forbidden censorship. Lest it be overlooked, the sanction imposed is not per se for petitioners exercise of his freedom of speech via television, but for the indecent contents of his utterances in a G rated TV program. More importantly, petitioner is deemed to have yielded his right to his full enjoyment of his freedom of speech to regulation under PD 1986 and its IRR as television station owners, program producers, and hosts have impliedly accepted the power of MTRCB to regulate the broadcast industry. Neither can petitioners virtual inability to speak in his program during the period of suspension be plausibly treated as prior restraint on future speech. For viewed in its proper perspective, the suspension is in the nature of an intermediate penalty for uttering an unprotected form of speech. It is definitely a lesser punishment than the permissible cancellation of exhibition or broadcast permit or license. In fine, the suspension meted was simply part of the duties of the MTRCB in the enforcement and administration of the law which it is tasked to implement. Viewed in its proper context, the suspension sought to penalize past speech made on prime-time G rated TV program; it does not bar future speech of petitioner in other television programs; it is a permissible subsequent administrative sanction; it should not be confused with a prior restraint on speech. While not on all fours, the Court, in MTRCB,153[66] sustained the power of the MTRCB to penalize a broadcast company for exhibiting/airing a pre-taped TV episode without Board authorization in violation of Sec. 7 of PD 1986. Any simplistic suggestion, however, that the MTRCB would be crossing the limits of its authority were it to regulate and even restrain the prime-time television broadcast of indecent or obscene speech in a G
153[66]

Supra note 65.

217

218

rated program is not acceptable. As made clear in Eastern Broadcasting Corporation, the freedom of television and radio broadcasting is somewhat lesser in scope than the freedom accorded to newspaper and print media. The MTRCB, as a regulatory agency, must have the wherewithal to enforce its mandate, which would not be effective if its punitive actions would be limited to mere fines. Television broadcasts should be subject to some form of regulation, considering the ease with which they can be accessed, and violations of the regulations must be met with appropriate and proportional disciplinary action. The suspension of a violating television program would be a sufficient punishment and serve as a deterrent for those responsible. The prevention of the broadcast of petitioners television program is justified, and does not constitute prohibited prior restraint. It behooves the Court to respond to the needs of the changing times, and craft jurisprudence to reflect these times. Finally, petitioner argues that there has been undue delegation of legislative power, as PD 1986 does not provide for the range of imposable penalties that may be applied with respect to violations of the provisions of the law. The argument is without merit. In Edu v. Ericta, the Court discussed the matter of undue delegation of legislative power in the following wise: It is a fundamental principle flowing from the doctrine of separation of powers that Congress may not delegate its legislative power to the two other branches of the government, subject to the exception that local governments may over local affairs participate in its exercise. What cannot be delegated is the authority under the Constitution to make laws and to alter and repeal them; the test is the completeness of the statute in all its term and provisions when it leaves the hands of the legislature. To determine whether or not there is an undue delegation of legislative power, the inquiry must be directed to the scope and definiteness of the measure enacted. The legislature does not abdicate its functions when it describes what job must be done, who is to do it, and what is the scope of his authority. For a complex economy, that may indeed be the only way in which the legislative process can go forward. A distinction has rightfully been made between delegation of power to make laws which necessarily involves a discretion as to what it shall be, which constitutionally may not be done, and delegation of authority or discretion as to its execution to be exercised under and in pursuance of the law, to which no valid objection can be made. The Constitution is thus not to be regarded as denying the legislature the necessary resources of flexibility and practicability. To avoid the taint of unlawful delegation, there must be a standard, which implies at the very least that the legislature itself determines matters of principle and lays down fundamental policy. Otherwise, the charge of complete abdication may be hard to repel. A standard thus defines legislative policy, marks its limits, maps out its boundaries and specifies the public agency to apply it. It indicates the circumstances under which the legislative command is to be effected. It is the criterion by which legislative purpose may be carried out. Thereafter, the executive or administrative office designated may in pursuance of the above guidelines promulgate supplemental rules and regulations.154[67]
154[67]

No. L-32096, October 24, 1970, 35 SCRA 481, 496-497.

218

219

Based on the foregoing pronouncements and analyzing the law in question, petitioners protestation about undue delegation of legislative power for the sole reason that PD 1986 does not provide for a range of penalties for violation of the law is untenable. His thesis is that MTRCB, in promulgating the IRR of PD 1986, prescribing a schedule of penalties for violation of the provisions of the decree, went beyond the terms of the law. Petitioners posture is flawed by the erroneous assumptions holding it together, the first assumption being that PD 1986 does not prescribe the imposition of, or authorize the MTRCB to impose, penalties for violators of PD 1986. As earlier indicated, however, the MTRCB, by express and direct conferment of power and functions, is charged with supervising and regulating, granting, denying, or canceling permits for the exhibition and/or television broadcast of all motion pictures, television programs, and publicity materials to the end that no such objectionable pictures, programs, and materials shall be exhibited and/or broadcast by television. Complementing this provision is Sec. 3(k) of the decree authorizing the MTRCB to exercise such powers and functions as may be necessary or incidental to the attainment of the purpose and objectives of [the law]. As earlier explained, the investiture of supervisory, regulatory, and disciplinary power would surely be a meaningless grant if it did not carry with it the power to penalize the supervised or the regulated as may be proportionate to the offense committed, charged, and proved. As the Court said in Chavez v. National Housing Authority: x x x [W]hen a general grant of power is conferred or duty enjoined, every particular power necessary for the exercise of the one or the performance of the other is also conferred. x x x [W]hen the statute does not specify the particular method to be followed or used by a government agency in the exercise of the power vested in it by law, said agency has the authority to adopt any reasonable method to carry out its function.155[68] Given the foregoing perspective, it stands to reason that the power of the MTRCB to regulate and supervise the exhibition of TV programs carries with it or necessarily implies the authority to take effective punitive action for violation of the law sought to be enforced. And would it not be logical too to say that the power to deny or cancel a permit for the exhibition of a TV program or broadcast necessarily includes the lesser power to suspend? The MTRCB promulgated the IRR of PD 1986 in accordance with Sec. 3(a) which, for reference, provides that agency with the power [to] promulgate such rules and regulations as are necessary or proper for the implementation of this Act, and the accomplishment of its purposes and objectives x x x. And Chapter XIII, Sec. 1 of the IRR providing: Section 1. VIOLATIONS AND ADMINISTRATIVE SANCTIONS.Without prejudice to the immediate filing of the appropriate criminal action and the immediate seizure of the pertinent articles pursuant to Section 13, any violation of PD 1986 and its Implementing Rules and Regulations governing motion pictures,
Supra note 17; citing Angara v. Electoral Commission, 63 Phil. 139 (1936); Provident Tree Farms, Inc. v. Batario, Jr., G.R. No. 92285, March 28, 1994, 231 SCRA 463.
155[68]

219

220

television programs, and related promotional materials shall be penalized with suspension or cancellation of permits and/or licenses issued by the Board and/or with the imposition of fines and other administrative penalty/penalties. The Board recognizes the existing Table of Administrative Penalties attached without prejudice to the power of the Board to amend it when the need arises. In the meantime the existing revised Table of Administrative Penalties shall be enforced. (Emphasis added.) This is, in the final analysis, no more than a measure to specifically implement the aforequoted provisions of Sec. 3(d) and (k). Contrary to what petitioner implies, the IRR does not expand the mandate of the MTRCB under the law or partake of the nature of an unauthorized administrative legislation. The MTRCB cannot shirk its responsibility to regulate the public airwaves and employ such means as it can as a guardian of the public. In Sec. 3(c), one can already find the permissible actions of the MTRCB, along with the standards to be applied to determine whether there have been statutory breaches. The MTRCB may evaluate motion pictures, television programs, and publicity materials applying contemporary Filipino cultural values as standard, and, from there, determine whether these audio and video materials are objectionable for being immoral, indecent, contrary to law and/or good customs, [etc.] x x x and apply the sanctions it deems proper. The lawmaking body cannot possibly provide for all the details in the enforcement of a particular statute.156[69] The grant of the rule-making power to administrative agencies is a relaxation of the principle of separation of powers and is an exception to the non-delegation of legislative powers.157[70] Administrative regulations or subordinate legislation calculated to promote the public interest are necessary because of the growing complexity of modern life, the multiplication of the subjects of governmental regulations, and the increased difficulty of administering the law.158[71] Allowing the MTRCB some reasonable elbow-room in its operations and, in the exercise of its statutory disciplinary functions, according it ample latitude in fixing, by way of an appropriate issuance, administrative penalties with due regard for the severity of the offense and attending mitigating or aggravating circumstances, as the case may be, would be consistent with its mandate to effectively and efficiently regulate the movie and television industry. But even as we uphold the power of the MTRCB to review and impose sanctions for violations of PD 1986, its decision to suspend petitioner must be modified, for nowhere in that issuance, particularly the power-defining Sec. 3 nor in the MTRCB Schedule of Administrative Penalties effective January 1, 1999 is the Board empowered to suspend the program host or even to prevent certain people from appearing in television programs. The MTRCB, to be sure, may prohibit the broadcast of such television programs or cancel permits for exhibition, but it may not suspend television personalities, for such would be beyond its jurisdiction. The MTRCB cannot extend its exercise of regulation beyond what the law provides. Only persons, offenses, and penalties clearly falling clearly within the letter and spirit of PD 1986 will be considered to be within the decrees penal or disciplinary operation.
156[69] 157[70]

People v. Maceren, No. L-32166, October 18, 1977, 79 SCRA 450, 458. Id. 158[71] Id.

220

221

And when it exists, the reasonable doubt must be resolved in favor of the person charged with violating the statute and for whom the penalty is sought. Thus, the MTRCBs decision in Administrative Case No. 01 -04 dated September 27, 2004 and the subsequent order issued pursuant to said decision must be modified. The suspension should cover only the television program on which petitioner appeared and uttered the offensive and obscene language, which sanction is what the law and the facts obtaining call for. In ending, what petitioner obviously advocates is an unrestricted speech paradigm in which absolute permissiveness is the norm. Petitioners flawed belief that he may simply utter gutter profanity on television without adverse consequences, under the guise of free speech, does not lend itself to acceptance in this jurisdiction. We repeat: freedoms of speech and expression are not absolute freedoms. To say any act that restrains speech should be greeted with furrowed brows is not to say that any act that restrains or regulates speech or expression is per se invalid. This only recognizes the importance of freedoms of speech and expression, and indicates the necessity to carefully scrutinize acts that may restrain or regulate speech. WHEREFORE, the decision of the MTRCB in Adm. Case No. 01-04 dated September 27, 2004 is hereby AFFIRMED with the MODIFICATION of limiting the suspension to the program Ang Dating Daan. As thus modified, the fallo of the MTRCB shall read as follows: WHEREFORE, in view of all the foregoing, a Decision is hereby rendered, imposing a penalty of THREE (3) MONTHS SUSPENSION on the television program, Ang Dating Daan, subject of the instant petition. DISSENTING OPINION, Justice Antonio Carpio. I dissent because the three-month suspension of petitioners TV program Ang Dating Daan constitutes an unconstitutional prior restraint on freedom of expression. The suspension prevents petitioner from even reciting the Lords Prayer, or even saying hello to viewers, in his TV program. The suspension bars the public airing of petitioners TV program regardless of whatever subject matter petitioner, or anyone else, wishes to discuss in petitioners TV program. This is like suspending the publication of the Philippine Daily Inquirer for three months if its editorial describes a private person as masahol pa sa putang babae. This is also similar to suspending for three months the column of a newspaper columnist for using the expletive putang ina mo in his column. Such suspension is the censorship that the Constitution outlaws when it states that [n]o law shall be passed abridging the freedom of speech, of expression, or of the press x x x.159[1] The remedy of any aggrieved person is to file a libel or tort case after the utterance or publication of such cusswords. Our libels laws punish with fine, imprisonment or damages libelous language already

159[1]

Section 4, Article III, Constitution.

221

222

uttered or published.160[2] Our tort laws also allow recovery of damages for tortious speech already uttered or published.161[3] However, both our libel and tort laws never impose a gag order on future expression because that will constitute prior restraint or censorship. Thus, our libel and tort laws do not allow the filing of a suit to enjoin or punish an expression that has yet to be uttered or written. Indeed, there can never be a prior restraint on future expression, whether for fear of possible libelous utterance or publication, or as a punishment for past libelous utterance or publication. Otherwise, many of the radio and TV political programs will have to be banned for the frequent use of cusswords and other libelous language. Even politicians will have to be barred from addressing political rallies, or the rallies themselves will have to be banned, because politicians often use cusswords and other profanities during political rallies. In the present case, the three-month preventive suspension of petitioners TV program bars petitioner from talking about the weather, or from talking about the birds and the bees, or even from talking about nothingness, in his TV program. The public airing of the entire TV program, regardless of its content, is totally suppressed for three months. The Government has no power under the Constitution to so brazenly suppress freedom of expression. This Court should never give its imprimatur to such a blatant violation of a fundamental constitutional right, which has been described as the one basic right that makes all other civil, human and political rights possible. Prior Restraint on Expression The well-settled rule is there can be no prior restraint on expression. This rule emanates from the constitutional command that [n]o law shall be passed abridging the freedom of speech, of expression, or of the press x x x. The history of freedom of ex pression has been a constant struggle against the censors prior restraint on expression. The leading American case of Near v. Minnesota162[4] teaches us that the primordial purpose of the Free Expression Clause is to prevent prior restraint on expression. This well-settled rule, however, is subject to exceptions narrowly carved out by courts over time because of necessity. In this jurisdiction, we recognize only four exceptions, namely: pornography,163[5] false or misleading advertisement,164[6] advocacy of imminent lawless action,165[7] and danger to national security.166[8] Only in these
160[2]

Article 353-359, Revised Penal Code; Article 33, Civil Code.


161[3]

Article 26, Civil Code.


162[4]

283 U.S. 697 (1931).


163[5]

Gonzales v. Kalaw-Katigbak, 222 Phil. 225 (1985).


164[6]

Pharmaceutical and Health Care Association of the Philippines v. Duque III , G.R. No. 173034, 9 October 2007, 535 SCRA 265.
165[7]

Eastern Broadcasting Corporation v. Dans , No. 222 Phil. 151 (1985).

222

223

instances may expression be subject to prior restraint. expression is not subject to prior restraint.

All other

Although pornography, false or misleading advertisement, advocacy of imminent lawless action, and expression endangering national security may be subject to prior restraint, such prior restraint must hurdle a high barrier. First, such prior restraint is strongly presumed as unconstitutional. Second, the government bears a heavy burden of justifying such prior restraint.167[9] The test to determine the constitutionality of prior restraint on pornography, advocacy of imminent lawless action, and expression endangering national security is the clear and present danger test. The expression subject to prior restraint must present a clear and present danger of bringing about a substantive evil the State has a right and duty to prevent, and such danger must be grave and imminent.168[10] The power of Congress to impose prior restraint on false or misleading advertisements emanates from the constitutional provision that the advertising industry is impressed with public interest, and shall be regulated by law for the protection of consumers and the promotion of the general welfare.169[11] Prior restraint on expression may be either content-based or content-neutral. Content-based prior restraint is aimed at suppressing the message or idea contained in the expression. Courts subject content-based restraint to strict scrutiny. Content-neutral restraint on expression is restraint that regulates the time, place or manner of expression in public places without any restraint on the content of the expression. Courts subject content-neutral restraint to intermediate scrutiny. Subsequent Punishment of Expression The rule is also well-settled that expression cannot be subject to subsequent punishment. This rule also emanates from the constitutional command that [n]o law shall be passed abridging the freedom of speech, of expression, or of the press x x x. However, courts again have carved out narrow exceptions to this rule out of necessity. The exceptions start with the four types of expression that may be subject to prior restraint. If a certain expression is subject to prior restraint, its utterance or publication in violation of the lawful restraint naturally subjects the person responsible to subsequent punishment. Thus, acts of pornography,170[12] false or misleading advertisement,171[13]
166[8]

Id.
167[9]

Iglesia ni Cristo (INC) v. Court of Appeals, G.R. No. 119673, 26 July 1996, 259 SCRA 529; New York Times v. United States, 403 U.S. 713 (1971).
168[10]

Bayan v. Ermita, G.R. Nos. 169838, 169848 and 169881, 25 April 2006, 488 SCRA 226.
169[11]

Section 11(2), Article XVI, Constitution.


170[12]

Article 201, Revised Penal Code.


171[13]

Section 6(a), Milk Code.

223

224

advocacy of imminent lawless action,172[14] and endangering national security,173[15] are all punishable under the law. Two other exceptions are defamation,174[16] which includes libel and slander, and tortious speech.175[17] Defamatory and tortious speech, per se, are not subject to prior restraint because by definition they do not constitute a clear and present danger to the State that is grave and imminent. Once defamatory or tortuous speech rises to the level of advocacy of imminent lawless action, then it may be subject to prior restraint because it is seditious176[18] but not because it is defamatory or tortious. Defamation and tortious conduct, however, may be subject to subsequent punishment, civilly or criminally. Fighting words are not subject to subsequent punishment unless they are defamatory or tortious. Fighting words refer to profane or vulgar words that are likely to provoke a violent response from an audience. Profane or vulgar words like Fuck the draft, when not directed at any particular person, ethnic or religious group, are not subject to subsequent punishment.177[19] As aptly stated, one mans vulgarity may be another mans lyric.178[20] If profane or vulgar language like Fuck the draft is not subject to subsequent punishment, then with more reason it cannot be subject to prior restraint. Without a law punishing the actual utterance or publication of an expression, an expression cannot be subject to prior restraint because such expression is not unlawful or illegal. Prior restraint is more deleterious to freedom of expression than subsequent punishment. Although subsequent punishment also deters expression, still the ideas are disseminated to the public. Prior restraint prevents even the dissemination of ideas to the public. Thus, the threemonth suspension of petitioners TV program, being a prior restraint on expression, has far graver ramifications than any possible subsequent punishment of petitioner. Three-Month Suspension is a Prohibited Prior Restraint The three-month suspension of petitioners TV program is indisputably a prior restraint on expression. During the three-month suspension, petitioner cannot utter a single word in his TV program because the program is totally suppressed. A prior restraint may be justified only if the expression falls under any of the four types of
172[14]

Article 142, Revised Penal Code.


173[15]

Article 138, Revised Penal Code.


174[16]

See note 2.
175[17]

See note 3.
176[18]

Articles 138 and 142, Revised Penal Code.


177[19]

Cohen v. California, 403 U.S. 15 (1971).


178[20]

Id.

224

225

expression that may be subject to prior restraint, namely, pornography, false or misleading advertisement, advocacy of imminent lawless action, and danger to national security. Obviously, what petitioner uttered does not fall under any of the four types of expression that may be subject to prior restraint. What respondents assail is the following ranting of petitioner: Lehitimong anak ng demonyo; sinungaling; Gago ka talaga Michael, masahol ka pa sa putang babae o di ba. Yung putang babae ang gumagana lang doon yung ibaba, [dito] kay Michael ang gumagana ang itaas, o di ba! O, masahol pa sa putang babae yan. Sabi ng lola ko masahol pa sa putang babae yan. Sobra ang kasinungalingan ng mga demonyong ito No matter how offensive, profane or vulgar petitioners words may be, they do not constitute pornography, false or misleading advertisement, advocacy of imminent lawless action, or danger to national security. Thus, petitioners offensive, profane or vulgar language cannot be subject to prior restraint but may be subject to subsequent punishment if defamatory or tortious. Any prior restraint is strongly presumed to be unconstitutional and the government bears a heavy burden of justifying such prior restraint.179[21] Such prior restraint must pass the clear and present danger test. The majority opinion, which imposes a prior restraint on expression, is totally bereft of any discussion that petitioners ranting poses a clear and present danger to the State that is grave and imminent. The respondents have not presented any credible justification to overcome the strong presumption of unconstitutionality accorded to the three-month suspension order. The three-month suspension cannot be passed off merely as a preventive suspension that does not partake of a penalty. The actual and real effect of the three-month suspension is a prior restraint on expression in violation of a fundamental constitutional right. Even Congress cannot validly pass a law imposing a three-month preventive suspension on freedom of expression for offensive or vulgar language uttered in the past. Congress may punish such offensive or vulgar language, after their utterance, with damages, fine or imprisonment but Congress has no power to suspend or suppress the peoples right to speak freely because of such past utterances. In short, Congress may pass a law punishing defamation or tortious speech but the punishment cannot be the suspension or suppression of the constitutional right to freedom of expression. Otherwise, such law would be abridging the freedom of speech, of expression, or of the press. If Congress cannot pass such a law, neither can respondent MTRCB promulgate a rule or a decision suspending for three months petitioners constitutional right to freedom of expression. And of course, neither can

179[21]

See note 9.

225

226

this Court give its stamp of imprimatur to such an unconstitutional MTRCB rule or decision. Read: 1. Gonzales vs. Kalaw Katigbak, 137 SCRA 717 2. New York Times vs. U.S., 403 U.S. 713 (Any system of prior restraints of expression comes to this Court bearing a heavy presumption against its validity) 3. Near vs. Minnesota, 283 U.S. 697 4. Times Film vs. City of Chicago, 365 U.S. 43 5. Freedman vs. Maryland, 380 U.S. 51 8. Clear and present danger and dangerous tendency rule (whether the words used in such circumstances and are of such a nature as to create a clear and present danger that they will bring about the substantive evils that the State has the right to prevent) 7-a. Dangerous tendency rule (If the words uttered create a dangerous tendency which the State has the right to prevent, then such words are punishable) Read: 1. Cabansag vs. Fernandez, 102 Phil. 152 2. Read again the Reyes and Ruiz cases, supra 3. Read again Zaldivar vs. Sandiganbayan, GR No. 7960-707& Zaldivar vs. Gonzales, GR No. 80578, February 1, 1989 8. The balancing-of-interest test (When a particular conduct is regulated in the interest of the public order, and the regulation results in an indirect, conditional, partial abridgment of speech, the duty of the courts is to determine which of the 2 conflicting interests demand greater protection under the circumstances presented.) Read: AYER PRODUCTION VS. JUDGE CAPULONG, JUAN PONCE ENRILE, ET AL., 160 SCRA 861 Read also: 1. Lagunzad vs. Gonzales, 92 SCRA 476 2. Gitlow vs. New York, 268 U.S. 652, including this test by Justice Holmes 3. See also Zaldivar case above

the criticism on

CHAPTER VI - THE NON-ESTABLISHMENT OF RELIGION CLAUSE Section 5. No law shall be made respecting the establishment of religion, or prohibiting the free exercise thereof. The free exercise and enjoyment of religious profession and worship, without discrimination or preference shall forever be allowed. No religious test shall be
226

227

required for the exercise of civil or political rights. ESTRADA VS. SOLEDAD ESCRITOR, 492 SCRA 1 (Resolution of the Motion for Reconsideration), 408 SCRA 1 Puno, J. Respondent is the Court interpreter of RTC Branch 253, Las Pinas City. Complainant requested for an investigation of respondent for living with a man not her husband while she was still legally married and having borne a child within this live-in arrangement. Estrada believes that Escritor is committing a grossly immoral act which tarnishes the image of the judiciary, thus she should not be allowed to remain employed therein as it might appear that the court condones her act. Respondent admitted she started living with Luciano Quilapio, Jr. more than 20 years ago when her husband was still alive but living with another woman. She likewise admitted having a son with Quilapio but denies any liability for alleged grossly immoral conduct because: She is a member of the Jehovahs Witnesses and the Watch Tower Society; That the conjugal arrangement was in conformity with their religious beliefs; That the conjugal arrangement with Quilapio has the approval of her congregation. Escritor likewise claimed that she had executed a DECLARATION OF PLEDGING FAITHFULNESS in accordance with her religion which allows members of the Jehovahs witnesses who have been abandoned by their spouses to enter into marital relations. The Declaration thus makes the resulting union moral and binding within the congregation all over the world except in countries where divorce is allowed. HELD: Escritors conjugal arrangement cannot be penalized as she has made out a case for exemption from the law based on her fundamental right to religion. The Court recognizes that state interests must be upheld in order that freedoms---including religious freedom---may be enjoyed. IN THE AREA OF RELIGIOUS EXERCISE AS A PREFERRED FREEDOM, HOWEVER, MAN STANDS ACCOUNTABLE TO AN AUTHORITY HIGHER THAN THE STATE, and so the state interest sought to be upheld must be so compelling that its violation will erode the very fabric of the state that will also protect the freedom. In the absence of a showing that the state interest exists, man must be allowed to subscribe to the Infinite. Escritor was therefore held not administratively liable for grossly immoral conduct. FREEDOM OF RELIGION -any specific system of belief, worship or conduct, often involving a code of ethics and philosophy.
227

228

-A profession of faith to an active power that binds and elevates man to his Creator. The existence of a Divine being is not necessarily inherent in religion; the Buddhists espouses a way of life without reference to an omnipotent God. Strong fences make good neighbors. The idea is to delineate the boundaries between two institutions and prevent encroachments by one against the other. The doctrine cuts both ways. It is not only the State that is prohibited from interfering in purely ecclesiastical affairs; the Church is likewise barred from meddling in purely secular matters. NON-STABLISHMENT CLAUSE: It simply means that the State cannot set up a church; nor pass laws which aids one religion; aid all religion, or prefer one religion over another nor force nor influence a person to go to or remain away from church against his will; or force him to profess a belief or disbelief; that the State cannot openly or secretly participate in the affairs of any religious organization or group and vice versa (EVERSON VS. BOARD OF EDUCATION, 330 US 1) This clause seeks to protect: Voluntarism---must come into existence through the voluntary support of its members; Insulation from political processgrowth through voluntary support of its members will not take place if there is intervention from the State. There will be no violation of the non-establishment clause if: the statute has a secular legislative purpose; its principal or primary effect is one that neither advances nor inhibits religion; and it does not foster an excessive government entanglement with religion. (LEMON VS. KURTZMAN, 403 US 602) The government is neutral and while protecting all, it prefers none and disparages none. All here applies both to the believer and the non-believer. FREEDOM OF RELIGION INCLUDES FREEDOM FROM RELIGION; THE RIGHT TO WORHIP INCLUDES THE RIGHT NOT TO WORSHIP. SCHOOL PRAYER CASE (ENGEL VS. VITALE, 370 US 421) It is unconstitutional for a school to require the students to recite a prayer composed by the Board of Regents at the starts of the days class. It is no part of the business of government to compose official prayers for any group of the American People. SCHOOL DISTRICT OF ABINGTON VS. SCHEMPP, 374 US 203
228

229

It is unconstitutional for a law to require that at least 10 verses from the Holy Bible be read daily without comment because the same constitute a religious exercise which violates the nonestablishment clause. BOARD OF EDUCATION VS. ALLEN, 392 US 236 A law requiring the Board of Education to lend textbooks free of charge to all students from grades 7-12 of parochial school. This is constitutional since it is not the parochial school which gets the benefits but the parents. EVERSON VS. BOARD OF EDUCATION, 330 US 1 The law authorizing reimbursement of transportation expenses of school children going to and from parochial schools is not violative of the non-establishment clause because it will be the parents who get benefits, not the parochial school. RIGHT TO RELIGIOUS PROFESSION AND WORSHIP HAS TWO ASPECTS: a. Freedom to believe; and b. Freedom to act. IN the first, such freedom is absolute. He may indulge in his own theories about life and death; worship any god he chooses, or none at all. He may not be punished even if he cannot prove what he believes. In the second, if the individual externalizes what he believes, his freedom to do so becomes subject to the authority of the State. This is so because religious freedom can be exercised only with due regard to the rights of others. Example: Go forth and multiply---cannot marry several times just to comply. PEOPLE VS. LAGMAN & ZOSA, 38 O.G. 1676 Avoiding military duties based on religious grounds is not allowed in the Philippines because of Section 4, Article IIThe state is the protector of the people and it is the prime duty of the people to defend the State and in the fulfillment of this duty, the State may call all citizens to render military or civil service. IN RE SUMMERS, 325 US 561 The act of the Illinois Supreme Court denying admission to the bar because of his refusal to take in good faith an oath to support the Constitution of the State of Illinois which requires mandatory service in the military in times of war was reversed by the US Supreme Court stating that this constitutes a violation of the 1st Amendment which guarantees religious freedom. 1. Religious freedom in relation to impairment of the right to join associations,36 SCRA 445 contracts and

229

230

2. Read: 1. Aglipay vs. Ruiz, 64 Phil. 201 2. Garces vs. Estenzo, 104 SCRA 510 3. INK vs. Gironella, 106 SCRA 1 4. American Bible Society vs. City of Manila, 101 5. Gerona vs. Sec. of Education, 106 Phil. 11 6. Pamil vs. Teleron, November 20, 1978 7. Victoriano vs. Elizalde Rope, 59 SCRA 54 7. German vs. Barangan, 135 SCRA 514

Phil. 398

ANG LADLAD LGBT PARTY VS. COMELEC, G.R. No. 190582, April 7, 2010 DEL CASTILLO, J.: This is a Petition for Certiorari under Rule 65 of the Rules of Court, with an application for a writ of preliminary mandatory injunction, filed by Ang Ladlad LGBT Party (Ang Ladlad) against the Resolutions of the Commission on Elections (COMELEC) dated November 11, 2009 (the First Assailed Resolution) and December 16, 2009 (the Second Assailed Resolution) in SPP No. 09-228 (PL) (collectively, the Assailed Resolutions). The case has its roots in the COMELECs refusal to accredit Ang Ladlad as a party-list organization under Republic Act (RA) No. 7941, otherwise known as the Party-List System Act. Ang Ladlad is an organization composed of men and women who identify themselves as lesbians, gays, bisexuals, or trans-gendered individuals (LGBTs). Incorporated in 2003, Ang Ladlad first applied for registration with the COMELEC in 2006. The application for accreditation was denied on the ground that the organization had no substantial membership base. On August 17, 2009, Ang Ladlad again filed a Petition for registration with the COMELEC. Before the COMELEC, petitioner argued that the LGBT community is a marginalized and under-represented sector that is particularly disadvantaged because of their sexual orientation and gender identity; that LGBTs are victims of exclusion, discrimination, and violence; that because of negative societal attitudes, LGBTs are constrained to hide their sexual orientation; and that Ang Ladlad complied with the 8point guidelines enunciated by this Court in Ang Bagong Bayani-OFW Labor Party v. Commission on Elections. Ang Ladlad laid out its national membership base consisting of individual members and organizational supporters, and outlined its platform of governance. On November 11, 2009, after admitting the petitioners evidence, the COMELEC (Second Division) dismissed the Petition on moral grounds, stating that: x x x This Petition is dismissible on moral grounds. Petitioner defines the Filipino Lesbian, Gay, Bisexual and Transgender (LGBT) Community, thus: x x x a marginalized and under-represented sector that is particularly disadvantaged because of their sexual orientation and gender identity. and proceeded to define sexual orientation as that which: x x x refers to a persons capacity for profound emotional, affectional and sexual attraction to, and intimate and sexual relations with, individuals of a different gender, of the same gender, or more than one gender.

230

231

This definition of the LGBT sector makes it crystal clear that petitioner tolerates immorality which offends religious beliefs. The ANG LADLAD apparently advocates sexual immorality as indicated in the Petitions par. 6F: Consensual partnerships or relationships by gays and lesbians who are already of age. It is further indicated in par. 24 of the Petition which waves for the record: In 2007, Men Having Sex with Men or MSMs in the Philippines were estimated as 670,000 (Genesis 19 is the history of Sodom and Gomorrah). Laws are deemed incorporated in every contract, permit, license, relationship, or accreditation. Hence, pertinent provisions of the Civil Code and the Revised Penal Code are deemed part of the requirement to be complied with for accreditation. ANG LADLAD collides with Article 695 of the Civil Code which defines nuisance as Any act, omission, establishment, business, condition of property, or anything else which x x x (3) shocks, defies; or disregards decency or morality x x x It also collides with Article 1306 of the Civil Code: The contracting parties may establish such stipulations, clauses, terms and conditions as they may deem convenient, provided they are not contrary to law, morals, good customs, public order or public policy. Art 1409 of the Civil Code provides that Contracts whose cause, object or purpose is contrary to law, morals, good customs, public order or public policy are inexistent and void from the beginning. Finally to safeguard the morality of the Filipino community, the Revised Penal Code, as amended, penalizes Immoral doctrines, obscene publications and exhibitions and indecent shows as follows: Art. 201. Immoral doctrines, obscene publications and exhibitions, and indecent shows. The penalty of prision mayor or a fine ranging from six thousand to twelve thousand pesos, or both such imprisonment and fine, shall be imposed upon: 1. Those who shall publicly expound or proclaim doctrines openly contrary to public morals;

When Ang Ladlad sought reconsideration to the COMELEC EN BANC, three commissioners voted to overturn the First Assailed Resolution (Commissioners Gregorio Y. Larrazabal, Rene V. Sarmiento, and Armando Velasco), while three commissioners voted to deny Ang Ladlads Motion for Reconsideration (Commissioners Nicodemo T. Ferrer, Lucenito N. Tagle, and Elias R. Yusoph). The COMELEC Chairman, breaking the tie and speaking for the majority in his Separate Opinion, upheld the First Assailed Resolution, stating that: Ladlad is applying for accreditation as a sectoral party in the party-list system. Even assuming that it has properly proven its under-representation and marginalization, it cannot be said that Ladlads expressed sexual orientations per se would benefit the nation as a whole. Section 2 of the party-list law unequivocally states that the purpose of the partylist system of electing congressional representatives is to enable Filipino citizens belonging to marginalized and under-represented sectors, organizations and parties, and who lack well-defined political constituencies but who could contribute to the formulation and enactment of appropriate legislation that will benefit the nation as a whole, to become members of the House of Representatives.

231

232

If entry into the party-list system would depend only on the ability of an organization to represent its constituencies, then all representative organizations would have found themselves into the party-list race. But that is not the intention of the framers of the law. The party-list system is not a tool to advocate tolerance and acceptance of misunderstood persons or groups of persons. Rather, the party-list system is a tool for the realization of aspirations of marginalized individuals whose interests are also the nations only that their interests have not been brought to the attention of the nation because of their under representation. Until the time comes when Ladlad is able to justify that having mixed sexual orientations and transgender identities is beneficial to the nation, its application for accreditation under the party-list system will remain just that. Thus, even if societys understanding, tolerance, and acceptance of LGBTs is elevated, there can be no denying that Ladlad constituencies are still males and females, and they will remain either male or female protected by the same Bill of Rights that applies to all citizens alike. The COMELEC likewise used the Holy Bible and the Koran in denying Ladlads application. On January 4, 2010, Ang Ladlad filed this Petition, praying that the Court annul the Assailed Resolutions and direct the COMELEC to grant Ang Ladlads application for accreditation. Ang Ladlad also sought the issuance ex parte of a preliminary mandatory injunction against the COMELEC, which had previously announced that it would begin printing the final ballots for the May 2010 elections by January 25, 2010. On January 6, 2010, the Office of the Solicitor General (OSG was ordered to file its Comment on behalf of COMELEC not later than 12:00 noon of January 11, 2010. Instead of filing a Comment, however, the OSG filed a Motion for Extension, requesting that it be given until January 16, 2010 to Comment. Somewhat surprisingly, the OSG later filed a Comment in support of petitioners application. Thus, in order to give COMELEC the opportunity to fully ventilate its position, we required it to file its own comment. The COMELEC, through its Law Department, filed its Comment on February 2, 2010. In the meantime, due to the urgency of the petition, a temporary restraining order was issued on January 12, 2010, effective immediately and continuing until further orders from this Court, directing the COMELEC to cease and desist from implementing the Assailed Resolutions. Also, on January 13, 2010, the Commission on Human Rights (CHR) filed a Motion to Intervene or to Appear as Amicus Curiae, attaching thereto its Comment-inIntervention. The CHR opined that the denial of Ang Ladlads petition on moral grounds violated the standards and principles of the Constitution, the Universal Declaration of Human Rights (UDHR), and the International Covenant on Civil and Political Rights (ICCPR). On January 19, 2010, we granted the CHRs motion to intervene. HELD: We grant the petition. Compliance with the Requirements of the Constitution and Republic Act No. 7941 The COMELEC denied Ang Ladlads application for registration on the ground that the LGBT sector is neither enumerated in the Constitution and RA 7941, nor is it associated with or related to any of the sectors in the enumeration.
232

233

Respondent mistakenly opines that our ruling in Ang Bagong Bayani stands for the proposition that only those sectors specifically enumerated in the law or related to said sectors (labor, peasant, fisherfolk, urban poor, indigenous cultural communities, elderly, handicapped, women, youth, veterans, overseas workers, and professionals) may be registered under the party-list system. As we explicitly ruled in Ang Bagong Bayani-OFW Labor Party v. Commission on Elections, the enumeration of marginalized and under-represented sectors is not exclusive. The crucial element is not whether a sector is specifically enumerated, but whether a particular organization complies with the requirements of the Constitution and RA 7941.

A cursory perusal of Ang Ladlads initial petition shows that it never claimed to exist in each province of the Philippines. Rather, petitioner alleged that the LGBT community in the Philippines was estimated to constitute at least 670,000 persons; that it had 16,100 affiliates and members around the country, and 4,044 members in its electronic discussion group. Ang Ladlad also represented itself to be a national LGBT umbrella organization with affiliates around the Philippines composed of the following LGBT networks: Abra Gay Association Aklan Butterfly Brigade (ABB) Aklan Albay Gay Association Arts Center of Cabanatuan City Nueva Ecija Boys Legion Metro Manila Cagayan de Oro People Like Us (CDO PLUS) Cant Live in the Closet, Inc. (CLIC) Metro Manila Cebu Pride Cebu City Circle of Friends Dipolog Gay Association Zamboanga del Norte Gay, Bisexual, & Transgender Youth Association (GABAY) Gay and Lesbian Activists Network for Gender Equality (GALANG) Metro Manila Gay Mens Support Group (GMSG) Metro Manila Gay United for Peace and Solidarity (GUPS) Lanao del Norte Iloilo City Gay Association Iloilo City Kabulig Writers Group Camarines Sur Lesbian Advocates Philippines, Inc. (LEAP) LUMINA Baguio City Marikina Gay Association Metro Manila Metropolitan Community Church (MCC) Metro Manila Naga City Gay Association Naga City ONE BACARDI Order of St. Aelred (OSAe) Metro Manila PUP LAKAN RADAR PRIDEWEAR Rainbow Rights Project (R-Rights), Inc. Metro Manila San Jose del Monte Gay Association Bulacan Sining Kayumanggi Royal Family Rizal Society of Transexual Women of the Philippines (STRAP) Metro Manila Soul Jive Antipolo, Rizal The Link Davao City Tayabas Gay Association Quezon Womens Bisexual Network Metro Manila Zamboanga Gay Association Zamboanga City Against this backdrop, we find that Ang Ladlad has sufficiently demonstrated its compliance with the legal requirements for accreditation. Indeed, aside from
233

234

COMELECs moral objection and the belated allegation of non-existence, nowhere in the records has the respondent ever found/ruled that Ang Ladlad is not qualified to register as a party-list organization under any of the requisites under RA 7941 or the guidelines in Ang Bagong Bayani. The difference, COMELEC claims, lies in Ang Ladlads morality, or lack thereof. Our Constitution provides in Article III, Section 5 that [n]o law shall be made respecting an establishment of religion, or prohibiting the free exercise thereof. At bottom, what our non-establishment clause calls for is government neutrality in religious matters. Clearly, governmental reliance on religious justification is inconsistent with this policy of neutrality. We thus find that it was grave violation of the non-establishment clause for the COMELEC to utilize the Bible and the Koran to justify the exclusion of Ang Ladlad. Rather than relying on religious belief, the legitimacy of the Assailed Resolutions should depend, instead, on whether the COMELEC is able to advance some justification for its rulings beyond mere conformity to religious doctrine. Otherwise stated, government must act for secular purposes and in ways that have primarily secular effects. As we held in Estrada v. Escritor: x x x The morality referred to in the law is public and necessarily secular, not religious as the dissent of Mr. Justice Carpio holds. "Religious teachings as expressed in public debate may influence the civil public order but public moral disputes may be resolved only on grounds articulable in secular terms." Otherwise, if government relies upon religious beliefs in formulating public policies and morals, the resulting policies and morals would require conformity to what some might regard as religious programs or agenda. The non-believers would therefore be compelled to conform to a standard of conduct buttressed by a religious belief, i.e., to a "compelled religion," anathema to religious freedom. Likewise, if government based its actions upon religious beliefs, it would tacitly approve or endorse that belief and thereby also tacitly disapprove contrary religious or non-religious views that would not support the policy. As a result, government will not provide full religious freedom for all its citizens, or even make it appear that those whose beliefs are disapproved are second-class citizens. We are not blind to the fact that, through the years, homosexual conduct, and perhaps homosexuals themselves, have borne the brunt of societal disapproval. It is not difficult to imagine the reasons behind this censure religious beliefs, convictions about the preservation of marriage, family, and procreation, even dislike or distrust of homosexuals themselves and their perceived lifestyle. Nonetheless, we recall that the Philippines has not seen fit to criminalize homosexual conduct. Evidently, therefore, these generally accepted public morals have not been convincingly transplanted into the realm of law. The Assailed Resolutions have not identified any specific overt immoral act performed by Ang Ladlad. Even the OSG agrees that there should have been a finding by the COMELEC that the groups members have committed or are committing immoral acts. The OSG argues: x x x A person may be sexually attracted to a person of the same gender, of a different gender, or more than one gender, but mere attraction does not translate to immoral acts. There is a great divide between thought and action. Reduction ad absurdum. If immoral thoughts could be penalized, COMELEC would have its hands full of disqualification cases against both the straights and the gays. Certainly this is not the intendment of the law. Respondent has failed to explain what societal ills are sought to be prevented, or why special protection is required for the youth. Neither has the COMELEC
234

235

condescended to justify its position that petitioners admission into the party-list system would be so harmful as to irreparably damage the moral fabric of society. We, of course, do not suggest that the state is wholly without authority to regulate matters concerning morality, sexuality, and sexual relations, and we recognize that the government will and should continue to restrict behavior considered detrimental to society. Nonetheless, we cannot countenance advocates who, undoubtedly with the loftiest of intentions, situate morality on one end of an argument or another, without bothering to go through the rigors of legal reasoning and explanation. In this, the notion of morality is robbed of all value. Clearly then, the bare invocation of morality will not remove an issue from our scrutiny. We also find the COMELECs reference to purported violations of our penal and civil laws flimsy, at best; disingenuous, at worst. Article 694 of the Civil Code defines a nuisance as any act, omission, establishment, condition of property, or anything else which shocks, defies, or disregards decency or morality, the remedies for which are a prosecution under the Revised Penal Code or any local ordinance, a civil action, or abatement without judicial proceedings. A violation of Article 201 of the Revised Penal Code, on the other hand, requires proof beyond reasonable doubt to support a criminal conviction. It hardly needs to be emphasized that mere allegation of violation of laws is not proof, and a mere blanket invocation of public morals cannot replace the institution of civil or criminal proceedings and a judicial determination of liability or culpability. As such, we hold that moral disapproval, without more, is not a sufficient governmental interest to justify exclusion of homosexuals from participation in the partylist system. The denial of Ang Ladlads registration on purely moral grounds amounts more to a statement of dislike and disapproval of homosexuals, rather than a tool to further any substantial public interest. Respondents blanket justifications give rise to the inevitable conclusion that the COMELEC targets homosexuals themselves as a class, not because of any particular morally reprehensible act. It is this selective targeting that implicates our equal protection clause. Equal Protection Despite the absolutism of Article III, Section 1 of our Constitution, which provides nor shall any person be denied equal protection of the laws, courts have never interpreted the provision as an absolute prohibition on classification. Equality, said Aristotle, consists in the same treatment of similar persons. The equal protection clause guarantees that no person or class of persons shall be deprived of the same protection of laws which is enjoyed by other persons or other classes in the same place and in like circumstances. Recent jurisprudence has affirmed that if a law neither burdens a fundamental right nor targets a suspect class, we will uphold the classification as long as it bears a rational relationship to some legitimate government end. In Central Bank Employees Association, Inc. v. Banko Sentral ng Pilipinas, we declared that [i]n our jurisdiction, the standard of analysis of equal protection challenges x x x have followed the rational basis test, coupled with a deferential attitude to legislative classifications and a reluctance to invalidate a law unless there is a showing of a clear and unequivocal breach of the Constitution.

235

236

Freedom of Expression and Association Freedom of expression constitutes one of the essential foundations of a democratic society, and this freedom applies not only to those that are favorably received but also to those that offend, shock, or disturb. Any restriction imposed in this sphere must be proportionate to the legitimate aim pursued. Absent any compelling state interest, it is not for the COMELEC or this Court to impose its views on the populace. Otherwise stated, the COMELEC is certainly not free to interfere with speech for no better reason than promoting an approved message or discouraging a disfavored one. This position gains even more force if one considers that homosexual conduct is not illegal in this country. It follows that both expressions concerning ones homosexuality and the activity of forming a political association that supports LGBT individuals are protected as well.

In the area of freedom of expression, for instance, United States courts have ruled that existing free speech doctrines protect gay and lesbian rights to expressive conduct. In order to justify the prohibition of a particular expression of opinion, public institutions must show that their actions were caused by something more than a mere desire to avoid the discomfort and unpleasantness that always accompany an unpopular viewpoint.

236

237

Of course, none of this suggests the impending arrival of a golden age for gay rights litigants. It well may be that this Decision will only serve to highlight the discrepancy between the rigid constitutional analysis of this Court and the more complex moral sentiments of Filipinos. We do not suggest that public opinion, even at its most liberal, reflect a clear-cut strong consensus favorable to gay rights claims and we neither attempt nor expect to affect individual perceptions of homosexuality through this Decision. WHEREFORE, the Petition is hereby GRANTED.

ROEL EBRALINAG, ET AL VS. THE DIVISION SUPERINTENDENT OF SCHOOLS OF CEBU, March 1, 1993

Grino--Aquino, J. Facts: 1. The petitioners are high school and grade schools students enrolled in the different public schools of the Province of Cebu and who belong to the religious group known as the Jehovah's Witnesses; 2. That they rrefused to take part in the flag ceremony which includes playing by a band or singing the Philippine National Anthem, saluting the Philippine Flag and reciting the patriotic pledge because they considered the flag as an image and they should not worship it except GOD; 3. That because of their refusal to perform the foregoing acts as required by RA 1265 of July 11, 1955 and by Department Order No. 8 dated July 21, 1955 of the DECS making the flag ceremony compulsory in all educational institutions, they were expelled by the respondent school authorities. Hence this petition. Issue: -----237

238

May the petitioners be expelled for refusing to salute the flag, recite the patriotic pledge or sing the national anthem in order to follow their religious beliefs? Held: The same issue was raised in Gerona vs. Secretary of Education, 106 Phil. 2 (1959) and Balbuna vs. Secretary of Education, 110 Phil. 150 (1960) where the SC held that: The flag is not an image but a symbol of the Republic of the Philippines, an emblem of national sovereignty, of national unity and cohesion and of freedom and liberty which it and the Constitution guarantee and protect. Under a system of complete separation of church and state in the government, the flag is utterly devoid of any religious significance. The law, RA 1265 was likewise incorporated in Executive Order No. 297, September 21, 1988. Our task is extremely difficult for the 30-year old decision of this Court in GERONA upholding the salute law and approving the expulsion of students who refuse to obey it, is not lightly to be trifled with. The idea that one may be compelled to salute the flag, sing the national anthem, and recite the patriotic pledge, during flag ceremony on pain of being dismissed from one's job or be expelled in school, IS ALIEN TO THE CONSCIENCE OF THE PRESENT GENERATION OF FILIPINOS WHO CUT THEIR TEETH ON THE BILL OF RIGHTS WHICH GUARANTEES THEIR RIGHTS TO FREE SPEECH AND THE FREE EXERCISE OF RELIGIOUS PROFESSION AND WORSHIP (Section 5, Art. III, 1987 Constitution). Religious freedom is a fundamental right which is entitled to the highest priority and the amplest protection among human rights, for it involves the relationship of man and his Creator (Chief Justice Fernando's separate opinion in German vs. Barangan, 135 SCRA 530). The right to religious profession has a two-fold aspect, vis., freedom to believe and freedom to act on one's belief. The first is absolute as long as the belief is confined within the realm of the thought. The second is subject to regulation where the belief is translated into external acts that affect the public welfare. The sole justification for a prior restraint or limitation on the exercise of religious freedom (according the Former Chief justice Teehankee in his dissenting opinion in German vs. Baranagan) is the existence of a grave and present danger of a character both grave and imminent, of a serious evil to public safety, public morals, public health or any other legitimate public interest, that the State has the right and duty to presvent. Absent such a threat to public safety, the expulsion of the petitioners from the schools is not justified since they are not doing anything that could warrant their expulsion since during flag ceremonies, they just quietly stand at attention to show their respect for the rights of others who choose to participate in the solemn proceedings.

238

239

In Victoriano vs. Elizalde Rope Workers Union, 59 SCRA 54, we upheld the exemption of the members of the Iglesia ni Kristo from the coverage of the closed-shop agreement between the labor union and the company because it would violate the teaching of their church not to join any labor group. We hold that a similar exemption may be accorded to the Jehovah's Witnesses with regard to the observance of the flag ceremony out of respect to their religious beliefs, however "bizarre" those beliefs may seem to others

CHAPTER VII - THE CONSTITUTIONAL RIGHT TO TRAVEL Section 6. The liberty of abode and of changing the same within the limits prescribed by law shall not be impaired except upon lawful order of the court. Neither shall the right to travel be impaired except in the interest of national security, public safety, or public health, as may be provided by law. NOTE: THE APPLICABLE PROVISION OF THE HUMAN SECURITY ACT ON THE RIGHT TO TRAVEL Section 26 provides that persons who have been charged with terrorism or conspiracy to commit terrorism---even if they have been granted bail because evidence of guilt is not strongcan be: Detained under house arrest; Restricted from traveling; and/or Prohibited from using any cellular phones, computers, or other means of communications with people outside their residence. Upon application of the prosecutor, the suspects right to travel shall be limited to the municipality or city where he resides or where the case is pending, in the interest of national security and public safety. Travel outside of said municipality or city, without the authorization of the court, shall be deemed a violation of the terms and conditions of the bail which shall then be forfeited as provided in the Rules of Court. These restrictions shall be terminated upon acquittal of the accused; or the dismissal of the case filed against him; or earlier upon the discretion of the court or upon motion of the prosecutor. 1. The constitutional as well as human right to travel, 2. Read: FERDINAND MARCOS, ET AL. VS. HON. RAUL MANGLAPUS, ET AL., G.R. NO. 88211, September 15, 1989 and the Resolution of the Motion for Reconsideration dated October 27, 1989 right to travel; liberty of abode
239

129 SCRA

240

and "right to return" En banc Cortes, J. This is a petition for mandamus and prohibition asking the Supreme Court to Order the respondents to issue travel documents to the petitioners and to enjoin the implementation of the President's decision to bar their return to the Philippines. The case for the petitioners is founded on the assertion that their right to return to the Philippines is guaranteed by the following provisions of the Constitution: Section 1. No person shall be deprived of life liberty or property without due process of law, nor shall any person be denied equal protection of the laws. Section 6. The liberty of abode and of changing the same within the limits prescribed by law shall not be impaired except in the interest of national security, public safety or public health, as may be provided by law. The petitioners contend that the President has no power to impair the liberty of abode of the Marcoses because only the Courts may do so "within the limits prescribed by law". Nor may the President impair the right to travel because no law has authorized her to do so. Also, the petitioners claim that under international law, particularly the Universal Declaration of Humjan Rights guaranteed the right of the Marcoses to return to the Philippines. Thus: Art. 13 (1) Everyone has the right to freedom of movement and residence within the borders of each state. (2) Everyone has the right to leave any country, including his own, AND TO RETURN TO HIS COUNTRY. Likewise, under the International Covenant on Civil and Political Rights, which had been ratified by the Philippines, provides: Art. 12 4) No one shall be arbitrarily deprived of the right to enter his own country. The respondents argue that the issue in this case involves a political question which is therefore beyond the jurisdiction of the Court. Furthermore, they argue that the right of the state to national security prevails over individual rights, citing Section 4, Art. II of the 1987 Philippine Constitution. Issue: Whether or not, in the exercise of the powers granted in the Constitution, the President may prohibit the Marcoses from returning to the Philippines.
240

241

The sub-issues, which could help in the determination of the main issue, are: 1. Does the President have the power to bar the Marcoses to return to the Philippines? a. Is this a political question? 2. Assuming that the President has the power to bar former Pres. Marcos and his family from returning to the Philippines, in the interest of national security, public safety or public health, has the President made a finding that the return of the petitioners to the Philippines is a clear and present danger to national security, public welfare or public health. And if she has made that finding, have the requirements of due process been complied with in making such finding? Has there been prior notice to the petitioners? Held: It must be emphasized that the individual right involved in this case is not the right to travel from the Philippines to other countries or within the Philippines. These are what the right to travel connote. Essentially, the right to return to one's country, a totally distinct right under international law, independent from, though related to the right to travel. Thus, even the Universal declaration of Human Rights and the International Covenant on Civil and Political Rights treat the right to freedom of movement and abode within the territory of the state, the right to leave a country and the right to enter one's country as separate and distinct rights. THE RIGHT TO RETURN TO ONE'S COUNTRY IS NOT AMONG THE RIGHTS SPECIFICALLY GUARANTEED BY THE BILL OF RIGHTS, WHICH TREATS ONLY OF THE LIBERTY OF ABODE AND THE RIGHT TO TRAVEL, BUT IT IS OUR WELL-CONSIDERED VIEW THAT THE RIGHT TO RETURN MAY BE CONSIDERED AS A GENERALLY ACCEPTED PRINCIPLE OF INTERNATIONAL LAW, UNDER OUR CONSTITUTION, IS PART OF THE LAW OF THE LAND. To the President, the problem is one of balancing the general welfare and the common good against the exercise of rights of certain individuals. The power involved is the President's RESIDUAL POWER to protect the general welfare of the people. The court cannot close its eyes to present realities and pretend that the country is not besieged by the insurgency, separatist movement in Mindanao, rightist conspiracies to grab power, etc. With these before her, the President cannot be said to have acted arbitrarily, capriciously and whimsically. Lastly, the issue involved in the case at bar is not political in nature since under Section 1, Art. VIII of the Constitution, judicial power now includes the duty to "determine whether or not there has been a grave abuse of discretion amounting to lack of jurisdiction on the part of any branch or instrumentality of the government." NOTE:
241

242

The main opinion was concurred in by 7 justices (CJ Fernan, Narvasa, Melencio-Herrera, Gancayco, Grino-Aquino, Medialdea and Regalado) or a total of 8 justices in voting in favor of DISMISSING the petition. Seven justices filed separate dissenting opinions (Gutierrez, Jr., Cruz, Paras, Feliciano, Padilla, Bidin and Sarmiento). *********************** Gutierrez, Jr., J., dissenting. With all due respect for the majority in the Court that the main issue in this case is not one of power but one on RIGHTS. If he comes home, the government has the power to arrest and punish him but does it have the power to deny him his right to come home and die among familiar surroundings? x x x The government has more than ample powers under existing laws to deal with a person who transgresses the peace and imperils public safety. BUT THE DENIAL OF TRAVEL PAPERS IS NOT ONE OF THOSE POWERS BECAUSE THE BILL OF RIGHTS SAY SO. THERE IS NO LAW PRESCRIBING EXILE IN FOREIGN LAND AS THE PENALTY FOR HURTING THE NATION. . The fears expressed by its representatives were based on mere conjectures of political and economic destabilization without any single piece of concrete evidence to back up their apprehensions. Amazingly, however, the majority has come to the conclusion that there exist "factual bases for the President's decision" to bar Marcos's return. That is not my recollection of the impressions of the Court after the hearing. 2. Silverio vs. CA, April 8, 1991 Read also: 1. Caunca vs. Salazar, 82 Phil. 851 2. Kwong vs. PCGG, December 7,l987 Manotoc vs. CA, 142 SCRA 149 1. Petitioner Ricardo Manotoc, Jr. has 6 criminal cases for estafa pending against him. In said cases he was admitted to bail with the FGU Insurance Corporation as surety. He is also involved in a case pending before the Securities and Exchange Commission. 2. The SEC requested the Commissioner on Immigration not to clear petitioner for departure pending disposition of the case involving him. The same was granted by the Commissioner. 3. Petitioner subsequently filed before the trial courts a motion entitled "motion for permission to leave the country" stating as ground therefor his desire to go to the United States, "relative to his business transactions and opportunities". 4. The motion was denied by the lower courts and the matter was elevated to the Court of Appeals which also denied the same. Petitioner brings the matter to the S.C. claiming his constitutional right to travel and
242

243

also contending that having been admitted to bail as a matter of right, neither the courts which granted him bail nor the SEC would have jurisdiction over his liberty. HELD: Petition denied. a. A court has the power to prohibit a person admitted to bail from leaving the Philippines. This is a necessary consequence of the nature and function of a bail bond. The condition imposed upon petitioner to make himself available at all times whenever the court requires his presence operates as a valid restriction on his right to travel. b. "x x x the result of the obligation assumed by appellee to hold the accused amenable at all times to the orders and processes of the lower court, was to prohibit the accused from leaving the jurisdiction of the Philippines, because, otherwise, said orders and processes will be nugatory, and inasmuch as the jurisdiction of the courts from which they issued does not extend beyond that of the Philippines they would have no binding force outside of said jurisdiction."(People vs. Uy Tuising, 61 Phil. 404 (l935) c. To allow the petitioner to leave the Philippines without sufficient reason would place him beyond the reach of the courts. d. Petitioner cites the Court of Appeals case of People vs. Shepherd (C.A.-G.R. No. 23505-R, Feb. 13, 1980) as authority for his claim that he could travel. The S.C. held however that said case is not squarely on all fours with the case at bar. Unlike the Shepherd case, petitioner has failed to satisfy the courts of the urgency of his travel, the duration thereof, as well as the consent of his surety to the proposed travel. e. It may thus be inferred that the fact that a criminal case is pending against an accused does not automatically bar him from travelling abroad. He must however convince the courts of the urgency of his travel, the duration thereof, and that his sureties are willing to undertake the responsibility of allowing him to travel. 4. Villavicencio vs. Lukban, 39 Phil. 778 5. Roan vs. Gonzales, supra. 6. Salonga vs. Hermoso, 97 SCRA 121 7. Read also the Ferdinand Marcos Cases of August 1989 CHAPTER VIII - THE CONSTITUTIONAL RIGHT TO INFORMATION Section 7. The right of the people to information on matters of public concern shall be recognized. Access to official recordsshall be afforded the citizen subject to such limitations as may be provided by law. 1. Read:
243

& October,

244

Right to Privacy; right to information on matters of public concern; CAMILO L. SABIO vs. GORDON, G.R. No. 174340, October 17, 2006, 504 SCRA 704 Sandoval-Gutierrez, J. The Facts:

On February 20, 2006, Senator Miriam Defensor Santiago introduced Philippine Senate Resolution No. 455 (Senate Res. No. 455),180[4] directing an inquiry in aid of legislation on the anomalous losses incurred by the Philippines Overseas Telecommunications Corporation (POTC), Philippine Communications Satellite Corporation (PHILCOMSAT), and PHILCOMSAT Holdings Corporation (PHC) due to the alleged improprieties in their operations by their respective Board of Directors. The pertinent portions of the Resolution read:

WHEREAS, in the last quarter of 2005, the representation and entertainment expense of the PHC skyrocketed to P4.3 million, as compared to the previous years mere P106 thousand; WHEREAS, some board members established wholly owned PHC subsidiary called Telecommunications Center, Inc. (TCI), where PHC funds are allegedly siphoned; in 18 months, over P73 million had been allegedly advanced to TCI without any accountability report given to PHC and PHILCOMSAT; WHEREAS, the Philippine Star, in its 12 February 2002 issue reported that the executive committee of Philcomsat has precipitately released P265 million and granted P125 million loan to a relative of an executive committee member; to date there have been no payments given, subjecting the company to an estimated interest income loss of P11.25 million in 2004; WHEREFORE, be it resolved that the proper Senate Committee shall conduct an inquiry in aid of legislation, on the anomalous losses incurred by the Philippine Overseas Telecommunications Corporation (POTC), Philippine Communications Satellite Corporation (PHILCOMSAT), and Philcomsat Holdings Corporations (PHC) due to the alleged improprieties in the operations by their respective board of directors. On May 8, 2006, Chief of Staff Rio C. Inocencio, under the authority of Senator Richard J. Gordon, wrote Chairman Camilo L. Sabio of the PCGG, one of the herein petitioners, inviting him to be one of the resource persons in the public meeting jointly conducted by the Committee on Government Corporations and Public Enterprises and Committee on

180[4]

Annex E of the Petition in G.R. No. 174318.

244

245

Public Services. The purpose of the public meeting was to deliberate on Senate Res. No. 455.181[6]

On May 9, 2006, Chairman Sabio declined the invitation because of prior commitment.182[7] At the same time, he invoked Section 4(b) of E.O. No. 1 earlier quoted. On September 12, 2006, at around 10:45 a.m., Major General Balajadia arrested Chairman Sabio in his office at IRC Building, No. 82 EDSA, Mandaluyong City and brought him to the Senate premises where he was detained. Hence, Chairman Sabio filed with the Supreme Court a petition for habeas corpus against the Senate Committee on Government Corporations and Public Enterprises and Committee on Public Services, their Chairmen, Senators Richard Gordon and Joker P. Arroyo and Members. The case was docketed as G.R. No. 174340.

I S S U E S:

Is the refusal of the petitioners to testify in Congress by virtue of EO No. 1, Section 4 [b] violates the constitutional provision on information on matters of public concern?

H E L D:

Yes.

Section 4(b) of E.O. No.1 which was invoked by the petitioners in support of their refusal to testify in the Senate limits the power of legislative inquiry by exempting all PCGG members or staff from testifying in any judicial, legislative or administrative proceeding, thus:

No member or staff of the Commission shall be required to testify or produce evidence in any judicial, legislative or administrative proceeding concerning matters within its official cognizance.

Such provision of EO No. 1 is unconstitutional because it violates the constitutional provision ensuring the peoples access to information on matters of public

BANTAY REPUBLIC ACT SCRA 1

VS. COMELEC, MAY 4, 2007, 523

The petitioner requested the COMELEC to publish the individual nominees of all the party-list groups in order that they will
181[6] 182[7]

Annex F of the Petition in G.R. No. 174318. Annex G of the Petition in G.R. No. 174318.

245

246

be guided on what party-list group shall be supported by them. The COMELEC held that under the Party-list Act, such list of nominees is confidential and should not be published. Held: The COMELEC should publish the list of nominees of all the party-list groups. This is in accordance with the right to information on matters of public concern which shall be accorded to every citizen.

VALMONTE VS. BELMONTE, GR NO. 74930, FEBRUARY 13, 1989 in relation to the Right to Privacy Cortes, J. Facts: 1. On June 4, 1986, petitioner Valmonte wrote the respondent asking the latter to furnish him copies of former members of the Batasang Pambansa who were able to secure a "clean loan" from the GSIS prior to the February 7, 1986 elections; 2. On June 17, 1986, respondent through counsel refused to give petitioner a list of said lawmakers who obtained "clean loans" from GSIS on the ground that there is a confidential relationship between GSIS and its borrowers and it would be proper for them to preserve same; 3. On July 19, 1986, the petitioners filed this instant petition. Issues: 1. Whether or not the case should be dismissed for failure to exhaust administrative remedies? 2. Whether or not the petitioners are entitled to the documents sought in accordance with their constitutional right to information? Held: 1. It is well-settled in our jurisdiction that before a party can be allowed to resort to the courts, he is expected to have exhausted all means of administrative redress available under the law. In the case at bar, the decision of the General Manager of the GSIS is appealable/reviewable by the GSIS Board of Trustees. Petitioners did not ask the Board of Trustees to review the decision of the respondent. However, the rule on exhaustion of administrative remedies is not applicable when only questions of law is involved. (Pascual vs. Provincial Board, 106 Phil. 466; Aguilar vs. Valencia, 40 SCRA 210; Malabanan vs. Ramento, 129 SCRA 359. This is not the first time that the court is confronted with a case involving the right to information. In Tanada vs. Tuvera, 136 SCRA 27, we upheld the citizen's right to information as well as in Legaspi vs. CSC,
246

the the the the

247

150 SCRA 530 and ordered the government officers involved to act as prayed for by the petitioners. The pertinent provision of the Constitution is Section 7, Art. III which provides: The right of the people to information on matters of public concern shall be recognized. Access to official records, and to documents, and papers pertaining to official acts, transactions x x x shall be afforded the citizen, subject to such limitations as may be provided for by law. The postulate of public office is a public trust as institutionalized in the Constitution (Sec. 1, Art. XI) to protect the people from abuse of governmental power, would certainly be empty words if access to information of public concern is denied except under limitations prescribed by law. Petitioners are members of the media. As such, they have both the right to gather and the obligation to check the accuracy of the information they disseminate x x x The right to information is an essential premise of a meaningful right to speech and expression. But this is not to say that the right to information is merely an adjunct of and therefore restricted in application by the exercise of the freedom of speech and of the press. Far from it. The right to information goes hand in hand with the constitutional policies of "full public disclosure" and "honesty in the public service". Yet, like all the constitutional guarantees, the right to information is not absolute. It is subject to limitations provided for by law and the people's right to information is limited to "matters of public concern". Similarly, the State's policy of full disclosure is limited to "transactions involving public interest" and subject to "reasonable conditions prescribed by law." The information sought to be obtained by the petitioners affect public interest since the GSIS is the trustee of contributions from the government and its employees. The funds of the GSIS assume a public character and that its obligations are guaranteed by the government. The petitioners are entitled to access to documents sought subject to reasonable regulations that the respondent may impose relating to manner and hours of examination, to the end that damage or loss of the records may be avoided, that undue interference with the duties of the custodian of the records may be prevented and that the right of other persons entitled to inspect the records may be insured [Legaspi vs. CSC, supra; Subido vs. Ozaeta, 80 Phil. 383] he petitioners, however, are not entitled to be furnished copies of list of alleged members of the Batasang Pambansa who were able to secure clean loans through the intercessions of Pres. Marcos and the First Lady. This is so because access to public records does not include the right to compel custodians of official records to prepare lists, abstracts, summaries and the like in their desire to acquire information on matters of public concern. The respondent is therefore ordered to allow petitioners access to documents and records evidencing loans granted to members of the Batasang Pambansa, as petitioners may specify, subject to reasonable rules and regulations as the GSIS may deem necessary.
247

248

CARPIO MORALES, J.: The Facts: In the exercise of its legislative power, the Senate of the Philippines, through its various Senate Committees, conducts inquiries or investigations in aid of legislation which call for, inter alia, the attendance of officials and employees of the executive department, bureaus, and offices including those employed in Government Owned and Controlled Corporations, the Armed Forces of the Philippines (AFP), and the Philippine National Police (PNP). On September 21 to 23, 2005, the Committee of the Senate as a whole issued invitations to various officials of the Executive Department for them to appear on September 29, 2005 as resource speakers in a public hearing on the railway project of the North Luzon Railways Corporation with the China National Machinery and Equipment Group (hereinafter North Rail Project). The public hearing was sparked by a privilege speech of Senator Juan Ponce Enrile urging the Senate to investigate the alleged overpricing and other unlawful provisions of the contract covering the North Rail Project. On September 28, 2005, the President of the Philippines issued E.O. 464, ENSURING OBSERVANCE OF THE PRINCIPLE OF SEPARATION OF POWERS, ADHERENCE TO THE RULE ON EXECUTIVE PRIVILEGE AND RESPECT FOR THE RIGHTS OF PUBLIC OFFICIALS APPEARING IN LEGISLATIVE INQUIRIES IN AID OF LEGISLATION UNDER THE CONSTITUTION, AND FOR OTHER PURPOSES, which, pursuant to Section 6 thereof, took effect immediately. The salient provisions of the Order are as follows: SECTION 1. Appearance by Heads of Departments Before Congress. In accordance with Article VI, Section 22 of the Constitution and to implement the Constitutional provisions on the separation of powers between co-equal branches of the government, all heads of departments of the Executive Branch of the government shall secure the consent of the President prior to appearing before either House of Congress. When the security of the State or the public interest so requires and the President so states in writing, the appearance shall only be conducted in executive session. SECTION. 2. Nature, Scope and Coverage of Executive Privilege. (a) Nature and Scope. - The rule of confidentiality based on executive privilege is fundamental to the operation of government and rooted in the separation of powers under the Constitution (Almonte vs. Vasquez, G.R. No. 95367, 23 May 1995). Further, Republic Act No. 6713 or the Code of Conduct and Ethical Standards for Public Officials and Employees provides that Public Officials and Employees shall not use or divulge confidential or classified information officially known to them by reason
248

249

of their office and not made available to the public to prejudice the public interest. (b) Who are covered. The following are covered by this executive order: 1. Senior officials of executive departments who in the judgment of the department heads are covered by the executive privilege; 2. Generals and flag officers of the Armed Forces of the Philippines and such other officers who in the judgment of the Chief of Staff are covered by the executive privilege; 3. Philippine National Police (PNP) officers with rank of chief superintendent or higher and such other officers who in the judgment of the Chief of the PNP are covered by the executive privilege; 4. Senior national security officials who in the judgment of the National Security Adviser are covered by the executive privilege; and 5. Such other officers as may be determined by the President.

I S S U E S: 1. Whether E.O. 464 violates the right of the people to information on matters of public concern; and

H E L D: E.O 464 likewise violates the constitutional provision on the right to information on matters of public concern. There are clear distinctions between the right of Congress to information which underlies the power of inquiry and the right of the people to information on matters of public concern. For one, the demand of a citizen for the production of documents pursuant to his right to information does not have the same obligatory force as a subpoena duces tecum issued by Congress. Neither does the right to information grant a citizen the power to exact testimony from government officials. These powers belong only to Congress and not to an individual citizen. To the extent that investigations in aid of legislation are generally conducted in public, however, any executive issuance tending to unduly limit disclosures of information in such investigations necessarily deprives the people of information which, being presumed to be in aid of legislation, is presumed to be a matter of public concern. The citizens are thereby denied access to information which they can use in formulating their own opinions on the matter before Congress opinions which they can then communicate to their representatives and other government officials through the various legal means allowed by their freedom of expression. Thus holds Valmonte v. Belmonte: It is in the interest of the State that the channels for free political discussion be maintained to the end that the government may perceive and be responsive to the peoples will. Yet, this open dialogue can be effective only to the extent that the citizenry is informed and thus able to formulate its will intelligently. Only when the participants in the discussion are aware of the issues and have access to information relating thereto can such bear fruit.183 (Emphasis and underscoring supplied)
183

G.R. No. 74930, February 13, 1989, 170 SCRA 256.

249

250

The impairment of the right of the people to information as a consequence of E.O. 464 is, therefore, in the sense explained above, just as direct as its violation of the legislatures power of inquiry. 1-a. Legaspi vs. CSC, 150 SCRA 530 1-b. Brilliantes vs. Chang, Aug. 14, 1990 1-c. Canlas vs. Vazquez, July 3, 1990 1-d. Aquino-Sarmiento vs. Manuel Morato, November 13, 1991 2. Tanada vs. Tuvera, 146 SCRA 44 3. Baldoza vs. Dimaano, 71 SCRA 14 4. Lantaco vs. Lllamas, 108 SCRA 502 5. Subido vs. Ozaeta, 80 Phil. 383 CHAPTER IX - THE CONSTITUTIONAL RIGHT TO FORM AND JOIN ASSOCIATIONS Section 8. The right of the people, including those employed in the public and private sectors, to form unions, associations, societies for purposes not contrary to law shall not be abridged. 1. Freedom of Association, 100 SCRA 100 2. The fundamental right of self-organization,108 SCRA 3. The right of self-organization of managerial 434 4. Read: 1. In re: ATTY. EDILLON, 84 SCRA 554 2. Tarnate vs. Noriel, 100 SCRA 93 3. Samahan ng Manggagawa vs. Noriel, 108 SCRA 381 4. Villar vs. Inciong, April 20,l983 5. P. vs. Ferrer, 48 SCRA 382 6. P. vs. Ferrer, 56 SCRA 793 (Read the dissenting Justice FERNANDO in both cases) 390

employees,47 SCRA

opinion of

CHAPTER X - THE POWER OF EMINENT DOMAIN Section 9. Private property shall not be taken for public use without just compensation 1. The inherent power of eminent domain,93 SCRA 663 2. Who may exercise it? How about a barangay? Yes with the Presidents approval. Read: 1. Barangay Matictic vs. Elbinias, 148 SCRA 83
250

251

2. Procedure for the exercise of said power; Extent of payment to be made before writ of possession shall be issued in favor of the government. Value of property expropriated for national government projects; Writ of possession when it shall be issued by the court; when Rule 67 of the Rules of Court and when RA 8974 shall apply; full payment of just compensation before government takes over. REPUBLIC OF THE PHILIPPINES VS. JUDGE GINGOYON, 478 SCRA 474 Tinga, J. Facts: In 2003, the Supreme Court held in AGAN VS. PIATCO, 402 SCRA 612 that the CONCESSION AGREEMENT FOR THE BUILD OPERATE TRANSFER ARRANGEMENT OF THE NINOY AQUINO INTERNATIONAL AIRPORT PASSENGER TERMINAL II between the Philippine Government and the Philippine International Air Terminals Co., Inc. (PIATCO) as well as the amendments thereto is void for being contrary to law and public policy. On Motion for Reconsideration (420 SCRA 420), the Supreme Court held that: This Court, however, is not unmindful of the reality that the structures comprising the NAIA IPT III facility are almost complete and that funds have been spent by PIATCO in their construction. For the government to take over the said facility, IT HAS TO COMPENSATE RESPONDENT PIATCO AS BUILDER OF THE SAID STRUCTURES. THE COMPENSATION MUST BE JUST AND IN ACCORDANCE WITH LAW AND EQUITY FOR THE GOVERNMENT CAN NOT UNJUSTLY ENRICH ITSELF AT THE EXPENSE OF PIATCO AND ITS INVESTORS. On December 21, 2004, the Government filed a complaint for expropriation with the RTC of Pasay City seeking a writ of possession authorizing to take immediate possession and control over NAIA 3 facilities and deposited the amount of P3.0B in cash with Land Bank of the Philippines representing the assessed value of the terminals assessed value for taxation purposes. On the same day, Judge Gingoyon issued an Order directing the issuance of a writ of possession to the government to take or enter upon the possession of the NAIA 3 facilities. It held that it is the ministerial duty of the government to issue writ of possession upon deposit of the assessed value of the property subject of expropriation. However, on January 4, 2005, Judge Gingoyon issued another Order supplementing the December 21, 2004 Order. It pointed out that the earlier orders to the amount to be deposited by the government was based on Section 2, Rule 67 when what should be applicable is RA 8974 and therefore ordered that the amount of US$62,343,175.77 be released to PIATCO instead of the amount in the December 21, 2004 Order.

251

252

On January 7, 2005, Judge Gingoyon issued another Order directing the appointment of three (3) Commissioners to determine just compensation for the NAIA 3 Complex. Both Orders were questioned by the government as having been issued with grave abuse of discretion. ISSUES: 1. What law is applicable in this expropriation case: Rule 67 of the Rules of Court or RA 8974? 2. If RA 8974 will be used, may the court used the provision of Rule 67 on the 3 commissioners to determine just compensation. HELD: 1. Application of Rule 67 would violate the AGAN Doctrine which provides that for the government to take over the said NAIA 3 facility, IT HAS TO COMPENSATE RESPONDENT PIATCO AS BUILDER OF THE SAID STRUCTURES. If Section 2, Rule 67 will be applied, PIATCO would be enjoined from receiving the just compensation even if the government takes over the NAIA 3 facility. It is sufficient that the government deposits the amount equal to the assessed value of the facilities. It would violate the proscription in the AGAN Decision that the government must pay first the just compensation before taking over the facilities. So when shall Rule 67 be used in expropriation cases and when shall RA 8974 be used? In all National government projects or national infrastructure projects, like those covered by the Build-Operate-Transfer, RA 8974 shall be followed. The rest, Rule 67 shall apply. Differences between the two laws on expropriation: a. Under Rule 67, the government merely deposits the assessed value of the property subject of expropriation and can have a writ of possession over the same while under RA 8974, the scheme of immediate payment (100%) shall be followed. b. Under Rule 67, there can be writ of possession even if the owner of the property has not received a single centavo while under RA 8974, as in this case, Writ of Possession may not be issued in favor of the government UNTIL ACTUAL RECEIPT by PIATCO of the preferred value of just compensation. Upon issuance of the writ in favor of the government, however, it could already exercise acts of ownership over the NAIA 3 facilities. The just compensation to be paid by the government shall be determined within 60 days from the finality of the decision based on Section 4, RA 8974. 2
252

253

Rule 67 on the appointment of three (3) commissioners to determine just compensation may be used since RA 8974 does not provide for such procedure. Just Compensation; Amount to be deposited in court before a Writ of Possession may be issued by the court in favor of the government; When to apply Rule 67 and when to apply RA No. 8974; Who owns the interest of the initial amount deposited for the purpose of issuing writ of possession REPUBLIC OF THE PHILIPPINES VS. HOLY TRINITY REALTY DEVELOPMENT CORPORATION, G.R. No. 172410, April 14, 2008 THE FACTS: On 29 December 2000, petitioner Republic of the Philippines, represented by the Toll Regulatory Board (TRB), filed with the RTC a Consolidated Complaint for Expropriation against landowners whose properties would be affected by the construction, rehabilitation and expansion of the North Luzon Expressway. The suit was docketed as Civil Case No. 869-M-2000 and raffled to Branch 85, Malolos, Bulacan. Respondent Holy Trinity Realty and Development Corporation (HTRDC) was one of the affected landowners. On 18 March 2002, TRB filed an Urgent Ex-Parte Motion for the issuance of a Writ of Possession, manifesting that it deposited a sufficient amount to cover the payment of 100% of the zonal value of the affected properties, in the total amount of P28,406,700.00, with the Land Bank of the Philippines, South Harbor Branch (LBP-South Harbor), an authorized government depository. TRB maintained that since it had already complied with the provisions of Section 4 of Republic Act No. 8974184[5] in relation to Section 2 of Rule 67 of the Rules of Court, the issuance of the writ of possession becomes ministerial on the part of the RTC. The RTC issued, on 19 March 2002, an Order for the Issuance of a Writ of Possession. On 3 March 2003, HTRDC filed with the RTC a Motion to Withdraw Deposit, praying that the respondent or its duly authorized representative be allowed to withdraw the amount of P22,968,000.00, out of TRBs advance deposit of P28,406,700.00 with LBP-South Harbor, including the interest which accrued thereon. Thereafter, the RTC allowed the release of the principal amount together with the interest to the respondent but on Motion for Reconsideration of the TRB, it disallowed the withdrawal of the interest reasoning out that the said issue will be included in the second stage of expropriation, that is, the determination of just compensation. The private respondent elevated the issue to the Court of Appeals which ruled that the respondent is entitled to the interest by way of accession.

253

254

Hence, this petition of the government before the Supreme Court. I S S U E: Who has the right over the interest of the amount deposited representing the zonal value of the property sought to be expropriated? The expropriator or the landowner? HELD: The petition is without merit. The TRB claims that there are two stages185[11] in expropriation proceedings, the determination of the authority to exercise eminent domain and the determination of just compensation. The TRB argues that it is only during the second stage when the court will appoint commissioners and determine claims for entitlement to interest, citing Land Bank of the Philippines v. Wycoco186[12] and National Power Corporation v. Angas.187[13] The TRB further points out that the expropriation account with LBP-South Harbor is not in the name of HTRDC, but of DPWH. Thus, the said expropriation account includes the compensation for the other landowners named defendants in Civil Case No. 869-M-2000, and does not exclusively belong to respondent. The said argument is without merit because it failed to distinguish between the expropriation procedures under Republic Act No. 8974 and Rule 67 of the Rules of Court. Republic Act No. 8974 and Rule 67 of the Rules of Court speak of different procedures, with the former specifically governing expropriation proceedings for national government infrastructure projects. Thus, in Republic v. Gingoyon,188[14] we held: There are at least two crucial differences between the respective procedures under Rep. Act No. 8974 and Rule 67. Under the statute, the Government is required to make immediate payment to the property owner upon the filing of the complaint to be entitled to a writ of possession, whereas in Rule 67, the Government is required only to make an initial deposit with an authorized government depositary. Moreover, Rule 67 prescribes that the initial deposit be equivalent to the assessed value of the property for purposes of taxation, unlike Rep. Act No. 8974 which provides, as the relevant standard for initial compensation, the market value of the property as stated in the tax declaration or the current relevant zonal valuation of the Bureau of Internal Revenue (BIR), whichever is higher, and the value of the improvements and/or structures using the replacement cost method. xxxx

254

255

Rule 67 outlines the procedure under which eminent domain may be exercised by the Government. Yet by no means does it serve at present as the solitary guideline through which the State may expropriate private property. For example, Section 19 of the Local Government Code governs as to the exercise by local government units of the power of eminent domain through an enabling ordinance. And then there is Rep. Act No. 8974, which covers expropriation proceedings intended for national government infrastructure projects. Rep. Act No. 8974, which provides for a procedure eminently more favorable to the property owner than Rule 67, inescapably applies in instances when the national government expropriates property for national government infrastructure projects. Thus, if expropriation is engaged in by the national government for purposes other than national infrastructure projects, the assessed value standard and the deposit mode prescribed in Rule 67 continues to apply. There is no question that the proceedings in this case deal with the expropriation of properties intended for a national government infrastructure project. Therefore, the RTC correctly applied the procedure laid out in Republic Act No. 8974, by requiring the deposit of the amount equivalent to 100% of the zonal value of the properties sought to be expropriated before the issuance of a writ of possession in favor of the Republic. The controversy, though, arises not from the amount of the deposit, but as to the ownership of the interest that had since accrued on the deposited amount. Whether the Court of Appeals was correct in holding that the interest earned by the deposited amount in the expropriation account would accrue to HRTDC by virtue of accession, hinges on the determination of who actually owns the deposited amount, since, under Article 440 of the Civil Code, the right of accession is conferred by ownership of the principal property: Art. 440. The ownership of property gives the right by accession to everything which is produced thereby, or which is incorporated or attached thereto, either naturally or artificially. The principal property in the case at bar is part of the deposited amount in the expropriation account of DPWH which pertains particularly to HTRDC. Such amount, determined to be P22,968,000.00 of the P28,406,700.00 total deposit, was already ordered by the RTC to be released to HTRDC or its authorized representative. The Court of Appeals further recognized that the deposit of the amount was already deemed a constructive delivery thereof to HTRDC: When the [herein petitioner] TRB deposited the money as advance payment for the expropriated property with an authorized government depositary bank for purposes of obtaining a writ of possession, it is deemed to be a constructive delivery of the amount corresponding to the 100% zonal valuation of the expropriated property. Since [HTRDC] is entitled thereto and indisputably the owner of the principal amount deposited by [herein petitioner] TRB, conversely, the interest yield, as
255

256

accession, in a bank deposit should likewise pertain to the owner of the money deposited.189[15] Since the Court of Appeals found that the HTRDC is the owner of the deposited amount, then the latter should also be entitled to the interest which accrued thereon. The deposit was made in order to comply with Section 4 of Republic Act No. 8974, which requires nothing less than the immediate payment of 100% of the value of the property, based on the current zonal valuation of the BIR, to the property owner. Thus, going back to our ruling in Republic v. Gingoyon190[16]: It is the plain intent of Rep. Act No. 8974 to supersede the system of deposit under Rule 67 with the scheme of immediate payment in cases involving national government infrastructure projects. The critical factor in the different modes of effecting delivery which gives legal effect to the act is the actual intention to deliver on the part of the party making such delivery.191[17] The intention of the TRB in depositing such amount through DPWH was clearly to comply with the requirement of immediate payment in Republic Act No. 8974, so that it could already secure a writ of possession over the properties subject of the expropriation and commence implementation of the project. In fact, TRB did not object to HTRDCs Motion to Withdraw Deposit with the RTC, for as long as HTRDC shows (1) that the property is free from any lien or encumbrance and (2) that respondent is the absolute owner thereof.192[18] A close scrutiny of TRBs arguments would further reveal that it does not directly challenge the Court of Appeals determinative pronouncement that the interest earned by the amount deposited in the expropriation account accrues to HTRDC by virtue of accession. TRB only asserts that HTRDC is entitled only to an amount equivalent to the zonal value of the expropriated property, nothing more and nothing less. We agree in TRBs statement since it is exactly how the amount of the immediate payment shall be determined in accordance with Section 4 of Republic Act No. 8974, i.e., an amount equivalent to 100% of the zonal value of the expropriated properties. However, TRB already complied therewith by depositing the required amount in the expropriation account of DPWH with LBP-South Harbor. By depositing the said amount, TRB is already considered to have paid the same to HTRDC, and HTRDC became the owner thereof. The amount earned interest after the deposit; hence, the interest should pertain to the owner of the principal who is already determined as HTRDC. The interest is paid by LBP-South Harbor on the deposit, and the TRB cannot claim that it paid an amount more than what it is required to do so by law.

256

257

Since the respondent is the owner of P22,968,000.00, it is entitled by right of accession to the interest that had accrued to the said amount only. We are not persuaded by TRBs citation of National Power Corporation v. Angas and Land Bank of the Philippines v. Wycoco, in support of its argument that the issue on interest is merely part and parcel of the determination of just compensation which should be determined in the second stage of the proceedings only. We find that neither case is applicable herein. The issue in Angas is whether or not, in the computation of the legal rate of interest on just compensation for expropriated lands, the applicable law is Article 2209 of the Civil Code which prescribes a 6% legal interest rate, or Central Bank Circular No. 416 which fixed the legal rate at 12% per annum. We ruled in Angas that since the kind of interest involved therein is interest by way of damages for delay in the payment thereof, and not as earnings from loans or forbearances of money, Article 2209 of the Civil Code prescribing the 6% interest shall apply. In Wycoco, on the other hand, we clarified that interests in the form of damages cannot be applied where there is prompt and valid payment of just compensation. The case at bar, however, does not involve interest as damages for delay in payment of just compensation. It concerns interest earned by the amount deposited in the expropriation account. Under Section 4 of Republic Act No. 8974, the implementing agency of the government pays just compensation twice: (1) immediately upon the filing of the complaint, where the amount to be paid is 100% of the value of the property based on the current relevant zonal valuation of the BIR (initial payment); and (2) when the decision of the court in the determination of just compensation becomes final and executory, where the implementing agency shall pay the owner the difference between the amount already paid and the just compensation as determined by the court (final payment) As a final note, TRB does not object to HTRDCs withdrawal of the amount of P22,968,000.00 from the expropriation account, provided that it is able to show (1) that the property is free from any lien or encumbrance and (2) that it is the absolute owner thereof. The said conditions do not put in abeyance the constructive delivery of the said amount to HTRDC pending the latters compliance therewith. Article 1187 of the Civil Code provides that the effects of a conditional obligation to give, once the condition has been fulfilled, shall retroact to the day of the constitution of the obligation. Hence, when HTRDC complied with the given conditions, as determined by the RTC in its Orderdated 21 April 2003, the effects of the constructive delivery retroacted to the actual date of the deposit of the amount in the expropriation account of DPWH.

BIGLANG-AWA VS. JUDGE BACALLA, 354 SCRA 562 PURSUANT TO SECTION 2, RULE 67 OF THE 1997 RULES OF CIVIL PROCEDURE AND THE DOCTRINE LAID DOWN IN THE ROBERN DEVELOPMENT CASE, THE ONLY
257

258

REQUISITES FOR THE IMMEDIATE ENTRY GOVERNMENT IN EXPROPRIATION CASES ARE:

BY

THE

a. the filing of a complaint for expropriation sufficient in form and substance; and b. the making of a deposit equivalent to the ASSESSED VALUE OF THE PROPERTY SUBJECT TO EXPROPRIATION. The owners of the expropriated land are entitled to legal interest on the compensation eventually adjudged from the date the condemnor takes possession of the land until the full compensation is paid to them or deposited in court. Requisites before immediate possession or writ of possession may be issued in expropriation cases, involving local government units as the expropriating agency: 1. Complaint sufficient in form and substance; and 2. Payment of 15% of the Market value as appearing in the latest Tax Declaration.

THE CITY OF ILOILO VS. JUDGE LEGASPI, RTC 22, ILOILO CITY, 444 SCRA 269 Requisites before the expropriator is allowed immediate entry on the property subject of expropriation if the expropriator is a local government unit.

1. the complaint for expropriation filed in court is sufficient in form and substance; and 2. the expropriator must deposit the amount equivalent to 15% of the fair market value of the property to be expropriated based on its current tax declaration. But if the government fails to use the property expropriated for the purpose for which it was intended, the landowner has the right to buy-back the same.

MACTAN-CEBU INTERNATIONAL AIRPORT AUTHORITY and AIR TRANSPORTATION OFFICE vs. BERNARDO LOZADA, JR., ET AL., G.R. No. 176625, February 25, 2010 NACHURA, J.: Subject of this case is Lot No. 88-SWO-25042 (Lot No. 88), with an area of 1,017 square meters, more or less, located in Lahug, Cebu City. Its original owner was Anastacio Deiparine when the same was subject to expropriation proceedings, initiated by the Republic of the Philippines (Republic), represented
258

259

by the then Civil Aeronautics Administration (CAA), for the expansion and improvement of the Lahug Airport. The case was filed with the then Court of First Instance of Cebu, Third Branch, and docketed as Civil Case No. R-1881. As early as 1947, the lots were already occupied by the U.S. Army. They were turned over to the Surplus Property Commission, the Bureau of Aeronautics, the National Airport Corporation and then to the CAA. During the pendency of the expropriation proceedings, respondent Bernardo L. Lozada, Sr. acquired Lot No. 88 from Deiparine. Consequently, Transfer Certificate of Title (TCT) No. 9045 was issued in Lozadas name. On December 29, 1961, the trial court rendered judgment in favor of the Republic and ordered the latter to pay Lozada the fair market value of Lot No. 88, adjudged at P3.00 per square meter, with consequential damages by way of legal interest computed from November 16, 1947the time when the lot was first occupied by the airport. Lozada received the amount of P3,018.00 by way of payment. The affected landowners appealed. Pending appeal, the Air Transportation Office (ATO), formerly CAA, proposed a compromise settlement whereby the owners of the lots affected by the expropriation proceedings would either not appeal or withdraw their respective appeals in consideration of a commitment that the expropriated lots would be resold at the price they were expropriated in the event that the ATO would abandon the Lahug Airport, pursuant to an established policy involving similar cases. Because of this promise, Lozada did not pursue his appeal. Thereafter, Lot No. 88 was transferred and registered in the name of the Republic under TCT No. 25057. The projected improvement and expansion plan of the old Lahug Airport, however, was not pursued. Lozada, with the other landowners, contacted then CAA Director Vicente Rivera, Jr., requesting to repurchase the lots, as per previous agreement. The CAA replied that there might still be a need for the Lahug Airport to be used as an emergency DC-3 airport. It reiterated, however, the assurance that should this Office dispose and resell the properties which may be found to be no longer necessary as an airport, then the policy of this Office is to give priority to the former owners subject to the approval of the President. On November 29, 1989, then President Corazon C. Aquino issued a Memorandum to the Department of Transportation, directing the transfer of general aviation operations of the Lahug Airport to the Mactan International Airport before the end of 1990 and, upon such transfer, the closure of the Lahug Airport. Sometime in 1990, the Congress of the Philippines passed Republic Act (R.A.) No. 6958, entitled An Act Creating the Mactan-Cebu International Airport Authority, Transferring Existing Assets of the Mactan International Airport and the Lahug Airport to the Authority, Vesting the Authority with Power to Administer and Operate the Mactan International Airport and the Lahug Airport, and For Other Purposes.

From the date of the institution of the expropriation proceedings up to the present, the public purpose of the said expropriation (expansion of the airport)
259

260

was never actually initiated, realized, or implemented. Instead, the old airport was converted into a commercial complex. Lot No. 88 became the site of a jail known as Bagong Buhay Rehabilitation Complex, while a portion thereof was occupied by squatters. The old airport was converted into what is now known as the Ayala I.T. Park, a commercial area. Thus, on June 4, 1996, petitioners initiated a complaint for the recovery of possession and reconveyance of ownership of Lot No. 88. The case was docketed as Civil Case No. CEB-18823 and was raffled to the Regional Trial Court (RTC), Branch 57, Cebu City. In their Answer, petitioners asked for the immediate dismissal of the complaint. They specifically denied that the Government had made assurances to reconvey Lot No. 88 to respondents in the event that the property would no longer be needed for airport operations. Petitioners instead asserted that the judgment of condemnation was unconditional, and respondents were, therefore, not entitled to recover the expropriated property notwithstanding non-use or abandonment thereof. After pretrial, but before trial on the merits, the parties stipulated on the following set of facts: (1) The lot involved is Lot No. 88-SWO-25042 of the Banilad Estate, situated in the City of Cebu, containing an area of One Thousand Seventeen (1,017) square meters, more or less; The property was expropriated among several other properties in Lahug in favor of the Republic of the Philippines by virtue of a Decision dated December 29, 1961 of the CFI of Cebu in Civil Case No. R-1881; The public purpose for which the property was expropriated was for the purpose of the Lahug Airport; After the expansion, the property was transferred in the name of MCIAA; [and] On November 29, 1989, then President Corazon C. Aquino directed the Department of Transportation and Communication to transfer general aviation operations of the Lahug Airport to the Mactan-Cebu International Airport Authority and to close the Lahug Airport after such transfer.

(2)

(3)

(4)

(5)

On October 22, 1999, the RTC rendered its Decision, disposing as follows: WHEREFORE, in the light of the foregoing, the Court hereby renders judgment in favor of the plaintiffs, Bernardo L. Lozada, Sr., and the heirs of Rosario Mercado, namely, Vicente M. Lozada, Marcia L. Godinez, Virginia L. Flores, Bernardo M. Lozada, Jr., Dolores L. Gacasan, Socorro L. Cafaro and Rosario M. Lozada, represented by their attorney-in-fact Marcia Lozada Godinez, and against defendants Cebu-Mactan International Airport Authority (MCIAA) and Air Transportation Office (ATO):

260

261

1. ordering MCIAA and ATO to restore to plaintiffs the possession and ownership of their land, Lot No. 88 Psd-821 (SWO-23803), upon payment of the expropriation price to plaintiffs; and 2. ordering the Register of Deeds to effect the transfer of the Certificate of Title from defendant[s] to plaintiffs on Lot No. [88], cancelling TCT No. 20357 in the name of defendant MCIAA and to issue a new title on the same lot in the name of Bernardo L. Lozada, Sr. and the heirs of Rosario Mercado, namely: Vicente M. Lozada, Mario M. Lozada, Marcia L. Godinez, Virginia L. Flores, Bernardo M. Lozada, Jr., Dolores L. Gacasan, Socorro L. Cafaro and Rosario M. Lozada. No pronouncement as to costs. Aggrieved, petitioners interposed an appeal to the CA. After the filing of the necessary appellate briefs, the CA rendered its assailed Decision dated February 28, 2006, denying petitioners appeal and affirming in toto the Decision of the RTC, Branch 57, Cebu City. Petitioners motion for reconsideration was, likewise, denied in the questioned CA Resolution dated February 7, 2007. Hence, this petition arguing that: (1) the respondents utterly failed to prove that there was a repurchase agreement or compromise settlement between them and the Government; (2) the judgment in Civil Case No. R-1881 was absolute and unconditional, giving title in fee simple to the Republic; and (3) the respondents claim of verbal assurances from government officials violates the Statute of Frauds. The petition should be denied. Petitioners anchor their claim to the controverted property on the supposition that the Decision in the pertinent expropriation proceedings did not provide for the condition that should the intended use of Lot No. 88 for the expansion of the Lahug Airport be aborted or abandoned, the property would revert to respondents, being its former owners. Petitioners cite, in support of this position, Fery v. Municipality of Cabanatuan, which declared that the Government acquires only such rights in expropriated parcels of land as may be allowed by the character of its title over the properties If x x x land is expropriated for a particular purpose, with the condition that when that purpose is ended or abandoned the property shall return to its former owner, then, of course, when the purpose is terminated or abandoned the former owner reacquires the property so expropriated. If x x x land is expropriated for a public street and the expropriation is granted upon condition that the city can only use it for a public street, then, of course, when the city abandons its use as a public street, it returns to the former owner, unless there is some statutory provision to the contrary. x x x. If, upon the contrary, however, the decree of expropriation gives to the entity a fee simple title, then, of course, the land becomes the absolute property of the expropriator, whether it be the State, a province, or municipality, and in that case the non-user does not have the effect of defeating the title acquired by the expropriation proceedings. x x x.

261

262

When land has been acquired for public use in fee simple, unconditionally, either by the exercise of eminent domain or by purchase, the former owner retains no right in the land, and the public use may be abandoned, or the land may be devoted to a different use, without any impairment of the estate or title acquired, or any reversion to the former owner. x x x. Contrary to the stance of petitioners, this Court had ruled otherwise in Heirs of Timoteo Moreno and Maria Rotea v. Mactan-Cebu International Airport Authority, thus Moreover, respondent MCIAA has brought to our attention a significant and telling portion in the Decision in Civil Case No. R-1881 validating our discernment that the expropriation by the predecessors of respondent was ordered under the running impression that Lahug Airport would continue in operation As for the public purpose of the expropriation proceeding, it cannot now be doubted. Although Mactan Airport is being constructed, it does not take away the actual usefulness and importance of the Lahug Airport: it is handling the air traffic both civilian and military. From it aircrafts fly to Mindanao and Visayas and pass thru it on their flights to the North and Manila. Then, no evidence was adduced to show how soon is the Mactan Airport to be placed in operation and whether the Lahug Airport will be closed immediately thereafter. It is up to the other departments of the Government to determine said matters. The Court cannot substitute its judgment for those of the said departments or agencies. In the absence of such showing, the Court will presume that the Lahug Airport will continue to be in operation (emphasis supplied). While in the trial in Civil Case No. R-1881 [we] could have simply acknowledged the presence of public purpose for the exercise of eminent domain regardless of the survival of Lahug Airport, the trial court in its Decision chose not to do so but instead prefixed its finding of public purpose upon its understanding that Lahug Airport will continue to be in operation. Verily, these meaningful statements in the body of the Decision warrant the conclusion that the expropriated properties would remain to be so until it was confirmed that Lahug Airport was no longer in operation. This inference further implies two (2) things: (a) after the Lahug Airport ceased its undertaking as such and the expropriated lots were not being used for any airport expansion project, the rights vis--vis the expropriated Lots Nos. 916 and 920 as between the State and their former owners, petitioners herein, must be equitably adjusted; and (b) the foregoing unmistakable declarations in the body of the Decision should merge with and become an intrinsic part of the fallo thereof which under the premises is clearly inadequate since the dispositive portion is not in accord with the findings as contained in the body thereof.

262

263

Indeed, the Decision in Civil Case No. R-1881 should be read in its entirety, wherein it is apparent that the acquisition by the Republic of the expropriated lots was subject to the condition that the Lahug Airport would continue its operation. The condition not having materialized because the airport had been abandoned, the former owner should then be allowed to reacquire the expropriated property. On this note, we take this opportunity to revisit our ruling in Fery, which involved an expropriation suit commenced upon parcels of land to be used as a site for a public market. Instead of putting up a public market, respondent Cabanatuan constructed residential houses for lease on the area. Claiming that the municipality lost its right to the property taken since it did not pursue its public purpose, petitioner Juan Fery, the former owner of the lots expropriated, sought to recover his properties. However, as he had admitted that, in 1915, respondent Cabanatuan acquired a fee simple title to the lands in question, judgment was rendered in favor of the municipality, following American jurisprudence, particularly City of Fort Wayne v. Lake Shore & M.S. RY. Co., McConihay v. Theodore Wright, and Reichling v. Covington Lumber Co., all uniformly holding that the transfer to a third party of the expropriated real property, which necessarily resulted in the abandonment of the particular public purpose for which the property was taken, is not a ground for the recovery of the same by its previous owner, the title of the expropriating agency being one of fee simple. Obviously, Fery was not decided pursuant to our now sacredly held constitutional right that private property shall not be taken for public use without just compensation. It is well settled that the taking of private property by the Governments power of eminent domain is subject to two mandatory requirements: (1) that it is for a particular public purpose; and (2) that just compensation be paid to the property owner. These requirements partake of the nature of implied conditions that should be complied with to enable the condemnor to keep the property expropriated. More particularly, with respect to the element of public use, the expropriator should commit to use the property pursuant to the purpose stated in the petition for expropriation filed, failing which, it should file another petition for the new purpose. If not, it is then incumbent upon the expropriator to return the said property to its private owner, if the latter desires to reacquire the same. Otherwise, the judgment of expropriation suffers an intrinsic flaw, as it would lack one indispensable element for the proper exercise of the power of eminent domain, namely, the particular public purpose for which the property will be devoted. Accordingly, the private property owner would be denied due process of law, and the judgment would violate the property owners right to justice, fairness, and equity. In light of these premises, we now expressly hold that the taking of private property, consequent to the Governments exercise of its power of eminent domain, is always subject to the condition that the property be devoted to the specific public purpose for which it was taken. Corollarily, if this particular purpose or intent is not initiated or not at all pursued, and is peremptorily abandoned, then the former owners, if they so desire, may seek the reversion of the property, subject to the return of the amount of just compensation received. In such a case, the exercise of the power of eminent domain has become improper for lack of the required factual justification. On the matter of the repurchase price, while petitioners are obliged to reconvey Lot No. 88 to respondents, the latter must return to the former what they received as just compensation for the expropriation of the property, plus legal
263

264

interest to be computed from default, which in this case runs from the time petitioners comply with their obligation to respondents. Respondents must likewise pay petitioners the necessary expenses they may have incurred in maintaining Lot No. 88, as well as the monetary value of their services in managing it to the extent that respondents were benefited thereby.

GABATIN VS. LAND BANK OF THE PHILIPPINES, 444 SCRA 176 What is the basis of the just compensation for expropriation proceedings in connection with the agrarian reform program of the government. Held:

BANK OF THE PHILIPPINE ISLANDS VS. COURT OF APPEALS, 441 SCRA 637 Just compensation in expropriation cases; value of the property when?

4. Basis of just compensation Read: a. NPC vs. Jocson, February 25, 1992 a-1. Ansaldo vs. Tantuico, Aug. 3, 1990 b. Mun. of Makati vs. CA, Oct. 1, 1990 c. Reublic vs. IAC, 185 SCRA 572 d. Mun. of Talisay vs. Ramirez, 183 SCRA 528 e. NPC vs. CA, 129 SCRA 665 f. Maddumba vs. GSIS, 182 SCRA 281 Read also:
264

265

1. Meaning of just compensation in eminent domain 29 SCRA 868 Basis of just compensation (Exceptional case)

proceedings,

BERKENKOTTER, INC. VS. COURT OF APPEALS AND REPUBLIC OF THE PHILIPPINES, December 14, 1992 Cruz, J. Facts: 1. On June 18, 1982, Vicente Viray, then President of Apolinario Apacible School of Fisheries, a government institution in Nasugbu, Batangas, sent the petitioner a written offer to buy the property of the latter with an area of 10,640 square meters for its 5-year expansion program; 2. That the petitioner expressed willingness to sell at P50.00 per square meter in its reply; 3. Viray then requested the Office of the Provincial Assessor of the Province of Batangas to appraise the land and the latter fixed its market value at P32.00 per square meter; 4. Viray then wrote the petitioner and expressed willingness to buy the latter's property at P32.00 per square meter. The petitioner, however, stuck to its original valuation. Later on, it said that its property had in fact appreciated to as much as P100.00 per square meter; 5. On October 28, 1983, the Republic of the Philippines filed a complaint for the expropriation of the petitioner's property and invoked the assessment made by the Provincial Appraisal Committee of the Provincial Assessor of Batangas in the amount of P32.00. The government likewise sought immediate possession of the property upon deposit of 10% of the total assessment in accordance with PD 48; 6. Berkenkotter originally questioned the purpose of the expropriation but later abandoned this objection and concentrated only on what it called the "underappraisal" of the subject land; 7. The RTC then appointed a panel of commissioners in accordance with Rule 67, ection 5, of the Rules of Court, to determine the just compensation to be paid for the land; 8. On September 23, 1985, the panel of commissioners submitted its report to the trial court and pegged the market value at P85.00 per square meter; 9. The Republic of the Philippines objected and pointed to three (3) contracts of sale executed by the petitioner in 1985 whereby it sold three (3) tracts of land similar in topography and adjacent to the property in question for the unit price of only P19.18 per square meter; 10. The court directed the commissioners to convene anew and to receive additional evidence. However, in its second report dated April 1, 1987, the panel reiterated its original recommendation of P85.00/sq. m. or a
265

266

total of P904,400.00 for the entire area sought to be expropriated. The trial court acting on this recommendation rendered judgment requiring the Republic to pay the petitioner the amount of P904,400.00 for the entire area sought to be expropriated; 11. The government appealed the trial court's decision to the Court of Appeals which rendered a decision REVERSING THE LOWER COURT'S DECISION and declaring that the fair market value which should be the basis in computing the amount to be paid by the government to the petitioner shall be P19.18, the market value according set by the petitioner if we follow the three (3) deeds of sale it executed in favor of three (3) different individuals; 12. The petitioner was therefore constrained to file this instant petition claiming that the Court of Appeals erred in holding that P19.18 per square meter should be the basis of the computation for the just compensation of its property because: a. Viray even offered the amount of P32.00 per squaremeter as the fair market value; b. that P32.00 per square meter was the appraised value made by the Office of the Provincial Assessor of Batangas; and c. the complaint itself prays that the market value be pegged at P32.00 per square meter. Issue: WHAT SHOULD BE THE BASIS IN THE COMPUTATION OF THE JUST COMPENSATION: P32.00/SQ. M. IN ACCORANCE WITH THE APPRAISAL OF THE PROVINCIAL ASSESSOR; P100.00/SQ.M. AS CLAIMED BY THE OWNER; P85.00/SQ. M. AS RECOMMENDED BY THE BOARD OF COMMISSIONERS APPOINTED BY THE COURT TO EVALUATE THE SAME, OR P19.18 PER SQUARE METER WHICH WAS THE SELLING PRICE IN AN ADJACENT LOT SOLD BY THE PETITIONER TO THREE PRIVATE INDIVIDUALS. Held. The basis in the computation of just compensation shall be P19.18 per square meter or the price which the petitioner sold its other lots to other individuals. This is so because there is no showing that the petitioner had any special reason for granting each of the individual vendees the extraordinary discount amounting to as much as 75% of its claimed real value of the land. To all appearances, they were ordinary buyers who bought the land for their own private purposes only and not for the public purpose invoked by the government. The petitioner's claim that the value as appearing in the deeds of sale in the three other parcels is not a reliable index of just compensation "because owners usually undervalue the selling price of the property to lower the expenses they would have to pay for capital gains tax and documentary stamps tax" is practically an admission that it did
266

267

not indicate the actual consideration in the three transactions where it was made to appear that the price per square meter was only P19.18. If this was the purpose of the petitioner when it executed the 3 deeds of sale, then IT IS SURELY HOIST NOW BY ITS OWN PETARD. AND RIGHTLY SO, FOR IT CANNOT BE ALLOWED TO PROFIT FROM ITS OWN DECEPTION AND CLAIM THAT THE SUBJECT PROPERTY SHOULD BE ASSESSED AT THE HIGHER RATE IT CLANDESTINELY AGREED UPON WITH THE BUYERS. The Court is disappointed that the petitioner should demand a higher price from the republic, which needs the land for a public purpose, when it was willing to accept less from the three individual buyers who had only their private interests to serve. The fact that the petitioner sold the 3 other parcels of land at P19.18 per square meter which are admittedly of the same topography as that subject of this case, it impliedly admitted that the price for the latter should be the same as the former. This rule of consistency is best expressed in the familiar saying, surely not unknown to the petitioner, THAT WHAT IS SAUCE FOR THE GOOSE IS ALSO SAUCE FOR THE GANDER. Just compensation is defined as the full and fair equivalent of the proerty sought to be expropriated (Association of Small Landowners vs. Secretary of Agrarian Reform, 175 SCRA 378). The measure is not the taker's gain but the owner's loss. he compensation, to be just, must be fair not only to the owner but also to the taker. To determine just compensation, the trial court should first ascertain the market value of the property, to which should be added the consequential benefits which may arise from the expropriation. The market value of the property is the price that may be agreed upon by the parties willing but not compelled to enter into a contract of sale. Among the factors to be considered in arriving at the fair market value are: 1. cost of acquisition; 2. the current value of like proerties; 3. its actual or potential uses; 4. particular case of lands; 5. their size, shape, location; and 6. the tax declarations thereon. Finally, note that as held in the case of Republic vs. Santos, 141 SCRA 30, the market value as recommended by the board of commissioners appointed by the court were at best only ADVISORY AND PERSUASIVE AND BY NO MEANS FINAL OR BINDING. 2. NHA vs. Reyes, 123 SCRA 245 3. Manotok vs. CA, May 21,1987 4. EPZA vs. Dulay, April 29,l987 5. Lagunzad vs. CA, 154 SCRA 199 When it is considered for "public use":
267

268

6. Sumulong vs. Guererro, 154 SCRA 461 7. Republic vs. CA, 154 SCRA 428 8.Cosculluela vs. CA, 164 SCRA 393 5. Requisite of "taking" in eminent domain cases Read: 1. Rep. vs. Castellvi, 58 SCRA 336 Requisites of taking: the expropriator must enter the property; the entrance must not be for just a momentary period; the entry must be under warrant of color or title; the property must be devoted for public use; and the owner must be ousted from beneficial use of his land. 2. Ignacio vs. Guererro, 150 SCRA 369 3. Garcia vs. CA, 102 SCRA 597 6. Not a valid exercise of eminent domain Read: 1. City of Manila vs. Chinese Community, 40 Phil. 349 ( A private property which is devoted to public use may not be expropriated for another public purpose.) 2. De Knecht vs. Bautista, 100 SCRA 660

a. b. c. d. e.

REPUBLIC OF THE PHILIPPINES VS. CRISTINA DE KNECHT AND THE COURT OF APPEALS, G.R. NO. 87335, February 12, 1989 Expropriation Gancayco, J. Facts: 1. On February 20, 1979, the Rep. of the Philippines initiated an expropriation proceedings against the owners of the houses standing along Fernando Rein-Del Pan streets, among them Cristina de Knecht together with Concepcion Cabarrus, and some other fifteen defendants in Civil Case No. 7001-P; 2. In June, 1979, the Republic of the Philippines prayed for the issuance of a writ of possession of the property to be expropriated on the ground that it had already deposited with the PNB 10% of the amount of compensation stated in the complaint; that on June 14, 1979, the Lower Court issued a writ of possession authorizing the Republic to enter into the properties condemned and created a committee to determine just compensation;

268

269

3. On July 16, 1979, De Knecht went to the Supreme Court on a petition for certiorari and prohibition directed against the June 14, 1979 order of the lower court; 4. On October 30, 1980, the Supreme Court rendered its decision granting the petition for certiorari and prohibition and directing that the Order of the respondent Judge dated June 14, 1979 be SET ASIDE and the respondent Judge is permanently enjoined from taking any further action on Civil Case No. 7001-P; 5. On August 8, 1981, the defendants in Civil Case No. 7001- moved for the dismissal of said case since the decision of the Supreme Court is already final; 6. On September 2, 1983, the Republic moved for the dismissal of the case due to the enactment of BP 340 expropriating the same properties for the same purpose. On the same date, the Court dismissed the case. The defendants moved for a reconsideration which the Court denied; 7. De Knecht appealed the Order dismissing the case to the Court of Appeals who on December 28, 1988 issued its decision setting aside the Order appealed from and dismissing the expropriation proceedings before the lower court on the ground that the choice of the above-mentioned streets as the line through which the EDSA should be extended is arbitrary and should not receive judicial approval; 8. The Republic of the Philippines filed a Petition for Review with the Supreme Court. Issue: Whether or not the legislature could still pass a law expropriating the lots of the private respondents despite the existence of a final decision of the Supreme Court which held that choice of their lot to be used as an extension of EDSA is arbitrary? Held: It is true that there is already a final decision of the Supreme Court to the effect that the choice of the Fernando Rein-Del Pan Streets is arbitrary and should not receive judicial approval. However, it is equally true that the Constitution and our laws may expropriate private properties after the payment of just compensation. When on February 17, 1983, the Batasang Pambansa passed BP 340 expropriating the same properties for the same purpose, IT APPEARS THAT THE SAME WAS BASED ON SUPERVENING EVENTS THAT OCCURRED after the decision of the SC in De Knecht vs. Bautista in 1980. The social impact factor which persuaded the Court to consider this extension has disappeared because of the fact that the residents of the area have been relocated and duly compensated and only DE KNECHT now is left while her property is only about 5% of the area to be expropriated. The Republic could continue it expropriation proceedings considering the supervening events after the decision was rendered. BP Bilang 340 THEREFORE EFFECTIVELY SUPERSEDED THE AFORESAID FINAL AND EXECUTORY DECISION OF THE SUPREME COURT. X x x THE COURT AGREES IN THE WISDOM
269

270

AND NECESSITY OF ENACTING BP 340. THUS THE ANTERIOR DECISION OF THIS COURT MUST YIELD TO THIS SUBSEQUENT LEGISLATIVE FIAT. ************************ Cruz, J., concurring Supervening events have changed the factual basis of the SC's decision to justify the subsequent enactment of the statute. If we are sustaining the legislation, it is not because we concede that the lawmakers can nullify the findings of the Court in the exercise of its discretion. It is simply because we ourselves have found that under the changed situation, the present expropriation is no longer arbitrary. I MUST ADD THAT THIS DECISION IS NOT A REVERSAL OF THE ORIGINAL DE KNECHT CASE, WHICH WAS DECIDED UNDER A DIFFERENT SET OF FACTS. 3. REPUBLIC OF THE PHILIPPINES VS. CRISTINA DE KNECHT AND THE COURT OF APPEALS, G.R. NO. 87335, February 12, 1989 3-a. Limitations of the power of expropriation, 3 SCRA 706

7. When shall we base the computation of the value of the property expropriated: at the time of taking or at the time of the institution of the expropriation proceedings? 8. Eminent domain cases, in general Read: 1. City of Baguio vs. NAWASA, 106 Phil. 144 2. Garcia vs. CA, 102 SCRA 620 3. Municipality of Daet vs. CA, 93 SCRA 503 4. Salas vs. Jarencio, 46 SCRA 734 5. Arce vs. Genito, Feb. 27, 1976 6. Guido vs. RPA, 84 Phil. 847 7. Rep. vs. Baylosis, 96 Phil. 461 8. Mataas na Lupa vs. Dimayuga, 130 SCRA 30 9. San Diego vs. Valdellon, 80 SCRA 305 10. Haguisan vs. Emilia, 131 SCRA 517 11. Heirs of Ardona vs. Reyes, 125 SCRA 220 12. Commissioner vs. Burgos, March 31,1980 13. Republic vs. Juan, 92 SCRA 29

CHAPTER XI - THE NON-IMPAIRMENT CLAUSE Section 10. No law impairing the obligation of contracts shall be passed. 1. Read: 1. Kabiling, et al., vs. NHA, December 18,l987 2. Clements vs. Nolting, 42 Phil. 702 3. Co vs. PNB, 114 SCRA 842
270

271

4. Lozano vs. Martinez,146 SCRA 323 5. Rutter vs. Esteban,93 Phil. 68 6. Ilusorio vs. CAR, 17 SCRA 25 7. Ortigas vs. Feati Bank, 94 SCRA 533 8. Ganzon vs. Insierto, 123 SCRA 713 9. Del Rosario vs. De los Santos, March 21, 1968 10. Abella vs. NLRC, 152 SCRA 140 11. PVBEU vs. PVB, 189 SCRA 14 Section 11. Free access to the courts and quasi-judicial bodies and adequate legal assistance shall not be denied to any person by reason of poverty.

CHAPTER XII - RIGHTS DURING CUSTODIAL INVESTIGATION Section 12. (1) Any person under investigation for the commission of an offense shall have the right to be informed of his right to remain silent and to have competent and independent counsel preferably of his own choice. If the person cannot afford the services of counsel, he must be provided with one. These rights cannot be waived except in writing and in the presence of counsel. (2) No torture, force, violence, threat, intimidation or any other means which vitiate the free will shall be used against him. Secret detention places, solitary, incommunicado, or other similar forms of detention are prohibited. (3) Any confession or admission obtained in violation of this or Section 17 hereof shall be inadmissible in evidence against him. Rights of a person under custodial detention for one suspected or arrested as a terrorist. NOTE: Applicable provisions of the Human Security Act/Anti-Terrorism Law, Republic Act No. 9372, Approved on March 6, 2007 and effective on July 15, 2007 (This Law shall be automatically suspended one (1) month before and two (2) months after the holding of any election) Section 21. Rights of a person under custodial detention.- The moment a person charged with or suspected of the crime of terrorism or the crime of conspiracy to commit terrorism is apprehended or arrested and detained, he shall forthwith be informed by the arresting police or law enforcement officers to whose custody the person concerned is brought, of his or her right:

271

272

1. to be informed of the nature and cause of his arrest, to remain silent and to have competent and independent counsel preferably of his own choice. If the person cannot afford the services of counsel of his or her choice, the police or law enforcement officers concerned shall immediately contact the free legal assistance unit of the IBP or the Public attorneys office (PAO). It shall be the duty of the free legal assistance unit of the IBP or the PAOs thus contacted to immediately visit the person detained and provide him with legal assistance. These rights cannot be waived except in writing and in the presence of the counsel of choice; 2. informed of the cause or causes of his detention in the presence of his legal counsel; 3. allowed to communicate freely with his legal counsel and to confer with them at any time without restriction; 4. allowed to communicate freely and privately without restrictions with the members of his family or with his nearest relatives and be visited by them; and 5. allowed freely to avail of the services of a physician or physicians of choice. Section 23. Requirement for an official custodial logbook and its contents.- The police or other law enforcement custodial unit in whose care and control the person charged with or suspected of the crime of terrorism or the crime of conspiracy to commit terrorism has been placed under custodial arrest and detention shall keep a securely and orderly maintained official logbook, which is hereby declared as public document and opened and made available for inspection and scrutiny of the lawyer or lawyers of the person under custody or any member of his family or relative by consanguinity within the fourth civil degree or his physician at any time of the day without any form of restriction. The logbook shall contain a clear and concise record of: 1. name, description, and address of the detained person; 2. date and exact time of his initial admission for custodial arrest and detention; 3. the name and address of the physician/s who examined him physically and medically; 4. the state of his health and his physical condition a the time of his initial admission for custodial detention; 5. the date and time of each removal of the detained person from his cell for interrogation or for any purpose; 6. the date and time of his return to his cell; 7. name and address of the physician who examined him physically and medically; 8. summary of the physical and medical findings after each interrogation; 9. names and addresses of the members of his family and relatives; 10. names and addresses of the persons who visited him; 11. date and time of such visits; 12. date and time when the detained person requested to communicate or confer with his lawyer; 13. the date and time of visits by his legal counsel and the date and time of departure; and 14. all other important events bearing on all relevant details regarding the treatment of the detained person while under custodial arrest or detention. Section 24. No torture or coercion in Investigation and interrogation. No threat, intimidation, or coercion, and no act which will inflict any form of physical pain or torment, or mental, moral, or
272

273

psychological pressure on the detained person which shall vitiate his free will shall be employed in his investigation and interrogation; otherwise, the evidence obtained from said detained person shall be in its entirety, absolutely not admissible and usable as evidence in any judicial, quasijudicial, legislative, or administrative investigation, inquiry, proceeding or hearing. The summary of the rights of an accused during custodial investigation (from the time of arrest) under the Constitution, laws and jurisprudence. THE PEOPLE OF THE PHILIPPINES VS. MAHINAY, G.R. No. 122485, February 1, 1999 Per Curiam: Considering the heavy penalty of death and in order to ensure that evidence against an accused were obtained through lawful means, the Court, as guardian of the rights of the people, lays down the PROCEDURE, GUIDELINES, AND DUTIES WHICH THE ARRESTING, DETAINING, INVITING OR INVESTIGATING OFFICER OR HIS COMPANIONS MUST OBSERVE AT THE TIME OF MAKING THE ARREST AND AGAIN AT AND DURING THE TIME OF THE CUSTODIAL INVESTIGATION OR INTERROGATION IN ACCORDANCE with the Constitution, jurisprudence and Republic Act No. 7438. It is high time to educate our law enforcement agencies who neglect either by ignorance or indifference the so-called Miranda rights which had become insufficient and which the court must update in the light of new legal developments. 1. The person arrested, detained, invited or under custodial investigation must be informed in a language known to and understood by him of the reason for the arrest and he must be shown a copy of the warrant of arrest, if any; Every other warnings, information or communication must be in a language known to and understood by said person; 2. He must be warned that he has the right to remain silent and that any statement he makes may be used as evidence against him; 3. He must be informed that he has the right to be assisted at all times and have the presence of an independent and competent lawyer, preferably of his own choice; 4. He must be informed that if he has no lawyer or cannot afford the services of a lawyer, one will be provided for him; and that a lawyer may also be engaged by any person in his behalf, or may be appointed by the court upon petition of the person arrested or one acting in his behalf; 5. That whether or not the person arrested has a lawyer, , he must be informed that no custodial investigation in any form shall be conducted except in the presence of his counsel or after a valid waiver has been made; 6. The person arrested must be informed that, at any time, he has the right to communicate or confer by the most expedient means---telephone, radio, letter or messenger---with his lawyer (either retained or appointed), any
273

274

member of his immediate family; or any medical doctor, priest or minister chosen by him or by any one from his immediate family or by his counsel, or be visited by/confer with duly accredited national or international nongovernmental organization. IT SHALL BE THE RESPONSIBILITY OF THE OFFICER TO ENSURE THAT THIS IS ACCOMPLISHED; 7. He must be informed that he has the right to waive any of said rights provided it is made voluntarily, knowingly and intelligently and ensure that he understood the same; 8. In addition, if the person arrested waives his right to a lawyer, he must be informed that it must be done in writing AND in the presence of counsel, otherwise, he must be warned that the waiver is void even if he insist on his waiver and chooses to speak; 9. That the person arrested must be informed that he may indicate in any manner at any time or state of the process that he does not wish to be questioned with the warning that once he makes such indication, the police may not interrogate him if the same had not yet commenced, or the interrogation has begun; 10. The person arrested must be informed that his initial waiver of his right to remain silent, the right to counsel or any of his rights does not bar him from invoking it at any other time during the process, regardless of whether he may have answered some questions or volunteered some information or statements; 11. He must be informed that any statement OR EVIDENCE, as the case may be, obtained in violation of any of the foregoing, whether inculpatory or exculpatory, in whole or in part, SHALL BE INADMISSIBLE IN EVIDENCE. (NOTE: Any violation of the foregoing rights of the accused shall entitle him to sue for damages against the arresting or investigating officers in accordance with RA7438, not to mention the possible criminal liability of said persons under existing laws). LATEST CASES ON THE RIGHTS OF A PERSON DURING CUSTODIAL INVESTIGATION Rights during custodial investigation; when custodial investigation is deemed to have started; right to be informed of the nature and cause of accusation against him. THE PEOPLE OF THE PHILIPPINES VS. JOSE TING LAN UY, JR., et al., 475 SCRA 248

Ynares-Santiago, J. Facts: For allegedly diverting and collecting funds of the National Power Corporation intended for the purchase of US Dollars from the United Coconut Planters Bank (UCPB), the accused-appellants were charged of Malversation through Falsification of Commercial Documents as defined and penalized under Arts. 217 and 171 [8] in relation to Article 48 of the
274

275

Revised Penal Code. After trial, all accused were convicted by the Sandiganbayan. While the Information charged the accused of willful and intentional commission of the acts complained of while the Decision found the accused guilty of inexcusable negligence. Accused Ochoa interposed an appeal and claimed that his conviction was based on his alleged sworn statement and the transcript of stenographic notes of a supposed interview with an NPC personnel and the report of the NBI. He maintains that he signed the sworn statement while confined a the Philippine heart center and upon assurance that it would not be used against him. He was not assisted by counsel nor he was apprised of his constitutional rights when he executed the affidavit. He likewise claimed that his constitutional rights to be informed of the nature and cause of accusation against and due process were violated. Held: 1. Even if the information charges willful malversation, conviction for malversation through negligence may still be adjudged if the evidence ultimately proves that mode of commission of the offense. (Diaz vs. Sandiganbayan, 302 SCRA 118). This was the doctrine laid down in the case of Samson vs. Court of appeals, 103 Phil. 277. 2. The claim that his affidavit is inadmissible in evidence in accordance with section 12 [1] of the Bill of Rights is not tenable. The investigation under said provision refers to custodial investigation where a suspect has already been taken into police custody and that the investigating officers begin to ask questions to elicit information and confessions or admissions from the suspect. Succinctly stated, custodial investigation refers to the critical pre-trial stage when the investigation ceases to be a general inquiry into an unsolved crime but has began to focus on a particular person as a suspect (People vs. Duenas, Jr., 426 SCRA 666). Clearly, therefore, the rights enumerated by the accused are not available BEFORE GOVERNMENT INVESTIGATORS ENTER THE PICTURE. The protective mantle of section 12, article III does not apply to administrative investigations (People vs. Judge Ayson, 175 SCRA 216); confession to a private individual (Kimpo vs. CA, 232 SCRA 53); verbal admission made to a radio announcer who was not a part of the investigation ( People vs. Ordono, 334 SCRA 673); or even to a Mayor approached as a personal confidante and not in his official capacity (People vs. Zuela, 323 SCRA 589). In fact, even a videotaped interview where the accused willingly admit his guilt in the presence of newsmen is not covered by the said provision though the trial courts were warned by the supreme Court to take extreme caution in admitting similar confessions because of the distinct possibility that the police, with the connivance of unscrupulous media practitioners, may attempt to legitimize coerced extrajudicial confessions and place them beyond the exclusionary rule by having an accused admit an offense on television (People vs. Endino, 353 SCRA 307). Clearly, the confession of the accused was obtained during an administrative investigation by NPC and therefore, the same was not covered by Section 12, Art. III of the Constitution.

275

276

(NOTE: In People vs. Andam, the confession made before a Municipal Mayor was held admissible as evidence). PEOPLE VS. FIGUEROA, 335 SCRA 349 Under Art. III, Section 12 [1] of the Constitution, a suspect in custodial investigation must be: 1. informed of his right to remain silent; 2. warned that anything he says can be and will be used against him; 3. told that he has the right to counsel, and that if he is indigent, a lawyer will be appointed to represent him. In this case, accused-appellant was given no more than a perfunctory recitation of his rights, signifying nothing more than a feigned compliance with the constitutional requirements. (People vs. Samolde, July 31, 2000) It is always incumbent on the prosecution to prove at the trial that, prior to in-custody questioning, the confessant was informed of his constitutional rights. The presumption of regularity of official acts does not prevail over the constitutional presumption of innocence. Hence, in the absence of proof that the arresting officers complied with the above constitutional safeguards, extrajudicial statements, whether inculpatory or exculpatory, made during the custodial investigation, are inadmissible not only against the DECLARANT but with more so against 3rd persons. THIS IS SO EVEN IF SUCH STATEMENTS ARE GOSPEL TRUTH AND VOLUNTARILY GIVEN. Such statements are useless EXCEPT AS EVIDENCE AGAINST THE VERY POLICE AUTHORITIES WHO VIOLATED THE SUSPECTS RIGHTS. PEOPLE VS. BARIQUIT, 341 SCRA 600 When custodial investigation is deemed to have started. The protection under Section 12 , Art. III of the Constitution begins when a person is taken into custody for investigation of his possible participation in the commission of a crime, or from the time he is singled out as a suspect in the commission of the crime, although not yet in custody. Custodial investigation begins when it is no longer a general inquiry into an unsolved crime but starts to focus on a particular person as a suspect, i.e., when the police investigator starts interrogating or exacting confession from the suspect in connection with an alleged offense. THE PLACE OF INTERROGATION IS NOT DETERMINATIVE OF THE EXISTENCE OR ABSENCE OF CUSTODIAL INVESTIGATION BUT THE TONE AND MANNER OF QUESTIONING BY THE POLICE AUTHORITIES. Thus, there was custodial investigation when the police authorities, upon their arrest of some of the accused, immediately asked them regarding their
276

277

participation in the commission of the crime , even while they were still walking along the highway on their way to the police station. This is line with the provisions of RA 7438 which makes it applicable even when a person is merely invited for questioning. PEOPLE VS. DANO, G.R. NO. 117690, 339 SCRA 515, SEPT. 1, 2000; PEOPLE VS. MAYORGA, G.R. NO. 135405, 346 SCRA 458, NOVEMBER 29, 2000. However, spontaneous statements voluntarily given, as where appellant orally admitted killing the victim before the barangay captain (who is neither a police officer nor a law enforcement agent), do not fall under custodial investigation. Such admission, even without the assistance of a lawyer, does not violate appellants constitutional rights AND THEREFORE ADMISSIBLE IN EVIDENCE.

PEOPLE VS. DANO, G.R. NO. 117690, 339 SCRA 515, SEPT. 1, 2000; PEOPLE VS. SAMOLDE, G.R. NO. 128551, 336 SCRA 632, JUL. 31, 2000. To be admissible in evidence, an extrajudicial confession must be: (i) voluntary; (ii) made with the assistance of competent and independent counsel; (iii) express; and (iv) in writing. A suspects confession, whether verbal or non-verbal, when taken without the assistance of counsel, without a valid waiver of such assistance, regardless of the absence of coercion or the fact that it had been voluntarily given, is inadmissible in evidence, even if appellants confession were gospel truth.

PEOPLE OF THE PHILIPPINES vs. ROMULO TUNIACO, ET AL., G.R. No. 185710, January 10, 2010 ABAD, J.: This case is about the requirements of a valid extrajudicial confession and the establishment of the existence of corpus delicti in murder cases. The Facts and the Case The city prosecutor of General Santos City charged the accused Romulo Tuniaco, Jeffrey Datulayta, and Alex Aleman with murder before the Regional Trial Court (RTC) of General Santos City in Criminal Case 8370. Based on the findings of the RTC, in the morning of June 13, 1992 some police officers from the Lagao Police Sub-Station requested police officer Jaime
277

278

Tabucon of the Central Police Station of General Santos City homicide division to take the statement of accused Alex Aleman regarding the slaying of a certain Dondon Cortez. On his arrival at the sub-station, Tabucon noted the presence of Atty. Ruperto Besinga, Jr. of the Public Attorneys Office (PAO) who was conversing with those taken into custody for the offense. When queried if the suspects would be willing to give their statements, Atty. Besinga said that they were. Some other police officer first took the statement of accused Jeffrey Datulayta. Officer Tabucon next took the statement of accused Aleman, whom he observed to be in good physical shape. Before anything else, officer Tabucon informed accused Aleman in Cebuano of his constitutional right to remain silent and to the assistance of counsel of his own choice and asked him if he was willing to give a statement. Aleman answered in the affirmative. When asked if he had any complaint to make, Aleman said that he had none. When Aleman said that he had no lawyer, Tabucon pointed to Atty. Besinga who claimed that he was assisting all the suspects in the case. Tabucon warned Aleman that anything he would say may be used against him later in court. Afterwards, the police officer started taking down Alemans statement. Accused Aleman said that in the course of a drinking bout with accused Datulayta and Tuniaco at around 9 p.m. on June 6, 1992, Dondon Cortez threatened to report his drinking companions illegal activities to the police unless they gave him money for his forthcoming marriage. According to Aleman, Datulayta and Tuniaco had already planned to kill Cortez in Tupi, South Cotabato, for making the same threats and now they decided to do it. They got Cortez drunk then led him out supposedly to get the money he needed. The three accused brought Cortez to Apopong near the dump site and, as they were walking, accused Aleman turned on Cortez and stabbed him on the stomach. Accused Datulayta, on the other hand, drew out his single shot homemade M16 pistol and shot Cortez on the head, causing him to fall. Datulayta handed over the gun to Aleman who fired another shot on Cortezs head. Accused Tuniaco used the same gun to pump some bullets into Cortezs body. Then they covered him with rice husks. After taking down the statement, Tabucon explained the substance of it to accused Aleman who then signed it in the presence of Atty. Besinga. On June 15, 1992 the police brought Aleman to the City Prosecutors Office where he swore to his statement before an assistant city prosecutor. In the afternoon, accused Datulayta and Aleman led Tabucon, the city prosecutor, and a police inspector, to the dump site where they left their victims body. After some search, the group found a spot covered with burnt rice husks and a partially burnt body of a man. About a foot from the body, they found the shells of a 5.56 caliber gun and an armalite rifle. On being arraigned, all three accused, assisted by Atty. Besinga, pleaded not guilty to the murder charge. After the prosecution rested its case, accused Tuniaco filed a demurrer to evidence which the Court granted, resulting in the dismissal of the case against him. On being re-arraigned at his request, accused Datulayta pleaded guilty to the lesser offense of Homicide. The trial court sentenced him to imprisonment of six years and one day and to pay P50,000.00 to the victims family.
278

279

For some reason, the trial court had Aleman subjected to psychiatric examination at the Davao Mental Hospital. But, shortly after, the hospital sent word that Aleman had escaped. He was later recaptured. When trial in the case resumed, Alemans new PAO lawyer raised the defense of insanity. This prompted the court to require the Provincial Jail Warden to issue a certification regarding Alemans behavior and mental condition while in jail to determine if he was fit to stand trial. The warden complied, stating that Aleman had been observed to have good mental condition and did not commit any infraction while in jail. Although the prosecution and defense stipulated that Atty. Besinga assisted accused Aleman during the taking of his extrajudicial confession, the latter, however, recanted what he said to the police during the trial. He testified that sometime in 1992, some police officers took him from his aunts house in Purok Palen, Labangal, General Santos City, and brought him to the Lagao police station. He was there asked to admit having taken part in the murder of Cortez. When he refused, they tortured him until he agreed to sign a document admitting his part in the crime. Accused Aleman also testified that he could not remember having been assisted by Atty. Besinga during the police investigation. He even denied ever knowing the lawyer. Aleman further denied prior association with accused Tuniaco and Datulayta. He said that he met them only at the city jail where they were detained for the death of Cortez. On October 8, 2001 the RTC rendered judgment, finding accused Aleman guilty beyond reasonable doubt of the crime charged, and sentenced him to suffer the penalty of reclusion perpetua. The court also ordered him to pay death indemnity of P70,000.00 and moral damages of P50,000.00 to the heirs of Cortez. On appeal to the Court of Appeals (CA) in CA-G.R. CR-HC 00311, the court rendered judgment on January 21, 2008, affirming the decision of the RTC with the modification that directed accused Aleman and Datulayta to indemnify the heirs of Cortez, jointly and severally, in the amounts of P50,000.00 as civil indemnity; P50,000.00 as moral damages; P25,000.00 as temperate damages; and P25,000.00 as exemplary damages. Aleman appealed to this Court. The Issues Presented Accused Aleman raises two issues: a) whether or not the prosecution was able to present evidence of corpus delicti; and b) whether or not accused Alemans extrajudicial confession is admissible in evidence. The Rulings of the Court 1. Corpus delicti has been defined as the body, foundation, or substance of a crime. The evidence of a dead body with a gunshot wound on its back would be evidence that murder has been committed. Corpus delicti has two elements: (a) that a certain result has been established, for example, that a man has died and (b) that some person is criminally responsible for it. The prosecution is burdened to prove corpus delicti beyond reasonable doubt either by direct evidence or by circumstantial or presumptive evidence. The defense claims that the prosecution failed to prove corpus delicti since it did not bother to present a medical certificate identifying the remains found at the dump site and an autopsy report showing such remains sustained gunshot and
279

280

stab wounds that resulted in death; and the shells of the guns used in killing the victim. But corpus delicti need not be proved by an autopsy report of the dead victims body or even by the testimony of the physician who examined such body. While such report or testimony is useful for understanding the nature of the injuries the victim suffered, they are not indispensable proof of such injuries or of the fact of death. Nor is the presentation of the murder weapons also indispensable since the physical existence of such weapons is not an element of the crime of murder. Here, the police authorities found the remains of Cortez at the place pointed to by accused Aleman. That physical confirmation, coming after his testimony of the gruesome murder, sufficiently establishes the corpus delicti of the crime. Of course, that statement must be admissible in evidence. 2. There is no reason for it not to be. Confession to be admissible must be a) voluntary; b) made with the assistance of a competent and independent counsel; c) express; and d) in writing. These requirements were met here. A lawyer, not working with or was not beholden to the police, Atty. Besinga, assisted accused Aleman during the custodial investigation. Officer Tabucon testified that he saw accused Aleman, before the taking of his statement, conversing with counsel at the police station. Atty. Besinga did not dispute this claim. Aleman alleges torture as the reason for the execution of the confession. The appellate court is correct in ruling that such allegation is baseless. It is a settled rule that where the defendant did not present evidence of compulsion, where he did not institute any criminal or administrative action against his supposed intimidators, where no physical evidence of violence was presented, all these will be considered as indicating voluntariness. Here, although Aleman claimed that he bore torture marks on his head, he never brought this to the attention of his counsel, his relatives, or the prosecutor who administered his oath. Accused Aleman claims, citing People v. Galit, that long questions followed by monosyllabic answers do not satisfy the requirement that the accused is amply informed of his rights. But this does not apply here. Tabucon testified that he spoke to Aleman clearly in the language he knew. Aleman, joined by Atty. Besinga, even signed a certification that the investigator sufficiently explained to him his constitutional rights and that he was still willing to give his statement. Further, Aleman asserts that he was lacking in education and so he did not fully realize the consequences of a confession. But as the CA said, no law or jurisprudence requires the police officer to ascertain the educational attainment of the accused. All that is needed is an effective communication between the interrogator and the suspect to the end that the latter is able to understand his rights. This appears to have been done in this case. Moreover, as the lower court noted, it is improbable that the police fabricated Alemans confession and just forced him to sign it. The confession has details that only the person who committed the crime could have possibly known. What is more, accused Datulaytas confession corroborate that of Aleman in important details. Under the doctrine of interlocking confessions, such corroboration is circumstantial evidence against the person implicated in it.
280

281

Custodial Investigation before Bantay Bayan Members requires that the suspect be informed of his Expanded Miranda Rights; otherwise, the evidence obtained shall be inadmissible in evidence.

PEOPLE OF THE PHILIPPINES VS. ANTONIO LAUGA, G.R. No. 186228, March 15, 2010 PEREZ, J.: Consistent with the ruling of this Court in People v. Cabalquinto, the real name and the personal circumstances of the victim, and any other information tending to establish or compromise her identity, including those of her immediate family or household members, are not disclosed in this decision. The Facts In an Information dated 21 September 2000, the appellant was accused of the crime of QUALIFIED RAPE allegedly committed as follows: That on or about the 15th day of March 2000, in the evening, at Barangay xxx, municipality of xxx, province of Bukidnon, Philippines, and within the jurisdiction of this Honorable Court, the above-named accused, being the father of AAA with lewd design, with the use of force and intimidation, did then and there, willfully, unlawfully and criminally have carnal knowledge with his own daughter AAA, a 13 year[s]old minor against her will. On 12 October 2000, appellant entered a plea of not guilty. During the pre-trial conference, the prosecution and the defense stipulated and admitted: (a) the correctness of the findings indicated in the medical certificate of the physician who examined AAA; (b) that AAA was only thirteen (13) years old when the alleged offense was committed; and (c) that AAA is the daughter of the appellant. On trial, three (3) witnesses testified for the prosecution, namely: victim AAA; her brother BBB; and one Moises Boy Banting, a bantay bayan in the barangay. Their testimonies revealed the following: In the afternoon of 15 March 2000, AAA was left alone at home. AAAs father, the appellant, was having a drinking spree at the neighbors place. Her mother decided to leave because when appellant gets drunk, he has the habit of mauling AAAs mother. Her only brother BBB also went out in the company of some neighbors. At around 10:00 oclock in the evening, appellant woke AAA up; removed his pants, slid inside the blanket covering AAA and removed her pants and underwear; warned her not to shout for help while threatening her with his fist; and told her that he had a knife placed above her head. He proceeded to mash her breast, kiss her repeatedly, and inserted his penis inside her vagina. Soon after, BBB arrived and found AAA crying. Appellant claimed he scolded her for staying out late. BBB decided to take AAA with him. While on their way to their maternal grandmothers house, AAA recounted her harrowing
281

282

experience with their father. Upon reaching their grandmothers house, they told their grandmother and uncle of the incident, after which, they sought the assistance of Moises Boy Banting. Moises Boy Banting found appellant in his house wearing only his underwear. He invited appellant to the police station, to which appellant obliged. At the police outpost, he admitted to him that he raped AAA because he was unable to control himself. The following day, AAA submitted herself to physical examination. Dra. Josefa Arlita L. Alsula, Municipal Health Officer of x x x, Bukidnon, issued the Medical Certificate, which reads: hyperemic vulvae with 4 oclock & 6 oclock freshly lacerated hymen; (+) minimal to moderate bloody discharges 2 to an alleged raping incident On the other hand, only appellant testified for the defense. He believed that the charge against him was ill-motivated because he sometimes physically abuses his wife in front of their children after engaging in a heated argument, and beats the children as a disciplinary measure. He went further to narrate how his day was on the date of the alleged rape. The lone assignment of error in the appellants brief is that, the trial court gravely erred in finding him guilty as charged despite the failure of the prosecution to establish his guilt beyond reasonable doubt, because: (1) there were inconsistencies in the testimonies of AAA and her brother BBB; (2) his extrajudicial confession before Moises Boy Banting was without the assistance of a counsel, in violation of his constitutional right; and (3) AAAs accusation was ill-motivated. HELD Appellant contests the admissibility in evidence of his alleged confession with a bantay bayan and the credibility of the witnesses for the prosecution. Admissibility in Evidence of an Extrajudicial Confession before a Bantay Bayan Appellant argues that even if he, indeed, confessed to Moises Boy Banting, a bantay bayan, the confession was inadmissible in evidence because he was not assisted by a lawyer and there was no valid waiver of such requirement. The case of People v. Malngan is the authority on the scope of Miranda doctrine provided for under Article III, Section 12(1) and (3) of Constitution. In Malngan, appellant questioned the admissibility of extrajudicial confessions given to the barangay chairman and a neighbor of private complainant. This Court distinguished. Thus: Arguably, the barangay tanods, including the Barangay Chairman, in this particular instance, may be deemed as law enforcement officer for purposes of applying Article III, Section 12(1) and (3), of the Constitution. When accusedappellant was brought to the barangay hall in the morning of 2 January 2001, she was already a suspect, actually the only one, in the fire that destroyed several houses x x x. She was, therefore, already under custodial investigation and the rights guaranteed by
282

the the her the

283

x x x [the] Constitution should have already been observed or applied to her. Accused-appellants confession to Barangay Chairman x x x was made in response to the interrogation made by the latter admittedly conducted without first informing accused-appellant of her rights under the Constitution or done in the presence of counsel. For this reason, the confession of accused-appellant, given to Barangay Chairman x x x, as well as the lighter found x x x in her bag are inadmissible in evidence against her x x x. [But such does] not automatically lead to her acquittal. x x x [T]he constitutional safeguards during custodial investigations do not apply to those not elicited through questioning by the police or their agents but given in an ordinary manner whereby the accused verbally admits x x x as x x x in the case at bar when accusedappellant admitted to Mercedita Mendoza, one of the neighbors x x x [of the private complainant]. (Emphasis supplied) Following the rationale behind the ruling in Malngan, this Court needs to ascertain whether or not a bantay bayan may be deemed a law enforcement officer within the contemplation of Article III, Section 12 of the Constitution. In People of the Philippines v. Buendia, this Court had the occasion to mention the nature of a bantay bayan, that is, a group of male residents living in [the] area organized for the purpose of keeping peace in their community[,which is] an accredited auxiliary of the x x x PNP. Also, it may be worthy to consider that pursuant to Section 1(g) of Executive Order No. 309 issued on 11 November 1987, as amended, a Peace and Order Committee in each barangay shall be organized to serve as implementing arm of the City/Municipal Peace and Order Council at the Barangay level. The composition of the Committee includes, among others: (1) the Punong Barangay as Chairman; (2) the Chairman of the Sangguniang Kabataan; (3) a Member of the Lupon Tagapamayapa; (4) a Barangay Tanod; and (5) at least three (3) Members of existing Barangay-Based Anti-Crime or neighborhood Watch Groups or a Non Government Organization Representative well-known in his community. This Court is, therefore, convinced that barangay-based volunteer organizations in the nature of watch groups, as in the case of the bantay bayan, are recognized by the local government unit to perform functions relating to the preservation of peace and order at the barangay level. Thus, without ruling on the legality of the actions taken by Moises Boy Banting, and the specific scope of duties and responsibilities delegated to a bantay bayan, particularly on the authority to conduct a custodial investigation, any inquiry he makes has the color of a state-related function and objective insofar as the entitlement of a suspect to his constitutional rights provided for under Article III, Section 12 of the Constitution, otherwise known as the Miranda Rights, is concerned. We, therefore, find the extrajudicial confession of appellant, which was taken without a counsel, inadmissible in evidence. Be that as it may, We agree with the Court of Appeals that the conviction of the appellant was not deduced solely from the assailed extrajudicial confession but from the confluence of evidence showing his guilt beyond reasonable doubt.
283

284

Voluntary and spontaneous confession of a suspect who is already under custody of the police is admissible in evidence even in the absence of counsel. PEOPLE OF THE PHILIPPINES VS. VICTOR VILLARINO, G.R.NO. 185012, MARCH 5, 2010 FACTS: On April 28, 1995, BBB, together with her 10-year old daughter AAA and her younger son CCC went to the house of their relative in Barangay D to attend the fiesta to be held the next day. On even date, from 7:00 oclock to 9:00 oclock in the evening, SPO4 Jesus Genoguin (SPO4 Genoguin) was in his house in Barangay D entertaining his guests, one of whom was appellant. While personally serving food and drinks to appellant, SP04 Genoguin noticed that the latter was wearing a bracelet and a necklace with pendant. Appellant even allowed SPO4 Genoguin to put on the bracelet. On April 29, 1995, at around 9:00 oclock in the morning, the appellant who was on his way to Barangay D, passed by the house of Rodrigo Olaje (Rodrigo). At that time, Rodrigo noticed appellant wearing a bracelet and a necklace with pendant. He was also wearing a white sleeveless t-shirt (sando). At 11:00 oclock in the morning, appellant was at the house of BBBs aunt. BBB offered him food. BBB also noticed that he was dressed in a white sando and that he wore jewelry consisting of a bracelet and a necklace with pendant. At 1:00 oclock in the afternoon, he was seen wearing the same sando and jewelry while drinking at the basketball court in Barangay D. At around 3:00 oclock in the afternoon, BBB told AAA to go home to Barangay D1 to get a t-shirt for her brother. AAA obeyed. However, she no longer returned. While BBB was anxiously waiting for AAA in the house of her aunt in Barangay D, she received information that a dead child had been found in Barangay D1. She proceeded to the area where she identified the childs body as that of her daughter, AAA. At around 4:00 oclock in the afternoon, Rodrigo, who was the barangay captain of Barangay D1 received information that a dead child was found in their barangay. He instructed a barangay tanod to inform the police about the incident. Thereafter, Rodrigo proceeded to the specified area together with other barangay tanods. SPO4 Genoguin also went to the crime scene after being informed by his commander. Upon arrival, he saw the corpse of a little girl behind a big boulder that was about 10 meters away from the trail junction of the barangays. People had gathered seven to 10 meters away from the dead body, but no one dared to approach. AAAs lifeless body lay face up with her buttocks on top of a small rock. Her body was slanted downward with her legs spread apart and dangling on the sides of the small boulder. She was no longer wearing short pants and panty, and blood oozed from her vagina. Wrapped around her right hand, which was positioned near her right ear, was a white sando. AAAs panty was found a meter away from her body, while her short pants was about two meters farther. A bracelet and a pendant were also recovered from the crime
284

285

scene. Rodrigo and BBB identified these pieces of jewelry as those seen on the appellant. They also identified the sando on AAAs arm as the appellants. Thus, the hunt for appellant began. On the same day, the appellant was found in the house of Aurelia Susmena near the seashore of Barangay D1. He was drunk and violent. He resisted arrest and had to be bodily carried to the motorboat that would take him to the municipal building in Almagro, Samar. The arresting team made the appellant take off his clothes since they were wet. When he complied, his briefs revealed bloodstains. On May 2, 1995, the police brought appellant to Calbayog City for medical examination since he had scratches and abrasions on his body. While waiting for a boat ride at 4:00 oclock in the morning, the police team took a coffee break. SPO4 Genoguin was momentarily left alone to guard the appellant. During this short period, the appellant voluntarily admitted to SPO4 Genoguin that he committed the crime charged. He also told SPO4 Genoguin that he could keep the pendant and bracelet if he would retrieve the t-shirt and throw it into the sea. SPO4 Genoguin rejected the appellants offer and reminded him of his right to a counsel and that everything the appellant said could be used against him in court. Unperturbed, the appellant reiterated his offer. When they boarded the motorboat, the appellant repeatedly offered to give SPO4 Genoguin P20,000.00 if he would throw the sando into the sea. However, the police officer ignored the offer and instead reported the matter to the Chief of Police of Almagro, SPO4 Basilio M. Yabao. Later, the appellants mother, Felicidad Mabute y Legaspi, asked him not to testify against her son. At the Calbayog District Hospital, Senior Resident Physician Dr. Jose V. Ong, found that appellants body had 10 healed abrasions and two linear abrasions or scratches, particularly, on his breast, knees, as well as right and left ears, that could have been caused by fingernails. On August 3, 1995, an Information was filed charging appellant Victor Villarino y Mabute with the special complex crime of rape with homicide. The Information contained the following accusatory allegations: That on or about the 29th day of April, 1995, at about 5:00 oclock in the afternoon, at Barangay D1, Municipality of Almagro, Province of Samar, Philippines, and within the jurisdiction of this Honorable Court, the above named accused, with lewd design, by means of force, violence and intimidation, did then and there, willfully, unlawfully and feloniously have carnal knowledge against a minor ten (10) years [sic], AAA, without the latters consent and against her will, and thereafter, with deliberate intent to kill, did then and there willfully, unlawfully and feloniously inflict upon the said AAA mortal wounds on x x x different parts of her body, which caused her untimely death. CONTRARY TO LAW. Appellant pleaded not guilty to the crime charged. After the termination of the pre-trial conference, trial ensued. The RTC found him guilty beyond reasonable doubt of the complex crime of Rape with Homicide and sentenced to Death. ISSUE: Is accused-appellants voluntary confession to SPO4 Genoguin admissible in evidence?
285

286

HELD: In the instant case, appellant voluntarily confessed to raping and killing AAA to SPO4 Genoguin. He even offered to give the pieces of jewelry to the latter if his sando is thrown into the sea. The appellant did not deny this accusation nor assail its truthfulness. When appellant confessed to the crime, he was alone with SPO4 Genoguin, and no force or intimidation was employed against him. The confession was spontaneously made and not elicited through questioning. The trial court did not, therefore, err in holding that compliance with the constitutional procedure on custodial interrogation is not applicable in the instant case. In People v. Dy, we held that: Contrary to the defense contention, the oral confession made by the Accused to Pat. Padilla that he had shot a tourist and that the gun he had used in shooting the victim was in his bar which he wanted surrendered to the Chief of Police (t.s.n., October 17, 1984, pp. 6-9), is competent evidence against him. The declaration of an accused acknowledging his guilt of the offense charged may be given in evidence against him (Sec. 29, Rule 130, Rules of Court). It may in a sense be also regarded as part of the res gestae. The rule is that, any person, otherwise competent as a witness, who heard the confession, is competent to testify as to the substance of what he heard if he heard and understood all of it. An oral confession need not be repeated verbatim, but in such a case it must be given in substance (23 C.J.S. 196, cited in People v. Tawat, G.R. No. 62871, May 25, 1985, 129 SCRA 431). What was told by the Accused to Pat. Padilla was a spontaneous statement not elicited through questioning, but given in an ordinary manner. No written confession was sought to be presented in evidence as a result of formal custodial investigation. (People v. Taylaran, G.R. No. L-19149, October 31, 1981, 108 SCRA 373). The Trial Court, therefore, cannot be held to have erred in holding that compliance with the constitutional procedure on custodial interrogation is not applicable in the instant case, as the defense alleges in its Error VII. At any rate, even without his confession, appellant could still be convicted of the complex crime of rape with homicide. The prosecution established his complicity in the crime through circumstantial evidence which were credible and sufficient, and led to the inescapable conclusion that the appellant committed the complex crime of rape with homicide. When considered together, the circumstances point to the appellant as the culprit.

2. Guidelines for police investigation Read: 1.Escobedo vs. Illinois, 378 US 478 2. Miranda vs. Arizona, 384 US 436 3. P. vs. Duero, 104 SCRA 379 2-a. Duties of the Police or Arresting Officers Read:
286

287

1. P. vs. Matos-Viduya, Sept. 11, 1990 1-a. P vs. Nicandro, 141 SCRA 289 2. P vs. Duhan, 142 SCRA 100 3. P vs. Caguioa, 95 SCRA 2 4. P vs. Ramos, 122 SCRA 312 3. To be informed of the Right to remain silent; cases Read: 1. Constitutional right to remain silent,104 SCRA 1-a. People vs. Marcos Jimenez, Dec. 10, 1991 Extrajudicial confession; counsel of choice Right to counsel during custodial investigation; while making an extrajudicial confession PEOPLE VS. PATUNGAN, 354 SCRA 413 The accused was under coercive and uncounselled custodial investigation by the police without a lawyer for 2 and a half days . Then, he was brought to the IBP Office where a lawyer assisted him in his extrajudicial confession. 391 in particular

The mere presence of a lawyer is not sufficient compliance with the constitutional requirement of assistance of counsel. Assistance of counsel must be effective, vigilant and independent. A lawyer who could just hear the investigation going on while working on another case hardly satisfies the minimum requirements of effective assistance of counsel. Not only was the accused subjected to custodial investigation without counsel, he was likewise denied effective assistance of counsel during the taking of his extra-judicial confession.

PEOPLE V. JIMENEZ G.R.No. 82604. December 10, 1991 NARVASA, J.: FACTS: On August 13, 1985, police authorities, acting upon a report, came upon the corpse of Pelagio Jimenez below a cliff near a balite tree. The police investigators learned that Marcos, the son of the deceased Pelagio Jimenez told his mother that his father had not come home the previous night: that the search for the deceased, who was living separately from them, commenced a day earlier but it was not until the morning of the following day, August 13, 1985, that deceased Pelagio was finally found dead. They also learned from the persons they interviewed of
287

288

circumstances that drew their suspicion to the son, Marcos and Robert, such as; the bathing at the artesian well "as if washing away stains of blood"; the deceased's violent quarrels with his children and occasions that he had been boxed and hit by his children. The police had invited the deceased's widow and her sons for questioning about the killing. A draft of the confession was prepared by the investigating officer but Marcos was not able to sign the same due to the absence of the judge before whom it is supposed to be sworn and signed. Marcos agreed to come back and sign his statement, but upon his return, he, assisted by a former judge whose presence was requested by the police authorities, refused to sign his statement. Subsequently, an information for parricide was filed against the widow and her sons, Marcos, Robert, and Wilkins. In an order dated July 21, 1986, the trial court absolved the widow and Wilkins of any participation in the filling for lack of proof. On December 12. 1986, the trial court found Marcos and Robert guilty beyond reasonable doubt of the crime of parricide, noting that the unsigned confession is admissible in evidence inasmuch as evidence aliunde corroborated such confession. Both accused contest such ruling. Hence this appeal. ISSUE: Is the extrajudicial confession of Marcos admissible in evidence? HELD: No. Decision reversed. Section 12 (1), Article III OF THE 1987 Constitution declares that a person being investigated by the police as a suspect in an offense has the right, among others, (1) to have a competent and independent counsel of his own choice and if he cannot afford the services of counsel, he must be provided with one; and that (2) said right cannot be waived except in writing and in the presence of counsel. The lawyer who assists the suspect under custodial interrogation should be of the latter's own choice, not one foisted on him by the police investigators or other parties. In this case, the former judge whose assistance was requested by the police was evidently not of Marcos Jimenez' own choice; she was the police officers' own choice; she did not ask Marcos if was is willing to have her represent him. This is not the mode of solicitation of legal assistance contemplated by the constitution. Furthermore, the former judge was not present when Marcos was being interrogated by the police. While she asked him if he had voluntarily given the statements contained in the typewritten document, this is far from being substantial compliance with the constitutional duty of police investigators during custodial interrogation. The typewritten confession is unsigned and was in fact expressly rejected by Marcos. Hence, the supposed waiver made therein of his constitutional right to counsel of his own choice. Neither can the confession prejudice his co-accused, his brother Robert, not only because it was obtained in violation of the constitution but also because of the principle of res inter alios acta.
288

289

The interrogation of Marcos Jimenez having been conducted without the assistance of counsel, and no valid waiver of such right to counsel have been made, not only the confession but also any admissible obtained in the course thereof are inadmissible against him or his co accused. In view of the inadmissibility in evidence of the confession, the rest of the evidence of the prosecution is inadequate to overcome the presumption of innocence raised by the fundamental law in favor of both the accused. Extrajudicial confession without the assistance of counsel, inadmissible as evidence; exception PEOPLE VS. PANFILO CABILES, 284 SCRA 199; PEOPLE VS. TAN, 286 SCRA 207 Melo, J. Even if the confession of the accused speaks of the truth, if it was made without the assistance of counsel, it is inadmissible in evidence regardless of the absence of coercion or even if it was voluntarily given. In order that a confession is admissible, the following requisites must be present: a. the confession must be voluntary; b. the confession must be made with the assistance of a competent and independent counsel; c. the confession must be express; and d. the confession must be in writing. The above requirements, however, are not applicable when the suspect makes an spontaneous statement, not elicited through questioning by the authorities, BUT GIVEN IN AN ORDINARY MANNER WHEREBY THE ACCUSED ORALLY ADMITTED HAVING COMMITTED THE CRIME. This was the decision of the Supreme Court in the case of PEOPLE VS. ANDAN, March 3, 1997 when the accused made a voluntary and verbal confession to the Municipal Mayor that he committed the crime imputed to him. As such, his uncounselled confession is admissible in evidence. PEOPLE VS. OBRERO, 332 SCRA 190 Mendoza, J. There are two (2) kinds of involuntary or coerced confessions under Art. III, Section 12 of the Constitution. These are: a. confession which are the product of third degree methods such as torture, force, violence, threat, intimidation; and b. those which are given without the benefit of Miranda Warnings. There is no compliance of the constitutional requirement of competent and independent counsel to assist an accused during custodial investigation when the accused was assisted by the Station Commander of the WPD, Atty. De los Reyes, while being investigated by other policemen of the same police station because the interest of the police is naturally adverse to the accused. In fact, the SC in the case of PEOPLE VS.
289

290

JANUARIO, 267 SCRA 608 held that a lawyer applying for a position in the NBI could not validly assist an accused being investigated then by the NBI. 1-b. P. vs. Aspili, November 21, 1990 1-c. People vs. Judge Ayson, 175 SCRA 216 (Confession made to the officials of Philippine Airlines during an investigation is admissible in evidence despite the fact that he was not informed of his rights during custodial investigations since said officials are not bound by the requirements of Section 12, Art. III of the Constitution) 1-d. P. vs. Pinlac, 165 SCRA 675 1-e. People vs. Loveria, 187 SCRA 47 1-f. Gamboa vs. Judge Cruz, 162 SCRA 675 2. P. vs. Galit, 135 SCRA 465 3. P vs. Alegre, 94 SCRA 109 4. Draculan vs. Donato, 85 SCRA 266 5. P. vs. Borromeo, June 29,l983 6. P vs. Camalog, GR No. 77116, January 31, 1989 (Including the duty of Police Officers in connection with said right) 7. P vs. Cui, Jr., 162 SCRA 220 3-a. How about if the accused gives an spontaneous statement before he could be advised of his right to remain silent? Read: Aballe vs. People, 183 SCRA 196 3-b. When shall the constitutional rights of the above demandable? During police line-up? Read: 1. P vs. Usman Hassan, 157 SCRA 261 2. Gamboa vs. Judge Cruz, 162 SCRA 642 3. DE LA TORRE VS. CA, 294 SCRA 196 4. PEOPLE VS. HATTON The right to counsel; accused as mentioned

PEOPLE VS. JEREZ, 285 SCRA 393 A lawyer provided by the investigators to the accused during the custodial investigation is deemed engaged by the accused where he never raised any objection against the formers appointment during the course of the investigation AND THE ACCUSED THEREAFTER SUBSCRIBES TO THE VERACITY OF HIS STATEMENT BEFORE THE SWEARING OFFICER. (NOTE: In the case of PEOPLE VS. JUANERIO, February 7, 1997, the SC held that a lawyer who was at the NBI Office applying for a position therein and who was appointed as counsel for a suspect being then investigated by the NBI could not be considered as the competent and independent counsel referred to in the Constitution especially so that later on, said lawyer was appointed by the NBI as one of its agents.)
290

291

The next case is very important. It diminishes the right to counsel during custodial investigation and makes the work of the investigator easier to make the confession of a suspect admissible as evidence. It is obviously a reversal of the People vs. Juanerio ruling. RIGHT TO A COMPETENT AND INDEPENDENT COUNSEL OF HIS OWN CHOICE. This case is different form the previous doctrines. PEOPLE OF THE PHILIPPINES VS. DOMINGO REYES, ET AL., G.R. No. 178300, March 17, 2009 CHICO-NAZARIO, J.: On 11 August 1999, an Information193[4] was filed before the RTC charging appellants with the special complex crime of kidnapping for ransom with homicide. The accusatory portion of the information reads: The undersigned State Prosecutor of the Department of Justice hereby accuses Domingo Reyes y Paje, Alvin Arnaldo y Avena and Joselito Flores y Victorio of the crime of kidnapping for ransom with homicide defined and penalized under Article 267 of the Revised Penal Code, as amended, committed as follows: That on or about 11:00 p.m. on July 16, 1999, at Sitio Lambakin, barangay Sto. Cristo, San Jose del Monte, Bulacan, Philippines and within the jurisdiction of this Honorable Court, the above-named accused conspiring, confederating and mutually helping one another and grouping themselves together with Juanito Pataray y Cayaban, Federico Pataray y Cabayan and Rommel Libarnes y Acejo, who are still at large, did then and there willfully, unlawfully and feloniously, by means of force and intimidation and with use of firearms, carry away and deprive Robert Yao, Yao San, Chua Ong Ping Sim, Raymond Yao, Ronald Matthew Yao, Lennie Yao, Charlene Yao, Jona Abagatnan ang Josephine Ortea against their will and consent on board their Mazda MVP van for the purpose of extorting money in the amount of Five Million Pesos (P5,000,000.00), that during the detention of Chua Ong Ping Sim and Raymong Yao, said accused with intent to kill, willfully and unlawfully strangled Chua Ong Ping Sim and Raymond Yao to death to the damage and prejudice of their heirs in such amount as may be awarded to them by this Honorable Court. The prosecution presented as witnesses Jona Abagatnan (Abagatnan), Robert Yao (Robert), Yao San, Police Officer 3 (PO3) Alex Alberto, PO3 Roberto Jabien, Atty. Florimond Rous (Atty. Rous) and Atty. Carlo Uminga (Atty. Uminga). Their testimonies, taken together, attest to the following: The Yao family is composed of Yao San (father), Chua Ong Ping Sim (mother), Robert and Raymond (children), Lenny (daughter-in-law, wife of Robert), Matthew and Charlene (grandchildren), and Jona Abagatnan and Josephine Ortea (housemaids). The Yao family owns and

193[4]

Records, pp. 42-43.

291

292

operates a poultry farm in Barangay Santo Cristo, San Jose del Monte, Bulacan. On 16 July 1999, at about 11:00 p.m., the Yao family, on board a Mazda MVP van, arrived at the their poultry farm in Barangay Sto. Cristo, San Jose del Monte, Bulacan. Yao San alighted from the van to open the gate of the farm. At this juncture, appellant Reyes and a certain Juanito Pataray (Pataray) approached, poked their guns at Yao San, and dragged him inside the van. Appellant Reyes and Pataray also boarded the van. Thereupon, appellants Arnaldo and Flores, with two male companions, all armed with guns, arrived and immediately boarded the van. Appellant Flores took the drivers seat and drove the van. Appellants Reyes and Arnaldo and their cohorts then blindfolded each member of the Yao family inside the van with packaging tape.194[6] After about 30 minutes of traveling on the road, the van stopped. Per order of appellants and their cohorts, Chua Ong Ping Sim, Robert, Raymond and Jona Abagatnan (Abagatnan) stepped out of the van with appellants Reyes and Arnaldo, Pataray and one of their male companions.195[7] Appellant Flores, with the other male companion, drove the van with the remaining members of the Yao family inside the vehicle.196[8] Later, the van stopped again. Appellant Flores and his male companion told Yao San to produce the amount of five million pesos (P5,000,000.00) as ransom in exchange for the release of Chua Ong Ping Sim, Robert, Raymond and Abagatnan. Thereafter, appellant Flores and his male companion left the van and fled; while Yao San, Lenny, Matthew, Charlene and Josephine remained inside the van. Upon sensing that the kidnappers had already left, Yao San drove the van towards the poultry farm and sought the help of relatives.197[9] Meanwhile, Chua Ong Ping Sim, Robert, Raymond and Abagatnan were taken on foot by appellants Reyes and Arnaldo, Pataray and one male companion to a safe-house situated in the mountainous part of San Jose Del Monte, Bulacan where they spent the whole night.198[10] On the morning of the following day, at around 4:00 a.m., appellants and their cohorts tried to contact Yao San regarding the ransom demanded, but the latter could not be reached. Thus, appellants instructed Abagatnan to look for Yao San in the poultry farm. Appellants Reyes and Arnaldo and one male companion escorted Abagatnan in proceeding to the poultry farm. Upon arriving therein, Abagatnan searched for Yao San, but the latter could not be found. Appellants Reyes and Arnaldo told Abagatnan to remind Yao San about the ransom demanded. Thereafter, appellants Reyes and Arnaldo and their male companion left Abagatnan in the poultry farm and went back to the safe-house.199[11] In the safe-house, appellants told Robert that they would release him so he could help Abagatnan in locating Yao San. Robert and
194[6]

195[7] 196[8] 197[9] 198[10]

199[11]

TSN, 26 October 1999, pp. 3-14; TSN, 11 August 2000, pp. 3-7; TSN, 21 September 2000, pp. 28. TSN, 26 October 1999, pp. 16-17; TSN, 11 August 2000, p. 7. Records, p. 34. Id. TSN, 26 October 1999, pp. 16-23; TSN, 7 December 1999, pp. 2-5; TSN, 11 August 2000, pp. 89. TSN, 7 December 1999, pp. 4-7.

292

293

appellants left the safe-house, and after 30 minutes of trekking, appellants abandoned Robert. Robert then ran towards the poultry farm. Upon arriving at the poultry farm, Robert found Yao San and informed him about the ransom demanded by the appellants. Robert also told Yao San that Chua Ong Ping Sim and Raymond were still held by appellants and their cohorts.200[12] On 18 July 1999, appellants called Yao San through a cellular phone and demanded the ransom of P5 million for Chua Ong Ping Sim and Raymond. Yao San acceded to appellants demand. Appellants allowed Yao San to talk with Chua Ong Ping Sim.201[13] On the morning of 19 July 1999, appellants again called Yao San via a cellular phone and threatened to kill Chua Ong Ping Sim and Raymond because of newspaper and radio reports regarding the incident. Yao San clarified to appellants that he did not report the incident to the police and also pleaded with them to spare the life of Chua Ong Ping Sim and Raymond. Appellants then instructed Yao San to appear and bring with him the ransom of P5 million at 3:00 p.m. in the Usan dumpsite, Litex Road, Fairview, Quezon City. Yao San arrived at the designated place of the pay-off at 4:00 p.m., but none of the appellants or their cohorts showed up. Yao San waited for appellants call, but none came. Thus, Yao San left.202[14] On 23 July 1999, the corpses of Chua Ong Ping Sim and Raymond were found at the La Mesa Dam, Novaliches, Quezon City.203[15] Both died of asphyxia by strangulation.204[16] On 26 July 1999, appellant Arnaldo surrendered to the Presidential Anti-Organized Crime Task Force (PAOCTF) at Camp Crame, Quezon City. Thereupon, appellant Arnaldo, with the assistance of Atty. Uminga, executed a written extra-judicial confession narrating his participation in the incident. Appellant Arnaldo identified appellants Reyes and Flores, Pataray and a certain Tata and Akey as his co-participants in the incident. Appellant Arnaldo also described the physical features of his cohorts and revealed their whereabouts.205[17] Subsequently, appellant Reyes was arrested in Sto. Cristo, San Jose del Monte, Bulacan. Thereafter, appellants Arnaldo and Reyes were identified in a police line-up by Yao San, Robert and Abagatnan as their kidnappers.206[18] On 10 August 1999, agents of the PAOCTF arrested appellant Flores in Balayan, Batangas. Afterwards, appellant Flores, with the assistance of Atty. Rous, executed a written extra-judicial confession detailing his participation in the incident. Appellant Flores identified appellants Reyes and Arnaldo, Pataray and a certain Tata and Akey as his co-participants in the incident. Appellant Flores was subsequently identified in a police line-up by Yao San, Robert and Abagatnan as one of their kidnappers.207[19]
200[12] 201[13] 202[14] 203[15] 204[16] 205[17] 206[18] 207[19]

Id. at 7-8; TSN, 11 August 2000, pp. 10-12. Records, p. 35. Id; TSN, 11 August 2000, pp. 12-14. TSN, 7 December 1999, pp. 8-9; TSN, 11 August 2000, pp. 14-15; Records, p. 35. Records, pp. 15-17. Id. at 5, 8, 12, & 24-28. Id. at 13-14 & 33, 35, & 38. Id. at 46-48, 63-64 & 302-306.

293

294

For its part, the defense presented the testimonies of appellants, Marina Reyes, Irene Flores Celestino, Wilfredo Celestino, Jr., Rachel C. Ramos, and Isidro Arnaldo. Appellants denied any liability and interposed alibis and the defense of frame-up. Their testimonies, as corroborated by their witnesses, are as follows: Appellant Arnaldo testified that he was an asset of the PAOCTF. He narrated that on 25 July 1999, while he was at the tricycle terminal of Brgy. Sto. Cristo, San Jose del Monte, Bulacan, a police officer named Liwanag of the PAOCTF approached and invited him to go to Camp Crame to shed light on a kidnapping case allegedly committed by a certain Brgy. Captain Ramos and by members of the Aguirre and Bautista families. He accepted the invitation. Subsequently, he proceeded to Camp Crame and met therein Colonel Cesar Mancao III (Colonel Mancao) of the PAOCTF. Colonel Mancao told him that the PAOCTF would arrest Brgy. Capt. Ramos and certain persons named Gerry Bautista and Dadie Bautista. Colonel Mancao instructed him to identify said persons as responsible for the kidnapping of the Yao family. He refused to do so because he feared Brgy. Capt. Ramos. The day after, Colonel Mancao called appellant Arnaldo to his office. Upon arriving thereat, the latter saw Yao San. Yao San promised him that if their kidnappers would be apprehended through his cooperation, he would give him P500,000.00. He accepted Yao Sans offer under the condition that he would identify a different set of suspects. Later, Colonel Mancao gave him P30,000.00.208[31] Subsequently, he pointed to appellants Reyes and Flores as his cohorts in kidnapping the Yao family. He implicated appellants Reyes and Flores to get even with them, since the two had previously mauled him after he sold their fighting cocks and failed to give them the proceeds of the sale.209[32] He denied having met with Atty. Uminga. He was not assisted by the latter when he was forced by the PAOCTF to make a written extrajudicial confession on the kidnapping of the Yao family. Further, he claimed that while he was under the custody of PAOCTF, a certain Major Paulino utilized him as a drug pusher. Upon failing to remit the proceeds of the drug sale, he was beaten up by PAOCTF agents and thereafter included as accused with appellants Reyes and Flores for the kidnapping of the Yao family.210[33] On the other hand, appellant Reyes testified that he slept in his house with his family from 6:00 p.m. of 16 July 1999 until the morning of the next day; that on the early morning of 26 July 1999, five policemen barged into his house and arrested him; that the policemen told him that he was a suspect in the kidnapping of the Yao family; that he was mauled by the policemen outside his house; that the policemen forcibly brought him to Camp Crame, where he was subsequently tortured; that he knew the Yao family because he worked as a carpenter in the familys poultry farm at Brgy. Sto. Cristo, San Jose del Monte, Bulacan; that he had no involvement in the kidnapping of the family; and that appellant Arnaldo

208[31] 209[32] 210[33]

TSN, 7 June 2001, pp. 3-21. TSN, 10 July 2001, pp. 3-6. Id. at 10-16; TSN, 21 August 2001, pp. 3-14.

294

295

implicated him in the kidnapping of the family because appellant Arnaldo held a grudge against him.211[34] For his part, appellant Flores testified that he stayed in his sisters house at Antipolo City from 12 July 1999 up to 30 July 1999; that he went to her house on 12 July 1999 because it was the birthday of her child; that he worked as a construction worker during his stay in his sisters house; that he was arrested in Batangas and thereafter brought to Camp Crame, where he was beaten up by policemen for refusing to admit involvement in the kidnapping of the Yao family; that after three days of beating, he was forced to sign a document which he later found out to be a written extrajudicial confession; that he never met nor did he know Atty. Rous; that he knew the Yao family because he lived near the familys poultry farm, and he used to work therein as a welder; that he had no participation in the kidnapping of the family; and that appellant Arnaldo implicated him in the kidnapping of the family because he and appellant Reyes had mauled appellant Arnaldo several years ago.212[35] The defense proffered documentary and object evidence to buttress their foregoing claims, to wit: (1) prayer booklet of appellant Arnaldo (Exhibit 1 for appellant Arnaldo);213[36] (2) calling card of Colonel Mancao (Exhibit 2 for appellant Arnaldo);214[37] and (3) pictures allegedly showing appellant Flores working as a carpenter in Antipolo City (Exhibits 1 & 2 for appellant Flores).215[38] After trial, the RTC rendered a Decision dated 26 February 2002 convicting appellants of the special complex crime of kidnapping for ransom with homicide and sentencing each of them to suffer the supreme penalty of death. Appellants were also ordered to pay jointly and severally the Yao family P150,000.00 as civil indemnity, P500,000.00 as moral damages and the costs of the proceedings. The dispositive portion of the RTC Decision reads: WHEREFORE, finding herein three (3) accused DOMINGO REYES y PAJE, ALVIN ARNALDO y AVENA, and JOSELITO FLORES y VICTORIO guilty as principals beyond reasonable doubt of the crime of KIDNAPPING FOR RANSOM WITH (DOUBLE) HOMICIDE as charged, they are hereby sentenced each to suffer the supreme penalty of DEATH as mandated by law, to jointly and severally indemnify the heirs of deceased Chua Ong Ping Sim and Raymond Yao in the amount of One Hundred Fifty Thousand Pesos (P150,000.00), and all the private offended parties or victims, including the heirs of the deceased, in the amount of Five Hundred Thousand Pesos (P500,000.00) as moral damages, subject to the corresponding filing fee as a first lien, and to pay the costs of the proceedings.216[39] By reason of the death penalty imposed on each of the appellants, the instant case was elevated to us for automatic review. However, pursuant to our ruling in People v. Mateo,217[40] we remanded the instant case to the Court of Appeals for proper disposition.
211[34] 212[35] 213[36] 214[37] 215[38] 216[39] 217[40]

TSN, 6 March 2001, pp. 3-10. TSN, 24 May 2001, pp. 2-9. Records, Volume VI, Index of Exhibits. Id. Records, p. 357. CA rollo, p. 61. G.R. Nos. 147678-87, 7 July 2004, 433 SCRA 640.

295

296

On 14 August 2006, the Court of Appeals promulgated its Decision affirming with modifications the RTC Decision. The appellate court reduced the penalty imposed by the RTC on each of the appellants from death penalty to reclusion perpetua without the possibility of parole. It also decreased the amount of civil indemnity from P150,000.00 to P100,000.00. Further, it directed appellants to pay jointly and severally the Yao family P100,000.00 as exemplary damages. The fallo of the Court of Appeals decision states: WHEREFORE, premises considered, the Decision of the Regional Trial Court of Malolos, Bulacan, Branch 12, dated February 26, 2002, in Criminal Case No. 1611-M-99 convicting accused-appellants of the crime of Kidnapping For Ransom with (Double) Homicide, is hereby AFFIRMED with MODIFICATIONS in that: 1) accused-appellants are instead sentenced to suffer the penalty of reclusion perpetua; 2) the award of civil indemnity ex delicto is hereby reduced to P100,000; and 3) accused-appellants are further ordered to pay private complainants the amount of P100,000.00 as exemplary damages.218[41] Appellants filed a motion for reconsideration of the Court of Appeals Decision but this was denied. Hence, appellants filed their Notice of Appeal on 25 August 2006. One of the main issues raised is: THE TRIAL COURT ERRED IN GIVING WEIGHT AND CREDENCE TO THE EXTRA-JUDICIAL CONFESSIONS OF APPELLANT ARNALDO AND APPELLANT FLORES; Appellant Reyes claims that his alleged participation in the kidnapping of the Yao family was based solely on the written extrajudicial confessions of appellants Arnaldo and Flores. He maintains, however, that said extra-judicial confessions are inadmissible in evidence, because they were obtained in violation of his co-appellants constitutional right to have an independent counsel of their own choice during custodial investigation. Appellant Reyes alleges that the agents of the PAOCTF did not ask his co-appellants during the custodial investigation whether they had a lawyer of their own choice, and whether they could afford to hire a lawyer; that the agents of the PAOCTF suggested the availability of Atty. Uminga and Atty. Rous to his co-appellants; and that Atty. Uminga and Atty. Rous were associates of the PAOCTF. Appellant Reyes also asseverates that the extra-judicial confessions of appellants Arnaldo and Flores cannot be utilized against him. Appellant Flores argues that his written extra-judicial confession is inadmissible in evidence, because it was obtained in violation of his constitutional right to have an independent counsel of his own choice during custodial investigation. He insists that his written extra-judicial confession was elicited through force, torture and without the assistance of
218[41]

Rollo, p. 34.

296

297

a lawyer. He avers that he was not assisted by any lawyer from the time he was arrested until he was coerced to sign the purported confession; that he was forced to sign it because he could not anymore endure the beatings he suffered at the hands of the PAOCTF agents; and that he never met or knew Atty. Rous who, according to the PAOCTF, had assisted him during the custodial investigation. Appellant Arnaldo contends that his written extra-judicial confession should be excluded as evidence, as it was procured in violation of his constitutional right to have an independent counsel of his own choice during custodial investigation. He claims that he was not given freedom to choose his counsel; that the agents of the PAOCTF did not ask him during the custodial investigation whether he had a lawyer of his own choice, and whether he could afford to hire a lawyer; and that the agents of the PAOCTF suggested the availability of Atty. Uminga to him. Thus, we have held that an extra-judicial confession is admissible in evidence if the following requisites have been satisfied: (1) it must be voluntary; (2) it must be made with the assistance of competent and independent counsel; (3) it must be express; and (4) it must be in writing.219[67] The right of an accused to be informed of the right to remain silent and to counsel contemplates the transmission of meaningful information rather than just the ceremonial and perfunctory recitation of an abstract constitutional principle.220[69] Such right contemplates effective communication which results in the subject understanding what is conveyed.221[70] The right to counsel is a fundamental right and is intended to preclude the slightest coercion as would lead the accused to admit something false.222[71] The right to counsel attaches upon the start of the investigation, i.e., when the investigating officer starts to ask questions to elicit information and/or confessions or admissions from the accused.223[72] The lawyer called to be present during such investigation should be, as far as reasonably possible, the choice of the accused. If the lawyer is one furnished in behalf of accused, he should be competent and independent; that is, he must be willing to fully safeguard the constitutional rights of the accused.224[73] A competent and independent counsel is logically required to be present and able to advice and assist his client from the time the latter answers the first question asked by the investigator until the signing of the confession. Moreover, the lawyer should ascertain that the confession was made voluntarily, and that the person under investigation fully understood the nature and the consequence of his extra-judicial confession vis-a-vis his constitutional rights. 225[74] However, the foregoing rule is not intended to deter to the accused from confessing guilt if he voluntarily and intelligently so desires, but to protect him from admitting what he is being coerced to admit although untrue. To be an effective counsel, a lawyer need not challenge all the
219[67] 220[69] 221[70] 222[71] 223[72] 224[73] 225[74]

People v. Base, 385 Phil. 803, 815 (2000). People v. Sayaboc, 464 Phil. 824, 839 (2004). People v. Agustin, 310 Phil. 594, 612 (1995). People v. Olermo, 454 Phil. 147, 165 (2003). Gamboa v. Cruz, G.R. No. L-56291, 27 June 1988, 162 SCRA 642, 653. People v. Deniega, G.R. No. 103499, 29 December 1995, 251 SCRA 626, 637. People v. Velarde, 434 Phil. 102, 119 (2002).

297

298

questions being propounded to his client. The presence of a lawyer is not intended to stop an accused from saying anything which might incriminate him; but, rather, it was adopted in our Constitution to preclude the slightest coercion on the accused to admit something false. The counsel should never prevent an accused from freely and voluntarily telling the truth.226[75] We have gone over the records and found that the PAOCTF investigators have duly apprised appellants Arnaldo and Flores of their constitutional rights to remain silent and to have competent and independent counsel of their own choice during their respective custodial investigations. The Pasubali227[76] of appellants Arnaldo and Floress written extra-judicial confessions clearly shows that before they made their respective confessions, the PAOCTF investigators had informed them that the interrogation about to be conducted on them referred to the kidnapping of the Yao family. Thereafter, the PAOCTF agents explained to them that they had a constitutional right to remain silent, and that anything they would say may be used against them in a court of law. They were also told that they were entitled to a counsel of their own choice, and that they would be provided with one if they had none. When asked if they had a lawyer of their own, appellant Arnaldo replied that he would be assisted by Atty. Uminga, while appellant Flores agreed to be represented by Atty. Rous. Thereafter, when asked if they understood their said rights, they replied in the affirmative. The appraisal of their constitutional rights was done in the presence of their respective lawyers and in the Tagalog dialect, the language spoken and understood by them. Appellants Arnaldo and Flores and their respective counsels, Atty. Uminga and Atty. Rous, also signed and thumbmarked the extra-judicial confessions. Atty. Uminga and Atty. Rous attested to the veracity of the afore-cited facts in their respective court testimonies.228[77] Indeed, the appraisal of appellants constitutional rights was not merely perfunctory, because it appeared certain that appellants had understood and, in fact, exercised their fundamental rights after being informed thereof. Records reflect that appellants Arnaldo and Reyes were likewise accorded their right to competent and independent counsel during their respective custodial investigations. As regards appellant Arnaldo, Atty. Uminga testified that prior to the questioning of appellant Arnaldo about the incident, Atty. Uminga told the PAOCTF investigators and agents to give him and appellant Arnaldo space and privacy, so that they could freely converse. After the PAOCTF investigators and agents left them, he and appellant Arnaldo went to a cubicle where only the two of them were present. He interviewed appellant Arnaldo in the Tagalog language regarding the latters personal circumstances and asked him why he was in the PAOCTF office and why he wanted a lawyer. Appellant Arnaldo replied that he wanted to make a confession about his participation in the kidnapping of the Yao family. Thereupon, he asked appellant Arnaldo if the latter would accept his assistance as his lawyer for purposes of his confession. Appellant Arnaldo agreed. He warned appellant Arnaldo that he might be sentenced to death if he confessed involvement in the incident. Appellant Arnaldo answered
226[75] 227[76] 228[77]

People v. Base, supra note 67. Records, pp. 312-318. TSN, 25 September 2001 and 27 September 2001.

298

299

that he would face the consequences because he was bothered by his conscience. He inquired from appellant Arnaldo if he was harmed or intimidated into giving self-incriminating statements to the PAOCTF investigators. Appellant Arnaldo answered in the negative. He requested appellant Arnaldo to remove his shirt for him to check if there were torture marks on his body, but he found none. He also observed that appellant Arnaldos appearance and movements were normal. His conference with appellant Arnaldo lasted for 15 minutes or more. Thereafter, he allowed the PAOCTF investigators to question appellant Arnaldo.229[78] Further, Atty. Uminga sat beside appellant Arnaldo during the inquiry and listened to the latters entire confession. After the taking of appellant Arnaldos confession, Atty. Uminga requested the PAOCTF investigators to give him a copy of appellant Arnaldos confession. Upon obtaining such copy, he read it entirely and thereafter gave it to appellant Arnaldo. He instructed appellant Arnaldo to read and comprehend the same carefully. He told appellant Arnaldo to ask him for clarification and comment if he did not agree or understand any part of his written confession. Appellant Arnaldo read his entire written confession and handed it to him. Atty. Uminga asked him if he had objections to it. Appellant Arnaldo replied in the negative. He then reminded appellant Arnaldo that the latter could still change his mind, and that he was not being forced to sign. Appellant Arnaldo manifested that he would sign his written confession. Later, he and appellant Arnaldo affixed their signatures to the written confession.230[79] With respect to appellant Flores, Atty. Rous declared that before the PAOCTF investigators began questioning appellant, Atty. Rous interviewed him in Tagalog inside a room, where only the two of them were present. He asked appellant Flores about his personal circumstances. Appellant Flores replied that he was a suspect in the kidnapping of the Yao family, and he wanted to give a confession regarding his involvement in the said incident. He asked appellant Flores whether he would accept his assistance as his lawyer. Appellant Flores affirmed that he would. He asked appellant Flores why he wanted to give such confession. Appellant Flores answered that he was bothered by his conscience. Atty. Rous warned appellant Flores that his confession would be used against him in a court of law, and that the death penalty might be imposed on him. Appellant Flores told him that he wanted to tell the truth and unload the burden on his mind. He requested appellant Flores to lift his shirt for the former to verify if there were torture marks or bruises on his body, but found none. Again, he cautioned appellant Flores about the serious consequences of his confession, but the latter maintained that he wanted to tell the truth. Thereafter, he permitted the PAOCTF investigators to question appellant Flores.231[80] Additionally, Atty. Rous stayed with appellant Flores while the latter was giving statements to the PAOCTF investigators. After the taking of appellant Flores statements, he instructed appellant Flores to read and check his written confession. Appellant Flores read the same and made some minor corrections. He also read appellant Flores written confession. Afterwards, he and appellant Flores signed the latters written confession.232[81]
229[78] 230[79] 231[80] 232[81]

TSN, 27 September 2001, pp. 5-9. Id. at 9-15. TSN, 25 September 2001, pp. 2-14. Id. at 14-19.

299

300

It is true that it was the PAOCTF which contacted and suggested the availability of Atty. Uminga and Atty. Rous to appellants Arnaldo and Flores, respectively. Nonetheless, this does not automatically imply that their right to counsel was violated. What the Constitution requires is the presence of competent and independent counsel, one who will effectively undertake his clients defense without any intervening conflict of interest.233[82] There was no conflict of interest with regard to the legal assistance rendered by Atty. Uminga and Atty. Rous. Both counsels had no interest adverse to appellants Arnaldo and Flores. Although Atty. Uminga testified that he was a former National Bureau of Investigation (NBI) agent, he, nevertheless, clarified that he had been separated therefrom since 1994234[83] when he went into private practice. Atty. Uminga declared under oath that he was a private practitioner when he assisted appellant Arnaldo during the custodial investigation.235[84] It appears that Atty. Uminga was called by the PAOCTF to assist appellant Arnaldo, because Atty. Umingas telephone number was listed on the directory of his former NBI officemates detailed at the PAOCTF. Atty. Rous, on the other hand, was a member of the Free Legal Aid Committee of the Integrated Bar of the Philippines, Quezon City at the time he rendered legal assistance to appellant Flores.236[85] Part of Atty. Rous duty as member of the said group was to render legal assistance to the indigents including suspects under custodial investigation. There was no evidence showing that Atty. Rous had organizational or personal links to the PAOCTF. In fact, he proceeded to the PAOCTF office to assist appellant Flores, because he happened to be the lawyer manning the office when the PAOCTF called.237[86] In People v. Fabro,238[87] we stated: The Constitution further requires that the counsel be independent; thus, he cannot be a special counsel, public or private prosecutor, counsel of the police, or a municipal attorney whose interest is admittedly adverse to that of the accused. Atty. Jungco does not fall under any of said enumeration. Nor is there any evidence that he had any interest adverse to that of the accused. The indelible fact is that he was president of the Zambales Chapter of the Integrated Bar of the Philippines, and not a lackey of the lawmen.

Further, as earlier stated, under Section 12(1), Article III of the 1987 Constitution, an accused is entitled to have competent and independent counsel preferably of his own choice. The phrase preferably of his own choice does not convey the message that the choice of a lawyer by a person under investigation is exclusive as to preclude other equally competent and independent attorneys from handling the defense. Otherwise, the tempo of custodial investigation would be solely in the hands of the accused who can impede, nay, obstruct, the progress of the interrogation by simply selecting a lawyer who, for one reason or another, is not available to protect his interest.239[88] While the choice of a lawyer in cases where the person under custodial interrogation cannot afford the services of counsel or where the
233[82] 234[83] 235[84] 236[85] 237[86] 238[87] 239[88]

People v. Velarde, supra note 74. TSN, 27 September 2001, p. 5. Id. TSN, 25 September 2001, pp. 4-5. Id. at 6. Supra note 65 at 726. People v. Mojello, 468 Phil. 944, 954 (2004).

300

301

preferred lawyer is not available is naturally lodged in the police investigators, the suspect has the final choice, as he may reject the counsel chosen for him and ask for another one. A lawyer provided by the investigators is deemed engaged by the accused when he does not raise any objection to the counsels appointment during the course of the investigation, and the accused thereafter subscribes to the veracity of the statement before the swearing officer.240[89] Appellants Arnaldo and Flores did not object to the appointment of Atty. Uminga and Atty. Rous as their lawyers, respectively, during their custodial investigation. Prior to their questioning, appellants Arnaldo and Flores conferred with Atty. Uminga and Atty. Rous. Appellant Arnaldo manifested that he would be assisted by Atty. Uminga, while appellant Flores agreed to be counseled by Atty. Rous. Atty. Uminga and Atty. Rous countersigned the written extra-judicial confessions of appellants Arnaldo and Flores, respectively. Hence, appellants Arnaldo and Flores are deemed to have engaged the services of Atty. Uminga and Atty. Rous, respectively. Since the prosecution has sufficiently established that the respective extra-judicial confessions of appellant Arnaldo and appellant Flores were obtained in accordance with the constitutional guarantees, these confessions are admissible. They are evidence of a high order because of the strong presumption that no person of normal mind would deliberately and knowingly confess to a crime, unless prompted by truth and conscience.241[90] Consequently, the burden of proving that undue pressure or duress was used to procure the confessions rests on appellants Arnaldo and Flores.242[91] In the case at bar, appellants Arnaldo and Flores failed to discharge their burden of proving that they were forced or coerced to make their respective confessions. Other than their self-serving statements that they were maltreated by the PAOCTF officers/agents, they did not present any plausible proof to substantiate their claims. They did not submit any medical report showing that their bodies were subjected to violence or torture. Neither did they file complaints against the persons who had allegedly beaten or forced them to execute their respective confessions despite several opportunities to do so. Appellants Arnaldo and Flores averred that they informed their family members/relatives of the alleged maltreatment, but the latter did not report such allegations to proper authorities. On the contrary, appellants Arnaldo and Flores declared in their respective confessions that they were not forced or harmed in giving their sworn statements, and that they were not promised or given any award in consideration of the same. Records also bear out that they were physically examined by doctors before they made their confessions.243[92] Their physical examination reports certify that no external signs of physical injury or any form of trauma were noted during their examination.244[93] In People v. Pia,245[94] we held that the following factors indicate voluntariness of an extra-judicial confession: (1) where the accused failed to present credible evidence of compulsion or duress or violence on their persons; (2) where they failed to complain to the officers who administered the oaths; (3) where they did not institute any criminal or administrative action against their alleged
240[89] 241[90] 242[91] 243[92] 244[93] 245[94]

People v. Base, supra note 67. People v. Bagnate, G.R. Nos. 133685-86, 20 May 2004, 428 SCRA 633, 651. People v. Fabro, supra note 65. Records, p. 18. Id. at 19. 229 Phil. 577, 582 (1986).

301

302

intimidators for maltreatment; (4) where there appeared to be no marks of violence on their bodies; and (5) where they did not have themselves examined by a reputable physician to buttress their claim. It should also be noted that the extra-judicial confessions of appellants Arnaldo and Flores are replete with details on the manner in which the kidnapping was committed, thereby ruling out the possibility that these were involuntarily made. Their extra-judicial confessions clearly state how appellants and their cohorts planned the kidnapping as well as the sequence of events before, during and after its occurrence. The voluntariness of a confession may be inferred from its language if, upon its face, the confession exhibits no suspicious circumstances tending to cast doubt upon its integrity, it being replete with details which could only be supplied by the accused.246[95] With respect to appellant Reyess claim that the extra-judicial confessions of appellants Arnaldo and Flores cannot be used in evidence against him, we have ruled that although an extra-judicial confession is admissible only against the confessant, jurisprudence makes it admissible as corroborative evidence of other facts that tend to establish the guilt of his co-accused.247[96] In People v. Alvarez,248[97] we ruled that where the confession is used as circumstantial evidence to show the probability of participation by the co-conspirator, that confession is receivable as evidence against a co-accused. In People v. Encipido249[98] we elucidated as follows: It is also to be noted that APPELLANTS extrajudicial confessions were independently made without collusion, are identical with each other in their material respects and confirmatory of the other. They are, therefore, also admissible as circumstantial evidence against their coaccused implicated therein to show the probability of the latters actual participation in the commission of the crime. They are also admissible as corroborative evidence against the others, it being clear from other facts and circumstances presented that persons other than the declarants themselves participated in the commission of the crime charged and proved. They are what is commonly known as interlocking confession and constitute an exception to the general rule that extrajudicial confessions/admissions are admissible in evidence only against the declarants thereof. Appellants Arnaldo and Flores stated in their respective confessions that appellant Reyes participated in their kidnapping of the Yao family. These statements are, therefore, admissible as corroborative and circumstantial evidence to prove appellant Reyes guilt. RIGHT TO COUNSEL The person who assisted him in court during his arraignment and pre-trial is not a lawyer. PEDRO CONSULTA VS. PEOPLE, G.R. No. 17942, February 12, 2009
246[95] 247[96] 248[97] 249[98]

People v. Bagnate, supra note 90. Santos v. Sandiganbayan, 400 Phil. 1175, 1206 (2000). G.R. No. 88451, 5 September 1991, 201 SCRA 364, 377. 230 Phil. 560, 574 (1986).

302

303

CARPIO MORALES, J.: On the matter of accused-appellants claim of having been denied due process, an examination of the records shows that while accusedappellant was represented by Atty. Jocelyn P. Reyes, who seems not a lawyer, during the early stages of trial, the latter withdrew her appearance with the conformity of the former as early as July 28, 2000 and subsequently, approved by the RTC in its Order dated August 4, 2000. Thereafter, accused-appellant was represented by Atty. Rainald C. Paggao from the Public Defenders (Attorneys) Office of Makati City. Since the accused-appellant was already represented by a member of the Philippine Bar who principally handled his defense, albeit unsuccessfully, then he cannot now be heard to complain about having been denied of due process.250[3] (Underscoring supplied) That appellants first counsel may not have been a member of the bar does not dent the proven fact that appellant prevented Nelia and company from proceeding to their destination. Further, appellant was afforded competent representation by the Public Attorneys Office during the presentation by the prosecution of the medico-legal officer and during the presentation of his evidence. People v. Elesterio251[4] enlightens: As for the circumstance that the defense counsel turned out later to be a non-lawyer, it is observed that he was chosen by the accused himself and that his representation does not change the fact that Elesterio was undeniably carrying an unlicensed firearm when he was arrested. At any rate, he has since been represented by a member of the Philippine bar, who prepared the petition for habeas corpus and the appellants brief. (Underscoring supplied) Read also: 1. The right to counsel, 57 SCRA 481 1-a. P vs. Nolasco, 163 SCRA 623 1-b. P vs. Hernandez, 162 SCRA 422 1-c. P. vs. Ampo-an, July 4, 1990 1-d. P. vs. Saludar, July 31, 1990 1-e. P. vs. Kidagan, August 20, 1990 1-f. Estacio vs. Sandiganbayan, 183 SCRA 12 1-g. P. vs. Buenaflor, 181 SCRA 225 2. P vs. Tampus, 96 SCRA 624 3. P vs. Taylaran, 108 SCRA 373 4. P vs. Tawat, 129 SCRA 431 5. P vs. Marcos, 147 SCRA 204 (Note that this criticized by constitutionalists) 6. P vs. Ladrera, 150 SCRA 113 7. P. Nulla, 153 SCRA 471 8. P vs. Marquez, 153 SCRA 700 9. P vs. Olvis, 154 SCRA 513 10. P vs. Caguioa, January 17, 1980 ll. P vs. Pecardal, 145 SCRA 624 12. P vs. Lasac, 148 SCRA 624 13. P vs. Pena, 80 SCRA 589
250[3] 251[4]

decision is widely

Rollo, p. 169 G.R. No. 63971, May 9, 1989, 173 SCRA 243, 249.

303

304

14. P vs. Jara, 144 SCRA 516 How about if the lawyer who assisted him during custodial investigation is a public attorney who was not chosen by the accused himself but given to him free of charge? The extrajudicial confession is inadmissible as evidence. Read: P. vs. Alegria, September 28, 1990 Could the Fiscal also represent the accused during custodial investigation to satisfy the requirement of the Constitution that the accused is assisted by counsel? No. The Fiscal is the counsel for the State, not the accused or the suspect. Read: P. vs. Matos-Viduaya, September 11, 1990 5. Right to remain silent and to counsel and the right to be informed of such rights; cases in general/when does these rights demandable? Effect of its non-observance by the investigator Read: 1. P vs. Albofera, 152 SCRA 123 1-a. P vs. Lasanas, 152 SCRA 27 1-b. P vs. Olvis, 154 SCRA 513 1-c. P vs. Capitin, 165 SCRA 47 1-a. Gamboa vs. Cruz, 162 SCRA 642 1-b. P vs. Hizon, 163 SCRA 760 1-c. P vs. Velasco, 110 SCRA 319 2. Diokno vs. Enrile, 110 SCRA 140 3. Morales vs. Ponce Enrile, 121 SCRA 538 4. P vs. Rojas, January 8, l987 5. P vs. Santiago, January 7,1987 6. P vs. Decierdo, 149 SCRA 496 5-a. Is the right to counsel indispensable in non-criminal proceedings? Read: 1. Nera vs. Auditor Genral, 164 SCRA 1 6. Presumptions on extrajudicial confessions(that official acts were regularly performed as against the presumption against waiver of constitutional rights) Read: 1. P vs. Duero, 104 SCRA 379 2. P vs. Jara, 144 SCRA 516 3. P vs. Abano, 145 SCRA 555 4. P vs. Tolentino, 145 SCRA 597 5. P vs. Salig, 133 SCRA 59 6. P vs. Cruz, 133 SCRA 426
304

305

7. P vs. Prudente,, 133 SCRA 651 8 P vs. Trinidad, 162 SCRA 714, when the regularity does not apply 7. Who can object to the admissibility of an Read: 1. Stonehill vs. Diokno, supra 2. P vs. Jara, 144 SCRA 576 3. P. vs. loveria, July 2, 1990 8. Inadmissible as evidence a. The doctrine of the "fruit of the poisoned tree"

presumption of

extrajudicial confession?

Even if the extrajudicial confession is inadmissible as evidence, accused still held guilty by the Supreme Court. PEOPLE VS. ROLANDO FELIXMINIA y CAMACHO GR No. 125333, March 20, 2002 En Banc Facts: 1. In the morning of September 19, 1995, accused-appellant and his cousin, RONNIE GARCIA were drinking gin in a canteen in Urdaneta, Pangasinan; 2. At around 10 a.m. of the same day, Rosita Mangunay saw both persons walking along Ambrosio St., in the poblacion and noticed that they smelled liquor when they greeted her; 3. In the early afternoon of the same day, accused-appellant and his cousin went to look for 6-year old Maria Lourdes Galinato, also known as Tisay and found her playing inside a jeepney and took her; 4. At around 2:45 p.m. of the same day, Mangunay again saw the accusedappellant walking along Ambrosio St., carrying Tisay who was crying and struggling. She claimed that she clearly saw the accused-appellant since they were walking towards each other coming from opposite directions; 5. Before 3 to 4 p.m., prosecution witness Natividad Bernardo, saw accusedappellant pass their house carrying a child who looked about 5-6 years old. 6. At about the same time, witness Leah Magno saw the accused-appellant carrying a child was seen heading towards the wooded area in the Macalong River; 7. By 5 p.m. to 6:30 p.m. of that same day, Magno saw accused-appellant walking alone to town coming from the direction of the Macalong River; 8. Meanwhile, the parents of Tisay were frantically searching for their child and when their search proved futile, they reported the matter to the Barangay Captain and to the Police;
305

306

9. Upon receipt of the information that the child was last seen with the accused-appellant, the police together with the Barangay Captains of Camantiles and Bayaoas, Urdaneta, Pangasinan, proceeded to the house of the accused-appellant; 10. As they approached the house, the accused-appellant jumped out of the window carrying a black bag. The police authorities gave chase and finally caught him after twenty (20) exhausting hours; 11. After his arrest, accused-appellant was brought to the Urdaneta Police Station where he admitted that he raped, killed and buried Maria Lourdes near the Macalong River in Barangay San Vicente, Urdaneta, Pangasinan, while UNDER INVESTIGATION WITHOUT THE ASSISTANCE OF A LAWYER. INDEED, THE BODY OF TISAY WAS FOUND IN THE PLACE WHICH HE DESCRIBED DURING HIS CUSTODIAL INVESTIGATION. 12. After trial, the trial court (RTC 45 presided over by JUDGE JOVEN COSTALES) rendered a judgment of conviction and imposing the penalty of death to the accused-appellant. The court admitted as evidence the extrajudicial confession of the accused-appellant and used the same as one of the grounds in support of the judgment of conviction. ISSUES 1. Is the extrajudicial confession of the accused appellant admissible in evidence? 2. Whether the lower court erred in convicting in convicting the accusedappellant? Held: 1 The alleged extrajudicial confession of the accused while under custodial investigation and without the assistance of counsel is inadmissible in evidence despite the fact that he was allegedly appraised of his constitutional rights to remain silent and to counsel. This is so because under the 1987 Constitution, the said rights could not be waived except in the presence of counsel. As such, in accordance with the doctrine of the fruit of the poisoned tree, the same is inadmissible in evidence. Any information or admission given by a person while in custody which may appear harmless or innocuous at the time without the competent assistance or an independent counsel should be struck down as inadmissible. 2 Though the extrajudicial confession of the accusedappellant is inadmissible as evidence, his conviction by the trial court is correct. This is so because [1] the compromising circumstances were duly proven which were consistent with each other and which lead with moral certainty to the conclusion that he was guilty of the crime charged; and [2] the totality of such circumstances eliminated beyond
306

307

doubt the possibility of his innocence. In People vs. Mahinay, it was held that conviction may be had on circumstantial evidence provided the following requisites are present: [a] there is more than one circumstance; [b] the facts from which the inferences are derived are proven; and [c] the combination of all circumstances is such as to produce a conviction beyond reasonable doubt. The evidence in this case are more than sufficient to prove the accused-appellants beyond reasonable doubt. Circumstantial evidence is not a weaker form of evidence vis--vis direct evidence and cases have recognized that circumstantial evidence in its weight and probative force, may surpass direct evidence in its effect upon the Supreme Court. (NOTE: The indemnification for the death of a person in a rape with Homicide cases was increased from P50,000.00 to P125,000.00. The said indemnity shall also be applicable where the death penalty is authorized by applicable amendatory laws)) b. The exclusionary rule, 145 SCRA 700 Read: 1. P vs. Burgos, 144 SCRA 516 2. P vs. Alcaraz,136 SCRA 74 3. Does it also include the confession of a accused? Read: 1. P vs. Bombesa, 162 SCRA 402 2. p. vs. Yutuc, July 26, 1990 9. Sec. 12(2) Read: 1. Dizon vs. Gen. Eduardo, May 3,1988 2. P vs. Eligino, August 11,1988 3. Contado vs. Tan, April 15, 1988 10. Extrajudicial confession; when admissible or Read: 1. The admissibility of an extrajudicial prosecution,142 SCRA 110 2. Admissibility of an extrajudicial 419 and 10 SCRA 520 confession in a criminal inadmissible

witness, not the

confession,135

SCRA

3. Inadmissibility of an admission obtained by SCRA 234 4. Confession as evidence against the accused, 96 Read:

force, 114

SCRA 637

307

308

5. P vs. Camalog, January 31, 1989 5-a. P vs. Capulong, 160 SCRA 533 5-b. P vs. Lagahan, December 8, 1988 5-c. P vs. Dino, 160 SCRA 197 5-d. P vs. Caramonte, 94 SCRA 150 5-e. P vs. Enciso, 160 SCRA 728 5-d. P vs. Abano, 145 SCRA 565 5-e. P vs. Quizon, 142 SCRA 362 5-f. P vs. Olvis, 154 SCRA 513 5-g. P vs. Robles, 104 SCRA 450 5-h. P vs. Eligino, 164 SCRA 260 5-i. P vs. Abejero, May 17,l980 5-j. P. vs. Bagano, 181 SCRA 34 5-k. P. vs. Estevan, 186 SCRA 184 5-l. P. vs. Ramos, 186 SCRA 184 5-m. P. vs. Flores, 186 SCRA 303 5-n. P. vs. Jungco, 186 SCRA 714 5-o. P. vs. Arsenio, 184 SCRA 205 6. P vs. Villanueva, 128 SCRA 488 7. P vs. Dejaresco, 129 SCRA 576 8. P vs. Tuvera, 130 SCRA 169 9. P vs. Maternal, 130 SCRA 625 10. P vs. Nilos, 127 SCRA 207 11. P vs. Sanchez, 132 SCRA 103 12. P vs. Pizarro, 131 SCRA 418 13. P vs. Sabilano, 132 SCRA 83 14. P vs. Veloso, 148 SCRA 60 15. Magtoto vs. Manguera, 63 SCRA 4 16. P vs. Gapasin, 145 SCRA 178 17. P vs. Palo, 147 SCRA 178 18. P. vs. De Jesus, 145 SCRA 521 19. P vs. Pia, 145 SCRA 581 20. P vs. Encipiado, 146 SCRA 478 21. P vs. Canumay, 130 SCRA 301 22. P vs. Marino, 130 SCRA 595 23. P vs. Natipravat, 145 SCRA 483 24. P vs. Cruz, 133 SCRa 426--when confession is valid 25. P. vs. De La Cruz, 183 SCRA 763---when confession is inadmissible but accused is still liable 11. Evidence of lack of cvoluntariness Read: 1. P vs. Jara, 144 SCRA 516 2. P vs. Abayon, 114 SCRA 197 12. Is the testimony of the arresting officer on the confession of the accused admissible? Read: 1. P vs. Dy, 158 SCRA 111 alleged oral

308

309

CHAPTER XIII - THE CONSTITUTIONAL RIGHT TO BAIL Section 13. All persons, except those charged with offenses punishable by reclusion perpetua when the evidence of guilt is strong, shall before conviction, be bailable by sufficient sureties, or be released on recognizance as may be provided by law. The right to be bail shall not be impaired even when the privilege of the writ of habeas corpus is suspended. Excessive bail shall not be required. 1. The right to bail, 104 SCRA 372 2. Bail, 81 SCRA 188 Kinds of bail; when not applicable. Recognizance/bail for a convict ATTY. JULIANA ADALIM-WHITE VS. JUDGE ARNULFO BUGTAS, RTC 2 BORONGAN, SAMAR, 475 SCRA 175 Austria-Martinez, J. Facts: Manuel Bagaporo, Jr. was convicted of frustrated murder and was sentenced four years and two months to eight years and one day of imprisonment. He started serving his sentence and subsequently, he filed an application for release on recognizance. In support of his application, the Provincial Jail Warden issued a certification that Bagaoporo has been confined at the Provincial Jail since February 9, 1996 and is already entitled to parole. Another certification was issued by the Supervising Parole and Probation Officer showing that Bagaporo applied for parole in lieu of the DOJs Maagang Paglaya Program. By virtue of the above certifications, respondent judge ordered the release of Bagaporo upon recognizance of the Provincial Jail Warden of Eastern Samar. He likewise justified the same based on the rule that bail is discretionary upon conviction by the RTC of an offense not punishable by death, reclusion perpetua or life imprisonment. Held: Respondent Judge is guilty of gross ignorance of the law for ordering the release of Bagaporo pending the approval of his application for parole and before the completion of the minimum period of the sentence imposed upon him. It is patently erroneous to release a convict on recognizance. Section 24, Rule 114 provides that there shall no bail for a convict after final judgment. The only exception is when the convict applies for Probation before he commences to serve his sentence and that the offense
309

310

and the penalty for the offense is within the purview of the Probation Law. Sections 5 and 16 of Rule 114 of the Rules of Court (on the different kinds of bail) APPLIES ONLY TO AN ACCUSED UNDERGOING PREVENTIVE IMPRISONMENT DURING TRIAL OR ON APPEAL. THEY DO NOT APPLY TO A PERSON CONVICTED BY FINAL JUSGMENT AND ALREADY SERVING SENTENCE. Judge Bugtas was therefore fined P40,000.00 for gross ignorance of the law and sternly warned that a repetition of the same or similar act shall be dealt with more severely. 3. Read: Excessive bail: 1. De la Camara vs. Enage, 41 SCRA 1 1-a. Pestano vs. Judge Velasco, July 3, 1990 Waiver of the right to bail: 1-b. P. vs. Donato, June 5, 1991 2. Almeda vs. Villaluz, 66 SCRA 38 3. Marcos vs. Cruz, 67 and 70 Phil. 4. Villasenor vs. Abano, 21 SCRA 312 5. P vs. IAC, January 10,1987, 147 SCRA 219 6. Manotoc vs. CA, May 30,1986 7. Garcia vs. Domingo, 52 SCRA 143 8. P vs. San Diego, 26 SCRA 522 4. See Section 10, Rule 114, 1985 Rules on Criminal Procedure a. Procedure when prosecutor does not object to the petition for bail in capital offenses: Bail in Extradition cases. 1 UNITED STATES VS. JUDGE PURUGGANAN & MARK JUMENEZ November, 2002

Right to notice and hearing before the issuance of a warrant of arrest in extradition case when earlier, the extraditee was allowed to be out on bail by the court. The bail could not be cancelled without hearing. 2
310

311

EDUARDO RODRIGUEZ VS. THE PRESIDING JUDGE, RTC 17, MANILA, 483 SCRA 290 Quisumbing, J. In SECRETARY OF JUSTICE VS. JUDGE LANTION, 322 SCRA 160 (The Mark Jimenez Case) , the Supreme Court on a 9-6 vote held that the extraditee is entitled to notice and hearing even when a request for extradition by another country is still being evaluated. However, on Motion for Reconsideration in the same case, in a 9-6 decision, the Supreme Court held that the prospective extraditee is not entitled to notice and hearing while his case is still under evaluation because this would defeat the purpose of the arrest warrant since it could give warning that respondents would be arrested and even encourage them to flee but entitled to notice and hearing if the case is already filed in court. It is a different matter if at first, the extraditee was allowed bail. The cancellation of his bail bond may be made only after notice and hearing. Otherwise, his right to due process of law will be violated. (NOTE: In the case of US vs. Judge Purugganan, 389 SCRA 623), the Supreme Court held that the extraditee is not entitled to post a bond even if the crime he was charged of abroad is a bailable offense. This is so because of the possibility of flight.) 3 GOVERNMENT OF HONGKONG SPECIAL ADMINISTRATIVE REGION VS. HON. FELIXBERTO OLALIA, JR., 521 SCRA 470 In UNITED STATES VS. JUDGE PURUGGANAN, 389 SCRA 623, it was held that the constitutional provision on bail does not available in extradition proceedings. It applies only in criminal proceedings because of the word conviction. However, the modern trend in public international law is the primacy placed on the worth of the individual person and the sanctity of human rights. As such, the right to bail was applied in a deportation proceedings, i.e., MEJOFF VS. DIRECTOR OF PRISONS, 90 Phil. 70 [1951] and justified the same with the Universal declaration of Human Rights. If the right to bail is available in deportation cases, then there is no reason why it is not allowed in extradition proceedings. BEFORE A PROSPECTIVE EXTRADITEE IS ALLOWED TO PUT UP BAIL, HE MUST PROVE BY CLEAR AND CONVINCING EVIDENCE THAT HE IS NOT A FLIGHT RISK AND WILL ABIDE WITH ALL ORDERS AND PROCESSES OF THE EXTRADITION COURT.

CHAPTER XIV - DUE PROCESS IN CRIMINAL PROCEEDINGS 1. In general:


311

312

1. P vs. Terrobias, 103 SCRA 321 Presumption of innocence prevails over the presumption of regularity in the performance of official duties of the police authorities and Presumption of innocence resulting in acquittal as a result on conflicting and inconsistent testimonies of the prosecutions witnesses: Non-compliance of Section 21, Republic Act No. 9165, violates the presumption of innocence on the part of the accused and therefore, he should be acquitted.

PEOPLE OF THE PHILIPPINES VS. RONALDO DE GUZMAN, G.R. No. 186498, March 26, 2010 On June 10, 2003, a confidential informant reported De Guzmans drug pushing activities to Alcala, Pangasinans Chief of Police, Sotero Soriano, Jr. Soriano immediately formed a team to conduct a buy-bust operation. After a short briefing, the team proceeded to De Guzmans house. Once there, the confidential informant introduced appellant to Senior Police Officer (SPO)1 Daniel Llanillo, who was designated as poseur-buyer. Llanillo tried to buy P200 worth of shabu. He handed two marked P100 bills to De Guzman, and the latter, in turn, gave him two heat-sealed transparent plastic sachets containing what was suspected as shabu. Thereafter, Llanillo gave the prearranged signal to the rest of the team. Appellant was arrested and frisked. The team recovered from De Guzman two packs of empty transparent sachets, three disposable lighters, and P3,380.00 in cash, which included the marked money paid by SPO1 Llanillo. The team then brought De Guzman to the police station in Alcala, Pangasinan. At the police station, De Guzman and the items seized during the buy-bust operation were turned over to the police investigator, SPO3 Eduardo Yadao. SPO3 Yadao entered the incident in the police blotter. He then placed his initials on the packets of suspected shabu, which were later submitted to the Philippine National Police (PNP) Crime Laboratory in Urdaneta City. Confirmatory tests revealed that the substance in the packets that appellant handed to SPO1 Llanillo was indeed shabu. At the trial, appellant denied the charges against him. He claimed that, on the morning of June 10, 2003, he was on the second floor of his house watching television when he was informed by his wife that police officers were looking for him. He claimed that SPO1 Llanillo informed him about a report that he (De Guzman) was repacking shabu, which he denied. Thereafter, the police officers frisked him and took the P3,000.00 from his pocket. The police officers also searched the cabinet, where his television was, and found a lighter. Then, he was handcuffed and brought to the police station. After trial, the RTC rendered a decision, finding De Guzman guilty beyond reasonable doubt of violating R.A. No. 9165. He was sentenced to life imprisonment and to pay a fine of P500,000.00. De Guzman elevated the matter to the Supreme Court on Petition for Review after the Court of Appeals affirmed the RTC Decision. He argues that the prosecution failed to show that the police officers complied with the mandatory procedures under R.A. No. 9165. In particular, he points to the fact that the
312

313

seized items were not marked immediately after his arrest; that the police officers failed to make an inventory of the seized items in his presence or in the presence of his counsel and of a representative from the media and from the Department of Justice (DOJ); and that no photographs were taken of the seized items and of appellant. Appellant also claims that the unbroken chain of custody of the evidence was not established. Further, appellant contends that the failure of the police officers to enter the buy-bust operation in the police blotter before the said operation, the lack of coordination with the Philippine Drug Enforcement Agency (PDEA), and the failure to observe the requirements of R.A. No. 9165 have effectively overturned the presumption of regularity in the performance of the police officers duties. HELD: A review of the records of this case reveals that circumstances warrant a reversal of the trial courts decision.

The Constitution mandates that an accused in a criminal case shall be presumed innocent until the contrary is proven beyond reasonable doubt. The prosecution is laden with the burden to overcome such presumption of innocence by presenting the quantum of evidence required. Consequently, courts are required to put the prosecution evidence through the crucible of a severe testing, and the constitutional right to presumption of innocence requires them to take a more than casual consideration of every circumstance or doubt favoring the innocence of the accused. When the circumstances are capable of two or more inferences, as in this case, one of which is consistent with innocence and the other is compatible with guilt, the presumption of innocence must prevail, and the court must acquit. The duty to prove the guilt of an accused is reposed in the State. Law enforcers and public officers have the duty to preserve the chain of custody over the seized drugs. This guarantee of the integrity of the evidence to be used against an accused goes to the very heart of his fundamental rights. In a prosecution for illegal sale of dangerous drugs, the following elements must be proven: (1) that the transaction or sale took place; (2) that the corpus delicti or the illicit drug was presented as evidence; and (3) that the buyer and seller were identified. What is material is the proof that the transaction or sale actually took place, coupled with the presentation in court of the prohibited or regulated drug. The delivery of the contraband to the poseur-buyer and the receipt of the marked money consummate the buy-bust transaction between the entrapping officers and the accused. The presentation in court of the corpus delicti the body or the substance of the crime establishes the fact that a crime has actually been committed. Contrary to De Guzmans contention, the trial court correctly found that the buy-bust transaction took place. The buyer (SPO1 Llanillo) and seller (De Guzman) were both identified and the circumstances of how the purported sale of the illegal drugs took place were clearly demonstrated. Thus, the prosecution successfully established the first and third elements of the crime. However, there is a problem in the prosecutions effort to establish the integrity of the corpus delicti.
313

314

The identity of the prohibited drug must be established with moral certainty. Apart from showing that the elements of possession or sale are present, the fact that the substance illegally possessed and sold in the first place is the same substance offered in court as exhibit must likewise be established with the same degree of certitude as that needed to sustain a guilty verdict. The corpus delicti should be identified with unwavering exactitude. The chain of custody requirement performs this function in that it ensures that unnecessary doubts concerning the identity of the evidence are removed. Section 21 of R.A. No. 9165 states: Section 21. Custody and Disposition of Confiscated, Seized, and/or Surrendered Dangerous Drugs, Plant Sources of Dangerous Drugs, Controlled Precursors and Essential Chemicals, Instruments/Paraphernalia and/or Laboratory Equipment. The PDEA shall take charge and have custody of all dangerous drugs, plant sources of dangerous drugs, controlled precursors and essential chemicals, as well as instruments/paraphernalia and/or laboratory equipment so confiscated, seized and/or surrendered, for proper disposition in the following manner: (1) The apprehending team having initial custody and control of the drugs shall, immediately after seizure and confiscation, physically inventory and photograph the same in the presence of the accused or the person/s from whom such items were confiscated and/or seized, or his/her representative or counsel, a representative from the media and the Department of Justice (DOJ), and any elected public official who shall be required to sign the copies of the inventory and be given a copy thereof.

The Court finds that the apprehending officers failed to comply with the guidelines set under R.A. No. 9165 and its IRR. SPO1 Llanillo himself admitted that the marking of the seized items was done in the police station and not immediately after the buy-bust operation. The failure to follow the procedure mandated under R.A. No. 9165 and its IRR must be adequately explained. The justifiable ground for non-compliance must be proven as a fact. The court cannot presume what these grounds are or that they even exist. Accordingly, non-compliance with the procedure shall not render void and invalid the seizure and custody of the drugs only when: (1) such non-compliance is attended by justifiable grounds; and (2) the integrity and the evidentiary value of the seized items are properly preserved by the apprehending team. There must be proof that these two (2) requirements were met before such non-compliance may be said to fall within the scope of the proviso. In this case, it was admitted that it was SPO3 Yadao, the assigned investigator, who marked the seized items, and only upon seeing the items for the first time at the police station. Moreover, there was no physical inventory made or photographs of the seized items taken under the circumstances required by R.A. No. 9165 and its IRR. There was also no mention that representatives from the
314

315

media and from the DOJ, and any elected official, were present during this inventory. The prosecution never explained the reasons for these lapses. As a method of authenticating evidence, the chain of custody rule requires that the admission of an exhibit be preceded by evidence sufficient to support a finding that the matter in question is what the proponent claims it to be. It would include testimony about every link in the chain, from the moment the item was picked up to the time it is offered in evidence, in such a way that every person who touched the exhibit would describe how and from whom it was received, where it was and what happened to it while in the witness possession, the condition in which it was received and the condition in which it was delivered to the next link in the chain. These witnesses would then describe the precautions taken to ensure that there had been no change in the condition of the item and no opportunity for someone not in the chain to have possession of the same. Indeed, it is from the testimony of every witness who handled the evidence that a reliable assurance can be derived that the evidence presented in court and that seized from the accused are one and the same. Accordingly, the failure to establish, through convincing proof, that the integrity of the seized items has been adequately preserved through an unbroken chain of custody is enough to engender reasonable doubt on the guilt of an accused. Reasonable doubt is that doubt engendered by an investigation of the whole proof and an inability after such investigation to let the mind rest upon the certainty of guilt. Absolute certainty of guilt is not demanded by the law to convict a person charged with a crime, but moral certainty is required as to every proposition of proof requisite to constitute the offense. A conviction cannot be sustained if there is a persistent doubt on the identity of the drug.

Indeed, the prosecutions failure to prove that the specimen submitted for laboratory examination was the same one allegedly seized from appellant is fatal to the prosecutions case. Finally, the prosecution cannot find solace in its invocation of the presumption of regularity in the apprehending officers performance of official duty. The presumption of regularity in the performance of official duty cannot by itself overcome the presumption of innocence nor constitute proof beyond reasonable doubt. Moreover, the failure to observe the proper procedure negates the operation of the presumption of regularity accorded to police officers. As a general rule, the testimonies of the police officers who apprehended the accused are accorded full faith and credit because of the presumption that they have performed their duties regularly. But when the performance of their duties is tainted with failure to comply with the procedure and guidelines prescribed, the presumption is effectively destroyed. Thus, even if the defense evidence is weak, the prosecutions whole case still falls. The evidence for the prosecution must stand or fall on its own weight and cannot be allowed to draw strength from the weakness of the defense.

ELPIDIO BONDAD, JR. VS. PEOPLE, G.R. No. 173804, EDecember 10, 2008

315

316

CARPIO MORALES, J.: Elpidio Bondad, Jr., y Burac (appellant) was charged before the Regional Trial Court (RTC) of Marikina City252[1] for violation of Section 5, paragraph 2(3), Article II of Republic Act No. 9165 (R.A. No. 9165) or the Comprehensive Dangerous Drugs Act of 2002, allegedly committed as follows:253[2] That on or about the 29th day of January 2004, in the City of Marikina, Philippines and within the jurisdiction of this Honorable Court, the above-named accused, without being authorized by law, did then and there willfully, unlawfully, feloniously and knowingly sell to poseur buyer 0.02 gram of Methamphetamine Hydrochloride (shabu) contained in one (1) heat-sealed transparent plastic sachet, a dangerous drug, in violation of the above-cited law.254[3] (Underscoring supplied) He was likewise charged for violation of Section 11, par. 2(3), Article II also of R.A. No. 9165, allegedly committed as follows: That on or about the 29th day of January 2004, in the City of Marikina, Philippines, and within the jurisdiction of this Honorable Court, the above-named accused, without being authorized by law to possess or otherwise use any dangerous drugs, did then and there willfully, unlawfully and feloniously have in his possession direct custody and control 0.04 gram of white crystalline substance contained in two (2) heat-sealed plastic sachets which gave positive result to the test for Methamphetamine Hydrochloride (shabu), which is a dangerous drug, in violation of the above-cited law.255[4] (Underscoring supplied) At around 7:05 p.m. of January 29, 2004, while PO2 Ferdinand Brubio, PO1 Christopher Anos, and PO1 Roberto Muega were at the Station Anti Illegal Drug Special Operations Task Force (SAIDSOTF), Office of the Marikina City Police Station, PO2 Nelson Arribay arrived together with a confidential informant. The confidential informant reported, among other things, about the rampant sale of shabu in a billiard hall along Bonifacio Avenue, Barangka, Marikina City and named a certain alias Jun as the vendor. The Chief of the SAIDSOTF, P/Sr. Insp. Ramchrisen Haveria, Jr., at once formed a buy-bust team composed of, among others, PO2 Ramiel Soriano and PO2 Dano who was designated as the poseur-buyer. PO2 Dano was given a one hundred peso bill bearing Serial No. Q487945 to be used as buy-bust money. It was agreed that PO2 Danos removal of his cap would signal that the buy-bust was consummated. The conduct of a buy-bust operation was recorded in the police blotter and was coordinated with the Philippine Drug Enforcement Agency (PDEA) which gave it control number NOC-012904-28. The buy-bust team, together with the confidential informant, proceeded to 3 Cs billiard hall at the corner of M. Cruz St. and Bonifacio Avenue in Barangka, Marikina City. On entering the hall, the confidential informant pointed to appellant who was then holding a cue stick beside the
252[1] 253[2] 254[3] 255[4]

Rollo, pp. 73-74. Records, p. 2. Records, p. 2 - Information dated February 2, 2004. Id. at p. 6.

316

317

billiard table as the alias Jun. The confidential informant approached appellant and talked to him. Within minutes, appellant approached PO2 Dano and asked him if he wanted to buy shabu, to which PO2 Dano answered piso lang. Appellant at once took out a Vicks container from his right front pocket256[5] which, when opened, yielded heat-sealed plastic sachets containing substances suspected to be shabu. From the container, appellant drew out one sachet in exchange for which PO2 Dano gave the marked one hundred peso bill. At that instant, PO2 Dano removed his cap. As the back-up police officers were closing-in, PO2 Dano grabbed appellants arm, identified himself, and apprised appellant of his constitutional rights. Upon PO2 Danos order, appellant returned the buybust money, handed the Vicks container, and gave his name as Elpidio Burac Bondad, Jr. Still at the place of arrest, PO2 Dano placed the markings EBBED BUYBUST 01/29/04 on the substance-filled sachet sold to him, and EBB-ED, POS 1 and 2, 01/29/04 on the sachets that remained inside the Vicks container. The buy-bust team thereupon brought appellant and the seized items to the Marikina City Police Station where a memorandum dated January 29, 2004257[6] was prepared by P/Sr. Insp. Chief Haveria, Jr., addressed to the Chief of the Eastern Police District Crime Laboratory Office, requesting for the conduct of laboratory examination on the seized items to determine the presence of dangerous drugs and their weight. PO2 Dano also requested that appellant be subjected to a drug test.258[7] The following day or on January 30, 2004, at 3:00 P.M., upon receipt of three sachets, a laboratory examination was conducted thereon by Police Senior Inspector Annalee R. Forro, Forensic Chemical Officer of the Eastern Police District Crime Laboratory Office, who, in Physical Science Report No. D-0094-04E259[8], recorded, among other things, the specimen submitted, her findings and conclusion as follows: SPECIMEN SUBMITTED: Three (3) heat-sealed transparent plastic sachets with markings marked as A through C respectively, each containing white crystalline substance with following recorded net weights and markings: A = 0.02 gram EBB-ED BUYBUST 01/29/04 B = 0.02 gram EBB-ED POSS 1 01/29/04 C = 0.02 gram EBB-ED POSS 2 01/29/04 x-x-x F I N D I N G S: x x x x-x-x x-x-x

Qualitative examination conducted on the above-stated specimen gave POSITIVE result to the tests for Methamphetamine Hydrochloride, a dangerous drug.

256[5] 257[6]

No specification if it was a pocket of the shirt or of the pants. Id. at p. 15 258[7] TSN, June 15, 2004, p. 41 259[8] Records, p. 17

317

318

x-x-x C O N C L U S I O N:

x-x-x

x-x-x

Specimens A through C contain Methamphetamine Hydrochloride, a dangerous drug. 260[9] (Italics and emphasis in the original) Denying the charges against him, appellant, a former police officer, claimed that he was framed up and gave the following version: On January 29, 2004, while he was playing inside 3 Cs billiard hall, PO2 Brubio, whom he knew was a policeman, entered the billiard hall. After greeting PO2 Brubio in Bicolano, he continued playing but PO2 Brubio suddenly handcuffed him and asked him Sumama ka muna. Another person who was at his back pushed him out of the billiard hall in the course of which he felt PO2 Brubio reaching his (appellants) right front pocket,261[10] drawing him to restrain the hand of PO2 Brubio, telling him pera ko yan! Aware that his son was inside the billiard hall, appellant summoned and handed him his wallet containing P2,000. PO2 Brubio, however, took the wallet from his son, telling him Huwag ka makialam dito. He was then made to board a car and taken to the Office of the SAIDSOTF at the police station. Appellants defense was corroborated by his son Christian Jeffrey C. Bondad, and Roberto U. Mata who was a spotter (referee) at the billiard hall at the time appellant was arrested. Finding for the prosecution, the trial court convicted appellant in both charges, disposing as follows: WHEREFORE, foregoing premises considered, the Court finds the accused ELPIDIO BONDAD, JR. y BURAC guilty beyond reasonable doubt of the crime of Violation of Sec. 11 par. 2(3), Art. II of R.A. 9165 and is sentenced to suffer the penalty of imprisonment for a period of TWELVE (12) YEARS and ONE (1) DAY and to pay the fine of THREE HUNDRED THOUSAND PESOS (PhP300,000.00) as provided for in Sec. 11 par. 2(3), Art. II of RA 9165. The accused is likewise found guilty of the crime of Violation of Sec. 5 Art. II of RA 9165 and is sentenced to suffer the penalty of LIFE IMPRISONMENT and fine of FIVE HUNDRED THOUSAND PESOS (PhP500,00.00) pursuant to Sec. 5, Art. II of RA 9165 the methamphetamine hydrochloride (shabu) is ordered confiscated in favor of the government for proper destruction by the proper agency. SO ORDERED.262[11] (Underscoring supplied) By Decision of February 8, 2006,263[12] the Court of Appeals affirmed the trial courts decision with modification, disposing as follows:

260[9]

Exhibit C, folder of exhibits, p. 2 There is also no specification if it was a pocket of the shirt or the pants 262[11] CA rollo, p. 124. 263[12] Penned by Justice Amelita G. Tolentino with the concurrence of Justices Portia Alio Hormachuelos and Vicente S.E. Veloso, CA rollo, pp. 232-254.
261[10]

318

319

WHEREFORE, in the light of the foregoing, the appeal is DISMISSED for lack of merit. The assailed decision is AFFIRMED with the MODIFICATION that the accused-appellant is sentenced to suffer an indeterminate penalty of imprisonment of twelve (12) years and one (1) day, as minimum, to thirteen (13) years, as maximum and to pay a fine of Three Hundred Thousand Pesos (P300,000.00). SO ORDERED.264[13] (Underscoring supplied)

Specifically with respect to the charge of possession of shabu, the appellate court held: The evidence for the prosecution fully proved beyond reasonable doubt the elements necessary to successfully prosecute a case for illegal possession of a prohibited drug, namely, (a) the accused is in possession of an item or an object identified to be a prohibited or a regulated drug, (b) such possession is not authorized by law and (c) the accused freely and consciously possessed said drug. Under Section 11, Par. 2 [3] of R.A. 9165, the mere act of possessing any dangerous drug consummates the crime. There is no doubt that the charge of illegal possession of shabu was proven beyond reasonable doubt since the accused-appellant knowingly possessed plastic sachets with white crystalline granules, without legal authority at the time he was caught during the buy-bust operation. The white crystalline granules found in his possession, upon laboratory examination, were positively identified as methamphetamine hydrochloride or shabu, a dangerous drug.265[14] (Italics in the original, underscoring supplied)

Hence, the present Petition for Review on Certiorari, appellant faulting the appellate court: II. . . . IN ADMITTING THE EVIDENCE OF THE PROSECUTION DESPITE CLEAR VIOLATION OF SECTION 21 (1) OF R.A. 9165; As the resolution of the second assignment of error is determinative of whether there is still necessity of segueing to the first and third assignments of error, it shall early on be passed upon. Appellant claims that there was failure to follow the requirements of Sec. 21 of R.A. No. 9165, hence, it compromised the integrity and evidentiary value of the allegedly seized items. Sec. 21 of R.A. No 9165 provides: Section 21. Custody and Disposition of Confiscated, Seized, and/or Surrendered Dangerous Drugs, Plant Sources of Dangerous Drugs, Controlled Precursors and Essential Chemicals, Instruments/Paraphernalia and/or Laboratory Equipment. The PDEA shall take charge and have custody of all dangerous drugs, plant sources or dangerous drugs, controlled precursors and essential chemicals, as well as
264[13] 265[14]

Rollo, p. 68. Id. at p. 66 (citations omitted)..

319

320

instruments/paraphernalia and/or laboratory equipment so confiscated, seized and or surrendered, for proper disposition in the following manner: (1) The apprehending team having initial custody and control of the drugs shall, immediately after seizure and confiscation, physically inventory and photograph the same in the presence of the accused or the persons/s from whom such items were confiscated and/or seized, or his/her representative or counsel, a representative from the media and the Department of Justice (DOJ), and any elected public official who shall be required to sign the copies of the inventory and be given a copy thereof; x x x (Emphasis and underscoring supplied) Appellant claims that no physical inventory and photographing of the drugs took place. A reading of the testimony of the poseur-buyer, PO2 Dano indeed confirms appellants claim, viz: Atty. Puentebella: When you brought him to the police, it was there that the items taken from him were inventoried, is it not? Witness: We did not make inventory because we simply brought the evidence confiscated. xxx Atty. Puentebella: You also did not take photographs of the items taken from the accused? Witness: Yes, sir. Atty. Puentebella: And you know for a fact that under the new drugs law, this is a requirement for the apprehending team to do, is it not? Pros. Gapuzan: Counsel is asking for a conclusion of law. I will object. Court: Witness may answer the question. Witness: Yes, sir. xxxx Atty. Puentebella: So it is very clear now Mr. Witness that at the time you apprehended the accused, you did not make an inventory in the presence of the accused nor you did not [sic] make a photograph of the items seized in the presence of the accused, an elective official, a representative from the Department of Justice, or the media, thats very clear? Witness: Yes, sir.
320

321

Atty. Puentebella: Since you did not make any inventory, it follows that you did not require them to sign your inventory as required by law? Witness: Yes, sir.266[16] (Emphasis and underscoring supplied)

Clearly then, the apprehending police officers failed to comply with the above-quoted provision of Section 21 of R.A. No. 9165. People v. Pringas holds, however: Non-compliance by the apprehending/buy-bust team with Section 21 is not fatal as long as there is justifiable ground therefor, and as long as the integrity and the evidentiary value of the confiscated/seized items, are properly preserved by the apprehending officer/team. Its non-compliance will not render an accused's arrest illegal or the items seized/confiscated from him inadmissible. What is of utmost importance is the preservation of the integrity and the evidentiary value of the seized items, as the same would be utilized in the determination of the guilt or innocence of the accused.267[17] (Citation omitted, emphasis, italics and underscoring supplied) The Courts pronouncement in Pringas is based on the provision of Section 21(a) of the Implementing Rules and Regulations268[18] of R.A. No. 9165, viz: x x x Provided, further, that non-compliance with these requirements under justifiable grounds, as long as the integrity and evidentiary value of the seized items are properly preserved by the apprehending officer/team, shall not render void and invalid such seizures of and custody over said items; (Emphasis and underscoring supplied) In the present case, by PO2 Danos claim, he immediately marked the seized items which were brought to the Crime Laboratory for examination. By his admission, however, he did not conduct an inventory of the items seized. Worse, no photograph of the items was taken. There was thus failure to faithfully follow the requirements of the law. Parenthetically, unlike in Pringas, the defense in the present case questioned early on, during the cross examination of PO2 Dano, the failure of the apprehending officers to comply with the inventory and photographing requirements of Section 21 of R.A. No. 9165269[19], despite PO2 Danos awareness of such requirements. And the defense raised it again during the offer of evidence by the prosecution, thus: Atty. Puentebella: xxxx

266[16] 267[17] 268[18] 269[19]

TSN, June 15, 2004, pp. 80-87. G.R. No. 175928. August 31, 2007, 531 SCRA 828, 842-843 Took effect on November 27, 2002. Vide TSN, June 15, 2004, pp. 81-85.

321

322

Exhibits B which is the brown envelope, B-1, B-2 and B-3 are objected to for being product of irregular functions of police and therefore fruit of poisonous thinking [sic] and they are not admissible and they were not photographed in the presence of the accused as provided for by Sec. 21, par.1, R.A. 9165;270[20] (emphasis supplied) IN FINE, as the failure to comply with the aforesaid requirements of the law compromised the identity of the items seized, which is the corpus delicti of each of the crimes charged against appellant,271[21] his acquittal is in order. This leaves it unnecessary to still dwell on the first and third assignments of error. WHEREFORE, the Petition is GRANTED. The assailed decision is REVERSED and SET ASIDE and appellant, Elpidio Bondad Jr., y Burac, is ACQUITED of the crimes charged. PEOPLE VS. SAMUEL OBMIRANIS, G.R. No. 181492, December 16, 2008 TINGA, J.: This is an appeal filed by Samuel Obmiranis y Oreta (appellant) who was charged with violation of Section 5 in relation to Section 26 of Republic Act (R.A.) No. 9165.272[1] He was allegedly caught in a buy-bust operation by elements of the Manila Western Police District (MWPD) while offering to sell methyl amphetamine hydrochloride, a dangerous drug locally known as shabu. The criminal information filed with the Regional Trial Court (RTC) of Manila, Branch 2273[2] accused him as follows: That on or about May 18, 2004, in the City of Manila, Philippines, the said accused, not having been authorized by law to sell, trade, deliver or give away to another any dangerous drug, did then and there willfully, unlawfully and knowingly attempt to sell or offer for sale one (1) transparent plastic sachet containing TWO POINT EIGHT ZERO ZERO (2.800) grams of white crystalline substance known as SHABU containing methylamphetamine hydrochloride, a dangerous drug. Contrary to law.274[3] At the pre-trial, both the prosecution and the defense stipulated on the qualification of Forensic Chemist Elisa Reyes and, thus, both parties dispensed with her testimony. The prosecution further admitted that the forensic chemist who analyzed the seized the confiscated substance

270[20] 271[21]

TSN, August 10, 2004, pp. 6-7. People v. Laxa, 414 Phil. 156, 170 (2001) citing People v. Rigodon, 238 SCRA 27 (1994).

322

323

which yielded positive for methylamphetamine hydrochloride content did not have personal knowledge of the ultimate source of the drug.275[4] Appellant was brought to trial after having entered a negative plea.276[5] The prosecution then proceeded to prove the charge against him through the lone testimony of police officer Jerry Velasco (Velasco). Velasco was the alleged leader of the raiding team that apprehended appellant on 18 May 2004 at the corner of G.Tuazon and Jhocson Streets in Sampaloc, Manila.277[6] The narrative woven by Velasco established the following facts: On 17 May 2004, Police Superintendent Marcelino Pedrozo (Pedrozo) of the MWPD organized a buy-bust team on the information of a confidential informant that the latter was able to place an order for half a bulto of shabu with appellant. Velasco was designated as the team leader and the poseur-buyer, with Police Officers Wilfredo Cinco, Edgardo Palabay, Roberto Benitez and one278[7]confidential informant as members.279[8] Pedrozo gave the team a marked 500-peso bill to be used as buy-bust money which was placed on top of a deck of boodle money. The team informed the Philippine Drug Enforcement Agency (PDEA) of the impending operation,280[9] entered the same in the blotter281[10] and proceeded to Bambang in G.Tuazon Street just before 12 a.m. of 18 May 2004the appointed time and date that the confidential informant and appellant had agreed to meet. The informant joined Velasco in his car, and they awaited the arrival of appellant at the corner of G.Tuazon and Jhocson Streets.282[11] At around 12:30 a.m., appellant on board a car arrived at the scene and seeing the informant he approached the latter. The informant introduced Velasco to appellant and said that Velasco would like to buy one-half bulto of shabu. Velasco negotiated with appellant to lower the price but the latter refused. Velasco then insisted that he must first see the merchandise. Appellant went back to his car, took the item and brought it to Velasco. Velasco readily recognized the item as a plastic sachet containing a white crystalline substance. When appellant asked for payment, he seemed to have recognized Velascos coofficer because he uttered the words, May pulis yata. At that point, he was arrested just as he was trying to get back to his car.283[12]

323

324

According to Velasco, he was the one who effected the arrest but it was Cinco who seized the plastic sachet from appellant. He further stated that immediately after the arrest, he and his team brought the seized item to the police headquarters and there, in his presence, Cinco marked the same with the initials SOO. At the trial, he identified the plastic sachet as that seized from appellant as well as the marking made by Cinco on it. Furthermore, he admitted on cross-examination that there was no evidence custodian designated and that he could not remember if the seized item had been inventoried and photographed in the presence of the accused; that Cinco put the item in his pocket after the same was recovered and did not mark it on the spot and that the markings made on the buy-bust money had not been entered in the blotter.284[13] The chemistry report issued at the instance of Pedrozo and signed by Forensic Chemical Officer Maritess Mariano of the PNP Crime Laboratory revealed that the specimen supposedly seized from appellant yielded positive of methylamphetamine hydrochloride content.285[14] Taking the stand, appellant boldly asserted that he was merely framed up by the buy-bust team, and strongly denied having transacted the alleged sale of shabu with Velasco and the confidential informant. He claimed that he was taken by Velasco and his team not on 18 May 2004 but rather on 17 May 2004 at 7:00 p.m. along Santa Teresita Street, Sampaloc, Manila;286[15] that he was there to see his girlfriend who was residing in that area; that when he was arrested by two men in civilian clothes, he was not committing any crime; that he asked them why they were arresting him but neither of them gave an answer and instead one of them grabbed him by his shoulder and ushered him inside a police car; that once inside the car, one of the men pulled out a gun with which he hit his neck, kicked him and uttered, Makulit ka ha, yuko!; that he asked them why they were doing that to him when in fact he merely told them to park their car properly on the street; that they cuffed his hands at the back and the driver, Velasco, asked if he could give them P200,000.00; that he answered he did not have that much money; that they drove the car around and told him that if he could not give them the money then he must just find for them someone who sells drugs in large-scale (Magturo ka ng nagbebenta ng droga, iyong malakihan ha!); that because he said he did not know anyone who was into selling drugs, he was taken to the U.N. Avenue police headquarters; that he was not detained at the headquarters but rather, he was brought to the second floor where the two arresting officers demanded P50,000.00 from him; that the demand was then reduced to P30,000.00 in exchange for the mitigation of his case.287[16] Olivia Ismael, another defense witness who introduced herself as a friend of appellants girlfriend and who admitted having witnessed appellants arrest, corroborated the material points of appellants testimony.288[17]

324

325

In its 23 February 2006 Decision, the RTC found appellant guilty beyond reasonable doubt of the offense charged. He was sentenced to suffer the penalty of life imprisonment, and to pay a P500,000.00 fine without subsidiary imprisonment as well as the costs.289[18] Appellant interposed an appeal with the Court of Appeals in which he reiterated that the prosecution was unable to establish his guilt beyond reasonable doubt in view of the failure to establish the chain of custody of the illegal drugs and that it was likewise unable to establish the consummation of the alleged sale of drugs.290[19] For its part, the People, through the Office of the Solicitor General (OSG), posited that the fact that all the essential elements of a consummated sale of dangerous drug had not been completely shown was immaterial because the charge involved a mere attempt or offer to sell which had been duly established by the prosecution.291[20] It also maintained that the chain of custody of the seized shabu had been duly established because the requirements in taking custody of seized narcotics provided for in Dangerous Drugs Board Regulation No. 1, series of 2002 292[21] admit of liberal interpretation.293[22] In its 4 September 2007 Decision,294[23] the Court of Appeals affirmed in toto the trial courts decision. The appeal has to be granted. In criminal prosecutions, fundamental is the requirement that the elemental acts constituting the offense be established with moral certainty as this is the critical and only requisite to a finding of guilt. In prosecutions involving narcotics, the narcotic substance itself constitutes the corpus delicti of the offense and the fact of its existence is vital to sustain a judgment of conviction beyond reasonable doubt.295[27] It is therefore of prime importance that in these cases, the identity of the dangerous drug be likewise established beyond reasonable doubt. 296[28] In other words, it must be established with unwavering exactitude that the dangerous drug presented in court as evidence against the accused is the same as that seized from him in the first place. The chain of custody requirement performs this function in that it ensures that unnecessary doubts concerning the identity of the evidence are removed.297[29]

325

326

Board Regulation No. 1, series of 2002 defines chain of custody as the duly recorded authorized movements and custody of seized drugs or controlled chemicals or plant sources of dangerous drugs or laboratory equipment of each stage, from the time of seizure/confiscation to receipt in the forensic laboratory to safekeeping to presentation in court for destruction. As a method of authenticating evidence, the chain of custody rule requires that the admission of the exhibit be preceded by evidence sufficient to support a finding that the matter in question is what the proponent claims it to be.298[30] It would thus include testimony about every link in the chain, from the moment the item was seized to the time it is offered in court as evidence, such that every person who handled the same would admit how and from whom it was received, where it was and what happened to it while in the witness possession, the condition in which it was received and the condition in which it was delivered to the next link in the chain. The same witnesses would then describe the precautions taken to ensure that there had been no change in the condition of the item and no opportunity for someone not in the chain to have possession of the same.299[31] It is from the testimony of every witness who handled the evidence from which a reliable assurance can be derived that the evidence presented in court is one and the same as that seized from the accused. The prosecution evidence in the case at bar, however, does not suffice to afford such assurance. Of all the people who came into direct contact with the sachet of shabu purportedly seized from appellant, only Velasco was able to observe the uniqueness thereof in court. Cinco, who, according to Velasco, took initial custody of the plastic sachet at the time of arrest and who allegedly marked the same with the initials SOO at the police station, was not even presented in court to directly observe the uniqueness of the specimen and, more importantly, to acknowledge the marking as his own. The same is true with respect to the laboratory personnel who could have but nevertheless failed to testify on the circumstances under which he received the specimen at the laboratory for analysis and testing, as well as on the conduct of the examination which was administered on the specimen and what he did with it at the time it was in his possession and custody. Aside from that, it was not reasonably explained why these same witnesses were not able to testify in court. While indeed the prosecution and the defense had stipulated on the qualification of the forensic chemist, dispensed with his testimony and admitted that said forensic chemist had no personal knowledge of the ultimate source of the drug submitted for examination, nevertheless, these stipulations and admission pertain only to a certain Elisa G. Reyes and not to Forensic Chemical Officer Maritess Mariano who, based on the chemistry report, was the one who examined the contents of the plastic sachet at the crime laboratory. In view of these loopholes in the evidence adduced against appellant, it can be reasonably concluded that the prosecution was unable to establish the identity of the dangerous drug and in effect failed to obliterate the hypothesis of appellants guiltlessness.

326

327

Be that as it may, although testimony about a perfect chain does not always have to be the standard because it is almost always impossible to obtain, an unbroken chain of custody indeed becomes indispensable and essential when the item of real evidence is a narcotic substance. A unique characteristic of narcotic substances such as shabu is that they are not distinctive and are not readily identifiable as in fact they are subject to scientific analysis to determine their composition and nature. 300[32] And because they cannot be readily and properly distinguished visually from other substances of the same physical and/or chemical nature, they are susceptible to alteration, tampering, contamination,301[33] substitution and exchange302[34] whether the alteration, tampering, contamination, substitution and exchange be inadvertent or otherwise not. 303[35] It is by reason of this distinctive quality that the condition of the exhibit at the time of testing and trial is critical.304[36] Hence, in authenticating narcotic specimens, a standard more stringent than that applied to objects which are readily identifiable must be applieda more exacting standard that entails a chain of custody of the item with sufficient completeness if only to render it improbable that the original item has either been exchanged with another or contaminated or tampered with.305[37] The Court certainly cannot reluctantly close its eyes to the possibility of substitution, alteration or contaminationwhether intentional or unintentionalof narcotic substances at any of the links in the chain of custody thereof especially because practically such possibility is great where the item of real evidence is small and is similar in form to other substances to which people are familiar in their daily lives.306[38] Graham v. State307[39] in fact acknowledged this danger. In that case, a substance later shown to be heroin was excluded from the prosecution evidence because prior to examination, it was handled by two police officers who, however, did not testify in court on the condition and whereabouts of the exhibit at the time it was in their possession. The court in that case pointed out that the white powder seized could have been indeed heroin or it could have been sugar or baking powder. It thus declared that the state must be able to show by records or testimony the continuous whereabouts of the exhibit at least between the time it came into the possession of police officers until it was tested in the laboratory to determine its composition.308[40]

327

328

Reasonable safeguards are provided for in our drugs laws to protect the identity and integrity of narcotic substances and dangerous drugs seized and/or recovered from drug offenders. Section 21309[41] of R.A. No. 9165 materially requires the apprehending team having initial custody and control of the drugs to, [1] immediately after seizure and confiscation, [2] physically inventory and [3] photograph the same in the presence of the accused or the person/s from whom such items were confiscated and/or seized, or his/her representative or counsel, a representative from the media and the Department of Justice, and any elected public official who [4] shall be required to sign the copies of the inventory and be given a copy thereof. The same requirements are also found in Section 2310[42] of its implementing rules311[43] as well as in Section 2312[44] of the Dangerous Drugs Board Regulation No. 1, series of 2002.313[45] These guidelines, however, were not shown to have been complied with by the members of the buy-bust team, and nothing on record suggests that they had extended reasonable efforts to comply with the statutory requirements in handling the evidence. Velasco, the leader of the raiding team, himself admitted that as soon as appellant was arrested, Cinco had taken custody of the plastic sachet of shabu, placed it in his pocket and brought the same together with appellant to the police station. It was at the police stationand not at the place where the item was seized from appellantwhere according to him (Velasco), Cinco had placed the initials SOO on the specimen. Velasco never even mentioned that the identifying mark on the specimen was placed in appellants presence; he could not even remember whether or not the specimen had been properly inventoried and photographed at least in appellants presence. Even more telling is the fact that, as elicited from Velasco himself during his crossexamination, no evidence custodian had been designated by the raiding team to safeguard the identity and integrity of the evidence supposedly seized from appellant.314[46] All these aforementioned flaws in the conduct of the post-seizure custody of the dangerous drug allegedly recovered from appellant, taken together with the failure of the key persons who handled the same to testify on the whereabouts of the exhibit before it was offered in evidence in court, militates against the prosecutions cause because it not only casts doubt on the identity of the corpus delicti but also tends to discredit, if not totally negate, the claim of regularity in the conduct of official police operation. What we can fairly assume is that the Court of Appeals had overlooked the significance of these glaring details in the records of the

328

329

case as it placed blind reliance right away on the credibility of Velascos testimony and on the presumption of regularity and thereby it failed to properly account for the missing substantial links in the chain of custody of the evidence. It needs no elucidation that the presumption of regularity in the performance of official duty must be seen in the context of an existing rule of law or statute authorizing the performance of an act or duty or prescribing a procedure in the performance thereof. The presumption, in other words, obtains only where nothing in the records is suggestive of the fact that the law enforcers involved deviated from the standard conduct of official duty as provided for in the law. Otherwise, where the official act in question is irregular on its face, an adverse presumption arises as a matter of course.315[48] There is indeed merit in the contention that where no ill motives to make false charges was successfully attributed to the members of the buy-bust team, the presumption prevails that said police operatives had regularly performed their duty, but the theory is correct only where there is no showing that the conduct of police duty was irregular. People v. Dulay316[49] and People v. Ganenas317[50] in fact both suggest that the presumption of regularity is disputed where there is deviation from the regular performance of duty. Suffice it to say at this point that the presumption of regularity in the conduct of police duty is merely just thata mere presumption disputable by contrary proof and which when challenged by the evidence cannot be regarded as binding truth.318[51] It must be emphasized at this juncture that what can reasonably be presumed based on the records of this case is that Velasco is aware of his duties and responsibilities as an agent of the government in its antinarcotics campaign. A member of the anti-narcotics division of the police since 1997,319[52] Velasco can be reasonably presumed to be adept in and mindful of the proper procedure in apprehending drug offenders, securing and taking custody of the evidence obtained in police operations such as this one and preserving the integrity of the evidence by protecting the chain of custody thereof.320[53] However, for reasons as obvious as intimated above, even this presumption is unworthy of credit. All told, in view of the deviation by the buy-bust team from the mandated conduct of taking post-seizure custody of the dangerous drug in this case, there is no way to presume that the members thereof had performed their duties regularly. Even granting that we must blindly rely on the credibility of Velascos testimony, still, the prosecution evidence would fall short of satisfying the quantum of evidence required to arrive at a finding of guilt beyond reasonable doubt inasmuch as the evidence chain

329

330

failed to solidly connect appellant with the seized drug in a way that would establish that the specimen is one and the same as that seized in the first place and offered in court as evidence. The Court cannot indulge in the presumption of regularity of official duty if only to obliterate the obvious infirmity of the evidence advanced to support appellants conviction. In Mallillin v. People,321[54] we categorically declared that the failure of the prosecution to offer in court the testimony of key witnesses for the basic purpose of establishing a sufficiently complete chain of custody of a specimen of shabu and the irregularity which characterized the handling of the evidence before the same was finally offered in court, materially conflict with every proposition as to the culpability of the accused. For the same plain but consequential reason, we will not hesitate to reverse the judgment of conviction in the present appeal. One final word. In no uncertain terms must it be stressed that basic and elementary is the presupposition that the burden of proving the guilt of an accused rests on the prosecution which must draw strength from its own evidence and not from the weakness of the defense. The rule, in a constitutional system like ours, is invariable regardless of the reputation of the accused because the law presumes his innocence until the contrary is shown. In dubio pro reo. When moral certainty as to culpability hangs in the balance, acquittal on reasonable doubt inevitably becomes a matter of right.322[55] WHEREFORE, the assailed Decision of the Court of Appeals in CA-G.R. CR.-H.C. No. 02158 affirming the judgment of conviction rendered by the Regional Trial Court of Manila, Branch 2, is REVERSED and SET ASIDE. Appellant Samuel Obmiranis y Oreta is ACQUITTED on reasonable doubt and is thus accordingly ordered released immediately from confinement, unless he is lawfully confined for another offense. JUNIE MALLILLIN Y. LOPEZ VS. PEOPLE, G.R. No. 172953 , April 30, 2008 THE FACTS: On the strength of a warrant of search and seizure issued by the RTC of Sorsogon City, Branch 52, a team of five police officers raided the residence of petitioner in Barangay Tugos, Sorsogon City on 4 February 2003. The team was headed by P/Insp. Catalino Bolanos (Bolanos), with PO3 Roberto Esternon (Esternon), SPO1 Pedro Docot, SPO1 Danilo Lasala and SPO2 Romeo Gallinera (Gallinera) as members. The search conducted in the presence of barangay kagawad Delfin Licup as well as petitioner himself, his wife Sheila and his mother, Normaallegedly yielded two (2) plastic sachets of shabu and five (5) empty plastic sachets containing residual morsels of the said substance. Accordingly, petitioner was charged with violation of Section 11, Article II of Republic Act No. 9165, otherwise known as The Comprehensive Dangerous Drugs Act of 2002.

330

331

That on or about the 4th day of February 2003, at about 8:45 in the morning in Barangay Tugos, Sorsogon City, Philippines, the said accused did then and there willfully, unlawfully and feloniously have in his possession, custody and control two (2) plastic sachets of methamphetamine hydrochloride [or] shabu with an aggregate weight of 0.0743 gram, and four empty sachets containing shabu residue, without having been previously authorized by law to possess the same. CONTRARY TO LAW. Petitioner entered a negative plea. At the ensuing trial, the prosecution presented Bolanos, Arroyo and Esternon as witnesses. Taking the witness stand, Bolanos, the leader of the raiding team, testified on the circumstances surrounding the search as follows: that he and his men were allowed entry into the house by petitioner after the latter was shown the search warrant; that upon entering the premises, he ordered Esternon and barangay kagawad Licup, whose assistance had previously been requested in executing the warrant, to conduct the search; that the rest of the police team positioned themselves outside the house to make sure that nobody flees; that he was observing the conduct of the search from about a meter away; that the search conducted inside the bedroom of petitioner yielded five empty plastic sachets with suspected shabu residue contained in a denim bag and kept in one of the cabinets, and two plastic sachets containing shabu which fell off from one of the pillows searched by Esternona discovery that was made in the presence of petitioner.323[10] On cross examination, Bolanos admitted that during the search, he was explaining its progress to petitioners mother, Norma, but that at the same time his eyes were fixed on the search being conducted by Esternon. Esternon testified that the denim bag containing the empty plastic sachets was found behind the door of the bedroom and not inside the cabinet; that he then found the two filled sachets under a pillow on the bed and forthwith called on Gallinera to have the items recorded and marked.324[12] On cross, he admitted that it was he alone who conducted the search because Bolanos was standing behind him in the living room portion of the house and that petitioner handed to him the things to be searched, which included the pillow in which the two sachets of shabu were kept;325[13] that he brought the seized items to the Balogo Police Station for a true inventory, then to the trial court326[14] and thereafter to the laboratory.327[15] Supt. Lorlie Arroyo (Arroyo), the forensic chemist who administered the examination on the seized items, was presented as an expert witness to identify the items submitted to the laboratory. She revealed that the two filled sachets were positive of shabu and that of the

323[10]

TSN, 22 April 2003, pp. 6-9.

324[12]

TSN, 23 July 2003, pp. 6-7, 10. Id. at 16-17. TSN, 23 July 2003, pp. 13-15. Id. at 9.

325[13]

326[14]

327[15]

331

332

five empty sachets, four were positive of containing residue of the same substance.328[16] She further admitted that all seven sachets were delivered to the laboratory by Esternon in the afternoon of the same day that the warrant was executed except that it was not she but rather a certain Mrs. Ofelia Garcia who received the items from Esternon at the laboratory .] The evidence for the defense focused on the irregularity of the search and seizure conducted by the police operatives. Petitioner testified that Esternon began the search of the bedroom with Licup and petitioner himself inside. However, it was momentarily interrupted when one of the police officers declared to Bolanos that petitioners wife, Sheila, was tucking something inside her underwear. Forthwith, a lady officer arrived to conduct the search of Sheilas body inside the same bedroom. At that point, everyone except Esternon was asked to step out of the room. So, it was in his presence that Sheila was searched by the lady officer. Petitioner was then asked by a police officer to buy cigarettes at a nearby store and when he returned from the errand, he was told that nothing was found on Sheilas body.329[18] Sheila was ordered to transfer to the other bedroom together with her children. Petitioner asserted that on his return from the errand, he was summoned by Esternon to the bedroom and once inside, the officer closed the door and asked him to lift the mattress on the bed. And as he was doing as told, Esternon stopped him and ordered him to lift the portion of the headboard. In that instant, Esternon showed him sachet of shabu which according to him came from a pillow on the bed. 330[20] Petitioners account in its entirety was corroborated in its material respects by Norma, barangay kagawad Licup and Sheila in their testimonies. Norma and Sheila positively declared that petitioner was not in the house for the entire duration of the search because at one point he was sent by Esternon to the store to buy cigarettes while Sheila was being searched by the lady officer. Licup for his part testified on the circumstances surrounding the discovery of the plastic sachets. He recounted that after the five empty sachets were found, he went out of the bedroom and into the living room and after about three minutes, Esternon, who was left inside the bedroom, exclaimed that he had just found two filled sachets.] On 20 June 2004 the trial court rendered its Decision declaring petitioner guilty beyond reasonable doubt of the offense charged. Petitioner was condemned to prison for twelve years (12) and one (1) day to twenty (20) years and to pay a fine of P300,000.00. The trial court reasoned that the fact that shabu was found in the house of petitioner was prima facie evidence of petitioners animus possidendi sufficient to convict him of the charge inasmuch as things which a person possesses or over which he exercises acts of ownership are presumptively owned by him. It also noted petitioners failure to ascribe ill motives to the police officers to fabricate charges against him. Hence, this Appeal.

TSN, 28 May 2003, p. 14. The results of the chemical analysis are embodied in Chemistry Report No. D-03703. See records, p. 18.
329[18]

328[16]

TSN, 2 December 2003, pp. 6-10. Id. at 11-12.

330[20]

332

333

HELD: Prosecutions for illegal possession of prohibited drugs necessitates that the elemental act of possession of a prohibited substance be established with moral certainty, together with the fact that the same is not authorized by law. The dangerous drug itself constitutes the very corpus delicti of the offense and the fact of its existence is vital to a judgment of conviction. Essential therefore in these cases is that the identity of the prohibited drug be established beyond doubt . Be that as it may, the mere fact of unauthorized possession will not suffice to create in a reasonable mind the moral certainty required to sustain a finding of guilt. More than just the fact of possession, the fact that the substance illegally possessed in the first place is the same substance offered in court as exhibit must also be established with the same unwavering exactitude as that requisite to make a finding of guilt. The chain of custody requirement performs this function in that it ensures that unnecessary doubts concerning the identity of the evidence are removed. As a method of authenticating evidence, the chain of custody rule requires that the admission of an exhibit be preceded by evidence sufficient to support a finding that the matter in question is what the proponent claims it to be. It would include testimony about every link in the chain, from the moment the item was picked up to the time it is offered into evidence, in such a way that every person who touched the exhibit would describe how and from whom it was received, where it was and what happened to it while in the witness possession, the condition in which it was received and the condition in which it was delivered to the next link in the chain. These witnesses would then describe the precautions taken to ensure that there had been no change in the condition of the item and no opportunity for someone not in the chain to have possession of the same. Indeed, the likelihood of tampering, loss or mistake with respect to an exhibit is greatest when the exhibit is small and is one that has physical characteristics fungible in nature and similar in form to substances familiar to people in their daily lives.331[41] Graham vs. State332[42] positively acknowledged this danger. In that case where a substance later analyzed as heroinwas handled by two police officers prior to examination who however did not testify in court on the condition and whereabouts of the exhibit at the time it was in their possessionwas excluded from the prosecution evidence, the court pointing out that the white powder seized could have been indeed heroin or it could have been sugar or baking powder. It ruled that unless the state can show by records or testimony, the continuous whereabouts of the exhibit at least between the time it came into the possession of police officers until it was tested in the laboratory to determine its composition, testimony of the state as to the laboratorys findings is inadmissible.333[43] A unique characteristic of narcotic substances is that they are not readily identifiable as in fact they are subject to scientific analysis to determine their composition and nature. The Court cannot reluctantly close its eyes to the likelihood, or at least the possibility, that at any of the
331[41]

Graham v. State, 255 N.E2d 652, 655. Graham v. State, 255 N.E2d 652. Graham v. State, 255 N.E2d 652, 655.

332[42]

333[43]

333

334

links in the chain of custody over the same there could have been tampering, alteration or substitution of substances from other casesby accident or otherwisein which similar evidence was seized or in which similar evidence was submitted for laboratory testing. Hence, in authenticating the same, a standard more stringent than that applied to cases involving objects which are readily identifiable must be applied, a more exacting standard that entails a chain of custody of the item with sufficient completeness if only to render it improbable that the original item has either been exchanged with another or been contaminated or tampered with. A mere fleeting glance at the records readily raises significant doubts as to the identity of the sachets of shabu allegedly seized from petitioner. Of the people who came into direct contact with the seized objects, only Esternon and Arroyo testified for the specific purpose of establishing the identity of the evidence. Gallinera, to whom Esternon supposedly handed over the confiscated sachets for recording and marking, as well as Garcia, the person to whom Esternon directly handed over the seized items for chemical analysis at the crime laboratory, were not presented in court to establish the circumstances under which they handled the subject items. Any reasonable mind might then ask the question: Are the sachets of shabu allegedly seized from petitioner the very same objects laboratory tested and offered in court as evidence? The prosecutions evidence is incomplete to provide an affirmative answer. Considering that it was Gallinera who recorded and marked the seized items, his testimony in court is crucial to affirm whether the exhibits were the same items handed over to him by Esternon at the place of seizure and acknowledge the initials marked thereon as his own. The same is true of Garcia who could have, but nevertheless failed, to testify on the circumstances under which she received the items from Esternon, what she did with them during the time they were in her possession until before she delivered the same to Arroyo for analysis. Given the foregoing deviations of police officer Esternon from the standard and normal procedure in the implementation of the warrant and in taking post-seizure custody of the evidence, the blind reliance by the trial court and the Court of Appeals on the presumption of regularity in the conduct of police duty is manifestly misplaced. The presumption of regularity is merely just thata mere presumption disputable by contrary proof and which when challenged by the evidence cannot be regarded as binding truth.334[52] Suffice it to say that this presumption cannot preponderate over the presumption of innocence that prevails if not overthrown by proof beyond reasonable doubt.335[53] In the present case the lack of conclusive identification of the illegal drugs allegedly seized from petitioner, coupled with the irregularity in the manner by which the same were placed under police custody before offered in court, strongly militates a finding of guilt. In our constitutional system, basic and elementary is the presupposition that the burden of proving the guilt of an accused lies on the prosecution which must rely on the strength of its own evidence and
334[52]

People v. Ambrosio, G.R. No. 135378, 14 April 2004, 427 SCRA 312, 318 citing People v. Tan, 382 SCRA

419 (2002).
335[53]

People v. Ambrosio, G.R. No. 135378, 14 April 2004, 427 SCRA 312, 318 citing People v. Tan, 382 SCRA

419 (2002).

334

335

not on the weakness of the defense. The rule is invariable whatever may be the reputation of the accused, for the law presumes his innocence unless and until the contrary is shown.336[54] In dubio pro reo. When moral certainty as to culpability hangs in the balance, acquittal on reasonable doubt inevitably becomes a matter of right. The Equipoise Rule; Evidence of guilt and evidence of innocence are about even; effect of non-presentation of forensic chemist who examined the shabu or marijuanaground for acquittal. PEOPLE OF THE PHILIPPINES vs. MONALYN CERVANTES, G.R. No. 181494, March 17, 2009 VELASCO, JR., J.: This is an appeal from the Decision dated July 19, 2007 of the Court of Appeals (CA) in CA-G.R. CR-H.C. No. 00476 which affirmed the April 23, 2004 Decision in Criminal Case No. 00-181929 of the Regional Trial Court (RTC), Branch 53 in Manila. The RTC found accused-appellant Monalyn Cervantes guilty beyond reasonable doubt of violation of Section 15, Article III of Republic Act No. (RA) 6425 or the Dangerous Drugs Act of 1972, as amended. The records show the following facts: In an Information dated April 7, 2000, accused-appellant and three others were charged with violation of Sec. 15, Art. III of RA 6425 (selling or distributing a regulated drug), allegedly committed as follows: That, on or about April 5, 2000, in the City of Manila, Philippines, and within the jurisdiction of this Honorable Court, accused ISIDRO ARGUSON y ARENDELA, @ Tisoy, MONALYN [CERVANTES] y SOLAR @ Mona, WILSON DEL MONTE @ Wilson and RICHARD REQUIZ @ Richard, conspiring, confederating and mutually helping one another, acting in common accord, did then and there, willfully, unlawfully and feloniously, for the amount of FIVE HUNDRED THOUSAND (P500,000.00) PESOS, Philippine Currency, sell, deliver and give away to a poseur-buyer, FOUR HUNDRED SEVENTY THREE POINT SEVENTY SIX (473.76) GRAMS OF METHAMPHETAMINE [HYDROCHLORIDE], commonly known as shabu, a regulated drug, without authority of law or the corresponding license therefor. CONTRARY TO LAW.337[1] Accused-appellant and her co-accused pleaded not guilty to the charge. In the ensuing trial, the prosecution presented in evidence the oral testimonies of William Todavia, PO3 Reynaldo Ramos of the Philippine National Police Regional Office IV (PNP R-IV), and P/Sr. Inspector Lorna Tria, a forensic chemical officer of the same regional office. The Peoples version of the incident, as summarized by the CA in the decision now on appeal, is as follows:

336[54]

People v. Laxa, id.

337[1]

Rollo, pp. 6-7.

335

336

On April 5, 2000, the Regional Special Operations Group IV (RSOG-IV), based at Camp Vicente Lim in Calamba, Laguna, received a tip from a deep penetration agent (DPA) about a group of drug traffickers led by Isidro Arguson operating in Cavite. Acting on this bit of information, a team led by SPO2 Geronimo Pastrana, PO3 Ramos, and PO2 Emerson Balosbalos arranged a buy-bust operation to be conducted at Argusons rest house in Barangay Lambingan, Tanza, Cavite.338[2] Upon arriving at the rest house, PO3 Ramos and PO2 Balosbalos, acting as poseur-buyers, were introduced by the DPA to Arguson as the buyers of PhP 500,000 worth of shabu, simultaneously showing him a bundle of money. Since Arguson did not have enough supply of shabu in the premises, he instructed the would-be-buyers to follow him to Pasay City. For the purpose, he hired a vehicle owned by Todavia. At about three oclock in the afternoon of that day, in front of the McDonalds branch in P. Ocampo St., Pasay City, 339[3] Arguson instructed the would-be-buyers to wait for someone who will come out from the nearby Estrella St. Very much later, accused-appellant emerged from Estrella St. and approached PO3 Ramos to check if he still had the money. After being shown the money bundle, accused-appellant left, only to return a few minutes later this time with Arguson, Wilson Del Monte, who was holding a black plastic bag, and Richard Requiz. Arguson then took from Del Monte the bag, later found to contain 473.76 grams of shabu packed in six small self-sealing transparent bags, and handed it to PO2 Balosbalos, who in turn gave him the bundle of boodle money. Finally, PO3 Ramos gave the pre-arranged signal to indicate the consummation of the drug deal and introduced himself as policeman. Accused-appellant and her scampering companions were later arrested and brought to and booked at Camp Vicente Lim. The black plastic bag containing the six small self-sealing bags of white crystalline substance was likewise taken to Camp Vicente Lim where PO3 Ramos prepared the booking sheets and arrest reports and the request for a qualitative analysis of the seized items. Regional Crime Laboratory Office IV Chief Inspector (C/I) Mary Jean Geronimo then conducted the standard physical and chemical examinations on the specimen referred to her. On April 6, 2000, C/I Geronimo prepared and completed Chemistry Report No. D-115800 on the crystalline substance. Per her report, the substance tested positive for methamphetamine hydrochloride or shabu. Apart from the witnesses affidavits and other documents, the prosecution, in the hearing of March 4, 2002, offered in evidence the following exhibits,340[4] inclusive of its sub markings, which, as may be expected, were objected to by the defense: (a) Exhibit B Chemistry Report No. D-115800 prepared by C/I Geronimo; (b) Exhibit C Memorandum of RSOG-IV dated April 5, 2000 to the Chief, Laboratory Service, requesting for qualitative analysis of the contents of the six transparent plastic bags; (c) Exhibits D and D-1 to D-6 Black plastic bag with markings; and six (6) self-sealing transparent bags allegedly containing the confiscated shabu; and (d) Exhibit F Receipt
338[2] 339[3]

Id. at 5. The McDonalds branch in P. Ocampo St. was later determined to be in Manila. 340[4] Records, pp. 185-187.

336

337

of property seized signed by PO2 Balosbalos and by Todavia and PO3 Ramos as witnesses. The CA decision likewise summarized the defenses account of what purportedly transpired, to wit: Accused-appellant testified that after she did laundry works at her house in Estrella Street near F.B. Harrison on April 4, 2000, her youngest child asked her to go to [McDonalds], Vito Cruz branch, to buy ice cream. When they arrived thereat at about 4:30 in the afternoon, there was a commotion going on in front of the restaurant. She then saw a woman who alighted from a nearby van and pointed her out to her companions, one of whom [was] an old man boarded her inside the van causing her to lose hold of her child. Thereafter, two (2) younger male persons, whom she later came to know as DEL MONTE and REQUIZ, were also boarded into the same van. They were taken to a cemetery where another vehicle came and took them to Camp Vicente Lim, where she allegedly met ARGUSON for the first time. On the other hand, accused DEL MONTE testified that he was a parking boy around Vito Cruz and that on the day in question, while he was watching a vehicle near [McDonalds], Vito Cruz branch, a commotion happened near his post. As he moved backward from where he stood, he was suddenly approached by a policeman who arrested him and boarded him inside a vehicle together with CERVANTES and REQUIZ, whom he did not know prior to that incident. For his part, accused REQUIZ testified that on the date and time in question, he was riding a borrowed bicycle on his way to the Cultural Center, passing by F.B. Harrison St., when he bumped a parked van, wherefrom a man alighted and cursed him, saying pulis ako wag kang aalis dyan[!] The man left and when he returned, accused CERVANTES was with him. Thereafter, he was boarded into the van together with the other accused.341[5] While not stated in the CA decision, Del Monte testified, like accused-appellant, that he was taken to a cemetery somewhere in Cavite where the arresting officers lingered for an hour before bringing him to Camp Vicente Lim.342[6] These testimonies remained uncontroverted. Arguson died during the course of the trial resulting in the dismissal of the case against him.343[7] On April 23, 2004, the RTC rendered judgment acquitting Del Monte and Requiz but finding accused-appellant guilty as charged and meting upon her the penalty of reclusion perpetua. The fallo of the RTC Decision reads: WHEREFORE, in view of the foregoing, judgment is hereby rendered: 1. Finding accused MONALYN CERVANTES Y SOLAR GUILTY beyond reasonable doubt of violation of Sec. 15, Article III, of Republic Act No.
341[5] 342[6]

Rollo, pp. 7-8. TSN, January 20, 2003, pp. 10-11. 343[7] Rollo, p. 8.

337

338

6425 as amended, and is sentenced to Reclusion Perpetua and to pay a fine in the amount of Php500,000.00; and 2. Finding the prosecutions evidence insufficient to prove the guilt of accused WILSON DEL MONTE and RICHARD REQUIZ beyond reasonable doubt, and who are hereby ACQUITTED. SO ORDERED.344[8] On May 18, 2004, accused-appellant filed a Notice of Appeal, pursuant to which the RTC forwarded the records of the case to this Court. Conformably with People v. Mateo,345[9] the Court directed the transfer of the case to the CA where it was docketed as CA-G.R. CR-H.C. No. 00476. Before the appellate court, accused-appellant urged her acquittal on the ground of insufficiency of evidence, particularly stating that the forensic chemist who actually conducted the laboratory examination on the specimens allegedly recovered from the accused was not presented in court x x x [and] hence, there was no clear identification of the contents of the confiscated sachets.346[10] By its Decision347[11] dated July 19, 2007, the CA, finding the elements necessary for the prosecution of illegal sale of drugs 348[12] to have sufficiently been satisfied and the identification of accused-appellant having been established, affirmed her conviction. The CA rejected accused-appellants lament about one Inspector Tria testifying on the chemistry report she did not prepare. As the appellate court stressed, C/I Geronimos forensic report carries the presumption of regularity in the performance of official functions [and] the entries thereon x x are prima facie evidence of the facts therein stated. The CA added the observation that absent any evidence overturning the presumption of regularity in the performance of official functions, the probative value and admissibility of the forensic report prepared by C/I Geronimo, who had resigned from the service, must be upheld even if she did not personally testify in court. On August 17, 2007, accused-appellant filed a Notice of Appeal of the CA affirmatory decision. On March 24, 2008, this Court required the parties to submit supplemental briefs if they so desired. The parties manifested their willingness to submit the case on the basis of the records already submitted, thus veritably reiterating their principal arguments raised in the CA, which on the part of accused-appellant would be:

CA rollo, p. 30. Penned by Judge Reynaldo A. Alhambra. G.R. Nos. 147678-87, July 7, 2004, 433 SCRA 640. 346[10] CA rollo, pp. 81-82. 347[11] Rollo, pp. 4-10. Penned by Associate Justice Estela M. Perlas-Bernabe and concurred in by Associate Justices Vicente Q. Roxas and Lucas P. Bersamin. 348[12] (a) identity of the buyer and the seller, the object and the consideration; and (b) the delivery of the thing sold and payment therefor.
345[9]

344[8]

338

339

For its part, the People, thru the Office of the Solicitor General, counters that the prosecution has established that the buy-bust transaction took place, has identified accused-appellant and her complicity in Argusons illegal trade, and has presented the corpus delicti, as evidence. The Courts Ruling After a circumspect study, the Court resolves to acquit accusedappellant, considering certain circumstances engendering reasonable doubt as to her guilt. We start off with the most basic, the testimony of the prosecutions principal witness, PO3 Ramos, who identified accused-appellant and described her role in the conspiracy to sell shabu. In the witness box, PO3 Ramos testified that, after being told by Arguson to wait for someone who will come out from the street whence Arguson would enter, accusedappellant emerged from said street, checked on the purchase money, asked the operatives to wait, and later re-appeared. What happened next is captured by the following answers of PO3 Ramos to the prosecutors questions: Q: What did you see when Cervantes already returned? A: When Monalyn return the one holding the plastic bag was Wilson, sir. Q: Wilson? A: Yes, sir, together with Richard, Wilson, Arguson, they were four (4). Atty. Cruz: Your honor, may we move to strike that out x x x. Fiscal Formoso: Thats part of the answer x x x now, when all these accused here return with Monalyn Cervantes, what happen[ed]? A: Arguson took the plastic bag from Wilson, sir and handed it to Balosbalos, Balosbalos gave Arguson the boodle money while I flash the signal x x x then we apprehended them.349[13] As may be noted, PO3 Ramos categorically stated that Del Monte was among the four who emerged with Arguson from a street. Without hesitation, PO3 Ramos pointed to Del Monte as the one holding the plastic bag allegedly containing the prohibited substance until Arguson took it from him and handed it over to PO2 Balosbalos. There is no suggestion that accused-appellant, while at the crime scene, ever handled the merchandise or its container. Yet, the trial court acquitted Requiz and Del Monte, but convicted accused-appellant, stating: Clearly, accused Monalyn Cervantes complicity with accused Isidro Arguson in the sale of shabu has been established by the testimony of PO3 Ramos.350[14] But two paragraphs later, the RTC went on to write: x x x While PO3 Ramos testified that the bag was initially held by accused Del Monte and then taken from him by accused Arguson, there is no other evidence which can support the charge of conspiracy with
349[13] 350[14]

TSN, October 23, 2001, pp. 12-16. CA rollo, p. 28.

339

340

Arguson and Cervantes x x x. The court does not find the evidence sufficient to pass the test of moral certainty to find accused Del Monte liable as charged. Even if PO3 Ramos saw him to have held the bag for Arguson, it could have been possible that he was merely asked by Cervantes or Arguson to carry the bag.351[15] Before us then is a situation where two personsaccusedappellant, a laundry woman; and Del Monte, a car park boy, in the company of the ostensible pusher, Arguson, during the actual buy bust are being indicted, on the basis alone of the testimony of a witness, with confederating with each and several others to sell shabu. The overt acts performed by accused-appellant, as indicia of conspiracy, consisted of allegedly verifying whether the poseur-buyer still had the purchase money, disappearing from the scene and then coming back with the principal player. On the other hand, Del Monte came accompanying Arguson carrying the drug-containing plastic bag no less. As between the two acts performed, carrying the bag would relatively have the more serious implication being in itself a punishable act of possession of regulated drugs. Both offered the defenses of denial and instigation, each testifying that they just happened to be near or passing by McDonalds at about 4:30 in the afternoon of April 4, 2000 when they were apprehended. But the trial court, in its observation that it could have been possible that [Del Monte] was merely asked by x x x Arguson to carry the bag, extended to Del Monte the benefit of the doubt, a benevolence denied to accusedappellant without so much of an acceptable explanation. Any reasonable mind might ask: Why the contrasting treatment? Why consider PO3 Ramos as a highly credible eyewitness as against accused-appellant, but an unreliable one as against Del Monte, when both accused are complete strangers to the policeman? To paraphrase an unyielding rule, if the inculpatory testimony is capable of two or more explanations, one consistent with the innocence of the accused persons and the other consistent with their guilt, then the evidence does not fulfill the test of moral certainty and is not sufficient to support a conviction.352[16] But even if we were to cast aside the foregoing equipoise rule, a reversal of the appealed decision is indicated on another but more compelling ground. We refer to the postulate that the prosecution, having failed to positively and convincingly prove the identity of the seized regulated substance, is deemed to have also failed to prove beyond reasonable doubt accused-appellants guilt. We shall explain. In every prosecution for illegal sale of dangerous drug, what is crucial is the identity of the buyer and seller, the object and its consideration, the delivery of the thing sold, and the payment for it. Implicit in these cases is first and foremost the identity and existence, coupled with the presentation to the court of the traded prohibited substance, this object evidence being an integral part of the corpus353[17] delicti354[18] of the crime of possession or selling of regulated/prohibited
Id. at 28-29. People v. Navarro, G.R. No. 173790, October 11, 2007, 535 SCRA 644, 653. 353[17] A Latin word which signifies body. 354[18] Literally body of the crime; in the legal sense, corpus delicti as referring to the fact of the commission of the crime charged or to the substance of the crime; it does not refer to the actual physical evidence, such as ransom money in the crime of kidnapping for ransom, the cadaver of the person murdered, or the confiscated cases of blue seal cigarettes in the crime of smuggling. See Rimorin, Sr. v. People, G.R. No. 146481, April 30, 2003, 402 SCRA 393, 400.
352[16] 351[15]

340

341

drug.355[19] There can be no such crime when nagging doubts persist on whether the specimen submitted for examination and presented in court was what was recovered from, or sold by, the accused.356[20] Essential, therefore, in appropriate cases is that the identity of the prohibited drug be established with moral certainty. This means that on top of the key elements of possession or sale, the fact that the substance illegally possessed and sold in the first place is the same substance offered in court as exhibit must likewise be established with the same degree of certitude as that needed to sustain a guilty verdict. And as we stressed in Malillin v. People, the chain of custody requirement performs this function in that it ensures that unnecessary doubts concerning the identity of the evidence are removed.357[21] So it is that in a slew of cases the Court has considered the prosecutions failure to adequately prove that the specimen submitted for laboratory examination was the same one supposedly seized from the offending seller or possessor as ground for acquittal.358[22] Sec. 1(b) of the Dangerous Drugs Board Regulation No. 1, Series of 2002, or the Guidelines on the Custody and Disposition of Seized Dangerous Drugs, Controlled Precursors and Essential Chemicals, and Laboratory Equipment, defines chain of custody, thusly: Chain of Custody means the duly recorded authorized movements and custody of seized drugs or controlled chemicals x x x from the time of seizure/confiscation to receipt in the forensic laboratory to safekeeping to presentation in court for destruction. Such record of movements and custody of seized item shall include the identity and signature of the person who held temporary custody of the seized item, the date and time when such transfer of custody [was] made in the course of safekeeping and use in court as evidence, and the final disposition.359[23] As a mode of authenticating evidence, the chain of custody rule requires that the admission of an exhibit be preceded by evidence sufficient to support a finding that the matter in question is what the proponent claims it to be. In context, this would ideally include testimony about every link in the chain, from the seizure of the prohibited drug up to the time it is offered into evidence, in such a way that everyone who touched the exhibit would describe how and from whom it was received, where it was and what happened to it while in the witness possession, the condition in which it was received, and the condition in which it was delivered to the next link in the chain.360[24] The need for the punctilious observance of the chain-of-custody process in drug-related cases is explained in Malillin in the following wise: While testimony about a perfect chain is not always the standard because it is almost always impossible to obtain, an unbroken chain of custody becomes indispensable and essential when the item of real evidence is not distinctive and is not really identifiable, or when its condition at the time of testing or trial is critical, or when a witness has failed to observe its uniqueness. The same standard likewise obtains in case the evidence is susceptible to alteration,
People v. Sanchez, G.R. No. 175832, October 10, 2008; citing Valdez v. People, G.R. No. 170180, November 23, 2007, 538 SCRA 611. 356[20] Valdez, supra note 19, at 628-629; citing People v. Ong, G.R. No. 137348, June 21, 2004, 432 SCRA 470. 357[21] G.R. No. 172953, April 30, 2008, 553 SCRA 619, 632; citing American jurisprudence. 358[22] Valdez, supra; Ong, supra note 20. 359[23] In accordance with Sec. 21, Art. II of the Implementing Rules and Regulations (IRR) of RA 9165 or the Comprehensive Dangerous Drugs Act of 2002 in relation to Sec. 81(b), Art. IX of RA 9165. 360[24] Malillin, supra note 21.
355[19]

341

342

tampering, contamination and even substitution and exchange. In other words, the exhibits level of susceptibility to fungibility, alteration or tamperingwithout regard to whether the same is advertent or otherwise notdictates the level of strictness in the application of the chain of custody rule. xxxx A unique characteristic of narcotic substances is that they are not readily identifiable as in fact they are subject to scientific analysis to determine their composition and nature. The Court cannot reluctantly close its eyes to the likelihood, or at least the possibility, that at any of the links in the chain of custody over the same there could have been tampering, alteration or substitution of substances from other casesby accident or otherwisein which similar evidence was seized or in which similar evidence was submitted for laboratory testing. Hence, in authenticating the same, a standard more stringent than that applied to cases involving objects which are readily identifiable must be applied, a more exacting standard that entails a chain of custody of the item with sufficient completeness if only to render it improbable that the original item has either been exchanged with another or been contaminated or tampered with.361[25] (Emphasis added.) As the Court distinctly notes in this case, of the individuals who came into direct contact with or had physical custody of the seized regulated items, only PO3 Ramos testified for the specific purpose of identifying the evidence. In the witness box, however, he did not indicate how he and his companions, right after the buy bust, handled the seized plastic bag and its contents. He did not name the duty desk officer at Camp Vicente Lim to whom he specifically turned over the confiscated bag and sachets at least for recording. What is on record is Exhibit C, which, as earlier described, is a memorandum362[26] PO3 Ramos prepared363[27] dated April 5, 2000 from the RSOG-IV Director to the Chief, PNP R-IV Crime Laboratory Service, submitting for qualitative analysis the white crystalline substance confiscated by the buy-bust group. Needless to stress, the unnamed person who delivered the suspected shabu and the recipient of it at the laboratory were no-show in court to testify on the circumstances under which they handled the specimen or whether other persons had access to the specimen before actual testing. And C/I Geronimo, the analyzing forensic chemist, was not also presented. Then, too, no one testified on how the specimen was cared after following the chemical analysis. As the Court observed aptly in People v. Ong, [T]hese questions should be answered satisfactorily to determine whether the integrity of the evidence was compromised in any way. Otherwise, the prosecution cannot maintain that it was able to prove the guilt of appellants beyond reasonable doubt.364[28] It cannot be overemphasized that Inspector Tria was really not part of the custodial chain. And she did not as she could not, even if she wanted to, testify on whether or not the specimen turned over for analysis and eventually offered in court as exhibit was the same substance received from Arguson.

361[25] 362[26]

Id. at 633-634. Records, p. 33. 363[27] TSN, October 23, 2001, p. 20. 364[28] Supra note 20, at 490.

342

343

Given the foregoing perspective, it is fairly evident that the police operatives trifled with the procedures in the custody of seized prohibited drugs in a buy-bust operation, as embodied in Sec. 21(1), Art. II of RA 9165, i.e., the apprehending officer/team having initial custody and control of the drug shall: immediately after seizure and confiscation, physically inventory and photograph the [drug] in the presence of the accused or the person/s from whom such items were confiscated and/or seized, or his/her representative or counsel, a representative from the media and the Department of Justice (DOJ), and any elected public official who shall be required to sign the copies of the inventory and be given a copy thereof.365[29] In this case, no physical inventory was made and no photograph taken nor markings made on the seized articles at the crime scene. PO3 Ramos admitted as much, thus: Q. Now, you were able to arrest all the accused here, after their arrest, what did you do? A. After informing their rights and the reason why we arrest them we brought them immediately to our office in Canlubang. xxxx Q. Now, what about this Shabu, who was in possession of this Shabu x x x when you left the place and proceeded to Canlubang? A. PO2 Balosbalos, sir. xxxx Q. Now, when you reach your office, what did you do there? A. I made the booking sheet and I requested for their medical/physical examination x x x.366[30] Just as clear is the fact that the exacting chain of custody rule was not observed. Withal, there is no reasonable assurance that no tampering or substitution occurred between the time the police seized the black bag in P. Ocampo St. in Manila until its contents were tested in the laboratory of the PNP R-IV headquarters in Canlubang, Laguna. In net effect, a heavy cloud of doubt hangs over the integrity and necessarily the evidentiary value of the seized items. The prosecution cannot, thus, rightfully assert that the six sachets seized from Arguson were the very same objects tested by C/I Geronimo and offered in court in proving the corpus delicti. Adding a negative dimension to the prosecutions case is the nonpresentation of C/I Geronimo and the presentation in her stead of Inspector Tria to testify on the chemical report C/I Geronimo prepared. While Inspector Tria can plausibly testify on the fact that C/I Geronimo prepared the chemical report in the regular course of her duties, she, Inspector Tria, was incompetent to state that the specimen her former
The IRR of RA 9165 provides further, non-compliance with these requirements under justifiable grounds, as long as the integrity and the evidentiary value of the seized items are properly preserved by the apprehending officer/team, shall not render void and invalid such seizures of and custody over said items. 366[30] TSN, October 23, 2001, pp. 18-19.
365[29]

343

344

colleague analyzed was in fact shabu and was the same specimen delivered to the laboratory for chemical analysis. To be sure, the Court, notably in People v. Bandang, has held that the non-presentation of the forensic chemist in illegal drug cases is an insufficient cause for acquittal. In it, the accused persons were convicted of illegal sale of shabu even if the forensic chemist who prepared the corresponding laboratory report was not presented. Thus, we wrote: x x x In People vs. Uy, we ruled that a forensic chemist is a public officer and as such, his report carries the presumption of regularity in the performance of his function and duties. Corollarily, under Section 44 of Rule 130, x x x entries in official records made in the performance of official duty are prima facie evidence of the facts therein stated. Omeros reports that the seven sachets of white crystalline substance were positive for methylamphetamine hydrochloride or shabu are, therefore, conclusive in the absence of evidence proving the contrary, as in this case. Second, it must be stressed that Atty. Enriquez raises his objection to the Initial Laboratory Report and Chemistry Report No. D1585-00 only now. He should have objected to their admissibility at the time they were being offered. Otherwise, the objection shall be considered waived and such evidence will form part of the records of the case as competent and admissible evidence. The familiar rule in this jurisdiction is that the admissibility of certain documents x x x cannot be raised for the first time on appeal.367[31] (Emphasis added.) It should be pointed out, however, that the Bandang ruling was cast against a different backdrop where: (1) the seized crystalline substance was the same item examined and tested positive for shabu and presented in court, implying that the identity and integrity of prohibited drug was safeguarded throughout, a circumstance not obtaining in this case; (2) there was a compelling reason for not presenting the examining forensic chemist, i.e., the parties stipulated that the confiscated seven plastic bags have been identified and examined and that the chemist stated in his report that the substance is positive for shabu. In this case, C/I Geronimos resignation from the service is not, standing alone, a justifying factor for the prosecution to dispense with her testimony; and (3) accused Bandang, et al. did not raise any objection to the chemical report during trial, unlike here where accused-appellant objected to Inspector Trias competency to testify on the Geronimo chemical report. At any rate, Inspector Trias testimony on, and the presentation of, the chemistry report in question only established, at best, the existence, due execution, and authenticity of the results of the chemistry analysis.368[32] It does not prove compliance with the requisite chain of custody over the confiscated substance from the time of seizure of the evidence. In this regard, the Court in effect stated in Malillin that unless the state can show by records or testimony that the integrity of the evidence has not been compromised by accounting for the continuous whereabouts of the object evidence at least between the time it came into the possession of the police officers until it was tested in the laboratory,369[33] then the prosecution cannot maintain that it was able to prove the guilt of the accused beyond reasonable doubt. So it was that in
367[31] 368[32]

G.R. No. 151314, June 3, 2004, 430 SCRA 570, 586-587. Sanchez, supra note 19. 369[33] Supra note 21, at 634.

344

345

People v. Kimura the Court said that in establishing the corpus delicti, proof beyond reasonable doubt demands that unwavering exactitude370[34] be observed, a demand which may be addressed by hewing to the chain-of-custody rule. Evidently, the prosecution has not proved that the substance seized in front of the McDonalds was the same substance adduced in evidence as an indispensable element of corpus delicti of the crime, which failure produces a serious doubt as to accusedappellants guilt.371[35] Both the trial and appellate courts made much of the presumption of regularity in the performance of official functions both with respect to the acts of PO3 Ramos and other PNP personnel at Camp Vicente Lim. To a point, the reliance on the presumptive regularity is tenable. This presumption is, however, disputable and may be overturned by affirmative evidence of irregularity or failure to perform a duty; 372[36] any taint of irregularity vitiates the performance and negates the presumption. And as earlier discussed, the buy bust team committed serious lapses in the handling of the prohibited item from the very start of its operation, the error of which the PNP R-IV command later compounded. The Court need not belabor this matter anew. Lest it be overlooked, the presumption of regularity in the performance of official duty always yields to the presumption of innocence and does not constitute proof beyond reasonable doubt.373[37] We held in one case: The presumption of regularity in the performance of official duty cannot be used as basis for affirming accused-appellants conviction because, [f]irst, the presumption is precisely just thata mere presumption. Once challenged by evidence, as in this case, x x x [it] cannot be regarded as binding truth. Second, the presumption of regularity in the performance of official functions cannot preponderate over the presumption of innocence that prevails if not overthrown by proof beyond reasonable doubt.374[38] For failure then of the prosecution to establish the guilt of accusedappellant beyond reasonable doubt, she must perforce be exonerated from criminal liability. The facts and the law of the case call for this kind of disposition. But a final consideration. The Court is cognizant of the campaign of the police and other drug enforcement agencies against the growing drug menace in the country. Unfortunately, their best efforts, particularly successful honest-to-goodness buy-bust operations, sometimes still end up in the acquittal of illegal drug manufacturers, distributors, pushers and/or lesser players, even when nabbed in flagrante, simply because drug enforcement operatives tend to compromise the integrity and evidentiary worth of the seized illegal items. This aberration is oftentimes in turn attributable to the unfamiliarity of police operatives of extant rules and procedures governing the custody, control, and handling of seized drugs. This is, thus, an opportune time to remind all concerned about these rules and
G.R. No. 130805, April 27, 2004, 428 SCRA 51, 70. Id. 372[36] Sevilla v. Cardenas, G.R. No. 167684, July 31, 2006, 497 SCRA 428, 443; citing Mabsucang v. Judge Balgos, 446 Phil. 217, 224 (2003). 373[37] People v. Caete, G.R. No. 138400, July 11, 2002, 384 SCRA 411, 424. 374[38] People v. Tan, G.R. No. 129376, May 29, 2002, 382 SCRA 419, 444.
371[35] 370[34]

345

346

procedures and the guiding jurisprudence. And to put things in the proper perspective, non-compliance with the legal prescriptions of the Dangerous Drugs Act, as amended, is, as we made abundantly clear in People v. Sanchez, not necessarily fatal to the prosecution of drugrelated cases; that police procedures may still have some lapses. These lapses, however, must be recognized, addressed, and explained in terms of their justifiable grounds, and the integrity and evidentiary value of the evidence seized must be shown to have been preserved by the apprehending officer or team. To be forewarned is to be forearmed. WHEREFORE, the CA Decision dated July 19, 2007 in CA-G.R. CR-H.C. No. 00476, affirming that of the RTC, Branch 53 in Manila which found her guilty of violating Sec. 15, Art. III of RA 6425 and imposed upon her the penalty of reclusion perpetua and a fine of PhP 500,000, is hereby REVERSED and SET ASIDE. Accused-appellant Monalyn Cervantes y Solar is ACQUITTED on the ground of reasonable doubt and is accordingly immediately RELEASED from custody unless she is being lawfully held for some lawful cause. Presumption of innocence leads to the accuseds acquittal due to inconsistent testimonies of prosecutions witnesses Inconsistent testimonies of prosecutions witnesses entitles the accused to acquittal based on his constitutional presumption of innocence. ELY AGUSTIN VS. PEOPLE OF THE PHILIPPINES, G.R. No. 158788, April 30, 2008 FACTS: On October 1, 1995, at 7:20 in the evening, armed men robbed the house of spouses George and Rosemarie Gante in Barangay Pug-os, Cabugao, Ilocos Sur, forcibly taking with them several valuables, including cash amounting to P600,000.00.375[3] Forthwith, the spouses reported the matter to the police, who, in turn, immediately applied for a search warrant with the Municipal Trial Court (MTC) of Cabugao, Ilocos Sur.376[4] The MTC issued Search Warrant No. 595,377[5] directing a search of the items stolen from the victims, as well as the firearms used by the perpetrators. One of the target premises was the residence of petitioner, named as one of the several suspects in the crime. On October 6, 1995, armed with the warrant, policemen searched the premises of petitioner's house located in Sitio Padual, Barangay Pug-os, Cabugao, Ilocos Sur. The search resulted in the recovery of a firearm and ammunitions which had no license nor authority to possess such weapon, and, consequently, the filing of a criminal case, docketed as Criminal Case No. 1651-K, for violation of P.D. No.

346

347

1866 or Illegal Possession of Firearms, against petitioner before the RTC. Thereafter, trial ensued. The prosecution presented eight witnesses namely: (1) P/Insp. Anselmo Baldovino378[7] (P/Insp. Baldovino), a police investigator and the applicant for the search warrant; (2) Rosemarie Gante (Gante), the victim of the robbery and private complainant; (3) Ignacio Yabes (Yabes), a Municipal Local Government Operations Officer of the Department of Interior and Local Government who was the civilian witness to the search; (4) P/Supt. Bonifacio Abian379[8] (P/Supt. Abian), Deputy Provincial Director of the Philippine National Police and part of the search team; (5) SPO4 Marino Peneyra (SPO4 Peneyra); (6) SPO1 Franklin Cabaya (SPO1 Cabaya); (7) SPO1 James Jara (SPO1 Jara); and (8) SPO2 Florentino Renon (SPO2 Renon). The prosecution's case centered mainly on evidence that during the enforcement of the search warrant against petitioner, a .38 caliber revolver firearm was found in the latter's house.380[9] In particular, SPO1 Cabaya testified that while poking at a closed rattan cabinet near the door, he saw a firearm on the lower shelf.381[10] The gun is a .38 caliber revolver382[11] with five live ammunitions,383[12] which he immediately turned over to his superior, P/Insp. Baldovino.384[13] Petitioner anchored his defense on denial and frame-up. The petitioner and his wife Lorna assert that petitioner does not own a gun.385[14] Lorna testified that she saw a military man planting the gun.386[15] After trial, the RTC rendered its Decision387[16] dated July 7, 1999, finding petitioner guilty beyond reasonable doubt. Petitioner filed an appeal with the CA, which rendered the assailed Decision388[18] dated January 22, 2003, affirming with modification the decision of the trial court, thus: WHEREFORE, except for the MODIFICATION reducing and changing the maximum of the prison term imposed to Five (5) Years Four (4) Months and Twenty (20) Days, the appealed Decision is otherwise AFFIRMED.

347

348

Hence, the instant Petition for Review, on the principal ground that the CA gravely erred in finding that the guilt of petitioner has been proven beyond reasonable doubt; and more specifically, in giving weight and credence to the testimonies of the police officers who searched the house of the petitioner which are replete with material and irreconcilable contradictions and in giving SPO1 Cabaya the presumption of regularity in the performance of duty despite the claim of Lorna that the .38 caliber revolver was planted. Petitioner insists that the trial court and the CA committed reversible error in giving little credence to his defense that the firearm found in his residence was planted by the policemen. He also alleges material inconsistencies in the testimonies of the policemen as witnesses for the prosecution, which amounted to failure by the prosecution to prove his guilt beyond reasonable doubt. HELD: The petition has merit. The paramount issue in the present case is whether the prosecution established the guilt of petitioner beyond reasonable doubt; and in the determination thereof, a factual issue, that is, whether a gun was found in the house of petitioner, must necessarily be resolved. It is a well-entrenched rule that appeal in criminal cases opens the whole case wide open for review.389[20] In convicting petitioner, the RTC relied heavily on the testimony of SPO1 Cabaya, who testified that he discovered the subject firearm in a closed cabinet inside the former's house. The trial court brushed aside petitioner's defense of denial and protestations of frame-up. The RTC justified giving full credence to Cabaya's testimony on the principles that the latter is presumed to have performed his official duties regularly; that he had no ill motive to frame-up petitioner; and that his affirmative testimony is stronger than petitioner's negative testimony.390[21] Weighing these findings of the lower courts against the petitioner's claim that the prosecution failed to prove its case beyond reasonable doubt due to the material inconsistencies in the testimonies of its witnesses, the Court finds, after a meticulous examination of the records that the lower courts, indeed, committed a reversible error in finding petitioner guilty beyond reasonable doubt of the crime he was charged with. The RTC and the CA have overlooked certain facts and circumstances that would have interjected serious apprehensions absolutely impairing the credibility of the witnesses for the prosecution. The conflicting testimonies of the prosecution witnesses as to who actually entered the house and conducted the search, who discovered the gun, and who witnessed the discovery are material matters because they relate directly to a fact in issue; in the present case, whether a gun has been found in the house of petitioner; or to a fact to which, by the process of logic, an inference may be made as to the existence or non-existence of a fact in issue.391[24] As held in United States

348

349

v. Estraa,392[25] a material matter is the main fact which is the subject of inquiry or any circumstance which tends to prove that fact or any fact or circumstance which tends to corroborate or strengthen the testimony relative to the subject of inquiry or which legitimately affects the credit of any witness who testifies. The evidence of prosecution is severely weakened by several contradictions in the testimonies of its witnesses. Especially damaged is the credibility of SPO1 Cabaya, none of whose declarations on material points jibes with those of the other prosecution witnesses. In the face of the vehement and consistent protestations of frame-up by petitioner and his wife, the trial court and the CA erred in overlooking or misappreciating these inconsistencies. The inconsistencies are material as they delve into the very bottom of the question of whether or not SPO1 Cabaya really found a firearm in the house of petitioner. SPO1 Cabaya testified that he entered the house with four other policemen, among whom were SPO1 Jara, SPO4 Peneyra, SPO3 Bernabe Ocado (SPO3 Ocado) and another one whose name he does not remember.393[26] While searching, he discovered the firearm in the kitchen, inside a closed cabinet near the door.394[27] He said that SPO1 Jara was standing right behind him, at a distance of just one meter, when he (Cabaya) saw the firearm;395[28] and that he picked up the gun, held it and showed it to SPO1 Jara.396[29] He asserted that SPO2 Renon was not one of those who went inside the house.397[30] However, SPO1 Jara, the best witness who could have corroborated SPO1 Cabaya's testimony, related a different story as to the circumstances of the firearm's discovery. SPO1 Jara testified that he merely conducted perimeter security during the search and did not enter or participate in searching the house.398[32] SPO1 Jara testified that he remained outside the house throughout the search, and when SPO1 Cabaya shouted and showed a gun, he was seven to eight meters away from him.399[33] He could not see the inside of the house and could see Cabaya only from his chest up.400[34] He did not see the firearm at the place where it was found, but saw it only when Cabaya raised his arm to show the gun, which was a revolver.401[35] He is certain that he was not with Cabaya at the time the latter discovered the firearm.402[36] He further testified that SPO3 Ocado, who, according to SPO1 Cabaya was one of those near him when he (Cabaya)

349

350

discovered the firearm, stayed outside and did not enter or search the house.403[37] P/Insp. Baldovino testified that only SPO2 Renon conducted the search and entered the house together with SPO1 Cabaya,404[38] directly contradicting SPO1 Cabaya's testimony that he, together with SPO1 Jara, SPO4 Peneyra, SPO3 Ocado, and another one whose name he cannot recall, were inside the house when he discovered the gun405[39] and that SPO2 Renon did not enter the house of petitioner.406[40] The testimonies of the other prosecution witnesses further muddled the prosecution evidence with more inconsistencies as to matters material to the determination of whether a gun had in fact been found in the house of petitioner. SPO4 Peneyra testified that Yabes stayed outside of the during the search;407[59] whereas SPO1 Jara testified that Yabes was inside, at the sala, but the latter saw the gun only when SPO1 Cabaya raised it.408[60] Although the Court has held that frame-up is inherently one of the weakest defenses,409[61] as it is both easily concocted and difficult to prove,410[62] in the present case, the lower courts seriously erred in ignoring the weakness of the prosecution's evidence and its failure to prove the guilt of petitioner beyond reasonable doubt. The rule requiring a claim of frame-up to be supported by clear and convincing evidence411[63] was never intended to shift to the accused the burden of proof in a criminal case. As the Court held in People of the Philippines v. Ambih:412[64] [W]hile the lone defense of the accused that he was the victim of a frame-up is easily fabricated, this claim assumes importance when faced with the rather shaky nature of the prosecution evidence. It is well to remember that the prosecution must rely, not on the weakness of the defense evidence, but rather on its own proof which must be strong enough to convince this Court that the prisoner in the dock deserves to be punished. The constitutional presumption is that the accused is innocent even if his defense is weak as long as the prosecution is not strong enough to convict him.413[65] (Emphasis supplied) In People of the Philippines v. Gonzales,414[66] the Court held that where there was material and unexplained inconsistency between the testimonies of two

350

351

principal prosecution witnesses relating not to inconsequential details but to the alleged transaction itself which is subject of the case, the inherent improbable character of the testimony given by one of the two principal prosecution witnesses had the effect of vitiating the testimony given by the other principal prosecution witness. The Court ruled that it cannot just discard the improbable testimony of one officer and adopt the testimony of the other that is more plausible. In such a situation, both testimonies lose their probative value. The Court further held: Why should two (2) police officers give two (2) contradictory descriptions of the same sale transaction, which allegedly took place before their very eyes, on the same physical location and on the same occasion? We must conclude that a reasonable doubt was generated as to whether or not the "buy-bust" operation ever took place.415[69] In the present case, to repeat, the glaring contradictory testimonies of the prosecution witnesses generate serious doubt as to whether a firearm was really found in the house of petitioner. The prosecution utterly failed to discharge its burden of proving that petitioner is guilty of illegal possession of firearms beyond reasonable doubt. The constitutional presumption of innocence of petitioner has not been demolished and therefore petitioner should be acquitted of the crime he was with. Read also: P. vs. Bernardino, January 28, 1991 1-a. P vs. Flores, 165 SCRA 71 1-b. Aguirre vs. P., 155 SCRA 337 1-c. P. vs. Guinto, 184 SCRA 287 1-d. P. vs. Solis, 182 SCRA 182 1-e. P. vs. Capilitan, 182 SCRA 313 2. Alonso vs. IAC, 151 SCRA 552 3. P vs. Lopez, 74 SCRA 205 4. P vs. Quiason, 78 SCRA 513 5. P vs. Jose, 37 SCRA 450 6. P vs. Poblador, 76 SCRA 634 7. Dumlao vs. Comelec, 95 SCRA 392 3. Presumption of innocence in general and in the order of trial PEOPLE VS. DE LOS SANTOS, 355 SCRA 415 PEOPLE VS. SATURNO, 355 SCRA 578 What is the EQUIPOISE RULE? A. If the evidence in a criminal case is evenly balanced, the constitutional presumption of innocence tilts the scale of justice in favor of the accused and he should be acquitted from the crime charged. Where the inculpatory facts and circumstances are capable of two or more interpretations one of which is consistent with the innocence of the accused and the other consistent with his guilt, then the evidence does not fulfill the test of moral certainty and is not sufficient to support a conviction. Read:

351

352

1. Alejandro vs. Pepito, 96 SCRA 322 3. Sacay vs. Sandiganbayan, July 10,l986 SACAY VS. SANDIGANBAYAN G.R. No. L-66497-98,July 10, 1986 FACTS: 1. At the initial hearing, the testimony of the prosecution witnesses was interrupted when the accused, through counsel, admitted that he shot the deceased but claimed that it was done in self-defense and fulfillment of duty. The prosecution then moved that the reverse procedure be adopted in view of the admission that the accused shot the deceased. No objection was interposed by the accused or his counsel. 2. On appeal with the S.C. after he was convicted the accused later claims that there was a violation of the order of trial provided for in Sec. 3, Rule 119 of the Rules of Court. He also cites the case of Alejandro vs. Pepito, 96 SCRA 322, wherein the S.C. ruled that : "It behooved the respondent Judge to have followed the sequence of trial set forth x x x the form of a trial is also a matter of public order and interest; the orderly course of procedure requires that the prosecution should go forward and present all of its proof in the first instance." HELD: The case of Alejandro vs. Pepito is not applicable inasmuch as the accused in the case at bar did not object to the procedure followed. In fact in the said Alejandro case, the Court also stated: "It is true that in the case of U.S. vs. Gaoiran, 17 Phil. 404 (l910), relied upon by the prosecution and the trial Court, the defense has produced its proofs before the prosecution presented its case, and it was held that no substantial rights of the accused were prejudiced. There is one radical difference, however, since in that case no objection was entered in the Court below to the procedure followed in the presentation of proof. In this case, the change in the order of trial made by respondent Judge was promptly and timely objected to by the defense." In fact it should be noted that under the newly adopted 1985 Rules of Criminal Procedure (Sec. 3e), Rule 119)the said procedure is now expressly sanctioned. Thus: "However, when the accused admits the act or omission charged in the complaint or information but interposes a lawful defense, the order of trial may be modified accordingly." 3.Sec. 3(3), Rule 119 , 1985 Rules on Criminal amended. 4. Other cases Read: 1. P vs. Opida, June 13,1986 2. P vs. Tempongko, October 2,1986
352

Procedure , as

353

3. P vs. Drammayo, 42 SCRA 59 4. P vs. Fernando, 145 SCRA 151 5. P vs. Tolentino, 145 SCRA 597 6. Castillo vs. Filtex, September 30,1983 7. Dumlao vs. COMELEC, supra 5. Right to counsel-during trial 1. Reason behind the requirement 2. Obligation of the judge to an accused who without a lawyer to assist him Read: 1. P vs. Holgado,85 Phil. 752 2. Delgado vs. CA, 145 SCRA 357 3. P vs. Cuison, 193 Phil. 296 5-a. The right to be heard by himself and counsel during trial Effect of the fact that accused was represented by a NON-LAWYER at the early part of the trial but a full-pledged lawyer took over as his counsel when he presented his evidence. (Also important in your criminal law as to the distinctions between robbery and grave coercion) PEDRO CONSULTA VS. PEOPLE, G.R. No. 179462, February 12, 2009 appears in court

CARPIO MORALES, J.: The Court of Appeals having, by Decision of April 23, 2007,416[1] affirmed the December 9, 2004 Decision of the Regional Trial Court of Makati City, Branch 139 convicting Pedro C. Consulta (appellant) of Robbery with Intimidation of Persons, appellant filed the present petition. The accusatory portion of the Information against appellant reads: That on or about the 7th day of June, 1999, in the City of Makati, Philippines and within the jurisdiction of this Honorable Court, the abovenamed accused, with intent of gain, and by means of force, violence and intimidation, did then and there willfully, unlawfully and feloniously take, steal and carry away complainants NELIA R. SILVESTRE gold necklace worth P3,500.00, belonging to said complainant, to the damage and prejudice of the owner thereof in the aforementioned amount of P3,500.00. CONTRARY TO LAW.417[2] underscoring supplied) (Emphasis in the original,

From the evidence for the prosecution, the following version is gathered:
416[1]

Penned by Associate Justice Estela M. Perlas-Bernabe, with the concurrence of Associate Justices Marina L. Buzon and Lucas P. Bersamin; CA rollo, pp. 166-176. 417[2] Records, p. 1.

353

354

At about 2:00 oclock in the afternoon of June 7, 1999, private complainant Nelia R. Silvestre (Nelia), together with Maria Viovicente (Maria) and Veronica Amar (Veronica), boarded a tricycle on their way to Pembo, Makati City. Upon reaching Ambel Street, appellant and his brother Edwin Consulta (Edwin) blocked the tricycle and under their threats, the driver alighted and left. Appellant and Edwin at once shouted invectives at Nelia, saying Putang ina mong matanda ka, walanghiya ka, kapal ng mukha mo, papatayin ka namin. Appellant added Putang ina kang matanda ka, wala kang kadala dala, sinabihan na kita na kahit saan kita matiempuhan, papatayin kita. Appellant thereafter grabbed Nelias 18K gold necklace with a crucifix pendant which, according to an alajera in the province, was of 18k gold, and which was worth P3,500, kicked the tricycle and left saying Putang ina kang matanda ka! Kayo mga nurses lang, anong ipinagmamalaki niyo, mga nurses lang kayo. Kami, marami kaming mga abogado. Hindi niyo kami maipapakulong kahit kailan! Nelia and her companions immediately went to the Pembo barangay hall where they were advised to undergo medical examination. They, however, repaired to the Police Station, Precinct 8 in Comembo, Makati City and reported the incident. They then proceeded to Camp Crame where they were advised to return in a few days when any injuries they suffered were expected to manifest. Nine days after the incident or on June 16, 1999, Nelia submitted a medico-legal report and gave her statement before a police investigator. Denying the charge, appellant branded it as fabricated to spite him and his family in light of the following antecedent facts: He and his family used to rent the ground floor of Nelias house in Pateros. Nelia is his godmother. The adjacent house was occupied by Nelias parents with whom she often quarreled as to whom the rental payments should be remitted. Because of the perception of the parents of Nelia that his family was partial towards her, her parents disliked his family. Nelias father even filed a case for maltreatment against him which was dismissed and, on learning of the maltreatment charge, Nelia ordered him and his family to move out of their house and filed a case against him for grave threats and another for light threats which were dismissed or in which he was acquitted. Appellant went on to claim that despite frequent transfers of residence to avoid Nelia, she would track his whereabouts and cause scandal. Appellants witness Darius Pacaa testified that on the date of the alleged robbery, Nelia, together with her two companions, approached him while he was at Ambel Street in the company of Michael Fontanilla and Jimmy Sembrano, and asked him (Pacaa) if he knew a bald man who is big/stout with a big tummy and with a sister named Maria. As he replied in the affirmative, Nelia at once asked him to accompany them to appellants house, to which he acceded. As soon as the group reached appellants house, appellant, on his (Pacaas) call, emerged and on seeing the group, told them to go away so as not to cause trouble. Retorting, Nelia uttered Mga hayop kayo, hindi ko kayo titigilan.
354

355

Another defense witness, Thelma Vuesa, corroborated Pacaas account. The trial court, holding that intent to gain on appellants part is presumed from the unlawful taking of the necklace, and brushing aside appellants denial and claim of harassment, convicted appellant of Robbery, disposing as follows: WHEREFORE, premises considered, this Court finds accused PEDRO C. CONSULTA guilty beyond reasonable doubt, as principal of the felony of Robbery with Intimidation of Persons defined and penalized under Article 294, paragraph No. 5, in relation to Article 293 of the Revised Penal Code and hereby sentences him to suffer the penalty of imprisonment from one (1) year, seven (7) months and eleven (11) days of arresto mayor, as minimum, to eight (8) years, eight (8) months and one (1) day of prision mayor, as maximum, applying the Indeterminate Sentence Law, there being no mitigating or aggravating circumstances which attended the commission of the said crime. The said accused is further ordered to pay unto the complainant Nelia Silvestre the amount of P3,500.00 representing the value of her necklace taken by him and to pay the costs of this suit. SO ORDERED. (Italics in the original, underscoring supplied) The appellate court affirmed appellants conviction with modification on the penalty. In his present appeal, appellant raises the following issues: (1) (2) Whether or not appellant was validly arraigned; Whether or not appellant was denied due process having been represented by a fake lawyer during arraignment, pre-trial and presentation of principal witnesses for the prosecution; The first two issues, which appellant raised before the appellate court only when he filed his Motion for Reconsideration of said courts decision, were resolved in the negative in this wise: On the matter of accused-appellants claim of having been denied due process, an examination of the records shows that while accusedappellant was represented by Atty. Jocelyn P. Reyes, who seems not a lawyer, during the early stages of trial, the latter withdrew her appearance with the conformity of the former as early as July 28, 2000 and subsequently, approved by the RTC in its Order dated August 4, 2000. Thereafter, accused-appellant was represented by Atty. Rainald C. Paggao from the Public Defenders (Attorneys) Office of Makati City. Since the accused-appellant was already represented by a member of the Philippine Bar who principally handled his defense, albeit unsuccessfully, then he cannot now be heard to complain about having been denied of due process.418[3] (Underscoring supplied)

418[3]

Rollo, p. 169

355

356

That appellants first counsel may not have been a member of the bar does not dent the proven fact that appellant prevented Nelia and company from proceeding to their destination. Further, appellant was afforded competent representation by the Public Attorneys Office during the presentation by the prosecution of the medico-legal officer and during the presentation of his evidence. People v. Elesterio419[4] enlightens: As for the circumstance that the defense counsel turned out later to be a non-lawyer, it is observed that he was chosen by the accused himself and that his representation does not change the fact that Elesterio was undeniably carrying an unlicensed firearm when he was arrested. At any rate, he has since been represented by a member of the Philippine bar, who prepared the petition for habeas corpus and the appellants brief. (Underscoring supplied) EVEN IF THE DECISION OF CONVICTION IS ALREADY FINAL AND EXECUTORY, THE CASE MAY STILL BE RE-OPENED IF THERE IS GROSSNEGLIGENCE ON THE PART OF ACCUSEDS COUNSEL THEREBY VIOLATING HIS RIGHT TO DUE PROCESS/COUNSEL. JOHN HILARIO VS. PEOPLE OF THE PHILIPPINES, G.R. No. 161070, April 14, 2008 THE FACTS: Petitioner, together with one Gilbert Alijid (Alijid), was charged with two counts420[3] of Murder in the Regional Trial Court (RTC), Branch 76, Quezon City to which petitioner, assisted by counsel de parte, pleaded not guilty. During trial, Atty. Raul Rivera of the Public Attorney's Office (PAO), counsel of Alijid, took over representing petitioner in view of the death of the latter's counsel. On December 5, 2001, the RTC rendered its Decision421[4] finding petitioner and his co-accused Alijid guilty beyond reasonable doubt of the crime of homicide and sentencing them to suffer imprisonment of eight (8) years and one (1) day of prision mayor to fourteen (14) years and eight (8) months of reclusion temporal in each count. On May 10, 2002, petitioner, this time unassisted by counsel, filed with the RTC a Petition for Relief422[5] from the Decision dated December 5, 2001 together with an affidavit of merit. In his petition, petitioner contended that at the time of the promulgation of the judgment, he was already confined at Quezon City Jail and was directed to be committed to the National Penitentiary in Muntinlupa; that he had no way of personally filing the notice of appeal thus he instructed his lawyer to file it on his behalf; that he had no choice but to repose his full trust and confidence to his lawyer; that he had instructed his lawyer to file the necessary motion for reconsideration or notice of appeal; that on May 2, 2002, he was already incarcerated at the New Bilibid Prisons, Muntinlupa City and learned from the
419[4]
420[3] 421[4] 422[5]

G.R. No. 63971, May 9, 1989, 173 SCRA 243, 249.


Docketed as Criminal Case Nos. Q-00-91647-48. Penned by Judge Monina A. Zenarosa, rollo, pp. 36-52. Id. at 53-60.

356

357

grapevine of his impending transfer to the Iwahig Penal Colony, Palawan; that believing that the notice of appeal filed by his counsel prevented the Decision dated December 5, 2001 from becoming final to warrant his transfer, he instructed his representative to get a copy of the notice of appeal from the RTC; that no notice of appeal was filed by his lawyer in defiance of his clear instructions; and that the RTC Decision showed that it was received by his counsel on February 1, 2002 and yet the counsel did not inform him of any action taken thereon. I S S U E: Whether or not the delay in appealing the instant case due to the defiance or failure of the petitioner's counsel de oficio to seasonably file a Notice of Appeal, constitutes excusable negligence to entitle the undersigned detention prisoner/ petitioner to pursue his appeal? Whether or not pro hac vice, the mere invocation of justice warrants the review of a final and executory judgment? HELD: Petitioner contends that the negligence of his counsel de oficio cannot be binding on him for the latter's defiance of his instruction to appeal automatically breaks the fiduciary relationship between counsel-client and cannot be against the client who was prejudiced; that this breach of trust cannot easily be concocted in this situation considering that it was a counsel de oficio, a lawyer from PAO, who broke the fiduciary relationship; that the assailed CA Resolutions both harped on technicalities to uphold the dismissal by the RTC of his petition for relief; that reliance on technicalities to the prejudice of petitioner who is serving 14 years imprisonment for a crime he did not commit is an affront to the policy promulgated by this Court that dismissal purely on technical grounds is frowned upon especially if it will result to unfairness; and that it would have been for the best interest of justice for the CA to have directed the petitioner to complete the records instead of dismissing the petition outright. In his Comment, the OSG argues that the mere invocation of justice does not warrant the review of an appeal from a final and executory judgment; that perfection of an appeal in the manner and within the period laid down by law is not only mandatory but jurisdictional and failure to perfect the appeal renders the judgment sought to be reviewed final and not appealable; and that petitioner's appeal after the finality of judgment of conviction is an exercise in futility, thus the RTC properly dismissed petitioner's petition for relief from judgment. The OSG further claims that notice to counsel is notice to clients and failure of counsel to notify his client of an adverse judgment would not constitute excusable negligence and therefore binding on the client. We grant the petition. A litigant who is not a lawyer is not expected to know the rules of procedure. In fact, even the most experienced lawyers get tangled in the web of procedure.423[12] We have held in a civil case that to demand as much from ordinary citizens whose only compelle intrare is their sense of right would turn the legal system into an intimidating monstrosity where an individual may be stripped of his property rights not because he has no right to the property but because he does not know how to establish such

423[12]

See Telan v. Court of Appeals, G.R. No. 95026, October 4, 1991, 202 SCRA 534, 541.

357

358

right.424[13] This finds application specially if the liberty of a person is at stake. As we held in Telan v. Court of Appeals: The right to counsel in civil cases exists just as forcefully as in criminal cases, especially so when as a consequence, life, liberty, or property is subjected to restraint or in danger of loss. In criminal cases, the right of an accused person to be assisted by a member of the bar is immutable. Otherwise, there would be a grave denial of due process. Thus, even if the judgment had become final and executory, it may still be recalled, and the accused afforded the opportunity to be heard by himself and counsel. xxxx Even the most experienced lawyers get tangled in the web of procedure. The demand as much from ordinary citizens whose only compelle intrare is their sense of right would turn the legal system into an intimidating monstrosity where an individual may be stripped of his property rights not because he has no right to the property but because he does not know how to establish such right. The right to counsel is absolute and may be invoked at all times. More so, in the case of an on-going litigation, it is a right that must be exercised at every step of the way, with the lawyer faithfully keeping his client company. No arrangement or interpretation of law could be as absurd as the position that the right to counsel exists only in the trial courts and that thereafter, the right ceases in the pursuit of the appeal.425[14] (Emphasis supplied) To repeat the ruling in Telan, no arrangement or interpretation of law could be as absurd as the position that the right to counsel exists only in the trial courts and that thereafter, the right ceases in the pursuit of the appeal.426[15] It is even more important to note that petitioner was not assisted by counsel when he filed his petition for relief from judgment with the RTC. It cannot be overstressed therefore, that in criminal cases, as held in Telan, the right of an accused person to be assisted by a member of the bar is immutable; otherwise, there would be a grave denial of due process. Cases should be determined on the merits after full opportunity to all parties for ventilation of their causes and defenses, rather than on technicality or some procedural imperfections. In that way, the ends of justice would be served better.427[16] While as a general rule, the failure of petitioner to file his motion for reconsideration within the 15-day reglementary period fixed by law rendered the resolution final and executory, we have on some occasions relaxed this rule. Thus, in Barnes v. Padilla428[17] we held: However, this Court has relaxed this rule in order to serve substantial justice considering (a) matters of life, liberty, honor or property, (b) the
424[13] 425[14] 426[15] 427[16] 428[17]

Id. Id. at 540-541. Id. at 541. Garcia v. Philippine Airlines, Inc., supra note 11, at 781.
G.R. No. 160753, September 30, 2004, 439 SCRA 675.

358

359

existence of special or compelling circumstances, (c) the merits of the case, (d) a cause not entirely attributable to the fault or negligence of the party favored by the suspension of the rules, (e) a lack of any showing that the review sought is merely frivolous and dilatory, and (f) the other party will not be unjustly prejudiced thereby. Invariably, rules of procedure should be viewed as mere tools designed to facilitate the attainment of justice. Their strict and rigid application, which would result in technicalities that tend to frustrate rather than promote substantial justice, must always be eschewed. Even the Rules of Court reflects this principle. The power to suspend or even disregard rules can be so pervasive and compelling as to alter even that which this Court itself had already declared to be final. In De Guzman v. Sandiganbayan, this Court, speaking through the late Justice Ricardo J. Francisco, had occasion to state: The Rules of Court was conceived and promulgated to set forth guidelines in the dispensation of justice but not to bind and chain the hand that dispenses it, for otherwise, courts will be mere slaves to or robots of technical rules, shorn of judicial discretion. That is precisely why courts in rendering justice have always been, as they ought to be guided by the norm that when on the balance, technicalities take a backseat against substantive rights, and not the other way around. Truly then, technicalities, in the appropriate language of Justice Makalintal, "should give way to the realities of the situation. Indeed, the emerging trend in the rulings of this Court is to afford every party litigant the amplest opportunity for the proper and just determination of his cause, free from the constraints of technicalities.429[18] Rules of procedure are mere tools designed to expedite the decision or resolution of cases and other matters pending in court. A strict and rigid application of rules that would result in technicalities that tend to frustrate rather than promote substantial justice must be avoided.430[21] Even if the judgment had become final and executory, it may still be recalled, and the accused afforded the opportunity to be heard by himself and counsel.431[22] However, instead of remanding the case to the CA for a decision on the merits, we opt to resolve the same so as not to further delay the final disposition of this case. In all criminal prosecutions, the accused shall have the right to appeal in the manner prescribed by law. The importance and real purpose of the remedy of appeal has been emphasized in Castro v. Court of Appeals432[27] where we ruled that an appeal is an essential part of our judicial system and trial courts are advised to proceed with caution so as not to deprive a party of the right to appeal and instructed that every party-litigant should be afforded the amplest opportunity for the proper and just disposition of his cause, freed from the constraints of technicalities. While this right is statutory, once it is granted by law, however, its suppression would be a violation of due process, a right guaranteed by the Constitution. Thus, the importance of finding out whether petitioner's loss of the right to appeal was due to the PAO lawyer's negligence and not at all attributed
429[18] 430[21] 431[22]

Id. at 686-687. Cusi-Hernandez v. Spouses Diaz, 390 Phil. 1245, 1252 (2000). Telan v. Court of Appeals, supra note 12, at 540-541; People of the Philippines v. Holgado, 85 Phil. 752, 756-757 (1950); Flores v. Judge Ruiz, 179 Phil. 351, 355 (1979); Delgado v. Court of Appeals, 229 Phil. 362, 366 (1986).

359

360

to petitioner.

PEOPLE VS. NADERA, JR., 324 SCRA 490 Mendoza, J. The cavalier attitude of Atty. Manolo Brotonel of the PAO cannot go unnoticed. It is discernible in [a] his refusal to cross-examine Oleby Nadera (the complainant for RAPE); [b] the manner in which he conducted Maricris Naderas cross-examination; and [c] his failure not only to present evidence for the accused but to inform the accused of his right to do so, if he desires. Only the faithful performance by counsel of his duty towards his client can give meaning and substance to the accuseds right to due process and to be presumed innocent until proven otherwise. Hence, a lawyers duty, especially that of a defense counsel, must not be taken lightly. It must be performed with all the zeal and vigor at his command to protect and safeguard the accuseds fundamental rights. It may be so that the defense counsel really found Olebys testimony to be believable. Nonetheless, he had the bounden duty to scrutinize private complainants testimony to ensure that the accuseds constitutional right to confront and examine the witnesses against him was not rendered for naught. It bears pointing out that in rape cases, it is often the words of the complainant against the accused, the two being the only persons present during the commission of the crime. This is so because the complainants testimony cannot be accepted with precipitate credulity without denying the accuseds constitutional right to be presumed innocent. This is where cross-examination becomes essential to test the credibility of the witnesses, expose falsehoods or half-truths, uncover the truth which rehearsed direct examination testimonies may successfully suppress, and demonstrate inconsistencies in substantial matters which create reasonable doubt as to the guilt of the accused and thus give substance to the constitutional right of the accused to confront the witnesses against him. For unless proven otherwise to be guilty beyond reasonable doubt, the accused is presumed innocent. (NOTE: For your Legal & Judicial Ethics) Atty. Brotonel as counsel de oficio, had the duty to defend his client and protect his rights, no matter how guilty or evil he perceives accused-appellant to be. The performance of this duty was all the more imperative because the life of the accused-appellant hangs in the balance. His duty was no less because he was counsel de oficio. The Decision of the RTC convicting the accused is SET ASIDE and the case is remanded for further proceedings consistent with this decision. Read: 1. P vs. Dischoso, 96 SCRA 957 2. Read also:

360

361

PEOPLE VS. YAMBOT, G.R. NO. 120350, 343 SCRA 20, OCT. 30, 2000; PEOPLE VS. BANIHIT, G.R. NO. 132045, 339 SCRA 86, AUG. 25, 2000. Right to be Heard by himself and counsel and to present evidence for his defense. In this case, the non-appearance of counsel for the accused on the scheduled hearing was not construed as waiver by the accused of his right to present evidence for his defense. Denial of due process can be successfully invoked where no valid waiver of rights had been made as in this case. In another case, the accused-appellant validly waived his right to present evidence. This is in consonance with the doctrine that everyone has a right to waive the advantage of a law or rule made solely for the benefit and protection of the individual in his private capacity, if it can be dispensed with and relinquished without infringing on any public right, and without detriment to the community at large. 6. The right to be present during trial Read: 1. Aquino vs. Military Commission, 63 SCRA 546 2. P vs. Judge, 125 SCRA 269 3. Waiver of the defendant's presence in a criminal prosecution,77 SCRA 430 The right to a speedy trial; not a case of; Requisites of double jeopardy; DANTE TAN VS. PEOPLE, G.R. No. 173637, April 21, 2009 CHICO-NAZARIO, J.: Before this Court is a Petition for Review on Certiorari filed under Rule 45 of the Revised Rules of Court seeking the reversal and setting aside of the Decision433[1] dated 22 February 2006 and Resolution434[2] dated 17 July 2006 issued by the Court of Appeals in CA-G.R. SP No. 83068 entitled, People of the Philippines v. Hon. Briccio C. Ygana, in his capacity as Presiding Judge of Branch 153, Regional Trial Court, Pasig City and Dante Tan. The assailed Decision reinstated Criminal Case No. 119830, earlier dismissed by the trial court due to an alleged violation of petitioner Dante T. Tans right to speedy trial. The assailed Resolution denied his Motion for Reconsideration and Motion to Inhibit. The factual and procedural antecedents of the instant petition are as follows:

433[1]

434[2]

Penned by Associate Justice Fernanda Lampas Peralta with Associate Justices Josefina GuevaraSalonga and Sesinando E. Villon, concurring; rollo, pp. 90-100. Id. at 102-112.

361

362

On 19 December 2000, a Panel of Prosecutors of the Department of Justice (DOJ), on behalf of the People of the Philippines (People), filed three Informations against Dante T. Tan (petitioner) before the Regional Trial Court (RTC) of Pasig City. The cases were docketed as Criminal Cases No. 119830, No. 119831 and No. 119832, all entitled, People of the Philippines v. Dante Tan. Criminal Case No. 119830435[3] pertains to allegations that petitioner employed manipulative devises in the purchase of Best World Resources Corporation (BW) shares. On the other hand, Criminal Cases No. 119831436[4] and No. 119832437[5] involve the alleged failure of petitioner to file with the Securities and Exchange Commission (SEC) a sworn statement of his beneficial ownership of BW shares. In two other related cases, two Informations were filed against a certain Jimmy Juan and Eduardo G. Lim for violation of the Revised Securities Act involving BW shares of stock. These were docketed as Criminal Cases No. 119828 and No. 119829. On the same day, the DOJ, through Assistant Chief State Prosecutor Nilo C. Mariano, filed a Motion for Consolidation praying that Criminal Cases No. 119830, No. 119831 and No. 119832 be consolidated together with Criminal Cases No. 119828 and No. 119829, which the trial court granted. On 21 December 2000, Criminal Cases No. 119830, No. 119831 and No. 119832 were raffled off to the Pasig RTC, Branch 153, presided by Judge Briccio C. Ygana. Criminal Cases No. 119828 and No. 119829 also went to the same court. Petitioner was arraigned on 16 January 2001, and pleaded not guilty to the charges.438[6] On 6 February 2001, the pre-trial was concluded, and a pre-trial order set, among other things, the first date of trial on 27 February 2001.439[7] Atty. Celia Sandejas of the Securities and Exchange Commission (SEC), under the direct control and supervision of Public Prosecutor Nestor Lazaro, entered her appearance for the People; Atty. Agnes Maranan for petitioner Dante Tan; Atty. Sigfrid Fortun for Eduardo Lim, Jr.; and Atty. Rudolf Brittanico for Jimmy Juan. State Prosecutors Susan Dacanay and Edna Villanueva later on took over as lawyers for the People. The People insists that during the pendency of the initial hearing on 27 February 2001, the parties agreed that Criminal Cases No. 119831 and No. 119832 would be tried ahead of Criminal Case No. 119830, and that petitioner would not interpose any objection to its manifestation, nor would the trial court disapprove it.

435[3] 436[4] 437[5] 438[6] 439[7]

Id. at 228-230. Id. at 231-232. Id. at 233-235. Records, p. 194. Id. at 253-259.

362

363

Thereafter, the People presented evidence for Criminal Cases No. 119831 and No. 119832. On 18 September 2001, the prosecution completed the presentation of its evidence and was ordered by the RTC to file its formal offer of evidence within thirty days. After being granted extensions to its filing of a formal offer of evidence, the prosecution was able to file said formal offer for Criminal Cases No. 119831 and No. 119832 on 25 November 2003.440[8] On 2 December 2003, petitioner moved to dismiss Criminal Case No. 119830 due to the Peoples alleged failure to prosecute. Claiming violation of his right to speedy trial, petitioner faults the People for failing to prosecute the case for an unreasonable length of time and without giving any excuse or justification for the delay. According to petitioner, he was persistent in asserting his right to speedy trial, which he had allegedly done on several instances. Finally, he claimed to have been substantially prejudiced by this delay. The prosecution opposed the Motion, insisting on its claim that the parties had an earlier agreement to defer the trial of Criminal Case No. 119830 until after that of Criminal Cases No. 119831-119832, as the presentation of evidence and prosecution in each of the five cases involved were to be done separately. The presentation of evidence in Criminal Cases No. 119831-119832, however, were done simultaneously, because they involved similar offenses of non-disclosure of beneficial ownership of stocks proscribed under Rule 36(a)-1441[9] in relation to Sections 32(a)1442[10] and 56443[11] of Batas Pambansa Bilang 178, otherwise known as the Revised Securities Act. Criminal Case No. 119830 pertains to alleged violation of Section 27 (b),444[12] in relation to Section 56 of said act.
440[8] 441[9]

442[10]

443[11]

444[12]

Rollo, pp. 247-253. Section 36. Directors, officers and principal stockholders. (a) Every person who is directly or indirectly the beneficial owner of more than ten per centum of any class of any equity security which is registered pursuant to this Act, or who is a director or an officer of the issuer of such security, shall file, at the time of the registration of such security on a securities exchange or by the effective date of a registration statement or within ten days after he becomes such a beneficial owner, director, or officer, a statement with the Commission and, if such security is registered on a securities exchange, also with the exchange, of the amount of all equity securities of such issuer of which he is the beneficial owner, and within ten days after the close of each calendar month thereafter, if there has been a change in such ownership during such month, shall file with the Commission, and if such security is registered on a securities exchange, shall also file with the exchange, a statement indicating his ownership at the close of the calendar month and such changes in his ownership as have occurred during such calendar month. Section 32. Reports. (a) (1) Any person who, after acquiring directly or indirectly the beneficial ownership of any equity security of a class which is registered pursuant to this Act, is directly or indirectly the beneficial owner of more than ten (10%) per centum of such class shall, within ten days after such acquisition or such reasonable time as fixed by the Commission, submit to the issuer of the security, to the stock exchanges where the security is traded, and to the Commission a sworn statement x x x. Penalties. Any person who violates any of the provisions of this Act, or the rules and regulations promulgated by the Commission under authority thereof, or any person who, in a registration statement filed under this Act, makes any untrue statement of a material fact of omits to state any material fact required to be stated therein or necessary to make the statements therein not misleading, shall, upon conviction, suffer a fine of not less than five thousand (P5,000.00) pesos nor more than five hundred thousand (P500,000.00) pesos or imprisonment of not less than seven (7) years nor more than twenty one (21) years, or both in the discretion of the court. If the offender is a corporation, partnership or association or other juridical entity, the penalty shall be imposed upon the officer or officers of the corporation, partnership, association or entity responsible for the violation, and if such officer is an alien, he shall, in addition to the penalties prescribed, be deported without further proceedings after service of sentence. Section 27. Manipulative and deceptive devices. It shall be unlawful for any person, directly or indirectly, by the use of any facility of any exchange xxxx

363

364

On 22 December 2003, Judge Briccio C. Ygana of the Pasig RTC, Branch 153, ruled that the delays which attended the proceedings of petitioners case (Criminal Case No. 119830) were vexatious, capricious and oppressive, resulting in violation of petitioners right to speedy trial. The RTC ordered445[13] the dismissal of Criminal Case No. 119830, disposing as follows: WHEREFORE, foregoing premises duly considered and finding the motion to dismiss to be meritorious, the Court hereby orders Criminal Case No. 119830 DISMISSED. On motion for reconsideration, the prosecution insisted that the parties agreed to hold separate trials of the BW cases, with petitioner acquiescing to the prosecution of Criminal Cases No. 119831 and No. 119832 ahead of Criminal Case No. 119830. In an Order dated 20 January 2004, the RTC denied the Motion for Reconsideration for lack of merit. The RTCs order of dismissal was elevated to the Court of Appeals via a petition for certiorari, with the People contending that: RESPONDENT JUDGE GRAVELY ABUSED HIS DISCRETION IN RULING THAT THE PEOPLE VIOLATED DANTE TANS RIGHT TO SPEEDY TRIAL, ALBEIT, THE LATTER AND RESPONDENT JUDGE HIMSELF HAVE CONFORMED TO THE DEFERMENT OF CRIMINAL CASE NO. 119830 PENDING HEARING OF THE TWO OTHER RELATED CASES. Setting aside the trial courts order of dismissal, the Court of Appeals granted the petition for certiorari in its Decision dated 22 February 2006. In resolving the petition, the appellate court reinstated Criminal Case No. 119830 in this wise: WHEREFORE, the petition is granted and the assailed Orders dated December 22, 2003 and January 20, 2004 are set aside. Criminal Case No. 119830 is reinstated and the trial court is ordered to conduct further proceedings in said case immediately.446[14] Petitioner moved for a reconsideration of the Decision and filed a motion for inhibition of the Justices who decided the case. On 17 July 2006, the Court of Appeals denied both motions. Petitioner Dante Tan, henceforth, filed the instant petition for review on certiorari, raising the following issues: I. WHETHER OR NOT THE ACTING SECRETARY OF JUSTICE MAY VALIDLY EXECUTE THE CERTIFICATE OF NON-FORUM SHOPPING ATTACHED TO THE PETITION FOR CERTIORARI FILED BY THE PEOPLE WITH THE COURT OF APPEALS EVEN
(b) To use or employ, in connection with the purchase or sale of any security, any manipulative or deceptive device or contrivance. Rollo, pp. 835-855. Id. at 99-100.

445[13] 446[14]

364

365

THOUGH THE CRIMINAL ACTION WAS INSTITUTED BY A COMPLAINT SUBSCRIBED BY THE AUTHORIZED OFFICERS OF THE SECURITIES AND EXCHANGE COMMISSION. II. WHETHER OR NOT THE PETITION FOR CERTIORARI VIOLATED TANS RIGHT AGAINST DOUBLE JEOPARDY. III. WHETHER OR NOT CRIMINAL CASE NO. 119830 WAS CORRECTLY DISMISSED BY THE TRIAL COURT ON THE GROUND OF VIOLATION OF TANS RIGHT TO SPEEDY TRIAL. IV. WHETHER OR NOT THE TRIAL COURT COMMITTED GRAVE ABUSE OF DISCRETION.

We first resolve the preliminary issues. In an attempt at having the instant petition dismissed, petitioner contends that the certificate of non-forum shopping attached to the Peoples appeal before the Court of Appeals should have been signed by the Chairman of the SEC as complainant in the cases instead of Acting DOJ Secretary Merceditas N. Gutierrez. Petitioners argument is futile. The Court of Appeals was correct in sustaining the authority of Acting DOJ Secretary Merceditas Gutierrez to sign the certificate of non-forum shopping of the petition for certiorari before said court. It must be stressed that the certification against forum shopping is required to be executed by the plaintiff.447[15] Although the complaint-affidavit was signed by the Prosecution and Enforcement Department of the SEC, the petition before the Court of Appeals originated from Criminal Case No. 119830, where the plaintiff or the party instituting the case was the People of the Philippines. Section 2, Rule 110 of the Rules of Court leaves no room for doubt and establishes that criminal cases are prosecuted in the name of the People of the Philippines, the offended party in criminal cases. Moreover, pursuant to Section 3, paragraph (2) of the Revised Administrative Code, the DOJ is the executive arm of the government mandated to investigate the commission of crimes, prosecute offenders and administer the probation and correction system. It is the DOJ, through its prosecutors, which is authorized to prosecute criminal cases on behalf of the People of the Philippines.448[16] Prosecutors control and direct the prosecution of criminal offenses, including the conduct of preliminary investigation, subject to review by the Secretary of Justice. Since it is the DOJ which is the government agency tasked to prosecute criminal cases before the trial court, the DOJ is best suited to attest whether a similar or related case has been filed or is pending in another court of tribunal. Acting DOJ Secretary Merceditas N. Gutierrez, being the head of the DOJ, therefore, had the authority to sign

447[15] 448[16]

Regalado, REMEDIAL LAW, p. 729. Revised Administrative Code, Section 3(2).

365

366

the certificate of non-forum shopping for Criminal Case No. 119830, which was filed on behalf of the People of the Philippines. The preliminary issues having been resolved, the Court shall proceed to discuss the main issues. At the crux of the controversy is the issue of whether there was a violation of petitioner Dante Tans right to speedy trial. Petitioner Dante Tan assails the Decision and Resolution of the Court of Appeals in CA-G.R. SP No. 83068. The appellate court determined that he impliedly agreed that Case No. 119830 would not be tried until after termination of Criminal Cases No. 119831-119832, which finding was grounded entirely on speculations, surmises and conjectures. Both parties concede that this issue is factual. It is a basic rule that factual issues are beyond the province of this Court in a petition for review, for it is not our function to review evidence all over again.449[17] Rule 45 of the Rules of Court provides that only questions of law may be raised in this Court in a petition for review on certiorari.450[18] The reason is that the Court is not a trier of facts.451[19] However, the rule is subject to several exceptions.452[20] Under these exceptions, the Court may delve into and resolve factual issues, such as in cases where the findings of the trial court and the Court of Appeals are absurd, contrary to the evidence on record, impossible, capricious or arbitrary, or based on a misappreciation of facts. In this case, the Court is convinced that the findings of the Court of Appeals on the substantial matters at hand, while conflicting with those of the RTC, are adequately supported by the evidence on record. We, therefore, find no reason to deviate from the jurisprudential holdings and treat the instant case differently. An accuseds right to have a speedy, impartial, and public trial is guaranteed in criminal cases by Section 14(2) of Article III of the Constitution. This right to a speedy trial may be defined as one free from vexatious, capricious and oppressive delays, its salutary objective being to assure that an innocent person may be free from the anxiety and expense of a court litigation or, if otherwise, of having his guilt determined within the shortest possible time compatible with the presentation and consideration of whatsoever legitimate defense he may interpose . Intimating historical perspective on the evolution of the right to speedy trial, we reiterate the old legal maxim, justice delayed is justice denied. This oft-repeated adage requires the expeditious resolution of disputes, much more so in criminal cases where an accused is constitutionally guaranteed the right to a speedy trial . Following the policies incorporated under the 1987 Constitution, Republic Act No. 8493, otherwise known as The Speedy Trial Act of 1998, was enacted, with Section 6 of said act limiting the trial period to 180 days from the first day of trial . Aware of problems resulting in the
449[17] 450[18]

451[19]

452[20]

Centeno v. Viray, 440 Phil. 881, 887 (2002). Busmente, Jr. v. National Labor Relations Commission, G.R. No. 73647, 8 April 1991, 195 SCRA 710, 713. Tad-y v. People, G.R. No. 148862, 11 August 2005, 466 SCRA 474, 492; Romago Electric Co., Inc. v. Court of Appeals, 388 Phil. 964, 975 (2000). Palon v. Nino, 405 Phil. 670, 681 (2001).

366

367

clogging of court dockets, the Court implemented the law by issuing Supreme Court Circular No. 38-98, which has been incorporated in the 2000 Rules of Criminal Procedure, Section 2 of Rule 119 . In Corpuz v. Sandiganbayan the Court had occasion to state The right of the accused to a speedy trial and to a speedy disposition of the case against him was designed to prevent the oppression of the citizen by holding criminal prosecution suspended over him for an indefinite time, and to prevent delays in the administration of justice by mandating the courts to proceed with reasonable dispatch in the trial of criminal cases. Such right to a speedy trial and a speedy disposition of a case is violated only when the proceeding is attended by vexatious, capricious and oppressive delays. The inquiry as to whether or not an accused has been denied such right is not susceptible by precise qualification. The concept of a speedy disposition is a relative term and must necessarily be a flexible concept. While justice is administered with dispatch, the essential ingredient is orderly, expeditious and not mere speed. It cannot be definitely said how long is too long in a system where justice is supposed to be swift, but deliberate. It is consistent with delays and depends upon circumstances. It secures rights to the accused, but it does not preclude the rights of public justice. Also, it must be borne in mind that the rights given to the accused by the Constitution and the Rules of Court are shields, not weapons; hence, courts are to give meaning to that intent. The Court emphasized in the same case that: A balancing test of applying societal interests and the rights of the accused necessarily compels the court to approach speedy trial cases on an ad hoc basis. In determining whether the accused has been deprived of his right to a speedy disposition of the case and to a speedy trial, four factors must be considered: (a) length of delay; (b) the reason for the delay; (c) the defendants assertion of his right; and (d) prejudice to the defendant. x x x. Closely related to the length of delay is the reason or justification of the State for such delay. Different weights should be assigned to different reasons or justifications invoked by the State. x x x.453[26] Exhaustively explained in Corpuz v. Sandiganbayan, an accuseds right to speedy trial is deemed violated only when the proceeding is attended by vexatious, capricious, and oppressive delays. In determining whether petitioner was deprived of this right, the factors to consider and balance are the following: (a) duration of the delay; (b) reason therefor; (c) assertion of the right or failure to assert it; and (d) prejudice caused by such delay.454[27] From the initial hearing on 27 February 2001 until the time the prosecution filed its formal offer of evidence for Criminal Cases No. 119831-119832 on 25 November 2003, both prosecution and defense
453[26] 454[27]

Id. at 313-314. Abardo v. Sandiganbayan, 407 Phil. 985, 999-1000 (2001); Dela Pena v. Sandiganbayan, 412 Phil. 921, 929 (2001).

367

368

admit that no evidence was presented for Criminal Case No. 119830. Hence, for a period of almost two years and eight months, the prosecution did not present a single evidence for Criminal Case No. 119830. The question we have to answer now is whether there was vexatious, capricious, and oppressive delay. To this, we apply the fourfactor test previously mentioned. We emphasize that in determining the right of an accused to speedy trial, courts are required to do more than a mathematical computation of the number of postponements of the scheduled hearings of the case. A mere mathematical reckoning of the time involved is clearly insufficient,455[28] and particular regard must be given to the facts and circumstances peculiar to each case.456[29] In Alvizo v. Sandiganbayan,457[30] the Court ruled that there was no violation of the right to speedy trial and speedy disposition. The Court took into account the reasons for the delay, i.e., the frequent amendments of procedural laws by presidential decrees, the structural reorganizations in existing prosecutorial agencies and the creation of new ones by executive fiat, resulting in changes of personnel, preliminary jurisdiction, and the functions and powers of prosecuting agencies. The Court also considered the failure of the accused to assert such right, and the lack of prejudice caused by the delay to the accused. In Defensor-Santiago v. Sandiganbayan,458[31] the complexity of the issues and the failure of the accused to invoke her right to speedy disposition at the appropriate time spelled defeat for her claim to the constitutional guarantee. In Cadalin v. Philippine Overseas Employment Administrations Administrator,459[32] the Court, considering also the complexity of the cases and the conduct of the parties lawyers, held that the right to speedy disposition was not violated therein. Petitioners objection to the prosecutions stand that he gave an implied consent to the separate trial of Criminal Case No. 119830 is belied by the records of the case. No objection was interposed by his defense counsel when this matter was discussed during the initial hearing.460[33] Petitioners conformity thereto can be deduced from his non-objection at the preliminary hearing when the prosecution manifested that the evidence to be presented would be only for Criminal Cases No. 119831-119832. His failure to object to the prosecutions manifestation that the cases be tried separately is fatal to his case. The acts, mistakes and negligence of counsel bind his client, except only when such mistakes would result in serious injustice.461[34] In fact, petitioners acquiescence is evident from the transcript of stenographic notes during the initial presentation of the Peoples evidence in the five BW cases on 27 February 2001, herein quoted below:
455[28]

456[29] 457[30] 458[31] 459[32] 460[33] 461[34]

Socrates v. Sandiganbayan, 324 Phil. 151, 170 (1996); Tai Lim v. Court of Appeals, 375 Phil. 971, 977 (1999). Santiago v. Garchitorena, G.R. No. 109266, 2 December 1993, 228 SCRA 214, 221. G.R. No. 101689, 17 March 1993, 220 SCRA 55. 408 Phil. 767 (2001). G.R. No. 104776, 5 December 1994, 238 SCRA 721. TSN, 27 February 2001. Producers Bank of the Philippines v. Court of Appeals, 430 Phil. 812, 823 (2002); People v. Hernandez, 328 Phil. 1123, 1143 (1996).

368

369

COURT: Atty. Sandejas, call your witness. ATTY. SANDEJAS [SEC Prosecuting Lawyer]: May we make some manifestation first, your Honor, before we continue presenting our witness. First of all, this witness will only be testifying as to two (2) of the charges: non-disclosure of beneficial ownership of Dante Tan x x x. xxxx COURT: (to Atty. Sandejas) Call your witness. ATTY. SANDEJAS: Our witness is Mr. Wilfredo Baltazar of the Securities and Exchange Commission, your Honor. We are presenting this witness for the purpose of non-disclosure of beneficial ownership case COURT: I would advise the counsel from the SEC to make it very clear your purpose in presenting your first witness. ATTY. SANDEJAS: Yes, your Honor. Can I borrow the file? COURT: Show it to counsel. ATTY. SANDEJAS: Crim. Case Nos. 119831 and 119832, for Violation of RA Rule 36(a)1, in relation to Sec. 32 (a)-1 of the Revised Securities Act when he failed to disclose his beneficial ownership amounting to more than 10% which requires disclosure of such fact.462[35]

During the same hearing, the People manifested in open court that the parties had agreed to the separate trials of the BW Cases: PROSECUTOR LAZARO: May we be allowed to speak, your Honor? Your Honor please, as we x x x understand, this is not a joint trial but a separate trial x x x so as manifested by the SEC lawyer, the witness is being presented insofar as 119831 and 119832 as against Dante Tan only x x x.463[36] The transcript of stenographic notes taken from the 3 April 2001 hearing further clarifies that only the two cases against Dante Tan were being prosecuted: ATTY. DE LA CRUZ [new counsel for accused Eduardo Lim, Jr.]: Your Honor, please, may I request clarification from the prosecutors regarding the purpose of the testimony of the witness in the stand. While the Private Prosecutor stated the purpose of the testimony of the witness. . PROSECUTOR LAZARO:

462[35] 463[36]

TSN, 27 February 2001, pp. 3-7; CA rollo, pp. 87-91. Id. at 71-74; id. at 155-156.

369

370

I was present during the last hearing. I was then going over the transcript of this case, well, I believe the testimony x x x mainly [is] on accused Dante Tan, your Honor. As a matter of fact, there was a clarification made by the parties and counsels after the witness had testified that the hearing in these cases is not a joint trial because it involves separate charges, involving different documents, your Honor. That is why the witness already testified only concerning Dante Tan. Per the query made by Atty. Fortun, because at that time, Atty. Fortun was still representing Mr. Lim, I believe, your Honor, then I understand that the testimony of this witness cannot just be adopted insofar as the other accused, your Honor. ATTY. MARANAN: We confirm that, your Honor, since x x x particularly since this is already cross, it is clear that the direct examination dealt exclusively with Mr. Dante Tan. PROS. LAZARO: Mr. Dante Tan, involving the 2 (two) cases.464[37] Moreover, although periods for trial have been stipulated, these periods are not absolute. Where periods have been set, certain exclusions are allowed by law.465[38] After all, this Court and the law recognize that it is but a fact that judicial proceedings do not exist in a vacuum and must contend with the realities of everyday life. In spite of the prescribed time limits, jurisprudence continues to adopt the view that the fundamentally recognized principle is that the concept of speedy trial is a relative term and must necessarily be a flexible concept.466[39] As to the assertion that delay in the presentation of evidence for Criminal Case No. 119830 has prejudiced petitioner because the witnesses for the defense may no longer be available at this time, suffice it to say that the burden of proving his guilt rests upon the prosecution.467[40] Should the prosecution fail for any reason to present evidence sufficient to show his guilt beyond reasonable doubt, petitioner will be acquitted. It is safely entrenched in our jurisprudence that unless the prosecution discharges its burden to prove the guilt of an accused beyond reasonable doubt, the latter need not even offer evidence in his behalf.468[41] In the cases involving petitioner, the length of delay, complexity of the issues and his failure to invoke said right to speedy trial at the appropriate time tolled the death knell on his claim to the constitutional guarantee.469[42] More importantly, in failing to interpose a timely objection to the prosecutions manifestation during the preliminary hearings that the cases be tried separately, one after the other, petitioner was deemed to have acquiesced and waived his objection thereto.

464[37] 465[38] 466[39] 467[40] 468[41]

469[42]

TSN, 3 April 2001, pp. 5-10; id. at 225-230. Solar Team Entertainment, Inc. v. Judge How, 393 Phil. 172, 184 (2000). Id. Republic v. Sandiganbayan and Marcos, 461 Phil. 598, 615 (2003). People v. Ganguso, G.R. No 115430, 23 November 1995, 250 SCRA 268, 274-275; People v. Abellanosa, 332 Phil. 760, 788 (1996), citing People v. Baclayon, G.R. No. 110837, 29 March 1994, 231 SCRA 578, 584, citing People v. Garcia, G.R. No. 94187, 4 November 1992, 215 SCRA 349, 358359. Santiago v. Garchitorena, supra note 29.

370

371

For the reasons above-stated, there is clearly insufficient ground to conclude that the prosecution is guilty of violating petitioners right to speedy trial. Grave abuse of discretion defies exact definition, but generally refers to capricious or whimsical exercise of judgment as is equivalent to lack of jurisdiction. Any capricious or whimsical exercise of judgment in dismissing a criminal case is equivalent to lack of jurisdiction. This is true in the instant case. There is also no merit to petitioners claim that a reversal of the RTCs Order dismissing Criminal Case No. 119830 is a violation of his constitutional right against double jeopardy which dismissal was founded on an alleged violation of his right to speedy trial. The constitutional protection against double jeopardy shields one from a second or later prosecution for the same offense. Article III, Section 21 of the 1987 Constitution declares that no person shall be twice put in jeopardy of punishment for the same offense, providing further that if an act is punished by a law and an ordinance, conviction or acquittal under either shall constitute a bar to another prosecution for the same act. Following the above constitutional provision, Section 7, Rule 117 of the Revised Rules of Court found it apt to stipulate: SEC. 7. Former conviction or acquittal; double jeopardy. When an accused has been convicted or acquitted, or the case against him dismissed or otherwise terminated without his express consent by a court of competent jurisdiction, upon a valid complaint or information or other formal charge sufficient in form and substance to sustain a conviction and after the accused had pleaded to the charge, the conviction or acquittal of the accused or the dismissal of the case shall be a bar to another prosecution for the offense charged, or for any attempt to commit the same or frustration thereof, or for any offense which necessarily includes or is necessarily included in the offense charged in the former complaint or information.

For double jeopardy to attach then, the following elements in the first criminal case must be present: (a) The complaint or information or other formal charge was sufficient in form and substance to sustain a conviction; (b) The court had jurisdiction; (c) The accused had been arraigned and had pleaded; and (d) He was convicted or acquitted or the case was dismissed or otherwise terminated without the express consent of the accused.470[43]

Among the above-cited elements, we are concerned with the fourth element, conviction or acquittal, or the case was dismissed or otherwise terminated without the express consent of the accused. This element is crucial since, as a general rule, the dismissal of a criminal case resulting in acquittal, made with the express consent of the accused or upon his own
470[43]

Condrada v. People, 446 Phil. 635, 641 (2003).

371

372

motion, will not place the accused in double jeopardy. 471[44] This rule, however, admits of two exceptions, namely: insufficiency of evidence and denial of the right to speedy trial.472[45] While indeed petitioner was in fact the one who filed the Motion to Dismiss Criminal Case No. 119830, the dismissal thereof was due to an alleged violation of his right to speedy trial, which would otherwise put him in double jeopardy should the same charges be revived. Petitioners situation is different. Double jeopard y has not attached, considering that the dismissal of Criminal Case No. 119830 on the ground of violation of his right to speedy trial was without basis and issued with grave abuse of discretion amounting to lack or excess of jurisdiction. Where the right of the accused to speedy trial has not been violated, there is no reason to support the initial order of dismissal. Following this Courts ruling in Almario v. Court of Appeals,473[46] as petitioners right to speedy trial was not transgressed, this exception to the fourth element of double jeopardy that the defendant was acquitted or convicted, or the case was dismissed or otherwise terminated without the express consent of the accused was not met. Where the dismissal of the case was allegedly capricious, certiorari lies from such order of dismissal and does not involve double jeopardy, as the petition challenges not the correctness but the validity of the order of dismissal; such grave abuse of discretion amounts to lack of jurisdiction, which prevents double jeopardy from attaching.474[47] As this Court ruled in People v. Tampal,475[48] reiterated in People v. Leviste,476[49] where we overturned an order of dismissal by the trial court predicated on the right to speedy trial It is true that in an unbroken line of cases, we have held that dismissal of cases on the ground of failure to prosecute is equivalent to an acquittal that would bar further prosecution of the accused for the same offense. It must be stressed, however, that these dismissals were predicated on the clear right of the accused to speedy trial. These cases are not applicable to the petition at bench considering that the right of the private respondents to speedy trial has not been violated by the State. x x x.

From the foregoing, it follows that petitioner cannot claim that double jeopardy attached when said RTC order was reversed by the Court of Appeals. Double jeopardy does not apply to this case, considering that there is no violation of petitioners right to speedy trial. The old adage that justice delayed is justice denied has never been more valid than in our jurisdiction, where it is not a rarity for a case to drag in our courts for years and years and even decades. It was this difficulty that inspired the constitutional requirement that the rules of court to be promulgated by the Supreme Court shall provide for a simplified and inexpensive procedure for the speedy trial and disposition of cases.477[50]
471[44] 472[45]

473[46] 474[47] 475[48] 476[49] 477[50]

Id. Id.; Philippine Savings Bank v. Bermoy, G.R. No. 151912, 26 September 2005, 471 SCRA 94, 106, citing People v. Bans, G.R. No. 104147, 8 December 1994, 239 SCRA 48, 55. 407 Phil. 279 (2002). Regalado, REMEDIAL LAW COMPENDIUM (Vol. II, 2001), p. 503. 314 Phil. 35, 45 (1995). 325 Phil. 525, 537 (1996). Justice Isagani Cruz, PHILIPPINE POLITICAL LAW, p. 292.

372

373

Indeed, for justice to prevail, the scales must balance, for justice is not to be dispensed for the accused alone.478[51] Evidently, the task of the pillars of the criminal justice system is to preserve our democratic society under the rule of law, ensuring that all those who appear before or are brought to the bar of justice are afforded a fair opportunity to present their side. As correctly observed by the Court of Appeals, Criminal Case No. 119830 is just one of the many controversial cases involving the BW shares scam where public interest is undoubtedly at stake. The State, like any other litigant, is entitled to its day in court, and to a reasonable opportunity to present its case. A hasty dismissal, instead of unclogging dockets, has actually increased the workload of the justice system and unwittingly prolonged the litigation.479[52] Finally, we reiterate that the rights given to the accused by the Constitution and the Rules of Court are shields, not weapons. Courts are tasked to give meaning to that intent. There being no capricious, vexatious, oppressive delay in the proceedings, and no postponements unjustifiably sought, we concur in the conclusions reached by the Court of Appeals. Speedy disposition of cases before administrative bodies; effect of undue delay

CAPT. WILFREDO ROQUERO VS. THE CHANCELLOR OF UP-MANILA, ET AL., G.R. No. 181851, March 9, 2010 PEREZ, J.: Petitioner Wildredo G. Roquero is an employee of UP-Manila assigned at the Philippine General Hospital (PGH) Security Division as Special Police Captain. Private respondent Imelda O. Abutal is a Lady Guard of Ex-Bataan Security Agency who was applying for a position in the security force assigned at UPPGH. The instant controversy arose from a complaint by private respondent Abutal with then Chancellor of UP-Manila Perla D. Santos-Ocampo for Grave Misconduct against petitioner Capt. Roquero. The formal charge filed on 1 October 1998 and docketed as ADM Case No. UPM-AC 97-007 reads as follows: After preliminary investigation duly conducted in accordance with the Rules and Regulations on the Discipline of UP Faculty and Employees, a prima facie case has been found to exist against you for GRAVE MISCONDUCT punishable under the University Rules and Regulations on the Discipline of UP Faculty and Employees in relation to the Civil Service Law, committed as follows:
478[51]

479[52]

Dimatulac v. Villon, 358 Phil. 328, 366 (1998); People v. Subida, G.R. No. 145945, 27 June 2006, 493 SCRA 125, 137. People v. Leviste, supra note 49.

373

374

That you, Capt. Wilfredo Roquero of the UP Manila Police Force, sometime in April 1996, while conducting an interview on MS. IMELDA ABUTAL who was then applying for the position of Lady Guard of Ex-Bataan Security Agency to be assigned at UP-PGH, proposed to her that if she agreed to be your mistress, you would facilitate her application and give her a permanent position; that despite the fact the MS. ABUTAL rejected your proposal, you still insisted on demanding said sexual favor from her; that you, therefore, are liable for GRAVE MISCONDUCT under Section 22, paragraph (c) of Rule XIV of the Omnibus Rules Implementing Book V of E.O. 292 on Civil Rules. On 1 October 1998, the petitioner was placed under preventive suspension for ninety (90) days by Chancellor Santos-Ocampo, the material portion of said Order reads: Considering the gravity of the offense charged and pursuant to Section 19 of Rules and Regulations on the Discipline of UP Faculty Members and Employees and Section 26 and 27 Rule XIV of Book V of Executive Order No. 292 and Omnibus Rules, you are hereby preventively suspended for ninety (90) days effective upon receipt hereof. While on preventive suspension, you are hereby required to appear before the Administrative Disciplinary Tribunal (ADT) whenever your presence is necessary. Thereafter, the Administrative Disciplinary Tribunal (ADT) composed of Atty. Zaldy B. Docena, Eden Perdido and Isabella Lara, was organized to hear the instant case. Atty. Paul A. Flor, as University Prosecutor, represented the prosecution. He was later on replaced by Atty. Asteria Felicen. Petitioner was represented by Atty. Leo G. Lee of the Public Attorneys Office (PAO) who was then replaced by Public Attorney Philger Inovejas. The Prosecution presented its only witness, private respondent Abutal. After the completion of the cross-examination on the prosecutions only witness, the prosecution agreed to submit its Formal Offer of Evidence on or before 16 July 1999. The prosecution, however, failed to submit its formal offer of evidence within the period agreed upon. Thereafter, on 10 August 1999, when the case was called, only petitioner and his counsel appeared. Atty. Flor merely called by telephone and requested Atty. Docena to reset the case to another date. Atty. Docena then ordered the resetting of the hearing on the following dates: 11 August and 21 August 1999. On 11 August 1999, only petitioner and his counsel came. No representative from the prosecution appeared before the ADT. Atty. Flor again called and asked
374

375

for the postponement of the hearing. By reason thereof, Atty. Docena issued an Order, which reads as follows: The continuation of the hearing of this case is hereby set to September 29, 1999 at 2:00 p.m., with the understanding that if and when the parties fail to appear at said hearing date, this case shall be deemed submitted for resolution based on the evidences already obtaining in the record of the case. SO ORDERED. 11 August 1999. On said date, the representative from the prosecution again failed to appear. On 22 October 1999, petitioner filed a Motion through counsel praying that complainant (private respondent herein) be declared to have waived her rights to formally offer her exhibits since complainant was not able to file her Formal Offer within the given period of fifteen (15) days from 1 July 1999 or up to 16 July 1999. The ADT was not able to act on the said Motion for almost five (5) years. Due to the unreasonable delay, petitioner, on 19 May 2004 filed another Motion asking for the dismissal of the administrative case against him. The Motion to Dismiss was anchored on the following reasons: that the prosecution had not formally offered its evidence; that the ADT had failed to act on the motion filed on 22 October 1999; that the unfounded charges in the administrative complaint were filed just to harass him; and that he is entitled to a just and speedy disposition of the case. On 26 May 2004, the prosecution, represented by Atty. Felicen in view of the resignation of Atty. Flor in August 1999, filed its Comment/Opposition to the Motion to Dismiss. The prosecution alleged that a Formal Offer of Documentary Exhibits had been filed on 24 January 2004, of which a copy thereof was received by Atty. Lee, petitioners counsel, on 30 January 2004, per registry return receipt. However, petitioner has not filed his comment to the said Formal Offer. Furthermore, the prosecution explained in its Comment/Opposition that in view of the resignation of Atty. Flor in August 1999 but who had been on leave by mid-July 1999, the Formal Offer could not be prepared by another counsel until all the transcript of stenographic notes have been furnished to the counsel that replaced Atty. Flor. Meanwhile, the stenographer, Jamie Limbaga, had been in and out of the hospital due to a serious illness, thus the delay in the filing of the prosecutors Formal Offer of Documentary Exhibits. On 8 June 2004, Atty. Docena issued the assailed Order denying petitioners motion to dismiss, to wit: Acting on respondents Motion to Dismiss, as well as the University Prosecutors Comment and/or Opposition to said Motion, and finding that said Motion to Dismiss to be bereft of merit, the same is hereby DENIED.
375

376

In view of the failure of the respondent to file his comment on the Prosecutions Formal Offer of Evidence, the Exhibits (A to G-1) of the Prosecution are hereby ADMITTED for the purpose for which the same have been offered. The respondent is hereby directed to present his evidence on June 22, 2004 at 10:30 in the morning. SO ORDERED. A motion for reconsideration was filed by petitioner but the same was denied in an Order dated 9 November 2004. Petitioner Captain Wilfredo Roquero then filed with the Court of Appeals a Petition for Certiorari under Rule 65, docketed as CA-G.R. SP No. 87776, alleging therein that the ADT committed grave abuse of discretion when it denied the motion to dismiss the administrative case filed against him. In a Decision dated 22 March 2007, the Honorable Court of Appeals denied the petition with prayer for TRO of Roquero reasoning that the ADT did not commit grave abuse of discretion in issuing the assailed orders. Hence, this Petition. The core issue of this case is whether the failure of the ADT to resolve Roqueros Motion (to declare complainant Imelda Abutal to have waived her right to submit her Formal Offer of Exhibit) which he seasonably filed on 22 October 1999 and the assailed Order of the ADT dated 8 June 2004 admitting the Formal Offer of Exhibit of complainant Imelda Abutal despite having filed after almost five years violated the constitutional right of Roquero to a speedy disposition of cases. HELD: Indeed, while Section 27 of the Uniform Rules on Administrative Cases in Civil Service states that the failure to submit the formal offer of evidence within the given period shall be considered as waiver thereof, the ADT in fact allowed the prosecution to present its formal offer almost five (5) years later or on 24 January 2004. Starting on that date, petitioner was presented with the choice to either present his evidence or to, as he did, file a motion to dismiss owing to the extraordinary length of time that ADT failed to rule on his motion. We cannot accept the finding of the Court of Appeals that there was no grave abuse of discretion on the part of the ADT because a formal offer of evidence was filed by the prosecution, a copy of which was received by petitioners counsel. The admission by ADT on 8 June 2004 of the formal offer of exhibits belatedly filed did not cure the 5-year delay in the resolution of petitioners 1999 motion to deem as waived such formal offer of evidence. Indeed, the delay of almost five (5) years cannot be justified.

The ADT admitted this explanation of the prosecutor hook, line and sinker without asking why it took him almost five (5) years to make that explanation. If
376

377

the excuses were true, the prosecution could have easily manifested with the ADT of its predicament right after Roquero filed his motion to declare the waiver of the formal offer. It is evident too that the prosecution failed to explain why it took them so long a time to find a replacement for the original prosecutor. And, the stenographer who had been in and out of the hospital due to serious illness should have been replaced sooner. While it is true that administrative investigations should not be bound by strict adherence to the technical rules of procedure and evidence applicable to judicial proceedings, the same however should not violate the constitutional right of respondents to a speedy disposition of cases. Section 16, Article III of the 1987 Constitution provides: Section 16. All person shall have the right to a speedy disposition of their cases before all judicial, quasi-judicial, or administrative bodies. The constitutional right to a speedy disposition of cases is not limited to the accused in criminal proceedings but extends to all parties in all cases, including civil and administrative cases, and in all proceedings, including judicial and quasi-judicial hearings. Hence, under the Constitution, any party to a case may demand expeditious action by all officials who are tasked with the administration of justice. The right to a speedy disposition of a case, like the right to a speedy trial, is deemed violated only when the proceedings are attended by vexatious, capricious, and oppressive delays; or when unjustified postponements of the trial are asked for and secured; or even without cause or justifiable motive, a long period of time is allowed to elapse without the party having his case tried. Equally applicable is the balancing test used to determine whether a defendant has been denied his right to a speedy trial, or a speedy disposition of a case for that matter, in which the conduct of both the prosecution and the defendant is weighed, and such factors as the length of the delay, the reasons for such delay, the assertion or failure to assert such right by the accused, and the prejudice caused by the delay. The concept of a speedy disposition is a relative term and must necessarily be a flexible concept. Hence, the doctrinal rule is that in the determination of whether that right has been violated, the factors that may be considered and balanced are as follows: (1) the length of delay; (2) the reasons for the delay; (3) the assertion or failure to assert such right by the accused; and (4) the prejudice caused by the delay. Applying the doctrinal ruling vis-a-vis the factual milieu of this case, the violation of the right to a speedy disposition of the case against petitioner is clear for the following reasons: (1) the delay of almost five (5) years on the part of ADT in resolving the motion of petitioner, which resolution petitioner reasonably found necessary before he could present his defense; (2) the unreasonableness of the delay; and (3) the timely assertions by petitioner of the right to an early disposition which he did through a motion to dismiss. Over and above this, the delay was prejudicial to petitioners cause as he was under preventive suspension for ninety (90) days, and during the interregnum of almost five years, the trial of the accusation against him remained stagnant at the prosecution stage. The Constitutional guarantee against unreasonable delay in the disposition of cases was intended to stem the tide of disenchantment among the people in the administration of justice by our judicial and quasi-judicial tribunals. The
377

378

adjudication of cases must not only be done in an orderly manner that is in accord with the established rules of procedure but must also be promptly decided to better serve the ends of justice. Excessive delay in the disposition of cases renders the rights of the people guaranteed by the Constitution and by various legislations inutile. WHEREFORE, the Petition is hereby GRANTED. The Administrative Disciplinary Tribunal (ADT) of the University of the Philippines-Manila, Atty. Zaldy B. Docena, Eden Perdido and Isabella Lara, in their capacities as Chairman and Members of the ADT respectively, are hereby ORDERED to DISMISS the administrative case against Capt. Wilfredo G. Roquero for violation of his constitutional right to a speedy disposition of cases. JAIME BERNAT VS. SANDIGANBAYAN, May 20, 2004 Right to speedy disposition of case. Facts: 1. On August 14, 1991, the petitioner and several others were charged of violation of Section 3 [e] of RA 3019, otherwise known as the Anti-graft and Corrupt Practices Act; 2. On August 23, 1994 after the presentation of the parties evidence, the case was deemed submitted for decision before the 2nd Division; 3. Thereafter, the case was unloaded to the newly created 5th Division, particularly to Justice Godofredo Legaspi and later re-assigned to Justice Ma. Cristina Cortez-Estrada upon her assumption of office on November 3, 1998. 4. In the early part of 2002 while Justice Estrada was writing the decision of the case, she found out that the November 26, 1993 transcript of stenographic notes, which was the cross-examination of the petitioner, was missing so she called the parties for a conference on April 19, 2002 to discuss the matter. 5. Instead of attending the conference, petitioner filed a motion to dismiss the case based on the alleged violation of his right to speedy trial. The Court denied the same as well as the subsequent Motion for Reconsideration. Hence, this Petition. Issue: Was there violation of the petitioners right to a speedy disposition of his case when the same was not decided for almost 8 years from the time it was deemed submitted for decision? Held: No. The right is violated only if the proceedings were attended by vexatious, capricious and oppressive delays. The determination of whether the delays are of said nature is relative and cannot be based on mere mathematical reckoning of time. Particular regard to the facts and circumstances of the case. As held in the case of DE LA PENA VS. SANDIGANBAYAN, certain factors shall be considered and balanced to determine if there is delay, as follows: 3. Length of the delay; 4. Reasons for the delay; 5. Assertion or failure to assert such right by the accused; and
378

379

6. Prejudiced caused by the delay. There is no violation of the right to speedy disposition of his case because petitioner failed to assert his constitutional right to a speedy disposition of his case. During the 8-year period prior to April 19, 2002, petitioner did not complain about the long delay in deciding his case. a. Read Admin. Circular No. 4 of the Supreme September 22, 1988 b. Department of Justice Circular No. 27, dated 1988 c. When shall this right starts Read: 1. P vs. Orsal, 113 SCRA 226 d. To what proceedings is this right available Read: 1. Caballero vs. Alfonso, 153 SCRA 153 e. In general Read: 1. The right to speedy trial, 28 SCRA 601 2. Conde vs. Rivera, 59 Phil. 650 3. Ventura vs. People, Nov. 6,1976 4. Martin vs. Ver, July 25, 1983 5. Bermisa vs. CA, 92 SCRa 6. Luneta vs. Mil. Com., 102 SCRA 56 7. P vs. Baladjay, 113 SCRA 284 8. P vs. Araula, 111 SCRA 598 9. Regaspi vs. Castillo, 69 SCRA 160 10. Acevedo vs. Sarmiento, 36 SCRA 247 11. Nepumuceno vs. Secretary,108 SCRA 658 12. Tatad vs. SB, 159 SCRA 70 13. P vs. CFI of Rizal, 161 SCRA 249 14. P vs. Laya, 161 SCRA 327 15. Salcedovs. Mendoza, 88 SCRA 811 16. DUTERTE VS. SANDIGANBAYAN, 289 SCRA 721 18. ANGCHANGCO VS. OMBUDSMAN, 269 SCRA 301 SUMBANG VS. GEN. COURT MARTIAL, G.R. NO. 140188, 337 SCRA 227, AUG. 3, 2000; BLANCO VS. SANDIGANBAYAN, G.R. NOS. 136757 58, 346 SCRA 108, NOV. 27, 2000; SOLAR TEAM ENTERTAINMENT, INC. HON. HOW, G.R. NO. 140863, 338 SCRA 51, AUG. 22, 2000. Speedy Disposition of Cases. (i) The determination of whether an accused had been denied the right to speedy trial depends on the surrounding circumstances of each case. Although it took about 8 years before the trial of this case
379

Court dated September 16,

380

was resumed, such delay did not amount to violation of petitioners right to speedy trial considering that such delay was not by attributable to the prosecution. Factors to consider in determining whether or not such right has been violated: 1. length of delay, 2. reasons for such delay, and 3. assertion or failure to assert such rights by the accused and the prejudice caused by the delay. (ii) Speedy Trial Act of 1998. The authority of the Secretary of Justice to review resolutions of his subordinates even after an information has already been filed in court does not present an irreconcilable conflict with the 30-day period prescribed in Sec. 7 of the Speedy Trial Act of 1998. 8. The right to an impartial trial Read: 1. P vs. Opida, June 13,1986 1-a. P vs. Tuazon, 159 SCRA 317 2. Olaguer vs. Chief of Staff, May 22, 1987 3. Mateo, Jr. vs. Villaluz,90 SCRA 16 4. P vs. Sendaydiego, 81 SCRA 120 5. Dimacuha vs. Concepcion, 117 SCRA 630 9. Right to a public trial Read: 1. Garcia vs. Domingo, July 25,1973 2. P vs. Tampus, March 28,1980 10. The right to be informed of the nature and cause of When the same is considered waived. accusation.

On March 17, 1999, appellant Jerry Nazareno was indicted for violation of Article 266-A of the Revised Penal Code in Criminal Case No. 2638 for the alleged rape of BBB, his daughter. The information reads: That sometime and between January 1992 up to December 06, 1998, in Barangay Codon, Municipality of San Andres, Province of Catanduanes, Philippines, and within the jurisdiction of this Honorable Court, the above-named accused by means of force, violence and intimidation did then and there willfully, unlawfully, feloniously and repeatedly made sexual intercourse with his daughter BBB at the age of 7 through 14 years old against her will.
380

381

The Information is worded thus: That from sometime in January 1990 up to December 1998 in Barangay Codon, municipality of San Andres, Catanduanes, and within the jurisdiction of the Honorable Court, the said accused, being the father of the complainant, did then and there willfully, feloniously and criminally repeatedly had sexual intercourse with her daughter AAA, then five years old up to the time when she was 15-years-old against her will. CONTRARY TO LAW.480[18] After trial , the accused was found guilty of qualified rape in both cases. He appealed his conviction to the Court of Appeals in accordance with the People vs. Mateo Doctrine but the Court of Appeals affirmed the RTC Decision. Hence, this Petition before the Supreme Court. I S S U E: Is the constitutional right of the petitioner to be informed of the nature and cause of accusation against him violated since the information failed to specify with certainty the approximate date of the commission of the offenses for rape which is a fatal defect. H E L D: The argument is specious. An information is intended to inform an accused of the accusations against him in order that he could adequately prepare his defense. Verily, an accused cannot be convicted of an offense unless it is clearly charged in the complaint or information. Thus, to ensure that the constitutional right of the accused to be informed of the nature and cause of the accusation against him is not violated, the information should state the name of the accused; the designation given to the offense by the statute; a statement of the acts or omissions so complained of as constituting the offense; the name of the offended party; the approximate time and date of the commission of the offense; and the place where the offense has been committed.481[27] Further, it must embody the essential elements of the crime charged by setting forth the facts and circumstances that have a bearing on the culpability and liability of the accused, so that he can properly prepare for and undertake his defense.482[28] However, it is not necessary for the information to allege the date and time of the commission of the crime with exactitude unless time is an essential ingredient of the offense.483[29] In People v. Bugayong,484[30] the Court held that when the time given in the information is not the essence of the offense, the time need not be proven as alleged; and that the complaint will be sustained if the proof shows that the offense was committed at any time within the period of the statute of limitations and before the commencement of the action.

480[18] 481[27]

Records, Vol. II, p. 18. People v. Quitlong, 354 Phil. 372, 388 (1998), citing Rules of Criminal Procedure (2000), Rule 110, Secs. 6 and 8. 482[28] Id. 483[29] People v. Santos, 390 Phil. 150, 161 (2000); Rules of Criminal Procedure (2000), Rule 110, Sec. 11 reads: Sec. 11. Date of commission of the offense. It is not necessary to state in the complaint or information the precise date the offense was committed except when it is a material ingredient of the offense. The offense may be alleged to have been committed on a date as near as possible to the actual date of its commission. 484[30] G.R. No. 126518, December 2, 1998, 299 SCRA 528.

381

382

In People v. Gianan,485[31] the Court ruled that the time of the commission of rape is not an element of the said crime as it is defined in Article 335 of the Revised Penal Code. The gravamen of the crime is the fact of carnal knowledge under any of the circumstances enumerated therein, i.e.: (1) by using force or intimidation; (2) when the woman is deprived of reason or otherwise unconscious; and (3) when the woman is under twelve years of age or is demented. In accordance with Rule 110, Section 11 of the 2000 Rules of Criminal Procedure, as long as it alleges that the offense was committed at any time as near to the actual date at which the offense was committed, an information is sufficient. The doctrine was reiterated with greater firmness in People v. Salalima486[32] and in People v. Lizada.487[33] In the case under review, the information in Criminal Case No. 2638 alleged that the rape of BBB transpired sometime and between January 1992 up to December 6, 1998 in Barangay Codon, Municipality of San Andres, Province of Catanduanes. In Criminal Case No. 2650, the information averred that from sometime in January 1990 up to December 1998 in Barangay Codon, Municipality of San Andres, Province of Catanduanes, AAA was raped by appellant. To the mind of the Court, the recitals in the informations sufficiently comply with the constitutional requirement that the accused be informed of the nature and cause of the accusation against him. In People v. Garcia,488[34] the Court upheld a conviction for ten counts of rape based on an Information which alleged that the accused committed multiple rapes from November 1990 up to July 21, 1994. In People v. Espejon,489[35] the Court found the appellant liable for rape under an information charging that he perpetrated the offense sometime in the year 1982 and dates subsequent thereto and sometime in the year 1995 and subsequent thereto. In the case under review, the information in Criminal Case No. 2638 alleged that the rape of BBB transpired sometime and between January 1992 up to December 6, 1998 in Barangay Codon, Municipality of San Andres, Province of Catanduanes. In Criminal Case No. 2650, the information averred that from sometime in January 1990 up to December 1998 in Barangay Codon, Municipality of San Andres, Province of Catanduanes, AAA was raped by appellant. To the mind of the Court, the recitals in the informations sufficiently comply with the constitutional requirement that the accused be informed of the nature and cause of the accusation against him. Indeed, this Court has ruled that allegations that rapes were committed before and until October 15, 1994,490[36] sometime in the year 1991 and the days thereafter,491[37] and on or about and sometime in the year 1988492[38] constitute sufficient compliance with Rule 110, Section 11 of the 2000 Rules of Criminal Procedure.

485[31] 486[32]

G.R. Nos. 135288-93, September 15, 2000, 340 SCRA 477. G.R. Nos. 137969-71, August 15, 2001, 363 SCRA 192. 487[33] G.R. Nos. 143468-71, January 24, 2003, 396 SCRA 62. 488[34] G.R. No. 120093, November 6, 1997, 281 SCRA 463. 489[35] G.R. No. 134767, February 20, 2002, 377 SCRA 412. 490[36] People v. Bugayong, supra note 30. 491[37] People v. Magbanua, G.R. No. 128888, December 3, 1999, 319 SCRA 719. 492[38] People v. Santos, G.R. Nos. 131103 & 143472, June 29, 2000, 334 SCRA 655.

382

383

More than that, the Court notes that the matter of particularity of the dates in the information is being raised for the first time on appeal. The rule is well-entrenched in this jurisdiction that objections as to matter of form or substance in the information cannot be made for the first time on appeal.493[39] Appellant failed to raise the issue of defective informations before the trial court. He could have moved to quash the informations or at least for a bill of particulars. He did not. Clearly, he slumbered on his rights and awakened too late. Too, appellant did not object to the presentation of the evidence for the People contending that the offenses were committed sometime and between January 1992 up to December 6, 1998 for Criminal Case No. 2632 and sometime in January 1990, up to December 1998 in Criminal Case No. 2650. On the contrary, appellant actively participated in the trial, offering denial and alibi as his defenses. Simply put, he cannot now be heard to complain that he was unable to defend himself in view of the vagueness of the recitals in the informations.

REASONS FOR THE CONSTITUTIONAL PROVISION ON THE RIGHT OF THE ACCUSED TO BE INFORMED OF THE NATURE AND CAUSE OF ACCUSATION DE LA CRUZ VS. PEOPLE OF THE PHILIPPINES , G.R. No. 175929, December 16, 2008 It is true that in all criminal prosecutions, the accused shall be informed of the nature and cause of the accusation against him. 494[88] The Constitution uses the word shall, hence, the same is mandatory. A violation of this right prevents the conviction of the accused with the crime charged in the Information. The constitutional guaranty has a three-fold purpose: First. To furnish the accused with such a description of the charge against him as will enable him to make his defense; and second, to avail himself of his conviction or acquittal for protection against a further prosecution for the same cause; and third, to inform the court of the facts alleged, so that it may decide whether they are sufficient in law to support a conviction.495[89] Read: 1. Sales vs. CA, 164 SCRA 717 1-a. P vs. Crisologo, 150 SCRA 653 1-b. P vs. Corral, 157 SCRA 678 1-c. P vs. Resavaga, 159 SCRA 426 1-d. Formilleza vs. SB, 159 SCRA 2. P vs. Labado, 98 SCRA 730 3. Ko Bu Lin vs. CA, 118 SCRA 573 4. P. vs. Cabale, 185 SCRA 140
493[39]

People v. Razonable, 386 Phil. 771, 780 (2000).

383

384

5. People vs. Regala, April 27, 1982 11. The right to meet witnesses face to face or the right of confrontation Read: 1. P. vs. Talingdan, Nov. 9, 1990 1-a. P vs. Villaluz, October 20, 1983 2. P vs. Valero, 112 SCRA 661 3. P vs. Bundalian, 117 SCRA 718 4. Talino vs. Sandiganbayan, March 16,1987 5. P vs. Seneris, 99 SCRA 92 6. Ortigas, JR. vs. Lufthansa, 64 SCRA 610 7. Toledo vs. People, 20 SCRA 54 8. P vs. Bardaje, 99 SCRA 388 9. P vs. Santos, 139 SCRA 383 10. Soliman vs. Sandiganbayan, 145 SCRA 640 11. P vs. Lacuna, 87 SCRA 364 12. P vs. Clores, 100 SCRA 227 13. Carredo vs. People, 183 SCRA 273 14. Fulgado vs. CA, 182 SCRA 81 12. Trial in absentia Read: 1. Borja vs. Mendoza, 77 SCRA 420 2. Nolasco vs. Enrile, 139 SCRA 502 3. P vs. Salas, 143 SCRA 163; Note the purpose of this provision) 4. P vs. Judge Prieto, July 21,1978 5. Gimenez vs. Nazareno, 160 SCRA 1 6. Carredo vs. People, 183 SCRA 273 13. Right to secure witnesses and production of Read: 1. Cavili vs. Hon. Florendo, 154 SCRA 610 2. Fajardo vs. Garcia, 98 SCRA 514 CHAPTER XV - HABEAS CORPUS Read: 1In the matter of the Petition for Habeas Corpus of Ferdinand Marcos, etc, GR No. 88079, May 18, 1989 and August & October, 1989. 1-a. Harvey vs. Santiago, supra 2. Cruz vs. Juan Ponce Enrile, April 15,1988 3. Abadilla vs. Fidel Ramos, December 1,1987 CHAPTER XVI - THE RIGHT AGAINST SELF-INCRIMINATION evidence.

384

385

SOCIAL JUSTICE SOCIETY (SJS) VS. DANGEROUS DRUGS BOARD, G.R. No. 157870, NOVEMBER 3, 2008; ATTY. MANUEL J. LASERNA, JR. VS. DANGEROUS DRUGS BOARD, G.R. No. 158633, NOVEMBER 3, 2008 As far as pertinent, the challenged section reads as follows: SEC. 36. Authorized Drug Testing.Authorized drug testing shall be done by any government forensic laboratories or by any of the drug testing laboratories accredited and monitored by the DOH to safeguard the quality of the test results. x x x The drug testing shall employ, among others, two (2) testing methods, the screening test which will determine the positive result as well as the type of drug used and the confirmatory test which will confirm a positive screening test. x x x The following shall be subjected to undergo drug testing: xxxx (c) Students of secondary and tertiary schools.Students of secondary and tertiary schools shall, pursuant to the related rules and regulations as contained in the schools student handbook and with notice to the parents, undergo a random drug testing x x x; (d) Officers and employees of public and private offices.Officers and employees of public and private offices, whether domestic or overseas, shall be subjected to undergo a random drug test as contained in the companys work rules and regulations, x x x for purposes of reducing the risk in the workplace. Any officer or employee found positive for use of dangerous drugs shall be dealt with administratively which shall be a ground for suspension or termination, subject to the provisions of Article 282 of the Labor Code and pertinent provisions of the Civil Service Law; xxxx (f) All persons charged before the prosecutors office with a criminal offense having an imposable penalty of imprisonment of not less than six (6) years and one (1) day shall undergo a mandatory drug test; G.R. No. 157870 (Social Justice Society v. Dangerous Drugs Board and Philippine Drug Enforcement Agency) In its Petition for Prohibition under Rule 65, petitioner Social Justice Society (SJS), a registered political party, seeks to prohibit the Dangerous Drugs Board (DDB) and the Philippine Drug Enforcement Agency (PDEA) from enforcing paragraphs (c), (d), (f), and (g) of Sec. 36 of RA 9165 on the ground that they are constitutionally infirm. For one, the provisions constitute undue delegation of legislative power when they give unbridled discretion to schools and employers to determine the manner of drug testing. For another, the provisions trench in the equal protection clause inasmuch as they can be used to harass a student or an employee deemed undesirable. And for a third, a persons constitutional right against unreasonable searches is also breached by said provisions. G.R. No. 158633 (Atty. Manuel J. Laserna, Jr. v. Dangerous Drugs Board and Philippine Drug Enforcement Agency)
385

386

Petitioner Atty. Manuel J. Laserna, Jr., as citizen and taxpayer, also seeks in his Petition for Certiorari and Prohibition under Rule 65 that Sec. 36(c), (d), (f), and (g) of RA 9165 be struck down as unconstitutional for infringing on the constitutional right to privacy, the right against unreasonable search and seizure, and the right against selfincrimination, and for being contrary to the due process and equal protection guarantees. SJS Petition (Constitutionality of Sec. 36[c], [d], [f], and [g] of RA 9165) The drug test prescribed under Sec. 36(c), (d), and (f) of RA 9165 for secondary and tertiary level students and public and private employees, while mandatory, is a random and suspicionless arrangement. The objective is to stamp out illegal drug and safeguard in the process the well being of [the] citizenry, particularly the youth, from the harmful effects of dangerous drugs. This statutory purpose, per the policy-declaration portion of the law, can be achieved via the pursuit by the state of an intensive and unrelenting campaign against the trafficking and use of dangerous drugs x x x through an integrated system of planning, implementation and enforcement of anti-drug abuse policies, programs and projects.496[14] The primary legislative intent is not criminal prosecution, as those found positive for illegal drug use as a result of this random testing are not necessarily treated as criminals. They may even be exempt from criminal liability should the illegal drug user consent to undergo rehabilitation. Secs. 54 and 55 of RA 9165 are clear on this point:

Sec. 54. Voluntary Submission of a Drug Dependent to Confinement, Treatment and Rehabilitation.A drug dependent or any person who violates Section 15 of this Act may, by himself/herself or through his/her parent, [close relatives] x x x apply to the Board x x x for treatment and rehabilitation of the drug dependency. Upon such application, the Board shall bring forth the matter to the Court which shall order that the applicant be examined for drug dependency. If the examination x x x results in the certification that the applicant is a drug dependent, he/she shall be ordered by the Court to undergo treatment and rehabilitation in a Center designated by the Board x x x. xxxx Sec. 55. Exemption from the Criminal Liability Under the Voluntary Submission Program.A drug dependent under the voluntary submission program, who is finally discharged from confinement, shall be exempt from the criminal liability under Section 15 of this Act subject to the following conditions: xxxx School children, the US Supreme Court noted, are most vulnerable to the physical, psychological, and addictive effects of drugs. Maturing nervous systems of the young are more critically impaired by intoxicants and are more inclined to drug dependency. Their recovery is also at a depressingly low rate.497[15] The right to privacy has been accorded recognition in this jurisdiction as a facet of the right protected by the guarantee against unreasonable search and seizure 498[16] under

RA 9165, Sec. 2. Vernonia School District 47J v. Acton, 515 U.S. 646 (1995), 661. 498[16] Ople v. Torres, G.R. No. 127685, July 23, 1998, 293 SCRA 141, 169; citing Morfe v. Mutuc, No. L20387, January 31, 1968, 22 SCRA 424, 444-445.
497[15]

496[14]

386

387

Sec. 2, Art. III499[17] of the Constitution. But while the right to privacy has long come into its own, this case appears to be the first time that the validity of a state-decreed search or intrusion through the medium of mandatory random drug testing among students and employees is, in this jurisdiction, made the focal point. Thus, the issue tendered in these proceedings is veritably one of first impression. US jurisprudence is, however, a rich source of persuasive jurisprudence. With respect to random drug testing among school children, we turn to the teachings of Vernonia School District 47J v. Acton (Vernonia) and Board of Education of Independent School District No. 92 of Pottawatomie County, et al. v. Earls, et al. (Board of Education),500[18] both fairly pertinent US Supreme Court-decided cases involving the constitutionality of governmental search. In Vernonia, school administrators in Vernonia, Oregon wanted to address the drug menace in their respective institutions following the discovery of frequent drug use by school athletes. After consultation with the parents, they required random urinalysis drug testing for the schools athletes. James Acton, a high school student, was denied participation in the football program after he refused to undertake the urinalysis drug testing. Acton forthwith sued, claiming that the schools drug testing policy violated, inter alia, the Fourth Amendment501[19] of the US Constitution. The US Supreme Court, in fashioning a solution to the issues raised in Vernonia, considered the following: (1) schools stand in loco parentis over their students; (2) school children, while not shedding their constitutional rights at the school gate, have less privacy rights; (3) athletes have less privacy rights than non-athletes since the former observe communal undress before and after sports events; (4) by joining the sports activity, the athletes voluntarily subjected themselves to a higher degree of school supervision and regulation; (5) requiring urine samples does not invade a students privacy since a student need not undress for this kind of drug testing; and (6) there is need for the drug testing because of the dangerous effects of illegal drugs on the young. The US Supreme Court held that the policy constituted reasonable search under the Fourth502[20] and 14th Amendments and declared the random drug-testing policy constitutional. In Board of Education, the Board of Education of a school in Tecumseh, Oklahoma required a drug test for high school students desiring to join extra-curricular activities. Lindsay Earls, a member of the show choir, marching band, and academic team declined to undergo a drug test and averred that the drug-testing policy made to apply to non-athletes violated the Fourth and 14th Amendments. As Earls argued, unlike athletes who routinely undergo physical examinations and undress before their peers in locker rooms, non-athletes are entitled to more privacy. The US Supreme Court, citing Vernonia, upheld the constitutionality of drug testing even among non-athletes on the basis of the schools custodial responsibility and authority. In so ruling, said court made no distinction between a non-athlete and an
499[17] Sec. 2. The right of the people to be secure in their persons, houses, papers, and effects against unreasonable searches and seizures of whatever nature and for any purpose shall be inviolable, and no search warrant or warrant of arrest shall issue except upon probable cause to be determined personally by the judge after examination under oath or affirmation of the complainant and the witnesses he may produce, and particularly describing the place to be searched and the person or things to be seized. 500[18] 536 U.S. 822 (2002); cited in 2 Bernas, CONSTITUTIONAL RIGHTS AND SOCIAL DEMANDS 224-227 (2004). 501[19] The right of the people to be secure in their persons, houses, papers, and effects, against unreasonable searches and seizures, shall not be violated, and no Warrants shall issue, but upon probable cause, supported by Oath or affirmation, and particularly describing the place to be searched, and the persons or things to be seized. 502[20] The Fourth Amendment is almost similar to Sec. 2, Art. III of the Constitution, except that the latter limited the determination of probable cause to a judge after an examination under oath of the complainant and his witnesses. Hence, pronouncements of the US Federal Supreme Court and State Appellate Court may be considered doctrinal in this jurisdiction, unless they are manifestly contrary to our Constitution. See Herrera, HANDBOOK ON ARREST, SEARCH AND SEIZURE 8 (2003).

387

388

athlete. It ratiocinated that schools and teachers act in place of the parents with a similar interest and duty of safeguarding the health of the students. And in holding that the school could implement its random drug-testing policy, the Court hinted that such a test was a kind of search in which even a reasonable parent might need to engage. In sum, what can reasonably be deduced from the above two cases and applied to this jurisdiction are: (1) schools and their administrators stand in loco parentis with respect to their students; (2) minor students have contextually fewer rights than an adult, and are subject to the custody and supervision of their parents, guardians, and schools; (3) schools, acting in loco parentis, have a duty to safeguard the health and well-being of their students and may adopt such measures as may reasonably be necessary to discharge such duty; and (4) schools have the right to impose conditions on applicants for admission that are fair, just, and non-discriminatory. Guided by Vernonia and Board of Education, the Court is of the view and so holds that the provisions of RA 9165 requiring mandatory, random, and suspicionless drug testing of students are constitutional. Indeed, it is within the prerogative of educational institutions to require, as a condition for admission, compliance with reasonable school rules and regulations and policies. To be sure, the right to enroll is not absolute; it is subject to fair, reasonable, and equitable requirements. The Court can take judicial notice of the proliferation of prohibited drugs in the country that threatens the well-being of the people,503[21] particularly the youth and school children who usually end up as victims. Accordingly, and until a more effective method is conceptualized and put in motion, a random drug testing of students in secondary and tertiary schools is not only acceptable but may even be necessary if the safety and interest of the student population, doubtless a legitimate concern of the government, are to be promoted and protected. To borrow from Vernonia, [d]eterring drug use by our Nations schoolchildren is as important as enhancing efficient enforcement of the Nations laws against the importation of drugs; the necessity for the State to act is magnified by the fact that the effects of a drug-infested school are visited not just upon the users, but upon the entire student body and faculty.504[22] Needless to stress, the random testing scheme provided under the law argues against the idea that the testing aims to incriminate unsuspecting individual students. Just as in the case of secondary and tertiary level students, the mandatory but random drug test prescribed by Sec. 36 of RA 9165 for officers and employees of public and private offices is justifiable, albeit not exactly for the same reason. The Court notes in this regard that petitioner SJS, other than saying that subjecting almost everybody to drug testing, without probable cause, is unreasonable, an unwarranted intrusion of the individual right to privacy,505[23] has failed to show how the mandatory, random, and suspicionless drug testing under Sec. 36(c) and (d) of RA 9165 violates the right to privacy and constitutes unlawful and/or unconsented search under Art. III, Secs. 1 and 2 of the Constitution.506[24] Petitioner Lasernas lament is just as simplistic, sweeping, and gratuitous and does not merit serious consideration. Consider what he wrote without elaboration:

Tolentino v. Alconcel, No. L-63400, March 18, 1983, 121 SCRA 92, 95-96. Rollo (G.R. No. 158633), p. 204, respondents Consolidated Memorandum. 505[23] Rollo (G.R. No. 157870), p. 10. 506[24] Section 1. No person shall be deprived of life, liberty, or property without due process of law, nor shall any person be denied the equal protection of the laws. Sec. 2. The right of the people to be secure in their persons, houses, papers, and effects against unreasonable searches and seizures of whatever nature and for any purpose shall be inviolable, and no search warrant or warrant of arrest shall issue except upon probable cause to be determined personally by the judge after examination under oath or affirmation of the complainant and the witnesses he may produce, and particularly describing the place to be searched and the person or things to be seized.
504[22]

503[21]

388

389

The US Supreme Court and US Circuit Courts of Appeals have made various rulings on the constitutionality of mandatory drug tests in the school and the workplaces. The US courts have been consistent in their rulings that the mandatory drug tests violate a citizens constitutional right to privacy and right against unreasonable search and seizure. They are quoted extensively hereinbelow.507[25] The essence of privacy is the right to be left alone.508[26] In context, the right to privacy means the right to be free from unwarranted exploitation of ones person or from intrusion into ones private activities in such a way as to cause humiliation to a persons ordinary sensibilities. 509[27] And while there has been general agreement as to the basic function of the guarantee against unwarranted search, translation of the abstract prohibition against unreasonable searches and seizures into workable broad guidelines for the decision of particular cases is a difficult task, to borrow from C. Camara v. Municipal Court.510[28] Authorities are agreed though that the right to privacy yields to certain paramount rights of the public and defers to the states exercise of police power.511[29] As the warrantless clause of Sec. 2, Art III of the Constitution is couched and as has been held, reasonableness is the touchstone of the validity of a government search or intrusion.512[30] And whether a search at issue hews to the reasonableness standard is judged by the balancing of the government-mandated intrusion on the individuals privacy interest against the promotion of some compelling state interest.513[31] In the criminal context, reasonableness requires showing of probable cause to be personally determined by a judge. Given that the drug-testing policy for employeesand students for that matterunder RA 9165 is in the nature of administrative search needing what was referred to in Vernonia as swift and informal disciplinary procedures, the probable-cause standard is not required or even practicable. Be that as it may, the review should focus on the reasonableness of the challenged administrative search in question. The first factor to consider in the matter of reasonableness is the nature of the privacy interest upon which the drug testing, which effects a search within the meaning of Sec. 2, Art. III of the Constitution, intrudes. In this case, the office or workplace serves as the backdrop for the analysis of the privacy expectation of the employees and the reasonableness of drug testing requirement. The employees privacy interest in an office is to a large extent circumscribed by the companys work policies, the collective bargaining agreement, if any, entered into by management and the bargaining unit, and the inherent right of the employer to maintain discipline and efficiency in the workplace. Their privacy expectation in a regulated office environment is, in fine, reduced; and a degree of impingement upon such privacy has been upheld. Just as defining as the first factor is the character of the intrusion authorized by the challenged law. Reduced to a question form, is the scope of the search or intrusion clearly set forth, or, as formulated in Ople v. Torres, is the enabling law authorizing a search narrowly drawn or narrowly focused?514[32] The poser should be answered in the affirmative. For one, Sec. 36 of RA 9165 and its implementing rules and regulations (IRR), as couched, contain provisions specifically directed towards preventing a situation that would unduly embarrass the employees or place them under a humiliating experience. While every officer and employee in a private
507[25] 508[26]

Rollo (G.R. No. 158633), p. 9. Ople, supra note 16, at 153; citing Cooley on Torts, Sec. 135, Vol. 1, 4th ed., [1932]. 509[27] 62 Am. Jur. 2d, Privacy, Sec. 1. 510[28] 387 U.S. 523; cited in 2 Bernas, supra note 18, at 232. 511[29] 62 Am. Jur. 2d, Privacy, Sec. 17. 512[30] Vernonia & Board of Education, supra notes 15 & 18. 513[31] Skinner v. Railway Labor Executives Assn., 489 U.S. 602, 619 (1989); cited in Vernonia, supra. 514[32] Supra note 16, at 166 & 169.

389

390

establishment is under the law deemed forewarned that he or she may be a possible subject of a drug test, nobody is really singled out in advance for drug testing. The goal is to discourage drug use by not telling in advance anyone when and who is to be tested. And as may be observed, Sec. 36(d) of RA 9165 itself prescribes what, in Ople, is a narrowing ingredient by providing that the employees concerned shall be subjected to random drug test as contained in the companys work rules and regulations x x x for purposes of reducing the risk in the work place. For another, the random drug testing shall be undertaken under conditions calculated to protect as much as possible the employees privacy and dignity. As to the mechanics of the test, the law specifies that the procedure shall employ two testing methods, i.e., the screening test and the confirmatory test, doubtless to ensure as much as possible the trustworthiness of the results. But the more important consideration lies in the fact that the test shall be conducted by trained professionals in access-controlled laboratories monitored by the Department of Health (DOH) to safeguard against results tampering and to ensure an accurate chain of custody.515[33] In addition, the IRR issued by the DOH provides that access to the drug results shall be on the need to know basis;516[34] that the drug test result and the records shall be [kept] confidential subject to the usual accepted practices to protect the confidentiality of the test results.517[35] Notably, RA 9165 does not oblige the employer concerned to report to the prosecuting agencies any information or evidence relating to the violation of the Comprehensive Dangerous Drugs Act received as a result of the operation of the drug testing. All told, therefore, the intrusion into the employees privacy, under RA 9165, is accompanied by proper safeguards, particularly against embarrassing leakages of test results, and is relatively minimal. To reiterate, RA 9165 was enacted as a measure to stamp out illegal drug in the country and thus protect the well-being of the citizens, especially the youth, from the deleterious effects of dangerous drugs. The law intends to achieve this through the medium, among others, of promoting and resolutely pursuing a national drug abuse policy in the workplace via a mandatory random drug test. 518[36] To the Court, the need for drug testing to at least minimize illegal drug use is substantial enough to override the individuals privacy interest under the premises. The Court can consider that the illegal drug menace cuts across gender, age group, and social- economic lines. And it may not be amiss to state that the sale, manufacture, or trafficking of illegal drugs, with their ready market, would be an investors dream were it not for the illegal and immoral components of any of such activities. The drug problem has hardly abated since the martial law public execution of a notorious drug trafficker. The state can no longer assume a laid back stance with respect to this modern-day scourge. Drug enforcement agencies perceive a mandatory random drug test to be an effective way of preventing and deterring drug use among employees in private offices, the threat of detection by random testing being higher than other modes. The Court holds that the chosen method is a reasonable and enough means to lick the problem. Taking into account the foregoing factors, i.e., the reduced expectation of privacy on the part of the employees, the compelling state concern likely to be met by the search, and the well-defined limits set forth in the law to properly guide authorities in the conduct of the random testing, we hold that the challenged drug test requirement is, under the limited context of the case, reasonable and, ergo, constitutional.
Under Sec. 7 [3] of the DOH IRR Governing Licensing and Accreditation of Drug Laboratories, a laboratory is required to use documented chain of custody procedures to maintain control and custody of specimens. 516[34] DOH IRR Governing Licensing and Accreditation of Drug Laboratories, Sec. 7 [10.3] provides that the original copy of the test results form shall be given to the client/donor, copy furnished the DOH and the requesting agency. 517[35] Id., Sec. 7 [10.4]. 518[36] Secs. 47 and 48 of RA 9165 charge the Department of Labor and Employment with the duty to develop and promote a national drug prevention program and the necessary guidelines in the work place, which shall include a mandatory drafting and adoption of policies to achieve a drug-free workplace.
515[33]

390

391

Like their counterparts in the private sector, government officials and employees also labor under reasonable supervision and restrictions imposed by the Civil Service law and other laws on public officers, all enacted to promote a high standard of ethics in the public service.519[37] And if RA 9165 passes the norm of reasonableness for private employees, the more reason that it should pass the test for civil servants, who, by constitutional command, are required to be accountable at all times to the people and to serve them with utmost responsibility and efficiency.520[38] Petitioner SJS next posture that Sec. 36 of RA 9165 is objectionable on the ground of undue delegation of power hardly commends itself for concurrence. Contrary to its position, the provision in question is not so extensively drawn as to give unbridled options to schools and employers to determine the manner of drug testing. Sec. 36 expressly provides how drug testing for students of secondary and tertiary schools and officers/employees of public/private offices should be conducted. It enumerates the persons who shall undergo drug testing. In the case of students, the testing shall be in accordance with the school rules as contained in the student handbook and with notice to parents. On the part of officers/employees, the testing shall take into account the companys work rules. In either case, the random procedure shall be observed, meaning that the persons to be subjected to drug test shall be picked by chance or in an unplanned way. And in all cases, safeguards against misusing and compromising the confidentiality of the test results are established. Lest it be overlooked, Sec. 94 of RA 9165 charges the DDB to issue, in consultation with the DOH, Department of the Interior and Local Government, Department of Education, and Department of Labor and Employment, among other agencies, the IRR necessary to enforce the law. In net effect then, the participation of schools and offices in the drug testing scheme shall always be subject to the IRR of RA 9165. It is, therefore, incorrect to say that schools and employers have unchecked discretion to determine how often, under what conditions, and where the drug tests shall be conducted. The validity of delegating legislative power is now a quiet area in the constitutional landscape.521[39] In the face of the increasing complexity of the task of the government and the increasing inability of the legislature to cope directly with the many problems demanding its attention, resort to delegation of power, or entrusting to administrative agencies the power of subordinate legislation, has become imperative, as here. Laserna Petition (Constitutionality of Sec. 36[c], [d], [f], and [g] of RA 9165) Unlike the situation covered by Sec. 36(c) and (d) of RA 9165, the Court finds no valid justification for mandatory drug testing for persons accused of crimes. In the case of students, the constitutional viability of the mandatory, random, and suspicionless drug testing for students emanates primarily from the waiver by the students of their right to privacy when they seek entry to the school, and from their voluntarily submitting their persons to the parental authority of school authorities. In the case of private and public employees, the constitutional soundness of the mandatory, random, and suspicionless drug testing proceeds from the reasonableness of the drug test policy and requirement. We find the situation entirely different in the case of persons charged before the public prosecutors office with criminal offenses punishable with six (6) years and one
CODE OF CONDUCT AND ETHICAL STANDARDS FOR PUBLIC OFFICERS AND EMPLOYEES, Sec. 2. 520[38] CONSTITUTION, Art. XI, Sec. 1. 521[39] Tatad, supra note 6, at 351.
519[37]

391

392

(1) day imprisonment. The operative concepts in the mandatory drug testing are randomness and suspicionless. In the case of persons charged with a crime before the prosecutors office, a mandatory drug testing can never be random or suspicionless. The ideas of randomness and being suspicionless are antithetical to their being made defendants in a criminal complaint. They are not randomly picked; neither are they beyond suspicion. When persons suspected of committing a crime are charged, they are singled out and are impleaded against their will. The persons thus charged, by the bare fact of being haled before the prosecutors office and peaceably submitting themselves to drug testing, if that be the case, do not necessarily consent to the procedure, let alone waive their right to privacy.522[40] To impose mandatory drug testing on the accused is a blatant attempt to harness a medical test as a tool for criminal prosecution, contrary to the stated objectives of RA 9165. Drug testing in this case would violate a persons right to privacy guaranteed under Sec. 2, Art. III of the Constitution. Worse still, the accused persons are veritably forced to incriminate themselves. 1. Self-incrimination, 24 SCRA 692 2. Read 1. Chavez vs. CA, 24 SCRA 663 2. Galman vs. Pamaran, 138 SCRA 294, read concurring and dissenting opinions 3. Villaflor vs. Summers, 41 Phil. 62 4. Beltran vs. Samson, 50 Phil. 570 5. Bagadiong vs. Gonzales, 94 SCRA 906 6. BASECO vs. PCGG, supra 7. Isabela Sugar vs. Macadaeg, 98 Phil. 995 8. Fernando vs. Maglanoc, 95 Phil. 431 9. US vs. Tang Teng, 23 Phil. 145 10. P vs. Otadora, 86 Phil. 244 11. P vs. Olvis, 154 SCRA 513 12. P vs. Boholst-Amadore, 152 SCRA 263 13. P vs. Rosas, 148 SCRA 464 14. P vs. Ruallo, 152 SCRA 635 15. P vs. Policarpio, 158 SCRA 85( Compare with Boholst cases) 16. P vs. Lumayok, 139 SCRA 1 17. Cabal vs. Kapunan, Jr. December 29, 1962

including the

the Rosas &

PEOPLE VS. BANIHIT, G.R. NO. 132045, 339 SCRA 86, AUG. 25, 2000; PEOPLE VS. CONTINENTE, G.R. NOS. 100801- 02, 339 SCRA 1, AUG. 25, 2000. The essence of this right against self-incrimination is testimonial compulsion or the giving of evidence against oneself through a testimonial act. Hence, an accused may be compelled to submit to physical examination and have a substance taken from his body for medical determination as to whether he was suffering from a disease that was contracted by his victim without violating this right. CHAPTER XVII - THE RIGHT AGAINST INVOLUNTARY SERVITUDE 1. Read:
522[40]

Leona Pasion Viuda de Garcia v. Locsin, 65 Phil. 689, 695 (1938); citing Cooley, CONST. LIM. 630

(8th ed.).

392

393

1. Aclaracion vs. Gatmaitan, 64 SCRA 131 2. Caunca vs. Salazar, supra CHAPTER XVIII - RIGHT AGAINST CRUEL AND UNUSUAL PUNISHMENT a. Is the Death Penalty already abolished by the Read: 1. P vs. Gavarra, 155 SCRa 327 2. P vs. Masangkay, 155 SCRA 113 3. P vs. Atencio, 156 SCRA 242 4. P vs. Intino, September 26, 1988 5. People vs. Munoz, 170 SCRA 107 b. Is death as a penalty a cruel or unuasual punishment? 1987 Constitution?

No. Death through lethal injection is the most humane way of implementing the death Penalty (Leo Echegaray vs. Secretary of Justice) Read: 1. P vs. Estoista, 93 Phil. 647 2. P vs. Villanueva,, 128 SCRA 488 3. Veniegas vs. People, 115 SCRA 79 4. P vs. Camano, 115 SCRA 688 2. On the death penalty whether it was abolished or not Read: a. P vs. Idnay, 164 SCRA 358

CHAPTER XIX - RIGHT AGAINST NON-IMPRISONMENT FOR DEBT 1. Read: 2. Ajeno vs. Incierto, 71 SCRA 166 CHAPTER XX - THE RIGHT AGAINST DOUBLE JEOPARDY 1. Requisites present before this right can be invoked PEOPLE VS. ALMARIO, 355 SCRA 1 There is double jeopardy when there is: [1] valid indictment; [2] before a competent court; [3] after arraignment; [4] when a valid plea has been entered; and
393

394

[5] when the defendant was convicted or acquitted, or the case was dismissed or otherwise terminated without the express consent of the accused. If the dismissal is through the instance of the accused or with his express consent, there is no double jeopardy and the case could be reinstated. However, this rule admits of two (2) exceptions: 1) the motion to dismiss is based on insufficiency of evidence; and 2) the motion to dismiss is based on the denial of the accuseds right to speedy trial. This is so because the dismissal is actually an acquittal and therefore, all the requisites of double jeopardy are complete.

JEFFREY RESO DAYAP vs. PRETZY-LOU SENDIONG, GENESA SENDIONG, ELVIE SY and DEXIE DURAN, G.R. No. 177960, January 29, 2009 The case had its origins in the filing of an Information523[4] on 29 December 2004 by the Provincial Prosecutors Office, Sibulan, Negros Oriental, charging herein petitioner Jeffrey Reso Dayap with the crime of Reckless Imprudence resulting to Homicide, Less Serious Physical Injuries, and Damage to Property. The pertinent portion of the information reads: That at about 11:55 oclock in the evening of 28 December 2004 at Brgy. Maslog, Sibulan, Negros Oriental, Philippines, and within the jurisdiction of this Honorable Court, the above-named accused, did then and there, willfully, unlawfully and feloniously drive in a reckless and imprudent manner a 10-wheeler cargo truck with plate number ULP-955, color blue, fully loaded with sacks of coconut shell, registered in the name of Ruben Villabeto of Sta. Agueda Pamplona, Negros Oriental, thereby hitting an automobile, a Colt Galant with plate number NLD-379 driven by Lou Gene R. Sendiong who was with two female passengers, namely: Dexie Duran and Elvie Sy, thus causing the instantaneous death of said Lou Gene R. Sendiong, less serious physical injuries on the bodies of Dexie Duran and Elvie Sy and extensive damage to the above-mentioned Colt Galant which is registered in the name of Cristina P. Weyer of 115 Dr. V. Locsin St., Dumaguete City, to the damage of the heirs of the same Lou Gene R. Sendiong and the other two offended parties abovementioned. An act defined and penalized by Article 365 of the Revised Penal Code. On 10 January 2005, before the Municipal Trial Court (MTC) of Sibulan, Negros Oriental, petitioner was arraigned and he pleaded not guilty to the charge.524[5]

523[4]

Records, p. 32. Rollo, p. 44; See Order dated 10 January 2005.

524[5]

394

395

On 17 January 2005, respondents Pretzy-Lou P. Sendiong, Genesa Sendiong and Dexie Duran filed a motion for leave of court to file an amended information.525[6] They sought to add the allegation of abandonment of the victims by petitioner, thus: The driver of the 10wheeler cargo truck abandoned the victims, at a time when said [LouGene] R. Sendiong was still alive inside the car; he was only extracted from the car by the by-standers.526[7] On 21 January 2005, however, the Provincial Prosecutor filed an Omnibus Motion praying that the motion to amend the information be considered withdrawn.527[8] On 21 January 2003, the MTC granted the withdrawal and the motion to amend was considered withdrawn.528[9] Pre-trial and trial of the case proceeded. Respondents testified for the prosecution. After the prosecution had rested its case, petitioner sought leave to file a demurrer to evidence which was granted. Petitioner filed his Demurrer to Evidence529[10] dated 15 April 2005 grounded on the prosecutions failure to prove beyond reasonable doubt that he is criminally liable for reckless imprudence, to which respondents filed a Comment530[11] dated 25 April 2005. In the Order531[12] dated 16 May 2005, the MTC granted the demurrer and acquitted petitioner of the crime of reckless imprudence. The MTC found that the evidence presented by respondents failed to establish the allegations in the Information. Pertinent portions of the order state: An examination of the allegations in the information and comparing the same with the evidence presented by the prosecution would reveal that the evidence presented has not established said allegations. The facts and circumstances constituting the allegations charged have not been proven. It is elementary in the rules of evidence that a party must prove his own affirmative allegations. xxxx Nowhere in the evidence of the prosecution can this Court find that it was the accused who committed the crime as charged. Its witnesses have never identified the accused as the one who has committed the crime. The prosecution never bothered to establish if indeed it was the accused who committed the crime or asked questions which would have proved the elements of the crime. The prosecution did not even establish if indeed it was the accused who was driving the truck at the time of the incident. The Court simply cannot find any evidence which would prove that a crime has been committed and that the accused is the person responsible for it.

525[6]

Records, pp. 34-36. Id. at 37. Id. at 41. Rollo, p. 55. Records, pp. 80-92. Id. at 93-94. Rollo, pp. 72-74.

526[7]

527[8]

528[9]

529[10]

530[11]

531[12]

395

396

There was no evidence on the allegation of the death of Lou Gene R. Sendiong as there was no death certificate that was offered in evidence. The alleged less serious physical injuries on the bodies of Dexie Duran and Elvie Sy were not also proven as no medical certificate was presented to state the same nor was a doctor presented to establish such injuries. The alleged damage to the [C]olt [G]alant was also not established in any manner as no witness ever testified on this aspect and no documentary evidence was also presented to state the damage. The prosecution therefore failed to establish if indeed it was the accused who was responsible for the death of Lou Gene R. Sendiong and the injuries to Dexie Duran and Elvie Sy, including the damage to the Colt Galant. The mother of the victim testified only on the expenses she incurred and the shock she and her family have suffered as a result of the incident. But sad to say, she could not also pinpoint if it was the accused who committed the crime and be held responsible for it. This Court could only say that the prosecution has practically bungled this case from its inception. xxxx The defense furthermore argued that on the contrary, the prosecutions [evidence] conclusively show that the swerving of vehicle 1 [the Colt Galant] to the lane of vehicle 2 [the cargo truck] is the proximate cause of the accident. The court again is inclined to agree with this argument of the defense. It has looked carefully into the sketch of the accident as indicated in the police blotter and can only conclude that the logical explanation of the accident is that vehicle 1 swerved into the lane of vehicle 2, thus hitting the latters inner fender and tires. Exhibit 7 which is a picture of vehicle 2 shows the extent of its damage which was the effect of vehicle 1s ramming into the rear left portion of vehicle 2 causing the differential guide of vehicle 2 to be cut, its tires busted and pulled out together with their axle. The cutting of the differential guide cause[d] the entire housing connecting the tires to the truck body to collapse, thus causing vehicle 2 to tilt to its left side and swerve towards the lane of vehicle 1. It was this accident that caused the swerving, not of [sic] any negligent act of the accused. xxxx Every criminal conviction requires of the prosecution to prove two thingsthe fact of the crime, i.e., the presence of all the elements of the crime for which the accused stands charged, and the fact that the accused is the perpetrator of the crime. Sad to say, the prosecution has miserably failed to prove these two things. When the prosecution fails to discharge its burden of establishing the guilt of the accused, an accused need not even offer evidence in his behalf. xxxx WHEREFORE, premises considered, the demurrer is granted and the accused JEFFREY RESO DAYAP is hereby acquitted for insufficiency of evidence. The bail bond posted for his temporary liberty is also hereby cancelled and ordered released to the accused or his duly authorized representative. SO ORDERED.532[13]

396

397

Respondents thereafter filed a petition for certiorari under Rule 65,533[14] alleging that the MTCs dismissal of the case was done without considering the evidence adduced by the prosecution. Respondents added that the MTC failed to observe the manner the trial of the case should proceed as provided in Sec. 11, Rule 119 of the Rules of Court as well as failed to rule on the civil liability of the accused in spite of the evidence presented. The case was raffled to the Regional Trial Court (RTC) of Negros Oriental, Br. 32. In the order534[15] dated 23 August 2005, the RTC affirmed the acquittal of petitioner but ordered the remand of the case to the MTC for further proceedings on the civil aspect of the case. The RTC ruled that the MTCs recital of every fact in arriving at its conclusions disproved the allegation that it failed to consider the evidence presented by the prosecution. The records also demonstrated that the MTC conducted the trial of the case in the manner dictated by Sec. 11, Rule 119 of the Rules of Court, except that the defense no longer presented its evidence after the MTC gave due course to the accuseds demurrer to evidence, the filing of which is allowed under Sec. 23, Rule 119. The RTC however agreed that the MTC failed to rule on the accuseds civil liability, especially since the judgment of acquittal did not include a declaration that the facts from which the civil liability might arise did not exist. Thus, the RTC declared that the aspect of civil liability was not passed upon and resolved to remand the issue to the MTC. The dispositive portion of the decision states: WHEREFORE, the questioned order of the Municipal Trial Court of Sibulan on accuseds acquittal is AFFIRMED. The case is REMANDED to the court of origin or its successor for further proceedings on the civil aspect of the case. No costs. SO ORDERED.535[16]

Both parties filed their motions for reconsideration of the RTC order, but these were denied for lack of merit in the order536[17] dated 12 September 2005. Respondents then filed a petition for review with the Court of Appeals under Rule 42, docketed as CA-G.R. SP. No. 01179. The appellate court subsequently rendered the assailed decision and resolution. The Court of Appeals ruled that there being no proof of the total value of the properties damaged, the criminal case falls under the jurisdiction of the RTC and the proceedings before the MTC are null and void. In so ruling, the appellate court cited Tulor v. Garcia (correct title of the case is Cuyos v. Garcia)537[18] which ruled that in complex crimes involving reckless imprudence resulting in homicide or physical injuries and damage
532[13]

Id. at 72 and 74. Records, pp. 3-11. Rollo, pp. 75-81. Id. at 81. Id. at 89-90. No. L-46934, 15 April 1998, .

533[14]

534[15]

535[16]

536[17]

537[18]

397

398

to property, the jurisdiction of the court to take cognizance of the case is determined by the fine imposable for the damage to property resulting from the reckless imprudence, not by the corresponding penalty for the physical injuries charged. It also found support in Sec. 36 of the Judiciary Reorganization Act of 1980 and the 1991 Rule 8 on Summary Procedure, which govern the summary procedure in first-level courts in offenses involving damage to property through criminal negligence where the imposable fine does not exceed P10,000.00. As there was no proof of the total value of the property damaged and respondents were claiming the amount of P1,500,000.00 as civil damages, the case falls within the RTCs jurisdiction. The dispositive portion of the Decision dated 17 August 2006 reads: WHEREFORE, premises considered, judgment is hereby rendered by Us REMANDING the case to the Regional Trial Court (RTC), Judicial Region, Branch 32, Negros Oriental for proper disposition of the merits of the case. SO ORDERED.538[19] Petitioner moved for reconsideration of the Court of Appeals decision,539[20] arguing that jurisdiction over the case is determined by the allegations in the information, and that neither the 1991 Rule on Summary Procedure nor Sec. 36 of the Judiciary Reorganization Act of 1980 can be the basis of the RTCs jurisdiction over the case. However, the Court of Appeals denied the motion for reconsideration for lack of merit in the Resolution dated 25 April 2007.540[21] It reiterated that it is the RTC that has proper jurisdiction considering that the information alleged a willful, unlawful, felonious killing as well as abandonment of the victims. In the present petition for review, petitioner argues that the MTC had jurisdiction to hear the criminal case for reckless imprudence, owing to the enactment of Republic Act (R.A.) No. 7691,541[22] which confers jurisdiction to first-level courts on offenses involving damage to property through criminal negligence. He asserts that the RTC could not have acquired jurisdiction on the basis of a legally unfiled and officially withdrawn amended information alleging abandonment. Respondents are also faulted for challenging the MTCs order acquitting petitioner through a special civil action for certiorari under Rule 65 in lieu of an ordinary appeal under Rule 42. The petition has merit. It should be granted. The first issue is whether the Court of Appeals erred in ruling that jurisdiction over the offense charged pertained to the RTC. Both the MTC and the RTC proceeded with the case on the basis of the Information dated 29 December 2004 charging petitioner only with
538[19]

Rollo, p. 35. Id. at 90-94. Supra note 2.

539[20]

540[21]

Entitled AN ACT EXPANDING THE JURISDICTION OF THE METROPOLITAN TRIAL COURTS, MUNICIPAL TRIAL COURTS AND MUNICIPAL CIRCUIT TRIAL COURTS, AMENDING FOR THE PURPOSE BATAS PAMBANSA BLG. 129, OTHERWISE KNOWN AS THE JUDICIARY REORGANIZATION ACT OF 1980, which took effect on 14 April 1994.

541[22]

398

399

the complex crime of reckless imprudence resulting to homicide, less serious physical injuries and damage to property. The Court of Appeals however declared in its decision that petitioner should have been charged with the same offense but aggravated by the circumstance of abandonment of the victims. It appears from the records however that respondents attempt to amend the information by charging the aggravated offense was unsuccessful as the MTC had approved the Provincial Prosecutors motion to withdraw their motion to amend the information. The information filed before the trial court had remained unamended.542[23] Thus, petitioner is deemed to have been charged only with the offense alleged in the original Information without any aggravating circumstance. Article 365 of the Revised Penal Code punishes any person who, by reckless imprudence, commits any act which, had it been intentional, would constitute a grave felony, with the penalty of arresto mayor in its maximum period to prision correccional in its medium period. When such reckless imprudence the use of a motor vehicle, resulting in the death of a person attended the same article imposes upon the defendant the penalty of prision correccional in its medium and maximum periods. The offense with which petitioner was charged is reckless imprudence resulting in homicide, less serious physical injuries and damage to property, a complex crime. Where a reckless, imprudent, or negligent act results in two or more grave or less grave felonies, a complex crime is committed.543[24] Article 48 of the Revised Penal Code provides that when the single act constitutes two or more grave or less grave felonies, or when an offense is a necessary means for committing the other, the penalty for the most serious crime shall be imposed, the same to be applied in its maximum period. Since Article 48 speaks of felonies, it is applicable to crimes through negligence in view of the definition of felonies in Article 3 as acts or omissions punishable by law committed either by means of deceit (dolo) or fault (culpa).544[25] Thus, the penalty imposable upon petitioner, were he to be found guilty, is prision correccional in its medium period (2 years, 4 months and 1 day to 4 years) and maximum period (4 years, 2 months and 1 day to 6 years). Applicable as well is the familiar rule that the jurisdiction of the court to hear and decide a case is conferred by the law in force at the time of the institution of the action, unless such statute provides for a retroactive application thereof.545[26] When this case was filed on 29 December 2004, Section 32(2) of Batas Pambansa Bilang 129 had already been amended by R.A. No. 7691. R.A. No. 7691 extended the jurisdiction of the first-level courts over criminal cases to include all offenses punishable with imprisonment not exceeding six (6) years irrespective of the amount of fine, and regardless of other imposable accessory or other penalties including those for civil liability. It explicitly states that in offenses involving damage to property through criminal

542[23]

See notes 8 and 9. People v. de los Santos, 407 Phil. 724, 744 (2001, citing Reodica v. Court of Appeals, 292 SCRA 87,

543[24]

102 (1998).
544[25]

People v. de los Santos, 407 Phil. 724 (2001).

Venancio Figueroa y Cervantes v. People of the Philippines, G.R. No. 147406, 14 July 2008, citing Alarilla v. Sandiganbayan, 393 Phil. 143, 155 (2000) and Escovar v. Justice Garchitorena, 466 Phil. 625, 635 (2004).

545[26]

399

400

negligence, they shall have exclusive original jurisdiction thereof. It follows that criminal cases for reckless imprudence punishable with prision correccional in its medium and maximum periods should fall within the jurisdiction of the MTC and not the RTC. Clearly, therefore, jurisdiction to hear and try the same pertained to the MTC and the RTC did not have original jurisdiction over the criminal case.546[27] Consequently, the MTC of Sibulan, Negros Oriental had properly taken cognizance of the case and the proceedings before it were valid and legal. As the records show, the MTC granted petitioners demurrer to evidence and acquitted him of the offense on the ground of insufficiency of evidence. The demurrer to evidence in criminal cases, such as the one at bar, is filed after the prosecution had rested its case, and when the same is granted, it calls for an appreciation of the evidence adduced by the prosecution and its sufficiency to warrant conviction beyond reasonable doubt, resulting in a dismissal of the case on the merits, tantamount to an acquittal of the accused.547[28] Such dismissal of a criminal case by the grant of demurrer to evidence may not be appealed, for to do so would be to place the accused in double jeopardy.548[29] But while the dismissal order consequent to a demurrer to evidence is not subject to appeal, the same is still reviewable but only by certiorari under Rule 65 of the Rules of Court. Thus, in such case, the factual findings of the trial court are conclusive upon the reviewing court, and the only legal basis to reverse and set aside the order of dismissal upon demurrer to evidence is by a clear showing that the trial court, in acquitting the accused, committed grave abuse of discretion amounting to lack or excess of jurisdiction or a denial of due process, thus rendering the assailed judgment void.549[30] Accordingly, respondents filed before the RTC the petition for certiorari alleging that the MTC gravely abused its discretion in dismissing the case and failing to consider the evidence of the prosecution in resolving the same, and in allegedly failing to follow the proper procedure as mandated by the Rules of Court. The RTC correctly ruled that the MTC did not abuse its discretion in dismissing the criminal complaint. The MTCs conclusions were based on facts diligently recited in the order thereby disproving that the MTC failed to consider the evidence presented by the prosecution. The records also show that the MTC correctly followed the procedure set forth in the Rules of Court. The second issue is whether the Court of Appeals erred in ordering the remand of the case of the matter of civil liability for the reception of evidence. We disagree with the Court of Appeals on directing the remand of the case to the RTC for further proceedings on the civil aspect, as well as with the RTC in directing a similar remand to the MTC.

546[27]

Venancio Figueroa y Cervantes v. People of the Philippines, G.R. No. 147406, 14 July 2008.

People v. Sandiganbayan, 488 Phil. 293, 310 (2004), citing People v. City of Silay, No. L-43790, 9 December 1976, 74 SCRA 247.
548[29]

547[28]

Id. People v. Uy, G.R. No. 158157, 30 September 2005, 471 SCRA 668.

549[30]

400

401

The acquittal of the accused does not automatically preclude a judgment against him on the civil aspect of the case. The extinction of the penal action does not carry with it the extinction of the civil liability where: (a) the acquittal is based on reasonable doubt as only preponderance of evidence is required; (b) the court declares that the liability of the accused is only civil; and (c) the civil liability of the accused does not arise from or is not based upon the crime of which the accused is acquitted. 550[31] However, the civil action based on delict may be deemed extinguished if there is a finding on the final judgment in the criminal action that the act or omission from which the civil liability may arise did not exist551[32] or where the accused did not commit the acts or omission imputed to him.552[33] Thus, if demurrer is granted and the accused is acquitted by the court, the accused has the right to adduce evidence on the civil aspect of the case unless the court also declares that the act or omission from which the civil liability may arise did not exist.553[34] This is because when the accused files a demurrer to evidence, he has not yet adduced evidence both on the criminal and civil aspects of the case. The only evidence on record is the evidence for the prosecution. What the trial court should do is issue an order or partial judgment granting the demurrer to evidence and acquitting the accused, and set the case for continuation of trial for the accused to adduce evidence on the civil aspect of the case and for the private complainant to adduce evidence by way of rebuttal. Thereafter, the court shall render judgment on the civil aspect of the case.554[35] A scrutiny of the MTCs decision supports the conclusion that the acquittal was based on the findings that the act or omission from which the civil liability may arise did not exist and that petitioner did not commit the acts or omission imputed to him; hence, petitioners civil liability has been extinguished by his acquittal. It should be noted that the MTC categorically stated that it cannot find any evidence which would prove that a crime had been committed and that accused was the person responsible for it. It added that the prosecution failed to establish that it was petitioner who committed the crime as charged since its witnesses never identified petitioner as the one who was driving the cargo truck at the time of the incident. Furthermore, the MTC found that the proximate cause of the accident is the damage to the rear portion of the truck caused by the swerving of the Colt Galant into the rear left portion of the cargo truck and not the reckless driving of the truck by petitioner, clearly establishing that petitioner is not guilty of reckless imprudence. Consequently, there is no more need to remand the case to the trial court for proceedings on the civil aspect of the case, since petitioners acquittal has extinguished his civil liability. ****************** It must be pointed out, however, that in PEOPLE VS. TAMPAL, 244 SCRA 202 and PEOPLE VS. LEVISTE, 255 SCRA 238, the SC

550[31]

Hun Hyung Park v. Eung Won Choi, G.R. No. 165496, 12 February 2007, 515 SCRA 502, 513. RULES OF COURT, Rule 111, Sec. 2, last par. Salazar v. People, 458 Phil. 504 (2003). Id. at 607. Id. at 518-519.

551[32]

552[33]

553[34]

554[35]

401

402

reversed the dismissal of the criminal case by the trial court based on speedy trial since the same was not predicated on the clear right of the accused to speedy trial. It is only when there is a clear violation of the accuseds right to speedy trial that the dismissal results in double jeopardy. 3. Double jeopardy, 102 SCRA 44 and 12 SCRA 561 4. When the act is punished by both a law and an ordinance: PEOPLE VS. RELOVA, 148 SCRA 292 If the accused was charged of theft of electricity based on the City Ordinance of Batangas and not based on the Revised Penal Code and later on the case is dismissed by the judge due to the fact that the crime has prescribed, the government can no longer charge the accused of the same crime under the Revised Penal Code since double jeopardy has set in. Read: 2. P vs. Duero, 104 SCRA 379 3. CUDIA VS. CA, 284 SCRA 173 4. CUISON VS. CA, 289 SCRA 159 2. P vs. Jara, 144 SCRA 516 3. P vs. Abano, 145 SCRA 555 4. P vs. Tolentino, 145 SCRA 597 5. P vs. Salig, 133 SCRA 59 6. P vs. Cruz, 133 SCRA 426 7. P vs. Prudente,, 133 SCRA 651 8 P vs. Trinidad, 162 SCRA 714, when the regularity does7, 1966 2. P vs. City Court,154 SCRA 175 3. Galman vs. Pamaran, 144 SCRA 43 4. P vs. Molero, 144 SCRA 397 5. P vs. Quibate, 131 SCRA 81 6. P vs. Obania, June 29,1968 7. Dionaldo vs. Dacuycuy, 108 SCRA 736 8. P vs. Judge Hernando, 108 SCRA 121 9. Esmena vs. Judge Pogoy, 102 SCRA 861 10. Mazo vs. Mun. Court, 113 SCRA 217 11. Andres vs. Cacdac, 113 SCRA 217 12. Buerano vs. CA, 115 SCRA 82 13. P vs. Militante, 117 SCRA 910 14. P vs. Fuentebella, 100 SCRA 672 15. Lazaro vs. P, 112 SCRA 430 16. Flores vs. Enrile, 115 SCRA 236 17. Bernarte vs. Sec. ,116 SCRA 43 18. Ko Bu Lin vs. CA, 118 SCRA 573 19. P vs. Duran, 1075 SCRA 979 20. P vs. Cuevo, 104 SCRA 312 21. Jimenez vs. Military Commission, 102 SCRA 39 22. P vs. Liwanag, 73 SCRA 473 23. P vs. Araula, January 30, 1982 24. P vs. Baladjay, March 30, 1982 25. P vs. City Court of Silay, 74 SCRA 247 28. P vs. Pilpa, 79 SCRA 81 29. P vs. Gloria, December 29, 1977 30. P vs. Galano, 75 SCRA 193 31. Tacas vs. Cariasco, 72 SCRA 527

presumption of

402

403

32. P vs. Ledesma, 73 SCRA 77 33. P vs. Consulta, 70 SCRA 277 34. P vs. Inting, 70 SCRA 289 35. De Guzman vs. Escalona, 97 SCRA 619 36. P vs. Pablo, 98 SCRA 289 37. Cruz vs. Enrile, 160 SCRA 700 38. Tangan vs. P, 155 SCRA 435 39. P vs. Quezada, 160 SCRA 516 40. Canizano vs. P, 159 SCRA 599 41. Bustamante vs. Maceren, 48 SCRA 144 There is no double jeopardy in this case: PEOPLE VS. MOLERO G.R No. L-67842, September 24, 1986 FACTS: 1. Molero was charged for having raped his daughter. The original complaint was dated March 22, 1977, the complainant charged Molero of having raped her on the "13th day of February 1976". 2. Molero was arraigned and pleaded "Not Guilty"; 3. During the trial, the complainant testified that she was raped by her father on February 5, 1976 and not February 13, 1976 as alleged in the complaint; 4. The Fiscal filed a motion for leave to amend the complaint. The motion was granted but was subsequently reconsidered. The lower court in its order dismissed the original complaint, but ordered the Fiscal to cause the filing of a new complaint charging the proper offense of rape committed on or before February 5, 1976; 5. A new complaint was therefore filed dated March 30, 1978 6. Molero claims that the new complaint places him in double jeopardy. HELD: There is no double jeopardy. a. Dismissal of the first case contemplated by the rule against double jeopardy presupposes a definite and unconditional dismissal which terminates the case.(Jaca vs. Blanco, 86 Phil. 452; People vs. Manlapas, 5 SCRA 883; People vs. Mogol, 131 SCRA 296) And "for dismissal to be a bar under the jeopardy clause of the Constitution, it must have the effect of acquittal.(People vs. Agoncillo, 40 SCRA 579); b. It is quite clear that the order of the trial court dismissal the original complaint was without prejudice to the filing of a new complaint and/or information charging Molero with the proper offense. The said dismissal did not therefore amount to an acquittal. c. In fact there was no need for the trial court to have adopted such a cumbersome procedure. It could have merely ordered an amendment of the complaint. Sec. 12, Rule 119 of the Revised Rules of Court applies
403

404

when there is a mistake in charging the proper offense, but not when an honest error of a few days is sought to be corrected and the change does not affect the rights of the accused. d. The precise time of the commission of the crime is not an essential element of the offense of rape. The amendment of the complaint changing the date of the commission of the crime of rape from February 13, 1976 to February 5, 1976 , a difference of 8 days was only a matter of form under the facts of this case and did not prejudice the rights of the accused. e. The reliance of the accused on the case of People vs. Opemia, 98 Phil. 698 is not well-taken. In the said case the proposed amendment was the changing of the date of the commission of the crime from June 18, 1952 to July 1947, or a difference of 5 years. The S.C. held that the amendment that would change the date of the commission of the offense from 1947 to 1952 is certainly not a matter of form. f. The dismissal of the first complaint did not amount to the appellant's acquittal. In effect, the order of dismissal does not constitute a proper basis for a claim of double jeopardy. (People vs. Bocar, 138 SCRA 166) 5. May the government appeal a judgment of acquittal or of the penalty imposed? No. for the increase

PEOPLE VS. HON. VELASCO, G.R. NO. 127444, 340 SCRA 207, SEPT. 13, 2000. Double Jeopardy. Evolution of doctrine. Appeal by the Government from verdicts of acquittal. As mandated by the Constitution, statutes and cognate jurisprudence, an acquittal is final and unappealable on the ground of double jeopardy, whether it happens at the trial court of a judgment of acquittal brought before the Supreme Court on certiorari cannot be had unless there is a finding of mistrial, as in Galman vs. Sandiganbayan. 6. May the appellate court of the Supreme Court increase the pernalty imposed by the trial court on appeal by the accused? Yes. PEOPLE VS. DOMINGO, G.R. No. 184343, March 2, 2009 Appellant Jesus Domingo assails the Decision555[1] of the Court of Appeals dated 30 April 2008 in CA-G.R. CR No. 30511, modifying the Decision556[2] dated 13 November 2006 of Branch 13 of the Regional Trial Court (RTC) of Malolos, Bulacan. The Court of Appeals found appellant guilty beyond reasonable doubt of murder in Criminal Cases No. 1496-M2000 and No. 1497-M-2000, attempted murder in Criminal Cases No. 1498-M-2000 and No. 1501-M-2000, frustrated murder in Criminal Case No. 1500-M-2000, and frustrated homicide in Criminal Case No. 1499-M2000.

555[1]

556[2]

Penned by Associate Justice Fernanda Lampas Peralta with Associate Justices Edgardo P. Cruz and Apolinario D. Bruselas, Jr., concurring; rollo, pp. 2-25. Penned by Presiding Judge Andres B. Soriano; CA rollo, pp. 11-23.

404

405

On 7 September 2000, appellant, with the assistance of counsel, was arraigned and he entered separate pleas of Not Guilty to the crimes charged. Thereafter, pre-trial conference was held, and trial ensued accordingly.557[4] The accused was convicted of Homicide, instead of Murder. On Appeal to the Court of Appeals, the CA held that the crime proven by the prosecution is Murder and therefore increased the penalty from Homicide to Murder. Is increase in the penalty valid? Yes because it was the accused who interposed the appeal making the court review the evidence. And it it finds the evidence sufficient for conviction of the crime of Murder, it could increase the penalty. Read: 1. Central Bank of the Philippines vs. CA, GR No. 8, 1989 1-a. P vs. Montemayor, January 30, 1969, 26 SCRA 2. P vs. Ruiz,81 SCRA 455 3. US vs. Yam Tung Way, 21 Phil. 67 4. P vs. Ang ho Kio, 95 Phil. 475 6. The "Supervening Fact Doctrine." Read: 1. 76 SCRA 469 2. P vs. Tarok, 73 Phil. 260 3. P vs. Villasis, 46 O.G. 268 4. Melo vs. People, 85 Phil. 766 5. P vs. Buling, 107 Phil. 712 5-a. P vs. Adil, 76 SCRA 462 5-b. P. vs. Tac-an, 182 SCRA 601 6. P vs. City Court of Manila, 121 SCRA 637 7. Read also Sec. 7, Rule 117, 1985 Rules on 41859, March 687

Criminal Procedure

CHAPTER XXI RIGHT AGAINST EX-POST FACTO LAW, BILL OF ATTAINER, ETC. Read: 1. Nunez vs. Sandiganbayan, 111 SCRA 433 2-LACSON VS. SANDIGANBAYAN, January 20, 1999 PANFILO M. LACSON VS. THE EXECUTIVE SECRETARY, THE SANDIGANBAYAN, ET AL. ROMEO ACOP & FRANCISCO ZUBIA, JR., PetitionersIntervenors G.R. No. 128096, January 20, 1999

557[4]

Id. at 13.

405

406

The petitioner seeks to stop the Sandiganbayan from trying the multiple murder case against him and 26 other police officers for the death of 11 Kuratong Baleleng members in the early morning of May 18, 1995 at Commonwealth Avenue, Quezon City. The police officers claimed that it was a shoot-out between them and the Kuratong Baleleng Members while SPO2 Eduardo de los Reyes claimed it was a summary execution or rub-out. The preliminary investigation conducted by the Deputy Ombudsman for Military Affairs resulted in the dismissal of the cases after finding that the incident was a legitimate police operation. However, the Review Board led by Deputy Ombudsman Francisco Villa resulted in the filing of multiple murder cases against the petitioner and his companion where he was indicted as a principal. Upon motion by the petitioner and his co-police officers with leave from the Sandiganbayan, a Motion for Reconsideration was filed with the Office of the Ombudsman who AMENDED the 11 informations on March 1, 1996 charging the petitioner , ROMEO ACOP and FRANCISCO ZUBIA, JR., as mere accessories. On March 5-6,1996, the accused questioned the jurisdiction of the Sandiganbayan over the 11 criminal cases since under Republic Act No. 7975, particularly Section 2, paragraphs [a] and [c], the said court has jurisdiction only if one or more of the principal accused has a rank of Brigadier General (Chief Superintendent) or higher and since the highest PNP officer charged as a principal accused is merely Chief Inspector, the Regional Trial Court of Quezon City has jurisdiction to try and decide the same. On May 8, 1996, the Sandiganbayan issued a Resolution transferring the case to the RTC of Quezon City which has original and exclusive jurisdiction over the cases under RA 7975. On May 17, 1996, the Office of the Special Prosecutor moved for a Reconsideration and insisted that the cases should remain with the Sandiganbayan which was opposed by the petitioner and his co-accused. While the Motions for Reconsideration were pending before the Sandiganbayan, Congress passed into law Republic Act No. 8249 which was approved by the President on February 5, 1997 entitled AN ACT FURTHER DEFINING THE JURISDICTION OF THE SANDIGANBAYAN, AMENDING FOR THIS PURPOSE PD 1606, AS AMENDED, PROVIDING FUNDS THEREFOR which deleted the word PRINCIPAL in Section 2, paragraphs [a] and [c] of RA 7975 thereby giving jurisdiction to the Sandiganbayan criminal cases involving police generals like the petitioners even though they are not charged as principals but merely accessories or accomplices. The new law further provides that it shall be applicable to all cases which are pending in court before the passage of the same provided trial has not begun at the time of its approval. On March 5, 1997, the Sandiganbayan issued its Resolution denying the Motion for Reconsideration of the Office of the Special Prosecutor and ruled that it stands pat in its Resolution dated May 8, 1996 ordering the transfer of the 11 criminal cases to the RTC of Quezon City. On the same day, however, the Sandiganbayan issued an ADDENDUM to its March 5, 1997 Resolution where it that with the
406

407

passage of RA 8249, the court admitted the amended informations in these cases and by the unanimous vote of 4 with 1 neither concurring nor dissenting, retained jurisdiction to try and decide the cases. The petitioner questioned the said Resolution Sandiganbayan to the Supreme Court on the following grounds: of the

1. their right to due process of law and equal protection of the law was violated as a result of the application of the new law by which restored to the Sandiganbayan jurisdiction over their cases especially so that the Sandiganbayan has foot-dragged for 9 months the resolution of the pending incident involving the transfer of these cases to the RTC of Quezon City and waited for the passage of the law to overtake such resolution and thereby rendering their vested rights under the old Sandiganbayan law moot; 2. the retroactive application of the new law violates their constitutional right against ex-post facto law; 3. the title of the law is misleading in that it contains the aforesaid innocuous provisions in Sections 4 and 7 which actually expands rather than defines the old Sandiganbayan law thereby violating the one title one subject requirement of Section 26 [1] Article VI of the Constitution. The petitioners-intervenors claimed that while the law (Sections 4 and 7) innocuously appears to have merely expanded the jurisdiction of the Sandiganbayan, it is in fact a class legislation and an ex-post facto law statute intended specifically to apply to all the accused in the Kuratong Baleleng case pending before the Sandiganbayan. Finally, if their case will be tried by the Sandiganbayan, they will be deprived of their two-tiered appeal to the Sandiganbayan which they acquire under RA 7975 before recourse to the Supreme Court could be made. Held: 1. The contention that the law violates petitioners right to due process and equal protection of the law is too shallow to deserve merit. It is an established precept in constitutional law that the guaranty of the equal protection of the laws is not violated by a legislation based on reasonable classification. The classification is reasonable and not arbitrary when there is concurrence of four elements, namely: a. b. c. d. it must rest on real and substantial distinctions; it must be germane to the purposes of the law; must not be limited to existing conditions only; and must apply equally to all members of the same classall of which are present in this case. The classification between those pending cases involving concerned public officials whose trial has not yet commenced and whose cases could have been affected by the amendments of the Sandiganbayan jurisdiction under RA 8249, as against those whose cases where trial has already started as of the approval of the law rests on substantial distinction that makes real differences. In the 1st instance, evidence against them were not yet presented, whereas in the latter the parties have already submitted their respective proofs, examined witnesses and presented documents.
407

408

Since it is within the power of Congress to define the jurisdiction of the courts, it can be reasonably anticipated that an alteration of that jurisdiction necessarily affect pending cases, which is why it has to provide for a remedy in the form of a transitory provision. The transitory provision does not only cover cases which are in the Sandiganbayan but also in any court. It just happened that the Kuratong Baleleng cases are one of those affected by the law. Moreover, those cases where trial has already begun are not affected by the transitory provision under Section 7 of the new law (RA 8249). 2. The petitioners argument that the retroactive application of the new law to the Kuratong Baleleng cases constitutes an ex post facto law for they are deprived of their right to due process as they can no longer avail of the two-tiered appeal which they had allegedly acquired under RA 7975 is without merit. In order that a law is an ex post facto law, the same must be one a. which makes an act done criminal before the passing of the law and which was innocent when committed, and punishes such action; b. which aggravates a crime or makes it greater than when it was committed; c. which changes the punishment and inflicts a greater punishment than the law annexed to the crime when it was committed; d. which alters the legal rules of evidence and receives less or different testimony than the law required a the time of the commission of the offense in order to convict the defendant; e. every law which, in relation to the offense or its consequences, alters the situation of a person to his disadvantage; f. that which assumes to regulate civil rights and remedies but in effect imposes a penalty or deprivation of a right which when done was lawful; g. deprives a person accused of a crime of some lawful protection to which he has become entitled, such as the protection of a former conviction or acquittal, or a proclamation of amnesty (KAY VILLEGAS KAMI, 35 SCRA 429; MEJIA VS. PAMARAN, 160 SCRA 457; TAN VS. BARRIOS, 190 SCRA 686; PEOPLE VS. SANDIGANBAYAN, 211 SCRA 241). Ex post facto law prohibits the retrospectivity of penal laws. RA 8249 is not a penal law. It is a substantive law on jurisdiction which is not penal in character. The other contention that their right to a two-tiered appeal which they acquired under RA 7975 has been diluted by the enactment of RA 8249 is incorrect. The same contention had been rejected by the court several times in the cases of RODRIGUEZ VS. SANDIGANBAYAN, 205 Phil. 567; ALVIAR VS. SANDIGANBAYAN, 137 SCRA 63; NUNEZ VS. SANDIGANBAYAN, 111 SCRA 433; DE GUZMAN VS. PEOPLE, December 15, 1982 considering that the right to appeal is not a natural right but statutory in nature that can be regulated by law. The mode of procedure provided for in the statutory right of appeal is not included in the prohibition against ex post facto laws. Moreover, the new law did not alter the rules of evidence or the mode of trial.
408

409

3. The contention that the new Sandiganbayan law violates the one title-one subject provision of the Constitution is without merit. The petitioners claim that the new does not define the jurisdiction of the Sandiganbayan but expands the same. But even assuming that that is true, the expansion of the jurisdiction, does not have to be expressly stated in the title of the law because such is the necessary consequence of the amendments. The requirement that every bill must only have one subject expressed in the title is satisfied if the title is comprehensive enough, as in this case, to include subjects related to the general purpose which the statute seeks to achieve. The Congress, in employing the word define in the title of the law, acted within its power since Section 2, Article VIII of the Constitution itself empowers the legislative body to define, prescribe and apportion the jurisdiction of various courts. (NOTE: Though the Supreme Court rejected all the above arguments raised by the petitioner and the intervenors who are against the trial of their cases with the Sandiganbayan and prefer to have their cases be tried and decided by the RTC of Quezon City, they got what they want in the end because it was held that the 11 criminal informations failed to alleged that they committed the crimes in relation to their public office which is a jurisdictional requirement in order that the same be tried by the Sandiganbayan. Finally, sometime in May, 1999, the Quezon City RTC to whom the said cases were raffled DISMISSED the 11 murder cases as a result of the retraction made by the eyewitnesses. The same was revived by the DOJ in April, 2001. The same was returned to the QC RTC to determine if the 2-year provisional rule under the 2000 Rules on Criminal Procedure is applicable) 2.a. Kay Villegas Kami, 35 SCRA 429 3. Sevilleja vs. COMELEC, 107 SCRA 141 4. P vs. Ferrer, 46 & 56 SCRA 5. Tan vs. Barrios, October 18, 1990

409

Vous aimerez peut-être aussi